Você está na página 1de 244

1

LIVRO OLÍMPICO -
VOLUME I

ELETROQUÍMICA
CINÉTICA QUÍMICA
TERMODINÂMICA

EXERCÍCIOS
OLÍMPICOS

ALEXANDRE
VARGAS GRILLO

LIVRO OLÍMPICO – VOLUME I


2

DADOS CADASTRAIS DA OBRA

Livro Olímpico – Volume I – Eletroquímica/Cinética

Química/Termodinâmica

Primeira edição – Fevereiro de 2021

Autor: Alexandre Vargas Grillo

Email: alexandre.grillo@ifrj.edu.br

Diagramação: Professor Alexandre Vargas Grillo

Revisão Ortográfica: Professor Alexandre Vargas Grillo

ISBN: 978-65-00-16293-6; Rio de Janeiro / Brasil

LIVRO OLÍMPICO – VOLUME I


3

AGRADECIMENTOS

Dedico este trabalho primeiramente а Deus e também os meus guias


espirituais presentes em todos os momentos da minha vida.

À minha mãе Estela Vargas Grillo, meus pais Vincenzo Grillo e Jorge Luiz
Zaupa e minha irmã Denise Vargas Grillo.

Dedico “In Memorian” este trabalho ao meu grande mestre, JOÃO ROBERTO
DA PACIÊNCIA NABUCO, por ter sido um amigo, pai, companheiro e também
mestre da vida e da química,

Ao meu amigo e mestre e eterno orientador, Professor Dr. Francisco José


Moura.

Agradecido por tudo e o meu muito obrigado.

“Assim como casas são feitas de pedras, a ciência não é feita de fatos. Mas
uma pliha de pedras não é uma casa e uma coleção de fatos não é,
necessariamente, ciência.”

Jules Henri Poicare (1854-1912)

LIVRO OLÍMPICO – VOLUME I


4

SOBRE O AUTOR

Alexandre Vargas Grillo é Doutor em Engenharia de Materiais e Processos

Químicos e Metalúrgicos pela PUC-Rio, Mestrado em Engenharia de

Materiais e Processos Químicos e Metalúrgicos pela mesma instituição (PUC-

Rio), pós-graduação em licenciatura de ensino fundamental e médio em

química pela Cândido Mendes e Engenheiro Químico pela Pontifícia

Universidade Católica do Rio de Janeiro.

Atualmente atua como Professor do Instituto Federal do Rio de Janeiro – IFRJ

– Campus Nilópolis, lecionando Físico-Química Avançada e Nanotecnologia

para os cursos de Bacharelado, Licenciatura e Técnico.

Na pesquisa atua como professor colaborador em Engenharia de Processos

Químicos e Metalúrgicos em Síntese de Nanopartículas pela PUC-Rio e

também no laboratório de Modelagem, Automação e Controle (LaMaC) da

mesma instituição. Revisor do Journal of Materials Science e consultor Ad

Hoc Faperj e membro da coordenação de Olimpíadas de Química do Estado

do Rio de Janeiro – OQRJ, atuando na área científica no preparo para a IChO

e Iberoamericana.

Coordenador da turma olímpica de Química do Instituto Federal do Rio de

Janeiro – Campus Nilópolis.

Área de atuação: Nanoparticles, Titanium Nitride, Characterization, Physical

Chemistry, Chemical Kinetics, Artificial Intelligence, Neural networks.

LIVRO OLÍMPICO – VOLUME I


5

APRESENTAÇÃO DA OBRA

O principal objetivo de realizar esta obra foi apresentar de maneira direta e


objetiva questões olímpicas, abordando os seguintes temas: o estudo da
eletroquímica, cinética química e a uma abordagem da termodinâmica. Diante
desta complexidade, foi bastante desafiador abordar temas tão complexos
como os apresentados.
O presente trabalho apresenta três capítulos bem importantes, sendo
apresentados na forma de exercícios (247 no total), abrangendo exercícios
olímpicos à nível nacional e internacional, com 100% do seu gabarito
comentado.
O principal objetivo de realizar esta obra foi compilar as principais olimpíadas
de Química à nível nacional e internacional, reunindo diversos exercícios de
alto nível, apresentando também exercícios do IME, ITA e ITT-JEE (questões
indianas). São mais de vinte anos pesquisando e estudando este mundo
olímpico da química. São problemas muito desafiadores que requer uma
abordagem que serve para todos os graus, tanto à nível de segundo grau
quanto à terceiro. Os temas escolhidos foram com profundo conhecimento e
sempre estão presentes nas diversas provas. Apesar do esforço imenso que
coloquei como desafiador, é de se esperar que uma obra dessa dimensão
deva conter inevitavelmente algum erro ou omissão. Diante disso, aceitarei
com muito apreço que sejam encaminhadas todas estas ideias para o
seguinte endereço eletrônico: alexandre.grillo@ifrj.edu.br. Espero que esta
obra seja útil a todos que queiram aprender todos estes fundamentos desta
disciplina fascinante que se trata da Química.

Fevereiro de 2021
Professor Alexandre Vargas Grillo

LIVRO OLÍMPICO – VOLUME I


6

SUMÁRIO DA OBRA
Capítulo 01 – Eletroquímica – Exercícios ................... página 6

Capítulo 02 – Cinética Química – Exercícios .............. página 94

Capítulo 03 – Termodinâmica – Exercícios ................. página 164

APÊNDICE ................................................................... página 234

REFERÊNCIAS BIBLIOGRÁFICAS ..............................página 241

LIVRO OLÍMPICO – VOLUME I


7

CAPÍTULO 01

ELETROQUÍMICA –
EXERCÍCIOS

ALEXANDRE
VARGAS GRILLO

LIVRO OLÍMPICO – VOLUME I


8

Capítulo 01 - Eletroquímica – Exercícios


1. DEDUÇÃO DA EQUAÇÃO DE NERNST
2. INSTITUTO MILITAR DE ENGENHARIA
3. OLIMPÍADA PORTUGUESA DE QUÍMICA
4. OLIMPÍADA BRASILEIRA DE QUÍMICA
5. OLIMPÍADA BRASILEIRA DE QUÍMICA
6. OLIMPÍADA BRASILEIRA DE QUÍMICA
7. OLIMPÍADA BRASILEIRA DE QUÍMICA
8. OLIMPÍADA BRASILEIRA DE QUÍMICA
9. OLIMPÍADA MATO-GROSSENSE DE QUÍMICA
10. OLIMPÍADA BAIANA DE QUÍMICA
11. INSTITUTO TECNOLÓGICO DA AERONÁUTICA
12. OLIMPÍADA BAIANA DE QUÍMICA
13. OLIMPÍADA BAIANA DE QUÍMICA
14. OLIMPÍADA BRASILEIRA DE QUÍMICA
15. OLIMPÍADA ALAGOANA DE QUÍMICA
16. INSTITUTO MILITAR DE ENGENHARIA
17. OLIMPÍADA ALAGOANA DE QUÍMICA
18. OLIMPÍADA MINEIRA DE QUÍMICA
19. OLIMPÍADA DE QUÍMICA DO RIO DE JANEIRO
20. GRILLO (QUESTÃO DE PRÓPRIA AUTORIA)
21. ITT-JEE
22. INSTITUTO MILITAR DE ENGENHARIA
23. INSTITUTO TECNOLÓGICO DA AERONÁUTICA
24. GRILLO (QUESTÃO DE PRÓPRIA AUTORIA)
25. INSTITUTO MILITAR DE ENGENHARIA
26. INSTITUTO TECNOLÓGICO DA AERONÁUTICA
27. INSTITUTO TECNOLÓGICO DA AERONÁUTICA
28. INSTITUTO MILITAR DE ENGENHARIA
29. OLIMPÍADA DE QUÍMICA DO RIO DE JANEIRO
30. OLIMPÍADA DE QUÍMICA DO RIO DE JANEIRO
31. U. S. NATIONAL CHEMISTRY OLYMPIAD
32. OLIMPÍADA RORAIMENSE DE QUÍMICA
33. INSTITUTO TECNOLÓGICO DA AERONÁUTICA
34. INSTITUTO TECNOLÓGICO DA AERONÁUTICA
35. GRILLO (QUESTÃO DE PRÓPRIA AUTORIA)
36. INSTITUTO MILITAR DE ENGENHARIA
37. INSTITUTO MILITAR DE ENGENHARIA
38. INSTITUTO MILITAR DE ENGENHARIA
39. OLIMPÍADA PERUANA DE QUÍMICA
40. OLIMPÍADA PERUANA DE QUÍMICA
41. OLIMPÍADA AUSTRALIANA DE QUÍMICA
42. OLIMPÍADA AUSTRALIANA DE QUÍMICA
43. OLIMPÍADA CANADENSE DE QUÍMICA
44. OLIMPÍADA AUSTRALIANA DE QUÍMICA
45. GRILLO (QUESTÃO DE PRÓPRIA AUTORIA)
46. GRILLO (QUESTÃO DE PRÓPRIA AUTORIA)
47. OLIMPÍADA CEARENSE DO ENSINO SUPERIOR DE QUÍMICA
48. OLIMPÍADA PERUANA DE QUÍMICA
49. OLIMPÍADA DE QUÍMICA DO RIO GRANDE DO SUL
50. INSTITUTO MILITAR DE ENGENHARIA
51. INSTITUTO MILITAR DE ENGENHARIA
52. OLIMPÍADA CEARENSE DO ENSINO SUPERIOR DE QUÍMICA
53. INSTITUTO MILITAR DE ENGENHARIA

LIVRO OLÍMPICO – VOLUME I


9

54. INSTITUTO TECNOLÓGICO DA AERONÁITICA


55. OLIMPÍADA DE QUÍMICA DO RIO DE JANEIRO
56. GRILLO (QUESTÃO DE PRÓPRIA AUTORIA)
57. ITT-JEE
58. ITT-JEE
59. ITT-JEE
60. ITT-JEE
61. ITT-JEE
62. ITT-JEE
63. INSTITUTO MILITAR DE ENGENHARIA
64. INSTITUTO MILITAR DE ENGENHARIA
65. INSTITUTO MILITAR DE ENGENHARIA
66. U. S NATIONAL CHEMISTRY OLYMPIAD
67. U. S NATIONAL CHEMISTRY OLYMPIAD
68. INSTITUTO MILITAR DE ENGENHARIA
69. INSTITUTO MILITAR DE ENGENHARIA
70. ITT-JEE
71. ITT-JEE
72. ITT-JEE
73. ITT-JEE
74. GRILLO (QUESTÃO DE PRÓPRIA AUTORIA)
75. ITT-JEE
76. OLIMPÍADA NORTE-NORDESTE DE QUÍMICA
77. OLIMPÍADA NORTE-NORDESTE DE QUÍMICA
78. OLIMPÍADA NORTE-NORDESTE DE QUÍMICA
79. ITT-JEE
80. INSTITUTO TECNOLÓGICO DA AERONÁUTICA
81. CYPRUS NATIONAL COMPETITION FOR THE INTERNATIONAL
CHEMISTRY OLYMPIAD
82. U. S NATIONAL CHEMISTRY OLYMPIAD
83. INSTITUTO MILITAR DE ENGENHARIA
84. INSTITUTO MILITAR DE ENGENHARIA
85. CONCURSO PARA DOCENTE – IFRJ – MODIFICADA
86. OLIMPÍADA BRASILEIRA DE QUÍMICA
87. INSTITUTO MILITAR DE ENGENHARIA
88. GRILLO (QUESTÃO DE PRÓPRIA AUTORIA)
89. GRILLO (QUESTÃO DE PRÓPRIA AUTORIA)
90. GRILLO (QUESTÃO DE PRÓPRIA AUTORIA)
91. ITT-JEE
92. U. S NATIONAL CHEMISTRY OLYMPIAD
93. U. S NATIONAL CHEMISTRY OLYMPIAD
94. QUALITYING EXAMINATIONS FOR APPLICATIONS FOR
JAPANESE GOVERNMENT SCHOLARSHIPS
95. GRILLO (QUESTÃO DE PRÓPRIA AUTORIA)
96. INSTITUTO TECNOLÓGICO DA AERONÁUTICA
97. ITT-JEE
98. ITT-JEE
99. ITT-JEE
100. ITT-JEE
101. ITT-JEE
102. U. S. NATIONAL CHEMISTRY OLYMPIAD
103. U. S. NATIONAL CHEMISTRY OLYMPIAD
104. GRILLO (QUESTÃO DE PRÓPRIA AUTORIA)
105. OLIMPÍADA BRASILEIRA DE QUÍMICA
106. GRILLO (QUESTÃO DE PRÓPRIA AUTORIA)

LIVRO OLÍMPICO – VOLUME I


10

107. ITT-JEE
108. ITT-JEE
109. GRILLO (QUESTÃO DE PRÓPRIA AUTORIA)
110. ITT-JEE

LIVRO OLÍMPICO – VOLUME I


11

1
Dedução da equação de Nernst.

Resposta: A dedução da equação de Nernst é proveniente da equação da


isoterma de van´t Hoff.

∆G = ∆G0 + R x T x ln Q, em que:
ΔG = variação da energia livre de Gibbs fora do estado padrão;
ΔGº = variação da energia livre de Gibbs padrão;
R = constante dos gases ideais (8,314 J/mol x K ou 1,987 cal/mol x K)
T = temperatura absoluta;
Q = coeficiente reacional.

Desenvolvendo a equação, temos: ∆G = ∆G0 + R x T x ln Q


− n x F x E = −n x F x E 0 + R x T x ln Q
Dividindo a equação acima por (n x F), temos:
− n x F x E −n x F x E 0 R x T
= + x ln Q
nxF nxF nxF
RxT
E = E0 − x ln Q
nxF
2
(INSTITUTO MILITAR DE ENGENHARIA) Em uma pilha Ni0 / Ni2+ // Ag+1 /
Ag0, os metais estão mergulhados em soluções aquosas 1,0 mol.L-1 de seus
respectivos sulfatos, a 25°C. Determine:
a) a equação global da pilha;
b) o sentido do fluxo de elétrons;
c) o valor da força eletromotriz (fem) da pilha. Dados:
Ni2+(aq) + 2e- → Ni0(s) (E0redução = - 0,25 V)
Ag+(aq) + 1e- → Ag0(s) (E0redução = + 0,80 V)

Resposta: Para a determinação e funcionamento da pilha, será necessário


inverter a semirreação do níquel e multiplicar por dois (x 2) a semirreação da
prata.

Ni0(s) → Ni2+(aq) + 2e- ∆G0 = −2 x F x (+0,25)

2Ag+(aq) + 2e- → 2Ag0(s) ∆G0 = −2 x F x (+0,80) +

Ni0(s) + 2Ag+(aq) → Ni2+(aq) + 2Ag0(s)


0
∆Gtotal = −2 x F x (+0,25) + [−2 x F x (+0,80)]
0
−2 x F x Etotal = −2 x F x (+0,25) + [−2 x F x (+0,80)]
0
Etotal = 0,25 + 0,80 = +1,05 V

LIVRO OLÍMPICO – VOLUME I


12

Item a) Equação global para o funcionamento da pilha galvânica:


Ni0(s) + 2Ag+(aq) → Ni2+(aq) + 2Ag0(s)
Item b) O fluxo de elétrons sai do ânodo (eletrodo de níquel) para o cátodo
(eletrodo de prata).
0
Item c) Etotal = +1,05 V

3
(OLIMPÍADA PORTUGUESA DE QUÍMICA) Tendo em conta os potenciais
de redução indicados:
E° (Fe+2/Fe) = - 0,44 V
E° (Fe+3/Fe+2) = + 0,77 V
O potencial do par Fe+3/Fe é:
a) + 0,037 V
b) – 0,037 V
c) + 0,330 V
d) + 0,110 V

Resposta: Alternativa B.
Organizando as semirreações:
Fe+2(aq) + 2e- → Fe(s) ∆G0 = −2 x F x (−0,44)

Fe+3(aq) + 1e- → Fe+2(aq) ∆G0 = −1 x F x (+0,77) +

Fe+3(aq) + 3e- → Fe(s)


0
∆Gtotal = −2 x F x (−0,44) + [−1 x F x (+0,77)]
0
−3 x F x Etotal = −2 x F x (−0,44) + [−1 x F x (+0,77)]
0
−3 x Etotal = −2 x (−0,44) + [−1 x (+0,77)]
0
−3 x Etotal = 0,88 − 0,77
0
Etotal = −0,0367 V

Este valor negativo implica que o processo não é espontâneo.

LIVRO OLÍMPICO – VOLUME I


13

4
(OLIMPÍADA BRASILEIRA DE QUÍMICA) A corrente necessária para, no
período de 100 horas, produzir 1 kg de magnésio a partir de cloreto de
magnésio fundido situa-se entre:
a) 5,0 e 10,0 A
b) 10,0 e 15,0 A
c) 15,0 e 20,0 A
d) 20,0 e 25,0 A
e) 25,0 e 30,0 A

Resposta: Alternativa D.
Estudando a eletrólise ígnea do cloreto de magnésio.

MgCl2(s) → Mg +2 −
(l) + 2 Cl(l)

Semirreação que ocorre no cátodo: Mg +2 −


(l) + 2e → Mg (s)

Semirreação que ocorre no ânodo: 2 Cl−


(l) → Cl2(g) + 2e

Somando as semirreações:

MgCl2(s) → Mg +2 −
(l) + 2 Cl(l)
Mg +2 −
(l) + 2e → Mg (s)
2 Cl−
(l) → Cl2(g) + 2e

+
MgCl2(s) → Mg (s) + Cl2(g)

A partir da relação estequiométrica, temos:


96500 𝐶
24 g de magnésio − − − − − 2 mol de elétrons x
mol de elétrons
3600 𝑠
1000 g de magnésio − − − − Q = 100 h x xi
1ℎ
1000 x 2 x 96500
i= = 22,34 A
24 x 100 x 3600

5
(OLIMPÍADA BRASILEIRA DE QUÍMICA) A reação básica que ocorre em
uma cela na qual Al2O3 e sais de alumínio são eletrolisados é:
Al+3 −
(aq) + 3e → Al(s) . Se a cela opera a 5,0 V e 1,0 x 10 A, quantos
5

gramas de alumínio metálico serão depositados em 8 horas de operação da


cela?
a) 27 kg
b) 85 kg
c) 180 kg
d) 270 kg
e) 540 kg

LIVRO OLÍMPICO – VOLUME I


14

Resposta: Alternativa D.
Estudando a eletrólise no cátodo – semirreação do alumínio.

Al+3 −
(aq) + 3e → Al(s)

A partir da relação estequiométrica, temos:

96500 𝐶
27 g de alumínio − − − − − 3 mol de elétrons x
mol de elétrons
3600 𝑠
Massa de alumínio − − − − Q = 8 h x x 1,0 x 105
1h

27 x 8 x 3600 x 1,0 x 105


Massa de alumínio =
3 x 96500

Massa de alumínio = 268601,04 g (268 kg)

6
(OLIMPÍADA BRASILEIRA DE QUÍMICA) Se a quantidade de elétrons,
assim como, a quantidade de cada uma das espécies químicas que intervêm
numa reação de uma pilha, são multiplicadas por dois, então o potencial da
pilha:
a) Aumenta para o dobro
b) Diminui para a metade
c) Eleva-se ao quadrado
d) Fica reduzido à raiz quadrada
e) Não varia

Resposta: Alternativa E.

O potencial químico caracteriza-se por ser uma propriedade termodinâmica


que não é uma função de estado. Diante disso, multiplicar o potencial por dois
não irá variar o seu valor.

7
(OLIMPÍADA BRASILEIRA DE QUÍMICA) Considere uma pilha formada por
duas lâminas metálicas, uma de zinco e outra de cobre imersos em suas
respectivas soluções de Zn2+ e Cu2+ separados por uma ponte salina,
conforme figura ao lado. Nessa pilha, é ligada uma lâmpada entre os
eletrodos e após certo tempo de funcionamento, observa-se que a lâmina de
zinco sofre uma diminuição de massa e a de cobre um aumento. Com relação
a esta pilha é correto afirmar que:

a) O cobre sofre oxidação

LIVRO OLÍMPICO – VOLUME I


15

b) O íon Cu2+ é o agente redutor


c) O eletrodo de zinco é o pólo (–)
d) No cátodo ocorre reação de oxidação
e) O sentido do fluxo de elétrons é do eletrodo de cobre para o de zinco
passando pelo circuito externo

Resposta: Alternativa C.

O eletrodo da esquerda trata-se da oxidação, enquanto o da direita é a


redução. Diante disso, as semirreações são as seguintes:

Cu+2 −
(aq) + 2e → Cu(s) (ânodo)
Zn(s) → Zn+2
(aq) + 2e

(cátodo)
Zn(s) + Cu+2 +2
(aq) → Cu(s) + Zn(aq) (equação química global)

Diante da equação eletroquímica global, o zinco é o agente redutor e íons


cobre é o agente oxidante.
Item a) Falso. O cobre sofre redução.
Item b) Falso. O íon Cu2+ é o agente oxidante.
Item c) Verdadeiro.
Item d) Falso. No cátodo ocorre reação de redução.
Item e) Falso. O sentido do fluxo de elétrons é do eletrodo de zinco para o
eletrodo de cobre.

8
(OLIMPÍADA BRASILEIRA DE QUÍMICA) Um eletrodo de vanádio é oxidado
eletroliticamente. A massa do eletrodo diminui de 114 mg após a passagem
de 650 Coulombs de corrente. Qual o número de oxidação do vanádio no
produto:
a) + 1
b) + 2
c) + 3
d) + 4
e) + 5

Resposta: Alternativa C.
Estudando a eletrólise no cátodo – semirreação do vanádio.
+a
V(aq) + a e− → V(s)

A partir da relação estequiométrica, temos:

96500 𝐶
50,94 g de alumínio − − − − a mol de elétrons x
mol de elétrons
114 x 10−3 g − − − − − − − 650 C

50,94 x 650
a= ≅ + 3,0
114 x 10−3 x 96500

LIVRO OLÍMPICO – VOLUME I


16

8
(OLIMPÍADA MATO-GROSSENSE DE QUÍMICA) O sal de cozinha,
indispensável para salgar e conservar os alimentos pode também ser
formador de um combustível: o gás hidrogênio. Ao colocarmos o sal de
cozinha em meio aquoso e passarmos por essa solução uma corrente
elétrica, é possível produzir H2(g) pela eletrólise. Apesar de ser uma possível
fonte energética para o futuro, o custo do projeto pode ser inviável devido ao
gasto com a energia elétrica para manter um gerador ligado. Uma opção para
sanar o problema seria o uso de fontes alternativas para alimentar o gerador,
como a eólica, a solar fotovoltaica e o biogás. Observe ao lado o esquema de
eletrólise de uma solução de sal de cozinha em meio aquoso usando
eletrodos de grafite. Sobre eletrólise representada, pode-se afirmar que:

a) Haverá formação de gás hidrogênio no ânodo.


b) Haverá formação de gás oxigênio cátodo.
c) Haverá formação de gás cloro no polo positivo.
d) Nessa eletrólise haverá a decomposição da água.

Resposta: Alternativa C.
O tipo de eletrólise que será necessário analisar é a eletrólise aquosa do
cloreto de sódio.

Equação de ionização do cloreto de sódio: NaCl(s) → Na+(aq) + Cl-(aq)


Equação de dissociação da água: H2O(l) → H+(aq) + OH-(aq)

Semirreação que ocorre no cátodo:


Na+(aq) + 1e- → Na (s)
2H+(aq) + 2e- → H2(g)

Quem descarrega com melhor facilidade no cátodo, Na+ ou H+?


Resposta: 2H+(aq) + 2e- → H2(g)

Semirreação que ocorre no ânodo:


2 OH-(aq) → ½ O2(g) + 2e- + 2OH-(aq)
2Cl-(aq) + 2e- → Cl2(g)

Quem descarrega com melhor facilidade no ânodo, Cl- ou OH-?


Resposta: 2Cl-(aq) → Cl2(g) + 2e-

LIVRO OLÍMPICO – VOLUME I


17

Somando todas as semirreações apresentadas.

2NaCl(s) → 2Na+(aq) + 2Cl-(aq)


2H2O(l) → 2H+(aq) + 2OH-(aq)
2H+(aq) + 2e- → H2(g)
2Cl-(aq) → Cl2(g) + 2e- +
+ −
2 NaCl(s) + H2 O(l) → 2Na
⏟ (aq) + 2OH(aq) + H2 (g) + Cl2 (g)
2 NaOH(aq)
Nesta eletrólise aquosa do cloreto de sódio há a formação de soda
cáustica, gás hidrogênio no cátodo (-) e produção de gás cloro no
ânodo (+).

9
(OLIMPÍADA MATO-GROSSENSE DE QUÍMICA) Considere a pilha a seguir
construída com os metais ferro e alumínio à 25oC, 1 atm e com concentrações
de soluções de 1mol.L-1:

Dados: Fe2+(aq) + 2e- → Fe(s) Eº = -0,44 V


Al (aq) + 3e → Al(s)
3+ - Eº = -1,66V
Pode-se afirmar que o ânodo, o cátodo e a diferença de potencial (ddp) dessa
pilha, respectivamente, são:
a) Aℓ; Fe; −2,1V
b) Fe; Aℓ; -1,22V
c) Aℓ; Fe; +1,22V
d) Fe; Aℓ; +2,0V

Resposta: Alternativa C.

Na figura, o ânodo é o eletrodo de alumínio, enquanto o cátodo é o eletrodo


de ferro. Organizando as semirreações, multiplicando a semirreação do ferro
por três e invertendo e multiplicando a semirreação do alumínio por dois,
temos:

3Fe+2(aq) + 6e- → 3Fe(s) ∆G0 = −6 x F x (−0,44)


2Al(s) → 2 Al+3(aq) + 6e- ∆G0 = −6 x F x (+1,66) +
3Fe+2(aq) + 2Al(s) → 3Fe(s) + 2 Al+3(aq)
0
∆Gtotal = −6 x F x (−0,44) + [−6 x F x (+1,66)]
0
−6 x F x Etotal = −6 x F x (−0,44) + [−6 x F x (+1,66)]
0
Etotal = −0,44 + 1,66 = +1,22 V
0
−3 x Etotal = 0,88 − 0,77

3Fe+2(aq) + 2Al(s) → 3Fe(s) + 2 Al+3(aq)

LIVRO OLÍMPICO – VOLUME I


18

A partir da equação química iônica, os íons ferro sofrem redução (agente


oxidante), enquanto o alumínio metálico sofre oxidação (agente redutor).

10
(OLIMPÍADA BAIANA DE QUÍMICA) Pretende-se avaliar se ocorre ou não
corrosão do ferro em meio ácido e determinar a força eletromotriz (f.e.m.)
padrão para esse processo. São dados os seguintes potenciais-padrões de
eletrodos, a 298,15 K:
Fe2+(aq) + 2e- → Fe(s) ε0 = - 0,440 V
2H (aq) + ½ O2(g) + 2e → H2O(l) ε0 = + 1,229 V
+ -

Com base nos dados apresentados, conclui-se que:


a) Ocorre corrosão em meio ácido e a f.e.m. padrão da pilha é de − 0,789 V
b) Ocorre corrosão em meio ácido e a f.e.m. padrão da pilha é de + 0,789 V
c) Ocorre corrosão em meio ácido, e a f.e.m. padrão da pilha é de + 1,669 V
d) O ferro é depositado em meio ácido e a f.e.m. padrão da pilha é de + 0,789
V
e) O ferro é depositado em meio ácido e a f.e.m. padrão da pilha é de + 1,669
V

Resposta: Alternativa C.

Organizando as semirreações, invertendo a semirreação do ferro, temos:

Fe(s) → Fe+2(aq) + 2e- ∆G0 = −2 x F x (+0,440)


2H+ (aq) + ½ O2(g) + 2e- → H2O(l) ∆G0 = −2 x F x (+1,229) +
Fe(s) + 2H+(aq) + ½ O2(g) → Fe+2(aq) + H2O(l))
0
∆Gtotal = −6 x F x (−0,44) + [−6 x F x (+1,66)]
0
−2 x F x Etotal = −2 x F x (+0,440) + [−2 x F x (+1,229)]
0
Etotal = +0,440 + 1,229 = +1,669 V

11
(INSTITUTO TECNOLÓGICO DA AERONÁUTICA) Descreva os
procedimentos utilizados na determinação do potencial de um eletrodo de
cobre Cu(s) | Cu2+(aq). De sua descrição devem constar:
a) A listagem de todo o material (soluções, medidores etc.) necessário para
realizar a medição do potencial do eletrodo em questão.
b) O desenho esquemático do elemento galvânico montado para realizar a
medição em questão. Deixe claro nesse desenho quais são os pólos positivo
e negativo e qual dos eletrodos será o anodo e qual será o catodo, quando
corrente elétrica circular por esse elemento galvânico. Neste último caso,
escreva as equações químicas que representam as reações anódicas e
catódicas, respectivamente.
c) A explicação de como um aumento do valor das grandezas seguintes afeta
o potencial do eletrodo de cobre (Aumenta? Diminui? Não altera?): área do
eletrodo, concentração de cobre no condutor metálico, concentração de íons
cobre no condutor eletrolítico e temperatura.

LIVRO OLÍMPICO – VOLUME I


19

Resposta:

Item a) Duas cubas de vidro, uma lâmina de cobre e uma lâmina de platina,
fio condutor (por exemplo, fio de cobre), ponte salina (tubo de vidro recurvado
contendo solução eletrolítica saturada, com as extremidades fechadas por
algodão), tubo de vidro com entrada para hidrogênio gasoso, soluções
apresentando [H+] = 1,0 mol x L-1 e [Cu2+] = 1,0 mol x L-1, voltímetro (o valor
obtido no voltímetro será positivo para metais que se reduzem no processo).
Temperatura: 25º C. Pressão do H2(g): 1 atm.

Item b)

Semirreação anódica: H2 → 2 H+ + 2 e-
Semirreação catódica: Cu2+ + 2 e- → Cu

Item c) Área do eletrodo: não altera

Concentração do cobre no condutor metálico: não altera

0,0592 [H+ ]
Concentrações de íons cobre: aumenta, pois E = E° − x log [Cu+2 ]
n

RxT
Aumento da temperatura: diminui, pois E = E° − n x F x lnQ

12
(OLIMPÍADA BAIANA DE QUÍMICA) Relativamente à pilha representada a
seguir, foram feitas as seguintes afirmações:
Cu(s) + 2 Ag+(aq) → Cu+2(aq) + 2 Ag(s)
I) O eletrodo de prata é o polo positivo.
II) No ânodo, ocorre a oxidação do cobre.
III) A concentração de íons Ag+ na solução irá diminuir.
IV) A massa da barra de cobre irá diminuir.
São corretas as afirmativas:
a) Somente I e IV
b) Somente II e III
c) Somente II e IV
d) Somente II, III e IV
e) I, II, III e IV

LIVRO OLÍMPICO – VOLUME I


20

Resposta: Alternativa E.

Cu(s) + 2 Ag+(aq) → Cu+2(aq) + 2 Ag(s)

Analisando a equação iônica global de funcionamento da pilha galvânica,


podemos tirar as seguintes conclusões.

1) O cobre sofre oxidação, sendo o cobre metálico o agente redutor.


Sua massa irá diminuir ao longo do funcionamento da pilha.
2) A prata irá sofrer um processo de redução (+), sendo os íons prata
o agente oxidante. Sua massa vai aumentar ao longo do processo
eletroquímico de funcionamento da pilha. A concentração de íons prata na
solução vai diminuir.

Item I) Verdadeiro.
Item II) Verdadeiro.
Item III) Verdadeiro.
Item IV) Verdadeiro.

13
(OLIMPÍADA BAIANA DE QUÍMICA) A corrosão eletroquímica opera como
uma pilha: ocorre transferência de elétrons quando dois metais de diferentes
potenciais são colocados em contato. É o que acontece quando se usa zinco
para proteger tubulações de ferro: o zinco funciona como anodo (sofre
corrosão) e o ferro como catodo. As semirreações são:
Zn2+ + 2e- → Zn(s) Eo = – 0,763V
Fe + 2e → Fe(s)
2+ - Eo = – 0,440 V
Qual é a ddp da pilha?
a) – 1,203 V
b) – 1,100 V
c) – 0,323 V
d) + 0,323 V
e) + 1,203 V

Resposta: Alternativa D.

Para o funcionamento da pilha, será necessário inverter a semirreação de


redução do zinco. Diante desta afirmativa, temos:
Zn(s) → Zn+2(aq) + 2e- ∆G0 = −2 x F x (+0,763)
Fe+2(aq) + 2e-→ Fe(s) ∆G0 = −2 x F x (−0,440) +
Zn(s) + Fe+2(aq) → Zn+2(aq) + Fe(s)
0
∆Gtotal = −2 x F x (+0,763) + [−2 x F x (−0,44)]
0
−2 x F x Etotal = −2 x F x (+0,763) + [−2 x F x (−0,44)]
0
Etotal = +0,763 − 0,44 = +0,323 V

LIVRO OLÍMPICO – VOLUME I


21

14
(OLIMPÍADA BRASILEIRA DE QUÍMICA) Que produtos são formados
durante a eletrólise de uma solução concentrada de cloreto de sódio?
I. Cl2(g)
II. NaOH(aq)
III. H2(g)
a) Somente I
b) Somente II
c) Somente I e II
d) Somente I e III
e) I, II e III

Resposta: Alternativa E.

Equação de ionização do cloreto de sódio: NaCl(s) → Na+(aq) + Cl-(aq)


Equação de dissociação da água: H2O(l) → H+(aq) + OH-(aq)

Semirreação que ocorre no cátodo:


Na+(aq) + 1e- → Na (s)
2H+(aq) + 2e- → H2(g)

Quem descarrega com melhor facilidade no cátodo, Na+ ou H+?


Resposta: 2H+(aq) + 2e- → H2(g)

Semirreação que ocorre no ânodo:


2 OH-(aq) → ½ O2(g) + 2e- + 2OH-(aq)
2Cl-(aq) + 2e- → Cl2(g)

Quem descarrega com melhor facilidade no ânodo, Cl- ou OH-?


Resposta: 2Cl-(aq) → Cl2(g) + 2e-
Somando todas as semirreações apresentadas.

2NaCl(s) → 2Na+(aq) + 2Cl-(aq)


2H2O(l) → 2H+(aq) + 2OH-(aq)
2H+(aq) + 2e- → H2(g)
2Cl-(aq) → Cl2(g) + 2e- +
+ −
2 NaCl(s) + H2 O(l) → 2Na
⏟ (aq) + 2OH(aq) + H2 (g) + Cl2 (g)
2 NaOH(aq)
Nesta eletrólise aquosa do cloreto de sódio há a formação de soda
caústica, gás hidrogênio no cátodo (-) e produção de gás cloro no
ânodo (+).

LIVRO OLÍMPICO – VOLUME I


22

15
(OLIMPÍADA ALAGOANA DE QUÍMICA) Considere uma pilha de
níquel/zinco e as semirreações representadas a seguir, com seus respectivos
potenciais de redução:
Ni+2 + 2e- → Ni Eo = - 0,25 V
Zn+2 + 2e- → Zn Eo = - 0,76 V
A diferença de potencial da pilha é:
a) 0,21 V
b) 0,31 V
c) 0,41 V
d) 0,50 V
e) 0,51 V

Resposta: Alternativa E.
Para o funcionamento da pilha, será necessário inverter a semirreação de
redução do níquel. Diante desta afirmativa, temos:
Zn(s) → Zn+2(aq) + 2e- ∆G0 = −2 x F x (+0,76)
Ni+2(aq) + 2e-→ Ni(s) ∆G0 = −2 x F x (−0,25) +
Zn(s) + Ni+2(aq) → Zn+2(aq) + Ni(s)
0
∆Gtotal = −2 x F x (+0,76) + [−2 x F x (−0,25)]
0
−2 x F x Etotal = −2 x F x (+0,76) + [−2 x F x (−0,25)]
0
Etotal = +0,76 − 0,25 = +0,51 V
16
A) (INSTITUTO MILITAR DE ENGENHARIA) O ácido clorídrico puro,
no estado líquido, pode ser eletrolizado? Por que?
B) (INSTITUTO MILITAR DE ENGENHARIA) As reações eletrolíticas
são, sempre, reações de oxidação e redução? Por que?

Resposta:
Item A) Não, pois não apresenta cargas iônicas (cátions e ânions).
Item B) Sim. Para processos eletrolíticos, uma das principais premissas é que
as reações químicas apresentem transferência de elétrons, ou seja, seja
reação de oxidação e redução.
17
(OLIMPÍADA ALAGOANA DE QUÍMICA) O trabalho produzido por uma pilha
é proporcional à diferença de potencial (ddp) nela desenvolvida quando se
une uma meia-pilha onde a reação eletrolítica de redução ocorre
espontaneamente (catodo) com outra meia pilha onde a reação eletrolítica de
oxidação, ocorre espontaneamente (anodo). Como exemplo temos:
Ag(s) → Ag+ + e- E° = - 0,80 V
Cu(s) → Cu2+ + 2e- E° = - 0,34 V
A ddp da pilha é de:
a) 0,28 V
b) 0,33 V
c) 0,41 V
d) 0,46 V
e) 0,56 V

LIVRO OLÍMPICO – VOLUME I


23

Resposta: Alternativa D.
Para o funcionamento da pilha, será necessário inverter e multiplicar por dois
a semirreação de redução da prata. Diante desta afirmativa, temos:
Cu(s) → Cu+2(aq) + 2e- ∆G0 = −2 x F x (−0,34)
2Ag (aq) + 2e → 2Ag(s)
+ - ∆G0 = −2 x F x (+0,80) +
Cu(s) + 2Ag+ (aq) → Cu+2(aq) + 2Ag(s))
0
∆Gtotal = −2 x F x (−0,34) + [−2 x F x (+0,80)]
0
−2 x F x Etotal = −2 x F x (−0,34) + [−2 x F x (+0,80)]
0
Etotal = −0,34 + 0,80 = + 0,46 V

18
(OLIMPÍADA MINEIRA DE QUÍMICA) As semirreações para as reações
químicas que ocorrem em uma pilha seca (pilha de Leclanché) e seus
respectivos potenciais padrão de redução são:
Zn+2(aq) + 2e- → Zn(s) Eo = - 0,76 V
2 NH4 (aq) + 2 MnO4(s) + 2e → Mn2O3(s) + H2O(l) + 2 NH3(aq)
+ -

Eo = + 0,74 V
Responda as questões que se seguem:
a) Escreva a equação química da reação global que ocorre no interior
de uma pilha seca.
b) Calcule a diferença de potencial eletroquímico (∆E) da pilha de
Leclanché.

Resposta: Para o funcionamento da pilha, será necessário inverter a


semirreação de redução do zinco. Diante desta afirmativa, temos:
Zn(s) → Zn+2(aq) + 2e-
2 NH4+(aq) + 2 MnO4(s) + 2e- → Mn2O3(s) + H2O(l) + 2 NH3(aq) +
Zn(s) + 2 NH4+(aq) + 2 MnO4(s) → Zn+2(aq) + Mn2O3(s) + H2O(l) + 2 NH3(aq)
0
∆Gtotal = −2 x F x (+0,76) + [−2 x F x (+0,74)]
0
−2 x F x Etotal = −2 x F x (+0,76) + [−2 x F x (+0,74)]
0
Etotal = +0,76 + 0,74 = + 1,50 V

Item a) Equação química global:


Zn(s) + 2 NH4+(aq) + 2 MnO4(s) → Zn+2(aq) + Mn2O3(s) + H2O(l) + 2 NH3(aq)
0
Item b) Etotal = +0,76 + 0,74 = + 1,50 V

LIVRO OLÍMPICO – VOLUME I


24

19
(OLIMPÍADA DE QUÍMICA DO RIO DE JANEIRO) Qual a massa de zinco
metálico depositada após a eletrólise de uma solução aquosa de sulfato de
zinco que durou aproximadamente 3,50 horas, sob corrente de 6,00 A?
a) 63,2 g
b) 51,2 g
c) 25,6 g
d) 7,12 mg
e) 427 mg

Resposta: Alternativa C.
Solução de sulfato de zinco (ZnSO4): ZnSO4(s) → Zn+2(aq) + SO4-2(aq)
Semirreação que acontece no cátodo: Zn+2(aq) + 2e- → Zn(s)
3600 s
Conversão do tempo para segundos: 3,50 horas x 1 hora = 12600 s

Cálculo da massa de zinco depositada:


96500 C
65,4 g − − − − − 2 mol de elétrons x
mol de elétrons
Massa − − − − − 12600 x 6,0

12600 x 6,0 x 65,40


Massa = = 25,62 g
2 x 96500

20
(GRILLO) Considere que o potencial-padrão do Cu2+(aq) / Cu é +0,340 V, e o
par Cu+(aq)/Cu é +0,552 V. Determine o valor do potencial – padrão da
seguinte semirreação (Cu2+(aq)/Cu+(aq)) e a sua respectiva constante de
equilíbrio.
Resposta: Organizando as semirreações:

Cu+2 −
(aq) + 2e → Cu(s) ∆G10 = −2 x F x (+0,340)

Cu+ −
(aq) + 1e → Cu(s) ∆G20 = −1 x F x (+0,552)

Invertendo a segunda semirreação:

Cu+2 −
(aq) + 2e → Cu(s) ∆G10 = −2 x F x (+0,340)

Cu(s) → Cu+
(aq) + 1e

∆G20 = −1 x F x (−0,552) +

Cu+2 − +
(aq) + 1e → Cu(aq)
0
∆Gtotal = ∆G10 + ∆G20
0
−1 x F x Etotal = −2 x F x (+0,340) + [−1 x F x (−0,552)]
0
−1 x Etotal = −2 x (+0,340) − (−0,552)
0
Etotal = +0,128 V
Cálculo da constante de equilíbrio (Keq):
0 0
∆Geletroquímica = ∆Gtermodinâmica

LIVRO OLÍMPICO – VOLUME I


25

0
− n x F x Etotal = −R x T x ln K eq
0
n x F x Etotal
ln K eq =
RxT
1 x 96500 x 0,128
ln K eq =
8,314 x (25 + 273)
ln K eq = 4,98

K eq = 𝑒 +4,98 = 146,28

21
(ITT-JEE) Calculate Gibb’s free energy change and equilibrium constant for
the reaction, Ni+2 + 2H2O → Ni(OH)2 + 2H+. Given that
NiO2 + 4H+ + 2e- → Ni+2 + 2H2O E° = + 1,678 V
and
NiO2 + 2H2O + 2e- → Ni(OH)2 + 2OH- E° = – 0,49 V
Resposta: Para a determinação do potencial padrão da equação iônica
apresentada, será necessário inverter a primeira equação iônica.

Ni+2 + 2H2O → NiO2 + 4H+ + 2e- ∆G10 = −2 x F x (−1,678)

NiO2 + 2H2O + 2e- → Ni(OH)2 + 2OH- ∆G20 = −2 x F x (−0,49)


Somando as semirreações, temos:

Ni+2 + 2H2O → NiO2 + 4H+ + 2e- ∆G10 = −2 x F x (−1,678)


NiO2 + 2H2O + 2e- → Ni(OH)2 + 2OH- ∆G20 = −2 x F x (−0,49) +
Ni+2 + 4 H2O → Ni(OH)2 + 4H+ + 2OH-
0
∆Gtotal = ∆G10 + ∆G20

−2 x F x E 0 = −2 x F x (−1,678) + [−2 x F x (−0,49)]

E 0 = −2,168
Cálculo da constante de equilíbrio, em função do potencial calculado:
0 0
∆Gtermodinâmico = ∆Geletroquímico

−R x T x ln K eq = −n x F x E 0

−8,314 x 298 x ln K eq = −2 x 96500 x (−2,168)

ln K eq = −168,88

K eq = 𝑒 (−168,88) = 4,51 𝑥 10−74

Logo, Ni+2 + 2 H2O → Ni(OH)2 + 4H+ + 2OH- e sua constante de equilíbrio é


igual a K eq = 𝑒 (−168,88) = 4,51 𝑥 10−74

LIVRO OLÍMPICO – VOLUME I


26

Para a determinação da constante de equilíbrio da questão, será necessário


levar a equação de dissociação da água com o valor da sua constante de
equilíbrio em consideração. Diante disso,
Ni+2 + 2 H2O → Ni(OH)2 + 4H+ + 2OH- K eq = 4,51 𝑥 10−74

H2O → H+ + OH- K eq = 10−14

Multiplicando por dois a segunda equação iônica, temos:

Ni+2 + 4 H2O → Ni(OH)2 + 4H+ + 2OH- K eq = 4,51 𝑥 10−74

2H2O → 2H+ + 2OH- K eq = (10−14 )2

Diminuindo as equações iônicas, temos:


Ni+2 + 4 H2O → Ni(OH)2 + 4H+ + 2OH- K eq = 4,51 𝑥 10−74
2H2O → 2H+ + 2OH- K eq = (10−14 )2 -
4,51 x 10−74
Ni+2 + 2H2O → Ni(OH)2 + 2H+ K total
eq = (10−14 )2

Cálculo da variação da energia livre de Gibbs padrão:


0
∆Gtermodinâmico = −R x T x ln K total
eq

0 4,51 x 10−74
∆Gtermodinâmico = −8,314 x 298 x ln
(10−14 )2

22
(INSTITUTO MILITAR DE ENGENHARIA) Em duas cubas eletrolíticas,
ligadas em série, ocorrem as reações, cujas equações são mostradas a
seguir, pela passagem de uma corrente elétrica de 1 Ampére:
Cuba A: Ag+(aq) + 1e- → Ag(s)
Cuba B: 2 H+(aq) + 2e- → H2(g)
Pede-se:
a) o tipo de reação que está ocorrendo;
b) a denominação do eletrodo onde ocorrem essas reações;
c) o tempo necessário para que ocorra a deposição de 1,08 g de prata;
d) O volume, em litros nas CNTP, do hidrogênio produzido durante o
tempo determinado na letra C.

Resposta:
Item a) As equações iônicas apresentadas são reações de redução.
Item b) A denominação que se dá no eletrodo se chama cátodo.
Item c) Cálculo do tempo para a deposição de prata:
96500 C
108 g − − − − − 1 mol de elétrons x
mol de elétrons
1,08 g − − − − − Q

1,08 x 1 x 96500
Q= = 965 C
108

LIVRO OLÍMPICO – VOLUME I


27

965
Cálculo do tempo: tempo = = 965 s
1

Item d) Cálculo do volume de gás hidrogênio nas CNTP, sabendo que são 2
mol de elétrons que participam na reação química.
96500 C
22,4 L − − − − − − − − − 2 mol de elétrons x
mol de elétrons
Volume − − − − − − − − 965 C

22,4 x 965
Volume = = 0,0112 L
2 x 96500

23
(INSTITUTO TECNOLÓGICO DA AERONÁUTICA) Descreva como o
hidróxido de sódio é obtido em escala industrial. Sua descrição deve incluir
as matérias-primas utilizadas, as equações das reações químicas envolvidas
no processo, as condições de operação e o aproveitamento de eventuais
subprodutos obtidos no processo.

Resposta:

Para a produção de hidróxido de sódio (soda cáustica) as matérias-primas


utilizadas são cloreto de sódio e água.

⏞ Na+
Equação de dissociação do cloreto de sódio: NaCl(aq) → −
(aq) + Cl(aq)

+ −
Equação de ionização da água: H2 O(l) → H(aq) + OH(aq)

Quem descarrega com melhor facilidade no cátodo, Na+ +


(aq) ou H(aq) ?
+
2H(aq) + 2e− → H2(g)

Quem descarrega com melhor facilidade no ânodo, Cl− −


(aq) ou OH(aq) ?
2Cl−
(aq) → Cl2 (𝑔) + 2𝑒

Multiplicando a primeira e a segunda semirreação por dois e somando todas


as reações, temos:
2 NaCl(aq) → 2 Na+ −
(aq) + 2 Cl(aq)
+ −
2H2 O(l) → 2H(aq) + 2OH(aq)
+
2H(aq) + 2e− → H2(g)
2Cl−(aq) → Cl2 (𝑔) + 2𝑒

+
+ −
2 NaCl(aq) + 2H2 O(l) → 2
⏟Na(aq) + 2OH(aq) + H2(g) + Cl2 (g)
2 NaOH(aq)
Observação: Esta é a forma eletroquímica para a produção de hidróxido de
sódio (soda cáustica) a partir do cloreto de sódio, havendo a liberação dos
gases H2(g) e Cl2(g).

LIVRO OLÍMPICO – VOLUME I


28

24
(GRILLO) Uma determinada corrente elétrica passa durante seis minutos
através de um voltâmetro contendo ácido sulfúrico diluído, liberando 40 x 10-
3 dm³ de hidrogênio gasoso medido a 15°C e pressão igual a 748 mmHg. A

partir destas informações, determine a corrente elétrica produzida por este


processo.

Resposta:

Analisando a eletrólise aquosa do ácido sulfúrico:

Equação de dissociação do ácido sulfúrico: H2SO4(aq) → 2 H+(aq) + SO4-2(aq)

Equação de ionização da água: H2O(aq) → 2 H+(aq) + OH-(aq)

O único que vai descarregar no cátodo, trata-se do hidrogênio: 2 H+(aq) + 2e-


→ H2(g)

Quem descarrega com maior facilidade no ânodo (pólo positivo), SO4-2(aq) ou


OH-(aq)?
2 OH-(aq) → ½ O2(g) + 2e- + H2O(l)

Somando as semirreações mencionadas acima, temos:


H2SO4(aq) → 2 H+(aq) + SO4-2(aq)
2 H2O(aq) → 2 H+(aq) + 2 OH-(aq)
2 H+(aq) + 2e- → H2(g)
2 OH-(aq) → ½ O2(g) + 2e- + H2O(l) +
H2SO4(aq) + H2O(l) → 2 H (aq) + SO4 (aq) + H2(g) + ½ O2(g)
+ -2

Através da eletrólise do ácido sulfúrico, observa-se que há sim a produção de


gás hidrogênio.

Cálculo do número de mol de gás hidrogênio, utilizando a equação dos gases


ideais, temos:
96500 C
1 mol de H2(g) − − − − − − − − − 2 mol de e− x
mol de e−
748
(760) 𝑥 40 𝑥 10−3
−−−−−−Q
0,08206 𝑥 (15 + 273)

748
(760) x 40 x 10−3 5774560
Q = 2 x 96500 x { }= = 321,50 C
0,08206 x 288 17961,29

Q 321,50
Cálculo da corrente (i): i = tempo = 360
= 0,893 A

LIVRO OLÍMPICO – VOLUME I


29

25
(INSTITUTO MILITAR DE ENGENHARIA) O alumínio pode ser produzido
industrialmente pela eletrólise do cloreto de alumínio fundido, o qual é obtido
a partir do minério bauxita, cujo principal componente é o óxido de alumínio.
Com base nas informações acima, calcule quantos dias são necessários para
produzir 1,00 tonelada de alumínio puro, operando-se uma cuba eletrolítica
com cloreto de alumínio fundido, na qual se faz passar uma corrente elétrica
constante de 10,0 kA.
Resposta: Estudando a eletrólise ígnea do cloreto de alumínio.

AlCl3(s) → Al+3 −
(l) + 3Cl(l)

Semirreação que ocorre no cátodo: Al+3 −


(l) + 3e → Al(s)

Semirreação que ocorre no ânodo: 2Cl− −


(l) + 2e → Cl2(g)

Somando as três semirreações, temos:

2AlCl3(s) → 2Al+3 −
(l) + 6Cl(l) (x 2)
2 Al+3 −
(l) + 6e → 2 Al(s) (x 2)
6Cl−
(l) → 3 Cl2(g) + 6e

(x 3) +
2AlCl3(s) → 2 Al(s) + 3 Cl2(g)

Estudando a semirreação de redução do alumínio: Al+3 −


(l) + 3e → Al(s)

Cálculo da carga (Q):


96500 C
27 g − − − − − − − − 3 mol de elétrons x
mol de elétrons
106 g − − − − − − − − Q

106 x 3 x 96500
Q= = 1,07 x 1010 C
27
1,07 x 1010 C
Cálculo do tempo: tempo = 10000
= 1,07 x106 s

Convertendo o tempo para dias:


1 hora 1 dia
tempo = 1,07 x106 s x x = 12,41 dias
3600 s 24 h
tempo = 12,41 dias

LIVRO OLÍMPICO – VOLUME I


30

26
(INSTITUTO TECNOLÓGICA DA AERONÁUTICA) A corrente elétrica que
passou através dos fios conectores de cobre do circuito durante a eletrólise
foi igual a 1,6 x 10-2 ampère. Qual das opções abaixo contém a conclusão
correta sobre o número de elétrons que passou, por segundo, através da
secção (X --- Y) do fio de cobre, conforme assinalado na figura?

a) 1,6.10-2
b) 1,0.10 12
c) 1,0.1017
d) 6,0.1020
e) 9,7.1021

Resposta: Alternativa C.
Sabendo que a equação matemática da corrente é dada por:
n x q e−
i=
tempo
1,60 x 10−19 x 𝑛
1,60 x 10−2 =
1
1,60 x 10−2
n = 1,60 x 10−19 = 10+17 elétrons

27
(INSTITUTO TECNOLÓGICA DA AERONÁUTICA) Escreva as equações
químicas das meia-reações que irão ocorrer em cada um dos eletrodos do
elemento galvânico esquematizado adiante e justifique porque a frase a
seguir está CERTA ou está ERRADA: "A concentração de ZnSO4 do lado
esquerdo vai aumentar."

LIVRO OLÍMPICO – VOLUME I


31

Resposta:

Semirreação da esquerda (oxidação - ânodo): Zn(s) → Zn+2


(aq) + 2e

Semirreação da direita (redução-cátodo): 2Ag +1 −


(l) + 2e → Ag (s)

Equação iônica global do funcionamento da pilha:

Zn(s) + 2Ag +1 +2
(l) → Ag (s) + Zn(aq)

"A concentração de ZnSO4 do lado esquerdo vai aumentar."


A frase está errada. A concentração do íon Zn+ aumenta por causa da
oxidação e a concentração de SO2-2 fica constante.

28
(INSTITUTO MILITAR DE ENGENHARIA) Calcule a intensidade da corrente
elétrica que deve ser utilizada para depositar 2,54 x 10-4 kg de cobre, de uma
solução de sulfato de cobre, no tempo, no tempo de 3 minutos e 20 segundos.

Resposta: Estudando a eletrólise no cátodo – semirreação do cobre.


Cu+2 −
(aq) + 2e → Cu(s)

60 s
Conversão do tempo: tempo = 3 min x 1 min + 20 s = 200 s

A partir da relação estequiométrica, temos:

96500 𝐶
63,55 g de Cu − − − − 2 mol de elétrons x
mol de elétrons
0,254 g − − − − − − − (200 x i)

0,254 x 2 x 96500
i= = 3,86 A
63,5 x 200

29
(OLIMPÍADA DE QUÍMICA DO RIO DE JANEIRO) Uma solução de sulfato
de níquel II foi eletrolisada durante 1,50 h entre eletrodos inertes. Se foram
depositados 35,0 g de níquel, qual o valor da corrente média?
a) 10,7 A
b) 12,3 A
c) 15,1 A
d) 21,3 A
e) 23,1 A

Resposta: Alternativa D.

Estudando a eletrólise no cátodo – semirreação do níquel:


Ni+2 −
(aq) + 2e → Ni(s)

3600 s
Conversão do tempo: tempo = 1,50 h x 1h
= 5400 s

A partir da relação estequiométrica, temos:


96500 𝐶
58,7 g de Ni − − − − 2 mol de elétrons x
mol de elétrons
35,0 g − − − − − − − (5400 x i)

LIVRO OLÍMPICO – VOLUME I


32

35,0 x 2 x 96500
i= = 21,31 A
58,7 x 5400

30
(OLIMPÍADA DE QUÍMICA DO RIO DE JANEIRO) A bateria de níquel
cádmio (também chamada NiCd) foi o segundo tipo de bateria recarregável a
ser desenvolvida. Atualmente, os celulares e tabletes utilizam baterias de
hidreto metálico ou de íons de lítio. As baterias NiCd estão em desuso, uma
vez que sua vida útil é pequena e, além de tudo, é muito poluente. Considere
uma célula eletroquímica de acordo com o esquema abaixo:

a) Escreva as semirreações envolvidas e a reação global balanceada, na


forma iônica, da célula.
b) Qual é a espécie reduzida e a oxidada? Qual é o agente oxidante e redutor?
Qual é o anodo e o catodo? Qual é a polaridade no eletrodo de cádmio?
c) Qual é o potencial padrão, E0, da célula?
d) Qual a direção do fluxo de elétrons no fio externo? Qual a direção da
migração dos íons presentes na ponte salina?
e) Se a concentração de Cd2+ for reduzida para 0,0010 mol/L e [Ni2+] = 1,0
mol/L, a fem da célula será maior ou menor do quando a concentração de
Cd2+ era 1,0 mol/L? Justifique sua resposta em termos do Princípio de Le
Chatelier.
f) Calcule o novo potencial da célula acima, considerando a temperatura de
25oC, e com as concentrações [Ni2+] = 0,1 mol/L e [Cd2+] = 0,0010 mol/L.
Dados do problema:
E0 (Cd2+/Cd) = – 0,402 V
E0 (Ni2+/Ni) = – 0,257 V

Resposta: Item a)

Semirreação do níquel: Ni+2 −


(aq) + 2e → Ni(s)
Semirreação do cádmio: Cd+2 −
(aq) + 2e → Cd(s)

Para o funcionamento da pilha, será necessário inverter a semirreação do


cádmio.

Equação iônica global para o funcionamento do dispositivo eletroquímico:


Ni+2 −
(aq) + 2e → Ni(s)
Cd(s) → Cd+2
(aq) + 2e

+
Cd(s) + Ni+2 +2
(aq) → Ni(s) + Cd(aq)

Item b) Cd(s) + Ni+2 +2


(aq) → Ni(s) + Cd(aq)

LIVRO OLÍMPICO – VOLUME I


33

Espécie reduzida = níquel


Espécie oxidada = cádmio
Agente oxidante = cátion níquel
Agente redutor = cádmio metálico
Ânodo = cádmio
Cátodo = níquel
Polaridade no eletrodo de cádmio = polaridade negativa

Item c) Qual é o potencial padrão, E0, da célula?

Semirreação da cela da esquerda: Ni+2 −


(aq) + 2e → Ni(s)
∆G10 = −2 x F x (− 0,258)

Semirreação da cela da direita: Cd(s) → Cd+2


(aq) + 2e

∆G20 = −2 x F x (+ 0,402)

Equação iônica global: Cd(s) + Ni+2 +2


(aq) → Ni(s) + Cd(aq)
0
∆Gtotal = ∆G10 + ∆G20

0
−2 x F x Etotal = −2 x F x (− 0,258) + [−2 x F x (+ 0,402)]

0
Etotal = − 0,258 + 0,402 = +0,144 V

Item d) Cd(s) + Ni+2 +2


(aq) → Ni(s) + Cd(aq)

Direção do fluxo de elétrons no fio externo = sai do ânodo (cádmio) para o


cátodo (níquel).
Migração dos íons presentes na ponte salina = o cátion potássio (K+) se
direciona para a cuba do níquel, enquanto o ânion nitrato (NO3-) migra para a
cuba do níquel.

Item e) Para a resolução deste item, será necessário utilizar a equação de


Nersnt.

Cd(s) + Ni+2 +2
(aq) → Ni(s) + Cd(aq)

RxT aNi x [Cd+2


(aq) ]
E = E0 − x ln
nxF a x [Ni+2 ]
Cd (aq)

8,314 x 298 1 x 0,0010


E = +0,144 − x ln = +0,233 V
2 x 96500 1 x 1,0

A partir do resultado obtido, o potencial será maior. Pelo princípio de Le


Chatelier, diminuindo a concentração da quantidade de matéria de cádmio, o
equilíbrio será deslocado para a direita.

LIVRO OLÍMPICO – VOLUME I


34

Item f) Cálculo do novo potencial da célula: [Ni2+] = 0,1 mol/L e [Cd2+] = 0,0010
mol/L. Para a resolução deste item, será necessário utilizar a equação de
Nersnt.

Cd(s) + Ni+2 +2
(aq) → Ni(s) + Cd(aq)

RxT aNi x [Cd+2


(aq) ]
E = E0 − x ln
nxF a x [Ni+2 ]
Cd (aq)

8,314 x 298 1 x 0,0010


E = +0,144 − x ln = +0,203 V
2 x 96500 1 x 0,10

31
(U. S. NATIONAL CHEMISTRY OLYMPIAD) A galvanic cell is based on the
half-reactions:
Cr3+ + 3e– → Cr E˚ = –0,744 V
Ni2+ + 2e– → Ni E˚ = –0,236 V
a) Write the balanced equation for the overall cell reaction.
b) State which electrode increases in mass as the cell operates. Explain
your answer.
c) Calculate E˚cell
d) Determine the value of ∆G˚ for the cell reaction at 25˚C.
e) Calculate the value of K for the cell reaction at 25˚C.
f) Find the voltage of the cell at 25˚C if [Cr3+] and [Ni2+] are both
changed to 0,010 M.

Resposta:

Cr3+ + 3e– → Cr E˚ = –0,744 V


Ni2+ + 2e– → Ni E˚ = –0,236 V

Invertendo a primeira semirreação e multiplicando por dois:


2Cr → 2Cr3+ + 6e– ∆G10 = −6 x F x (+ 0,744)

Multiplicando a segunda semirreação por três:


3Ni2+ + 6e– → 3Ni ∆G20 = −6 x F x (−0,236)

2Cr → 2Cr3+ + 6e– ∆G10 = −6 x F x (+ 0,744)


3Ni2+ + 6e– → 3Ni ∆G20 = −6 x F x (−0,236) +
2Cr + 3Ni2+→ 2Cr3+ + 3Ni
0
∆Gtotal = ∆G10 + ∆G20
−6 x F x E 0 = −6 x F x (+ 0,744) + −6 x F x (−0,236)
0
Etotal = + 0,744 − 0,236 = +0,508 V

Item a) Equação iônica para o funcionamento da célula galvânica:


2Cr + 3Ni2+→ 2Cr3+ + 3Ni

Item b) O eletro do que tem a sua massa aumentada é o eletrodo de níquel.

Item c) E 0 = +0,508 V

Item d) Cálculo do valor da energia livre de Gibbs padrão:

LIVRO OLÍMPICO – VOLUME I


35

0
∆Greação = −6 x 96500 x (+ 0,508) = −294132 𝐽 (− 294,1 kJ)

Item e) Cálculo da constante de equilíbrio:


0 0
∆Gtermodinâmico = ∆Geletroquímico

−R x T x ln K eq = −n x F x E 0

−8,314 x 298 x ln K eq = −294132

ln K eq = +118,72

K eq = 𝑒 (+118,72)

Item f) Cálculo da ddp, a partir da utilização da equação de Nersnt:

2Cr + 3Ni2+→ 2Cr3+ + 3Ni


1
+3 2
RxT a⏞
3
Ni x [Cr(aq) ]
E = E0 − x ln 3
nxF 2 +2
a⏟
Cr x [Ni(aq) ]
1

8,314 x 298 (0,010)2


E = +0,508 − x ln = +0,488 V
6 x 96500 (0,010)3

32
(OLIMPÍADA RORAIMENSE DE QUÍMICA) Uma pilha em solução aquosa
foi montada usando eletrodos de: Prata (Ag+ + e- → Ag, E° = + 0,80 V) e
Níquel (Ni2+ + 2e- → Ni, E° = - 0,25 V). Sobre a pilha montada, faça o que se
pede a seguir.
a) Escreva a equação global da pilha.
b) Determine o potencial-padrão E° da pilha e indique qual eletrodo será o
catodo e qual será o anodo.

Resposta: Para a determinação do potencial padrão e da equação iônica para


o funcionamento da pilha galvânica, será necessário inverter a semirreação
do níquel e multiplicar por dois a semirreação da prata.

Ni → Ni+2 + 2e- ∆G10 = −2 x F x (+0,25)


2 Ag+ + 2e- → 2Ag ∆G20 = −2 x F x (+0,80)

Somando as semirreações, temos:

Ni → Ni+2 + 2e- ∆G10 = −2 x F x (+0,25)


2 Ag+ + 2e- → 2Ag ∆G20 = −2 x F x (+0,80)
0
Ni + 2 Ag+ → Ni+2 + 2Ag ∆Gtotal = ∆G10 + ∆G20

−2 x F x E 0 = −2 x F x (+0,25) + [−2 x F x (+0,80)]


0
Etotal = +1,05

LIVRO OLÍMPICO – VOLUME I


36

Item a) Equação iônica para o funcionamento da pilha galvânica:


Ni + 2 Ag+ → Ni+2 + 2Ag
Item b) ddp = +1,05; ânodo = eletrodo de níquel e cátodo = eletrodo de prata.
33
(INSTITUTO TECNOLÓGICO DA AERONÁUTICA) Considere os dois
eletrodos (I e II) seguintes e seus respectivos potenciais na escala do eletrodo
de hidrogênio (Eº) e nas condições-padrão:
I) 2F-(aq) → 2e-(CM) + F2(g) EºI = 2,87 V
II) Mn (aq) + 4 H2O(l) → 5e (CM) + 8H (aq) + MnO4 (aq)
+2 - + - EºII = 1,51 V
A força eletromotriz de um elemento galvânico construído com os dois
eletrodos acima é de:
a) – 1,81 V
b) – 1,13 V
c) 0,68 V
d) 1,36 V
e) 4,38 V

Resposta: Alternativa D.
Para a determinação do potencial padrão, será necessário inverter a segunda
semirreação e multiplicar por dois e para a primeira semirreação será
necessário multiplicar por cinco.
10 F-(aq) → 10e-(CM) + 5F2(g) EºI = 2,87 V
10e (CM) + 16H (aq) + 2MnO4 (aq) → 2Mn (aq) + 8 H2O(l)
- + - +2 EºII = -1,51 V
10 F (aq) + 16H (aq) + 2MnO4 (aq) → 5F2(g) + 2Mn (aq) + 8 H2O(l)
- + - +2

0
−10 x F x Etotal = −10 x F x (+2,87) + [−10 x F x (−1,51)]
0
Etotal = +1,36 V

34
(INSTITUTO TECNOLÓGICO DA AERONÁUTICA) Por uma célula
eletrolítica passa uma corrente constante e igual a 0,965 ampère. Num dos
eletrodos, a reação que ocorre é a seguinte: Cr2O72- + 14 H+ + 6e- → 2 Cr3+
+ 7 H2O. Qual é o tempo certo durante o qual essa corrente deve passar para
que seja produzidos 0,400 mol de íons Cr3+?

Resposta:
96500 V
2 mol de Cr − − − − 6 mol de elétrons x
mol de elétrons
0,400 mol de Cr − − 0,965 x tempo

0,400 x 6 x 96500
tempo = 2 𝑥 0,965
= 120000 𝑠 (33,33 h)

LIVRO OLÍMPICO – VOLUME I


37

35
(GRILLO) Considere os seguintes potenciais apresentados a seguir: X2+(aq) /
X(s) é igual a + 0,34 V, e o par X+(aq) / X(s) é + 0,55 V. A partir desta informação,
calcule E°, ∆G° e Keq para o seguinte processo eletroquímico X2+(aq) / X+(aq).

Resposta: Arrumando as semirreações apresentadas pelos pares no


enunciado do exercício.
+2
X (aq) + 2e− → X (s)

96500 C
∆G° = −2 mol de elétrons x x (+0,34)
mol de elétrons
+1
X (aq) + 1e− → X (s)

96500 C
∆G° = −1 mol de elétrons x x (+0,55)
mol de elétrons
+2 +1
Para a determinação do potencial da semirreação X (aq) + 1e− → X (aq)
será necessário inverter a segunda semirreação. Realizando este processo,
temos:
+2
X (aq) + 2e− → X (s)
96500 C
∆G° = −2 mol de elétrons x x (+0,34)
mol de elétrons
X (s) → X +1
(aq) + 1e

96500 C
∆G° = −1 mol de elétrons x mol de elétrons x (−0,55) +
+2 +1
X (aq) + 1e− → X (aq)

96500 C
∆G° = −1 mol de elétrons x x E°
mol de elétrons

Determinação do potencial padrão (E°):

−1 x E° = −2 x (+0,34) + [−1 x (−0,55)]

E° = +0,13 V

+2
Cálculo da variação da energia livre de Gibbs padrão (X (aq) + 1e− →
+1
X (aq) ):

96500 C
∆G0 = −n x F x E° = −1 mol x x (+0,13)
mol

∆G0 = −12545 J

Este processo eletroquímico é espontâneo, pelo fato da variação da energia


livre de Gibbs for negativo.

Cálculo da constante de equilíbrio (Keq): ∆G0 = −12545 J

LIVRO OLÍMPICO – VOLUME I


38

−R x T x lnK eq = −12545 J

−8,314 x (25 + 273) x lnK eq = −12545 J

12545
lnK eq = = 5,06
8,314 x 298

K eq = 𝑒 (+5,06)

36
(INSTITUTO MILITAR DE ENGENHARIA) Um certo fabricante produz pilhas
comuns, nas quais o invólucro de zinco funciona como anodo, enquanto que
o catodo é inerte. Em cada uma, utilizam-se 5,87 g de dióxido de manganês,
9,2 g de cloreto de amônio e um invólucro de zinco de 80 g. As semirreações
dos eletrodos são:
Zn → Zn+2 + 2 e–
NH4+ + MnO2 + e– → 1/2 Mn2O3 + NH3 + 1/2 H2O
Determine o tempo que uma destas pilhas leva para perder 50% de sua carga,
fornecendo uma corrente constante de 0,08 A. Dado: F = 96.500 C.

Resposta: Multiplicando a segunda semirreação por dois e determinando a


equação eletroquímica global.
Zn → Zn+2 + 2 e–
2 NH4+ + 2 MnO2 + 2 e– → Mn2O3 + 2 NH3 + H2O
Zn + 2 NH4+ + 2 MnO2 → Zn+2 + Mn2O3 + 2 NH3 + H2O

Para a resolução do problema, será necessário determinar o reagente


limitante. Cálculo do número de mol para cada reagente.

80
nZn = = 1,22 mol
65,4
9,2 0,172
nNH4 Cl = = = 0,086 mol
53,5 2
5,87 0,0675
nMnO2 = 86,94 = 2
= 0,034 mol (reagente limitante)

Cálculo do tempo necessário:


96500 V
2 mol de MnO2 − − − 2 mol de 𝑒 − x x 0,50
mol de 𝑒 −
5,87
mol − − − − − 0,08 x tempo
86,94

5,87
86,94 x 2 x 96500 x 0,50
tempo = = 40710,95 s (≅ 11,31 h)
2 x 0,08

LIVRO OLÍMPICO – VOLUME I


39

37
(INSTITUTO MILITAR DE ENGENHARIA) Os eletrodos de uma bateria de
chumbo são Pb e PbO2. A reação global de descarga é Pb + PbO2 + 2H2SO4
→ 2PbSO4 + 2H2O. Admita que o “coeficiente de uso” seja de 25,0%. Este
coeficiente representa a fração do Pb e PbO2 presentes na bateria que são
realmente usados nas reações dos eletrodos. Calcule:
a) a massa mínima de chumbo em quilogramas (incluindo todas as
formas em que se encontra esse elemento) que deve existir numa bateria
para que ela possa fornecer uma carga de 38,6 x 104 C;
b) o valor aproximado da variação da energia livre da reação, sendo de
2,00 V a voltagem média da bateria quando fora de uso.

Resposta:
Item a) Cálculo da massa mínima de chumbo, a partir da seguinte relação
estequiométrica: Pb + PbO2 + 2H2SO4 → 2PbSO4 + 2H2O
Rendimento
do processo
eletroquímico
96500 𝐶
2 x 207,2 g de (Pb + PbO2 ) − − − − 2 x 𝑥 ⏞
0,25
mol de e−
Massa de Pb − − − − − − − − − − 38,6 x 104 𝐶
2 x 207,2 x 38,6 x 104
Massa = = 3315,20 g
2 x 96500 x 0,25
Item b) Cálculo da variação da energia livre de Gibbs padrão:
0
∆Greação = −n x F x E 0
0
∆Greação = −2 x 96500 x 2,0 = −3,86 x 105 J

38
(INSTITUTO MILITAR DE ENGENHARIA) Uma bateria de automóvel
apresenta as seguintes reações eletrodos durante a descarga:
no ânodo: Pb(s) + SO42-(aq) → PbSO4(s) + 2e-
no cátodo: PbO2(s) + 4H+(aq) + SO42-(aq) + 2e- → PbSO4(s) + 2H2O(l)
A solução inicial de ácido sulfúrico contido na bateria tem uma concentração
de 40%, em peso, de ácido sulfúrico e massa específica de 1,3 g/cm3. Após
abateria ter sido utilizada, a solução foi analisada e apresentou uma
concentração de 28,0%, em peso, de ácido sulfúrico com uma massa
específica de 1,2 g/cm3. Considerando fixo o volume da solução ácida na
bateria em 2,0 litros, determine o valor da carga fornecida pela bateria em
ampère-hora.

Resposta: Determinação da equação química iônica para o funcionamento da


pilha.

Pb(s) + SO42-(aq) → PbSO4(s) + 2e-


PbO2(s) + 4H+(aq) + SO42-(aq) + 2e- → PbSO4(s) + 2H2O(l)
Pb(s) + SO42-(aq) + PbO2(s) + 4H+(aq) + SO42-(aq) → PbSO4(s) + PbSO4(s) + 2H2O(l)

LIVRO OLÍMPICO – VOLUME I


40

Cálculo da massa de ácido sulfúrico inicial:


g 1 cm3
massainicial = 0,40 x 1,30 x x 2,0 L = 1040 g
cm3 10−3 dm3
g 1 cm3
massafinal = 0,28 x 1,20 3 x −3 3 x 2,0 L = 672 g
cm 10 dm
massaacumulada = massainicial − massafinal
massaacumulada = 1040 g − 672 g = 368 g
Cálculo da carga fornecida pela bateria em ampère-hora:
96500 𝐶
2 x 98 g de ácido − − − − − 2 mol de e− x
mol de e−
368 g de ácido − − − − − − Q

368 x 2 x 96500 1h
Q= 2 x 98
= 362367,35 C x s x 3600 s = 100,66 A x h

Q = 100,66 A x h
39
(OLIMPÍADA PERUANA DE QUÍMICA) Determine el potencial estándar de
reducción a 25oC para: Fe3+(ac) + 3e → Fe(s).
PAR REDOX Eº (REDUCCIÓN)
Fe3+ | Fe2+ + 0,77 V
Fe2+ | Fe - 0,44 V
a) 0,11 V
b) - 0,037 V
c) - 0,11 V
d) 0,037 V

Resposta: Alternativa B.
Somando as semirreações, temos:

Fe+3 + 1e- → Fe+2 ∆G10 = −1 x F x (+0,77)


Fe+2 + 2e- → Fe ∆G20 = −2 x F x (−0,44)
0
Fe+3 + 3e- → Fe ∆Gtotal = ∆G10 + ∆G20

0
−3 x F x Etotal = −1 x F x (+0,77) + [−2 x F x (−0,44)]
0
Etotal = −0,037 V

40
(OLIMPÍADA PERUANA DE QUÍMICA) Calcule el valor de la constante de
equilibrio de la celda electroquímica Zn(s)/Zn2+(ac)//Cu2+(ac)/Cu(s) a condiciones
estándar (1 atm, 25°C). Potenciales estándar de reducción (V)
Zn2+ + 2e- → Zn Eº = – 0,76V
Cu2+ + 2e- → Cu Eº = + 0,34V
a) 1,9 x 10–37
b) 4,4 x 1018
c) 1,6 x 1016
d) 1,5 x 1037

LIVRO OLÍMPICO – VOLUME I


41

Resposta: Alternativa E.

Zn2+ + 2e- → Zn Eº = – 0,76V


Cu2+ + 2e- → Cu Eº = + 0,34V

Invertendo a primeira semirreação:


Zn → Zn2+ + 2e- ∆G10 = −2 x F x (+ 0,76)

Zn → Zn2+ + 2e- ∆G10 = −2 x F x (+ 0,76)


Cu2+ + 2e- → Cu ∆G20 = −2 x F x (+0,34) +
Zn + Cu2+ → Zn2+ + Cu
0
∆Gtotal = ∆G10 + ∆G20
−2 x F x E 0 = −2 x F x (+ 0,76) + −2 x F x (+0,34)
0
Etotal = + 0,76 + 0,34 = +1,10 V

0 0
Cálculo da constante de equilíbrio: ∆Gtermodinâmico = ∆Geletroquímico
−R x T x ln K eq = −n x F x E 0

−8,314 x 298 x ln K eq = −2 𝑥 96500 𝑥 (+1,10)


−212300
ln K eq =
−2477,572
K eq = 𝑒 (+85,69) = 1,64 𝑥 10+37

41
(OLIMPÍADA AUSTRALIANA DE QUÍMICA) By referring to the standard
reduction potentials below, which of the species listed is the best oxidising
agent?
Cu2+ (aq) + 2e → Cu(s) E0 = +0,34 V
Ni (aq) + 2e → Ni(s)
2+ E0 = -0,23 V
Cd (aq) + 2e → Cd(s)
2+ E0 = -0,40 V
Fe2+ (aq) + 2e → Fe(s) E0 = -0,44 V
Zn2+ (aq) + 2e → Zn(s) E0 = -0,76 V
a) Cd(s)
b) Zn2+(aq)
c) Ni(s)
d) Cu2+(aq)
e) Fe(s)

Resposta: Alternativa D.

O melhor agente oxidante será aquele que apresenta a reação de redução


com o maior valor de potencial. Diante desta afirmativa, trata-se do potencial
do cátion cobre.

42
(OLIMPÍADA AUSTRALIANA DE QUÍMICA) The following is a list of selected
standard reduction potentials:
Cd2+(aq) + 2e– → Cd(s) Eo = -0,40 V
Zn (aq) + 2e → Zn(s)
2+ – Eo = -0,76 V
Ni (aq) + 2e → Ni(s)
2+ – Eo = -0,23 V

LIVRO OLÍMPICO – VOLUME I


42

By referring to the standard reduction potentials above, which of the following


species is the best oxidizing agent?
a) Cd(s)
b) Zn2+(aq)
c) Ni2+(aq)
d) Zn(s)
e) Ni(s)

Resposta: Alternativa C.

O melhor agente oxidante será aquele que vai apresentar a reação de


redução com o maior valor de potencial. Diante desta afirmativa, trata-se do
potencial do cátion níquel divalente.
43
(OLIMPÍADA CANADENSE DE QUÍMICA) Voici les demi-réactions de la
cellule illustrée:
Zn2+(aq) + 2e– → Zn(s) E° = –0,76 V
Cu (aq) + 2e → Cu(s)
2+ – E° = +0,34 V

Si la réaction est spontanée, lequel des énoncés suivants est vrai?


a) Le Zn est l’anode; les électrons vont de l’anode à la cathode; les anions
migrent à travers le pont salin de la demi-cellule Zn/Zn2+ à la demicellule
Cu/Cu2+ et les cations migrent dans la direction opposée.
b) Le Zn est la cathode; les électrons vont de l’anode à la cathode; les anions
migrent à travers le pont salin de la demi-cellule Zn/Zn2+ à la demi-cellule
Cu/Cu2+ et les cations migrent dans la direction opposée.
c) Le Zn est l’anode; les électrons vont de la cathode à l’anode; les anions
migrent à travers le pont salin de la demi-cellule Zn/Zn2+ à la demicellule
Cu/Cu2+ et les cations migrent dans la direction opposée.
d) Le Zn est la cathode; les électrons vont de l’anode à la cathode; les cations
migrent à travers le pont salin de la demi-cellule Zn/Zn2+ à la demi-cellule
Cu/Cu2+ et les anions migrent dans la direction opposée.
e) Le Zn est l’anode; les électrons vont de l’anode à la cathode; les cations
migrent à travers le pont salin de la demi-cellule Zn/Zn2+ à la demicellule
Cu/Cu2+ et les anions migrent dans la direction opposée.

Resposta: Alternativa E.

Zn2+(aq) + 2e– → Zn(s) E° = – 0,76 V


Cu2+(aq) + 2e– → Cu(s) E° = + 0,34 V

Invertendo a primeira semirreação.

LIVRO OLÍMPICO – VOLUME I


43

Zn → Zn2+(aq) + 2e– ∆G10 = −2 x F x (+ 0,76)


Cu2+(aq) + 2e– → Cu(s) ∆G20 = −2 x F x (+0,34) +
Zn(s) + Cu2+(aq) → Zn2+(aq) + Cu(s)
0
∆Gtotal = ∆G10 + ∆G20
0
−2 x F x Etotal = −2 x F x (+ 0,76) + −2 x F x (+0,34)
0
Etotal = + 0,76 + 0,34 = +1,10 V

O fluxo de elétrons sai do eletrodo do zinco para o cobre.


O ânodo é o eletrodo de zinco e o cátodo é o eletrodo de cobre.
Os cátions da ponte salina vão em direção à célula de cobre, enquanto
os ânions vão em direção à célula de zinco.
44
(OLIMPÍADA AUSTRALIANA DE QUÍMICA) A working galvanic cell is
constructed using an iron electrode in an iron(II) nitrate solution and a silver
electrode in a silver(I) nitrate solution. When the cell runs down, it can be
charged up again. Which of the following scenarios will occur during the
charging process? Eº(Ag+ (aq)/Ag(s) = + 0,80 V and Eº(Fe2+ (aq)/Fe(s) = – 0,41 V
a) A minimum of 1,21 volts is applied to the cell and electrons move
from the silver electrode to the iron electrode.
b) A minimum of 1,21 volts is applied to the cell and electrons move
from the iron electrode to the silver electrode.
c) A minimum of 0,39 volts is applied to the cell and electrons move
from the silver electrode to the iron electrode.
d) A minimum of 0,39 volts is applied to the cell and electrons move
from the iron electrode to the silver electrode.
e) A minimum of 2,01 volts is applied to the cell and electrons move
from the silver electrode to the iron electrode.

Resposta: Alternativa B.

Fe2+ + 2e- → Fe Eº = – 0,41 V


Ag+ + 1e- → Ag Eº = + 0,80 V

Invertendo a primeira semirreação e multiplicando a segunda semirreação.

Fe → Fe2+ + 2e- ∆G10 = −2 x F x (+ 0,41)


2Ag+ + 2e- → 2Ag ∆G20 = −2 x F x (+0,80) +
Fe + 2Ag+ → Fe2+ + 2Ag
0
∆Gtotal = ∆G10 + ∆G20
0
−2 x F x Etotal = −2 x F x (+ 0,41) + −2 x F x (+0,80)
0
Etotal = + 0,41 + 0,80 = +1,21 V

O fluxo de elétrons sai do eletrodo de ferro para o eletrodo de prata.

LIVRO OLÍMPICO – VOLUME I


44

45
(GRILLO) Considere uma pilha galvânica operando a uma temperatura
ambiente (250C) e pressão de 1 atm. A descrição da pilha está fundamentada
da seguinte maneira: PtSn2+(aq) (0,1 mol.L-1), Sn4+(aq) (0,05 mol.L-1)Fe3+(aq)
(0,01 mol.L-1), Fe2+(aq) (0,2 mol.L-1)Pt
a) Escreva a equação química iônica global de funcionamento da pilha
e o cálculo da sua ddp.
b) Calcule a variação da energia livre de Gibbs padrão da pilha para a
reação global.
c) Calcule o ΔG da reação no momento em que as concentrações das
espécies químicas são iguais às indicadas na notação da pilha, apresentada
no topo da questão. A pilha funciona nessas condições?
d) Calcule a constante de equilíbrio, K, quando a pilha parar de gerar
corrente elétrica. Dados:
Fe3+(aq) + e- → Fe2+(aq) E0 = + 0,77 V
Sn4+(aq) + 2e- → Sn2+(aq) E0 = + 0,15 V

Resposta: Invertendo a semirreação de estanho:

Sn2+(aq) → Sn4+(aq) + 2e- ∆G10 = −2 x F x (− 0,15)

Multiplicando por dois a semirreação do Ferro:


2Fe3+(aq) + 2e- → 2Fe2+(aq) ∆G20 = −2 x F x (+ 0,77)
Somando as duas semirreações, temos:

Sn2+(aq) → Sn4+(aq) + 2e-


2Fe3+(aq) + 2e- → 2Fe2+(aq) +
Sn2+(aq) + 2Fe3+(aq) → Sn4+(aq) + 2Fe2+(aq)
0
∆Gtotal = ∆G10 + ∆G20
0
−2 x F x Etotal = −2 x F x (− 0,15) + [−2 x F x (+ 0,77)]
0
−2 x F x Etotal = −2 x F x (− 0,15) + [−2 x F x (+ 0,77)]
0
Etotal = − 0,15 + 0,77 = +0,62 V

Item a) A equação iônica global para a funcionamento da pilha é seguinte:


Sn2+(aq) + 2 Fe3+(aq) → Sn4+(aq) + 2 Fe2+(aq)

Item b) Cálculo da variação da energia Livre de Gibbs padrão (∆G°):

96500 C
∆G° = −2 mol de elétrons x x (+0,62 V)
1 mol de elétrons

∆G° = −119660 J

Item c) Cálculo da variação da energia Livre de Gibbs (∆G):

[Fe+2 ]2 x [Sn+4 ]
∆G = ∆G° + R x T x ln { }
[Fe+3 ]2 x [Sn+2 ]

LIVRO OLÍMPICO – VOLUME I


45

[0,20]2 x 0,05
∆G = −119600 + 8,314 x 298 x ln { }
[0,01]2 x 0,10

∆G = −119600 + 2477,57 x ln(200) = −106533,04 J

Sim, a pilha galvânica funciona e de forma espontânea, pois a variação da


energia livre de Gibbs é negativa (∆G < 0).

Item d) Quando a pilha parar de gerar corrente elétrica, a partir da variação


da energia livre de Gibbs padrão, a constante de equilíbrio será a seguinte:
∆G° = −R x T x LnK eq

−119660 = −8,314 x 298 x LnK eq

LnK eq = 48,30

K eq = e48,30 = 9,44 x 1020

46
(GRILLO) Considere a seguinte célula galvânica que opera a uma
temperatura igual a 25°C:
Sn4+(aq, 0,020 mol.L-1), Sn2+(aq, 0,035 mol.L-1)  O2(g, 1,0 atm)  H+(aq, pH = 3,0) 
a) Escreva a reação global da célula e calcule o valor do potencial
padrão da célula galvânica.
b) Calcule o potencial da célula nas apresentadas acima.
c) Calcule a energia livre de Gibbs, a partir das condições
apresentadas acima.
Dados:
O2(g) + 4H+(aq) + 4e- → 2 H2O(l) E° = + 1,23 V
Sn+4(aq) + 2e- → Sn+2(aq) E° = + 0,15 V

Resposta:

Item a) Invertendo a segunda semirreação e multiplicando por dois, temos:

O2(g) + 4H+(aq) + 4e- → 2 H2O(l) ∆G10 = −4 x F x (+1,23)


2 Sn (aq) → 2Sn (aq) + 4e
+2 +4 - ∆G20 = −4 x F x (−0,15) +
O2(g) + 4H+(aq) + 2Sn+2(aq) → 2 H2O(l) + 2Sn+4(aq)
0
∆Gtotal = ∆G10 + ∆G20
0
−4 x F x Etotal = −4 x F x (+1,23) + [−4 x F x (−0,15)]
0
−4 x F x Etotal = −4 x F x (+1,23) + [−4 x F x (−0,15)]
0
Etotal = +1,08 V

0,0592
Item b) Utilizando a equação de Nernst, temos: E = E° − ( n
)x logQ

0,0592 [Sn+4 ]². a2H2 O


E = +1,08 − ( ) . log { +2 }
4 [Sn ]². PO2 . [H + ]4

LIVRO OLÍMPICO – VOLUME I


46

0,0592 [0,020]2 x (1)²


E = +1,08 − ( ) . log { }
4 [0,035]2 x 1,0 [10−3 ]4

E = +1,08 − 0,0148 x 11,51 = +0,91 V

Item c) Cálculo da energia livre de Gibbs (∆G):

∆G = ∆G° + R x T x lnQ
[Sn+4 ]². a2H2 O
∆G = −n x F x E° + R x T x ln { }
[Sn+2 ]²PO2 . [H + ]4

∆G = −4 x 96500 x (+1,08)
[0,020]2 x (1)²
+ 8,314 x 298 x ln { }
[0,035]2 x 1,0 x [10−3 ]4

∆G = −416880 + 8,314 x 298 x ln (3,26 x 1011 )

∆G = −416880 + 65684,87 = −351195,13 (−351,19 kJ)

47
(OLIMPÍADA CEARENSE DO ENSINO SUPERIOR DE QUÍMICA) Dadas as
concentrações não-padrão para a reação abaixo: Zn(s) +
Cu+2 −1 +2 −1
(aq) (0,100 mol. L ) → Zn(aq) (0,02 mol. L ) + Cu(s) .
Sabendo que os coeficientes de atividade iônica para Cu2+ e Zn2+, nessas
condições são 0,485 e 0,675 e que E°(Cu2+/Cu) = + 0,34 V e E°(Zn2+/Zn) = –
0,76 V, pode-se afirmar que E para esta pilha de Daniell é:
a) 1,10 V
b) 1,16 V
c) 1,04 V
d) 1,00 V
e) 0,96 V

Resposta: Alternativa A.
Organizando as semirreações e calculando a ddp da pilha nas condições
padrões.

Zn(s) → Zn+2
(aq) + 2e

∆G10 = −2 x F x (+0,76)
Cu+2 −
(aq) + 2e → Cu(s) ∆G20 = −2 x F x (+0,34)
Zn(s) + Cu+2 +2
(aq) → Zn(aq) + Cu(s)

0
∆Gtotal = ∆G10 + ∆G20
0
−2 x F x Etotal = −2 x F x (+ 0,76) + [−2 x F x (+ 0,34)]
0
Etotal = + 1,10 V

0,0592
Expressão da equação de Nernst: E = E 0 − ( n
) log Q

LIVRO OLÍMPICO – VOLUME I


47

1
0,0592 (0,675 𝑥 0,02) x a⏞
Cu
E = +1,10 − ( ) 𝑥 log { }
2 (0,485 𝑥 0,100) x a⏟Zn
⏟ 1
−0,555

E = +1,116 𝑉

48
(OLIMPÍADA PERUANA DE QUÍMICA) Determine el potencial estándar a
25°C para la óxido-reducción 2PH3(g) → 2P(s) + 3H2(g) Eo = ?, dadas las
siguientes reducciones a la misma temperatura:
P(s) + 3H+(ac) + 3e- → PH3(g) Eo = - 0,063 V
2H (ac) + 2e → H2(g)
+ - Eo = 0,000 V
a) E = 0,0063 V
o

b) Eo = - 0,0063 V
c) Eo = - 0,0126 V
d) Eo = 0,0126 V

Resposta: Alternativa A

Invertendo a primeira semirreação e multiplicando por dois:

2 PH3(g) → 2 P(s) + 6H+(ac) + 6e- ∆G10 = −6 x F x (+ 0,063)

Multiplicando a segunda semirreação por três:

6H+(ac) + 6e- → 3 H2(g) ∆G20 = −6 x F x (+ 0,00)

Somando as semirreações, temos:

2 PH3(g) → 2 P(s) + 6H+(ac) + 6e- ∆G10 = −6 x F x (+ 0,063)


6H+(ac) + 6e- → 3 H2(g) ∆G20 = −6 x F x (+ 0,00) +
2 PH3(g) → 2 P(s) + 3 H2(g)
0
∆Gtotal = ∆G10 + ∆G20
0
−6 x F x Etotal = −6 x F x (+ 0,063) + [−6 x F x (+ 0,00)]
0
Etotal = + 0,063 V

LIVRO OLÍMPICO – VOLUME I


48

49
(OLIMPÍADA DE QUÍMICA DO RIO GRANDE DO SUL) Sobre a pilha
representada abaixo, pede-se:

Potenciais de redução: EoCu = + 0,34 V; EoAg = + 0,80 V.


a) Seu cátodo e seu ânodo.
b) Quem se oxida e quem se reduz.
c) Qual eletrodo terá sua massa aumentada?
d) Qual das soluções irá diluir-se?
e) A reação catódica.
f) A reação anódica.
g) A reação global.
h) Sua ddp.

Resposta: É importante observar na figura ilustrada pela questão, que o fluxo


de elétrons sai do eletrodo de cobre para o eletrodo de prata.

Item a) Semirreação que ocorre no ânodo: Cu+2 −


(aq) + 2e → Cu(s)

Semirreação que ocorre no cátodo: Ag + −


(aq) + 1e → Ag (s)

Item g;h) Determinação da reação eletroquímica global e o valor da ddp:


Invertendo a semirreação do cobre e multiplicando por dois a semirreação da
prata.

Cu(s) → Cu+2
(aq) + 2e

∆G10 = −2 x F x (−0,34)
+ −
2Ag (aq) + 2e → 2Ag (s) ∆G20 = −2 x F x (+0,80)
Cu(s) + 2Ag + +2
(aq) → Cu(aq) + 2Ag (s)

0
∆Gtotal = ∆G10 + ∆G20
0
−2 x F x Etotal = −2 x F x (−0,34) + [−2 x F x (+ 0,80)]
0
Etotal = + 0,46 V

Item b)
Agente oxidante: Ag +
(aq)
Agente redutor: Cu(s)

Item c) O eletrodo que terá sua massa aumentada é o eletrodo de prata.

Item d) A solução de nitrato de prata.

LIVRO OLÍMPICO – VOLUME I


49

Item e) Reação catódica: Ag + −


(aq) + 1e → Ag (s)
Item f) Reação anódica: Cu(s) → Cu+2(aq) + 2e

50
(INSTITUTO MILITAR DE ENGENHARIA) Uma empresa de galvanoplastia
produz peças especiais recobertas com zinco. Sabendo que cada peça
recebe 7 g de Zn, que é utilizada uma corrente elétrica de 0,7 A e que a massa
molar do zinco é igual a 65 g.mol-1, qual o tempo necessário para o
recobrimento dessa peça especial?
a) 4 h e 45 min
b) 6 h e 30 min
c) 8 h e 15 min
d) 10 h e 30 min
e) 12 h e 45 min

Resposta: Alternativa C.

Semirreação que ocorre no cátodo: Zn+2 −


(aq) + 2e → Zn(aq)

Para a resolução do problema, será necessário utilizar o processo


estequiométrico da eletrólise.
96500 C
65 g de Zn − − − 2 mol de elétrons x
mol de elétrons
7,0 g de Zn − − − Q

7 x 2 x 96500
Q= = 20784,61 C
65
20784,61
Cálculo do tempo: 0,70 = tempo

20784,61
tempo = 0,70
= 29692,31 s

Conversão do tempo:
1h
tempo = 29692,31 s x 3600 s = 8,25 h
60 min
tempo = 8,0 h + 0,25 h x 1h
= 8,0 h + 15 min

51
(INSTITUTO MILITAR DE ENGENHARIA) Realiza-se a eletrólise de uma
solução aquosa diluída de ácido sulfúrico com eletrodos inertes durante 10
minutos. Determine a corrente elétrica média aplicada, sabendo-se que foram
produzidos no cátodo 300 mL de hidrogênio, coletados a uma pressão total
de 0,54 atm sobre a água, à temperatura de 300 K. Considere os seguintes
dados: Pressão de vapor da água a 300K = 0,060 atm; Constante de Faraday
= 1F = 96500 C.mol-1; Constante universal dos gases perfeitos = 0,08 atm.
L.mol-1.K.
a) 2,20 A
b) 1,93 A
c) 1,08 A
d) 0,97 A
e) 0,48 A

LIVRO OLÍMPICO – VOLUME I


50

Resposta: Alternativa B.
Processo de ionização do ácido sulfúrico:
+
H2 SO4(aq) → 2H(aq) + SO−2
4(aq)

+
Semirreação que ocorre no cátodo: 2H(aq) + 2e− → H2(g)
vapor
Cálculo da pressão de gás hidrogênio seco: Ptotal = PH2 + PH2 O

PH2 = 0,54 atm − 0,060 atm = 0,48 atm

Cálculo do número de mol de gás hidrogênio:


0,48 x 0,30
nH2 = = 5,85 x 10−3 mol
0,08206 x 300
Para a resolução do problema, será necessário utilizar o processo
estequiométrico da eletrólise. Cálculo da carga elétrica:
96500 C
1 mol de H2 − − − − − − − 2 mol de elétrons x
mol de elétrons
5,85 x 10−3 mol de H2 − − − Q

5,85 x 10−3 x 2 x 96500


Q= = 1128,93 C
1
Q
Cálculo da corrente: i = tempo

1128,93
i= 600
= 1,88 A

52
(OLIMPÍADA CEARENSE DO ENSINO SUPERIOR DE QUÍMICA) Suponha
que seja possível construir uma semicélula galvânica usando a semirreação
Cu2+(aq) + 1e− → Cu+(aq) e um eletrodo inerte. Na semicélula construída, as
atividades em solução do Cu+ e Cu2+ são de 0,01 e 0,50, respectivamente.
Dados:
Cu2+(aq) + 2e− ⇌ Cu(s) E° = 0,339 V
Cu+(aq) + 1e− ⇌ Cu(s) E° = 0,521 V
Se for utilizada sob pressão de 1,00 bar e a 25°C, a semicélula apresentará
potencial de:
a) 0,06 V
b) 0,18 V
c) 0,26 V
d) 0,43 V
e) 0,86 V

Resposta: Alternativa C.

LIVRO OLÍMPICO – VOLUME I


51

Para a resolução deste problema, será necessário inverter a segunda


semirreação.

Cu2+(aq) + 2e− ⇌ Cu(s) ∆G10 = −2 x F x (+ 0,339)


Cu(s) ⇌ Cu+(aq) + 1e− ∆G20 = −1 x F x (− 0,521)
Cu2+(aq) + 1e− ⇌ Cu+(aq)
0
∆Gtotal = ∆G10 + ∆G20
0
−1 x F x Etotal = −2 x F x (+ 0,339) + [−1 x F x (− 0,521)]
0
Etotal = + 0,157 V

Cálculo da ddp, levando em consideração aplicando a equação de Nernst:

0,0592 a1 +
E = +0,157 − ( ) . log { 1Cu }
1 aCu+2

0,0592 0,01
E = +0,157 − ( ) . log { }
1 ⏟ 0,50
−1,70

E = +0,157 + 0,100 = +0,257 V

53
(INSTITUTO MILITAR DE ENGENHARIA) Considere que a reação abaixo
ocorra em uma pilha: 2 Fe+3 + Cu → Cu+2 + 2 Fe+2. Assinale a alternativa que
indica o valor correto do potencial padrão dessa pilha. Dados:
Fe+2 → Fe+3 + e- E° = - 0,77V
Cu+2 + 2e- → Cu E° = + 0,34V
a) + 1,20 V
b) – 0,43 V
c) + 1,88 V
d) – 1,20 V
e) + 0,43 V

Resposta: Alternativa E.
Invertendo a primeira semirreação e multiplicando por dois:

2Fe+3 + 2e- → Fe+2 ∆G10 = −2 x F x (+ 0,77)


Invertendo a segunda semirreação por dois:

Cu → Cu+2 + 2e- ∆G20 = −2 x F x (− 0,34)

Somando as semirreações, temos:

2Fe+3 + 2e- → Fe+2 ∆G10 = −2 x F x (+ 0,77)


Cu → Cu+2 + 2e- ∆G20 = −2 x F x (− 0,34) +
2Fe+3 + Cu → Fe+2 + Cu+2

LIVRO OLÍMPICO – VOLUME I


52

0
∆Gtotal = ∆G10 + ∆G20
0
−2 x F x Etotal = −2 x F x (+ 0,77) + [−2 x F x (− 0,34)]
0
Etotal = + 0,43 V

54
(INSTITUTO TECNOLÓGICA DA AERONÁUTICA) A deposição eletrolítica
de 2,975 g de um metal de massa atômica 119 gerou 9650 coulombs. O
número de oxidação do metal é:
a) +1
b) +2
c) +3
d) +4
e) Nenhuma das respostas acima está certa

Resposta: Alternativa D.
Metal com massa atômica igual a 119 trata-se do estanho (Sn). Sabe-se que
o estanho apresenta dois números de oxidação, sendo igual a +2 ou +4.
Cálculo do número de oxidação (a) do estanho.
96500 C
119 g − − − − − − − a mol de elétrons x
1 mol de elétrons
2,975 g − − − − − − 9650 C

119 x 9650
a= = + 4; Sn+4
2,975 x 96500

55
(OLIMPÍADA DE QUÍMICA DO RIO DE JANEIRO) Em um projeto de feira
de ciências, alunos montaram diversas pilhas e determinaram as
diferenças de potencial (ddp) geradas. Todas as pilhas seguiram o
mesmo padrão de montagem, ilustrado na figura abaixo, onde cada
metal foi mergulhado numa solução de um sal do mesmo, em
concentração de 1,0 mol/L. A temperatura em que os experimentos
foram realizados foi de 25oC. Ao final de cada montagem, era indicado
que metal funcionava como catodo, que metal funcionava como anodo
e a ddp gerada. Com base nos dados fornecidos, a opção correta
referente às pilhas montadas é:

Potenciais:

LIVRO OLÍMPICO – VOLUME I


53

Cu2+ + 2e- → Cu E0 = +0,34V


Al + 3e → Al E = –1,67V
3+ - 0

Zn2+ + 2e- → Zn E0 = –0,76V


Ag+ + e- → Ag E0 = +0,80V
Pb2+ + 2e- → Pb E0 = –0,13V

a) Catodo = alumínio; Anodo = prata; ddp = + 2,47 V


b) Catodo = cobre; Anodo = alumínio; ddp = + 1,33 V
c) Catodo = alumínio; Anodo = zinco; ddp = + 2,43 V
d) Catodo = chumbo; Anodo = zinco; ddp = + 0,89 V
e) Catodo = prata; Anodo = cobre; ddp = + 0,46 V

Resposta: Alternativa E.
Para a resolução deste problema, será necessário analisar os pares indicados
pelas alternativas apresentadas.
Item a)
Al0(s) → Al3+(aq) + 3e- ∆G0 = −3 x F x (+1,67)
3Ag (aq) + 3e → 3Ag (s) ∆G0 = −3 x F x (+0,80)
+ - 0 +
Al0(s) + 3Ag+(aq) → Al3+(aq) + 3Ag0(s)
0
∆Gtotal = −3 x F x (+1,67) + [−3 x F x (+0,80)]
0
−3 x F x Etotal = −2 x F x (+1,67) + [−3 x F x (+0,80)]
0
Etotal = +1,67 + 0,80 = +2,47 V
Ânodo = alumínio
Cátodo = prata
Item b)
2Al0(s) → 2Al3+(aq) + 6e- ∆G0 = −6 x F x (+1,67)
3Cu+2(aq) + 6e- → 3Cu0(s) ∆G0 = −6 x F x (+0,34) +
2Al0(s) + 3Cu+2(aq) → 2Al3+(aq) + 3Cu0(s)
0
∆Gtotal = −6 x F x (+1,67) + [−6 x F x (+0,34)]
0
−6 x F x Etotal = −6 x F x (+1,67) + [−6 x F x (+0,34)]
0
Etotal = +1,67 + 0,34 = +2,01 V
Ânodo = alumínio
Cátodo = cobre
Item c)
2Al0(s) → 2Al3+(aq) + 6e- ∆G0 = −6 x F x (+1,67)
3Zn+2(aq) + 6e- → 3Zn0(s) ∆G0 = −6 x F x (−0,76) +
2Al0(s) + 3Zn+2(aq) → 2Al3+(aq) + 3Zn0(s)
0
∆Gtotal = −6 x F x (+1,67) + [−6 x F x (+0,34)]
0
−6 x F x Etotal = −6 x F x (+1,67) + [−6 x F x (−0,76)]

LIVRO OLÍMPICO – VOLUME I


54

0
Etotal = +1,67 + 0,34 = +0,91 V

Ânodo = alumínio
Cátodo = zinco
Item d)

Pb2+(aq) + 2e- → Pb0(s) ∆G0 = −2 x F x (−0,13)


Zn0(s) → Zn+2(aq) + 2e- ∆G0 = −2 x F x (+0,76) +
Zn0(s) + Pb+2(aq) → Zn2+(aq) + Pb0(s)
0
∆Gtotal = −2 x F x (−0,13) + [−2 x F x (+0,76)]
0
−2 x F x Etotal = −2 x F x (−0,13) + [−2 x F x (+0,76)]
0
Etotal = −0,13 + 0,76 = +0,63 V

Ânodo = zinco
Cátodo = chumbo
Item e)
Cu0(s) → Cu+2(aq) + 2e- ∆G0 = −2 x F x (−0,34)
2Ag+(aq) + 2e- → 2Ag0(s) ∆G0 = −2 x F x (+0,80) +
Cu0(s) + 2Ag+(aq) → Cu2+(aq) + 2Ag0(s)
0
∆Gtotal = −2 x F x (−0,34) + [−2 x F x (+0,80)]
0
−2 x F x Etotal = −2 x F x (−0,34) + [−2 x F x (+0,80)]
0
Etotal = −0,34 + 0,80 = +0,46 V

Ânodo = cobre
Cátodo = prata
56
(GRILLO) Foi necessário uma hora (1 h) para se eletrolisar completamente
100 mL de uma solução de nitrato de zinco com uma corrente de 3A. A partir
desta informação, calcule a normalidade desta solução.

Resposta: Estudo da eletrólise do nitrato de zinco: Zn(NO3 )2(aq) →



Zn+2
(aq) + 2 NO3 (aq)
Semirreação que ocorre no cátodo: Zn+2 −
(aq) + 2 e → Zn(s)

Cálculo da massa de zinco:


96500 C
65,41 g de Zn − − − − − − 2 mol de elétrons x
1 mol de elétrons
mZn − − − − − − − − − − 3 x 3600 C

65,41 x 3 x 3600 706428


mZn = = = 3,66 g
2 x 96500 193000
mZn 3,66 3,66
Cálculo da normalidade (N): N = eqg = 65,41 = 3,27 =
Zn x Vsolução x 0,100
2
1,12 egg. L−1

LIVRO OLÍMPICO – VOLUME I


55

57
(ITT-JEE) Calcule o tempo necessário para depositar um milimol de alumínio
metálico pela passagem de uma corrente de 9,65 A através de uma solução
aquosa de uma solução do íon alumínio.

Resposta: Semirreação que ocorre no alumínio: Al+3 −


(aq) + 3e → Al(s)

96500 C
1 mol − − − − − − − − 3 mol de elétrons x
mol de elétrons
1 x 10−3 mol − − − − − Q

Q = 289,50 C
Q
Cálculo do tempo: i = tempo

289,50 C
tempo = = 30 s
9,65
58
(ITT-JEE) Calculate the equilibrium constant for the reaction,

Fe+2 +4 +3 +3
(aq) + Ce(aq) → Fe(aq) + Ce(aq)

o o
(given ECe +4 / Ce+3 = +1,44 V; EFe+3 / Fe+2 = +0,68 V)

Resposta: Desenvolvendo as semirreações:

Fe+3 − +2
(aq) + 1e → Fe(aq) E 0 = +0,68 V

Ce+4 − +3
(aq) + 1e → Ce(aq) E 0 = +1,44 V

Invertendo a semirreação do ferro:

Fe+2 +3
(aq) → Fe(aq) + 1e

∆G0 = −1 x F x (−0,68)
Ce+4 − +3
(aq) + 1e → Ce(aq) ∆G0 = −1 x F x (+1,44) +
Fe+2 +4 +3 +3
(aq) + Ce(aq) → Fe(aq) + Ce(aq)

0
∆Gtotal = −1 x F x (−0,68) + [−1 x F x (+1,44)]
0
−1 x F x Etotal = −1 x F x (−0,68) + [−1 x F x (+1,44)]
0
Etotal = −0,68 + 1,44 = +0,46 V
0 0
Cálculo da constante de equilíbrio: ∆Geletroquímico = ∆Gtermodinâmico
0
−n x F x Etotal = −R x T x ln K eq
0
−n x F x Etotal
ln K eq =
−R x T
−1 x 96500 x 0,46
ln K eq =
−8,314 x 298
ln K eq = +17,92

K eq = e(+17,92)

LIVRO OLÍMPICO – VOLUME I


56

59
(ITT-JEE) Óxido de alumínio deve ser eletrolisado a uma temperatura igual a
1000ºC para produzir alumínio. Calcule a carga para preparar 5,12 kg de
alumínio.

Resposta: A esta temperatura, a eletrólise realizada foi a eletrólise ígnea.


Al2 O3(s) → 2Al+3 −2
(l) + 3O(l)

Semirreação que ocorre no cátodo: Al+3 −


(l) + 3e → Al(s)
1
Semirreação que ocorre no ânodo: O−2
(l) → 2 O2(g) + 2e

Multiplicando a semirreação do cátodo por dois e multiplicando a semirreação


do ânodo por três.

Al2 O3(s) → 2Al+3 −2


(l) + 3O(l)
2Al+3 −
(l) + 6e → 2Al(s)
3
3O−2
(l) → 2 O2(g) + 6e

+
Al2 O3(s) → 2Al(s) + 3O2(g)

Cálculo da carga (Q):


96500 C
27 g − − − − − − 3 mol de elétrons x
mol de elétrons
5120 g − − − − − Q

5120 𝑥 3 𝑥 96500
Q= = 5,49 x 10+7 C
27
60
(ITT-JEE) Find the equilibrium constant for the reaction, In+2 +2
(aq) + Cu(aq) →
o o
In+3 +
(aq) + Cu(aq) at 298 K. Given ECu+2 / Cu+ = +0,15 V; EIn+2 / In+ =
o
−0,40 V; EIn+3 / In+ = −0,42 V.

Resposta: Cálculo do potencial para a primeira parte:

In+2 − + 0
(aq) + 1e → In(aq) ∆G = −1 x F x (−0,40)
+ +3
In(aq) → In(aq) + 2e− ∆G0 = −2 x F x (+0,42) +
In+2
(aq) → In+3
(aq) + 1e −

0
∆Gtotal = −1 x F x (−0,40) + [−2 x F x (+0,42)]
0
−1 x F x Etotal = −1 x F x (−0,40) + [−2 x F x (+0,42)]
0
Etotal = +0,44 V

Cálculo do potencial para a segunda parte:

In+2 +3
(aq) → In(aq) + 1e

∆G0 = −1 x F x (+0,44)
Cu+2 − + 0
(aq) + 1e → Cu(aq) ∆G = −1 x F x (+0,15) +
In+2 +2 +3 +
(aq) + Cu(aq) → In(aq) + Cu(aq)

0
∆Gtotal = −1 x F x (−0,40) + [−1 x F x (+0,42)]

LIVRO OLÍMPICO – VOLUME I


57

0
−1 x F x Etotal = −1 x F x (+0,44) + [−1 x F x (+0,15)]
0
Etotal = +0,59 V
0 0
Cálculo da constante de equilíbrio: ∆Geletroquímico = ∆Gtermodinâmico
0
−n x F x Etotal = −R x T x ln K eq
0
−n x F x Etotal
ln K eq =
−R x T
−1 x 96500 x 0,59
ln K eq =
−8,314 x 298
ln K eq = +23,00

K eq = 𝑒 (+23,00)

61
(ITT-JEE) The standard electrode potentials of Mn3+/Mn2+ and Mn2+/Mn
electrodes at 25ºC are 1,51 V and –1,18V, respectively. Calculate the
standard electrode potential of Mn3+ + 3 e- → Mn and Keq.
Resposta: Cálculo do potencial para a primeira parte:

Mn+3 − +2
(aq) + 1e → Mn(aq) ∆G0 = −1 x F x (1,51)
Mn+2 − 0
(aq) + 2e → Mn(s) ∆G0 = −2 x F x (−1,18) +
Mn+3 − 0
(aq) + 3e → Mn(s)

0
∆Gtotal = −1 x F x (1,51) + [−2 x F x (−1,18)]
0
−3 x F x Etotal = −1 x F x (1,51) + [−2 x F x (−1,18)]
0
Etotal = − 0,283 V
0 0
Cálculo da constante de equilíbrio: ∆Geletroquímico = ∆Gtermodinâmico
0
−n x F x Etotal = −R x T x ln K eq
0
−n x F x Etotal
ln K eq =
−R x T
−3 x 96500 x (−0,283)
ln K eq =
−8,314 x 298
−81928,5
ln K eq =
−2477,572
ln K eq = +33,07

K eq = e(+33,07)

LIVRO OLÍMPICO – VOLUME I


58

62
(ITT-JEE) The density of Cu is 8,94 g cm–3. The quantity of electricity needed
to plate an area 10 cm x 10 cm to a thickness of 10–2 cm using CuSO4 solution
is:
a) 13586 C
b) 27172 C
c) 40758 C
d) 20348 C

Resposta: Alternativa B.

Semirreação que ocorre no cátodo: Cu+2 −


(aq) + 2e → Cu(s)

Cálculo da massa de cobre, a partir da densidade.


massa
d=
volume

massa = 8,94 x (10 x 10 x 10−2 ) = 8,94 g


96500 C
63,55 g − − − − − 2 mol de elétrons x
mol de elétrons
8,94 g − − − − − − Q

Q = 27150,59 C

63
(INSTITUTO MILITAR DE ENGENHARIA) São conhecidas três pilhas cujas
reações totais são:
3 Zn(s) + Cr2 O−2 + +2 +3
7(aq) + 14H(aq) → 3 Zn(aq) + 2Cr(aq) + 7H2 O(l)
+
Zn(s) + 2H(aq) → Zn+2(aq) + H2 (g)
Zn(s) + Cu+2 +2
(aq) → Zn(aq) + Cu(s)
E cujos potenciais observados nas condições padrões são respectivamente
2,09 V; 0,76 V e 1,10 V. A partir destes dados, determine os potenciais
padrões de oxidação para as semirreações do zinco, do cobre e do dicromato
envolvidas, e diga justificando se é espontânea a reação:

3 Cu+2 +3 −2 +
(aq) + 2 Cr(aq) + 7H2 O(l) → 3 Cu(s) + Cr2 O7(aq) + 14H(aq)

Resposta: Determinação do potencial de oxidação do zinco:


+
Zn(s) + 2H(aq) → Zn+2
(aq) + H2 (g) ∆G0 = −2 x F x (+ 0,76)
+
H2 (g) → 2H(aq) + 2 e− ∆G0 = −2 x F x (+ 0,00)
Zn(s) → Zn+2
(aq) + 2 e
− 0
∆Gtotal 0
= −2 x F x Etotal

0
−2 x F x Etotal = −2 x F x (+ 0,76) + [−2 x F x (+ 0,00)]
0
Etotal = +0,76 V

Determinação do potencial da semirreação de oxidação do cobre:

Zn(s) + Cu+2 +2
(aq) → Zn(aq) + Cu(s) ∆G0 = −2 x F x (+ 1,10)
Zn+2 −
(aq) + 2 e → Zn(s) ∆G0 = −2 x F x (−0,76)

LIVRO OLÍMPICO – VOLUME I


59

Cu+2 −
(aq) + 2 e → Cu(s)
0
∆Gtotal 0
= −2 x F x Etotal

0
−2 x F x Etotal = −2 x F x (+ 1,10) + [−2 x F x (− 0,76)]

0
Etotal = +0,34 V

Logo, o potencial de oxidação para a semirreação é:


Cu(s) → Cu+2 (aq) + 2 e
− 0
Etotal = −0,34 V

Determinação do potencial da semirreação de oxidação do dicromato:

3 Zn(s) + Cr2 O−2 + +2 +3


7(s) + 14H(aq) → 3 Zn(aq) + 2Cr(aq) + 7H2 O(l)
3Zn+2 −
(aq) + 6 e → 3Zn(s) +
Cr2 O−2 + +3
7(s) + 14H(aq) → 2Cr(aq) + 7H2 O(l)

0 0
∆Gtotal = −6 x F x Etotal

0
−6 x F x Etotal = −6 x F x (+ 1,10) + [−6 x F x (− 0,76)]

0
Etotal = +1,33 V

Determinação para analisar se o processo é espontâneo ou não espontâneo.

3 Cu+2 +3 −2 +
(aq) + 2 Cr(aq) + 7H2 O(l) → 3 Cu(s) + Cr2 O7(aq) + 14H(aq)

Cálculo do potencial:
+3
2Cr(aq) + 7H2 O(l) → Cr2 O−2 +
7(aq) + 14H(aq)
∆G10 = −6 x F x (−0,76)
3Cu+2 −
(aq) + 6e → 3Cu(s) ∆G20 = −6 x F x (+0,34)
3Cu+2 +3 −2 +
(aq) + 2Cr(aq) + 7H2 O(l) → 3Cu(s) + Cr2 O7(aq) + 14H(aq)

0
−6 x F x Etotal = −6 x F x (−0,76) + [−6 x F x (+0,34)]

0
Etotal = −0,76 + 0,34 = −0,42 V

Determinação da variação da energia livre de Gibbs:


0 0
∆Gtotal = −6 x F x Etotal

0
∆Gtotal = −6 x 96500 x (−0,42) = +243180 J

Processo não espontâneo, uma vez que a variação da energia livre de Gibbs
é maior que zero.

LIVRO OLÍMPICO – VOLUME I


60

64
(INSTITUTO MILITAR DE ENGENHARIA) Para a redução do Cr2 O−2 7(aq)

pelo Br(aq) em meio ácido, conhece-se as semirreações e os potenciais
padrões de oxidação:
+
2 Cr(aq) + 7H2 O(l) → Cr2 O−2 +
7(aq) + 14H(aq) + 6e

Eº = -1,33 V

2Br(aq) → Br2 (g) + 2e− Eº = -1,07 V

Pede-se:
a) Escrever a equação completa e balanceada.
b) Dizer se esta reação é espontânea, justificando.

Resposta:

Item a) Invertendo a primeira semirreação e multiplicando por três a segunda


semirreação:

Cr2 O−2 + − +
7(s) + 14H(aq) + 6e → 2 Cr(aq) + 7H2 O(l)
∆G10 = −6 x F x (+1,33)

6Br(aq) → 3Br2 (g) + 6e− ∆G20 = −6 x F x (−1,07)
Cr2 O−2 + − +
7(s) + 14H(aq) + 6Br(aq) → 2 Cr(aq) + 7H2 O(l) + 3Br2 (g)

0
−6 x F x Etotal = −6 x F x (+1,33) + [−6 x F x (−1,07)]

0
Etotal = +0,26 V

Cálculo da variação da energia de gibbs padrão da reação eletroquímica


global:
0 0
∆Gtotal = −6 x F x Etotal

0
∆Gtotal = −6 x 96500 x (+0,26) = −150540 J

Como o valor da energia livre de Gibbs é negativa, seu processo é


espontâneo.

65
(INSTITUTO MILITAR DE ENGENHARIA) Com base nos potenciais-padrão
de redução (Eored) disponíveis abaixo, determine a constante de equilíbrio
para a oxidação do íon Fe2+ por oxigênio, a 25ºC, em meio ácido, de acordo
com a reação: O2(g) + 4H+(aq) + 4Fe2+(aq) → 4Fe3+(aq) + 2H2O(l). Dados:
O2(g) + 4H+(aq) + 4e– → 2H2O(l) Eored = +1,23 V
Fe (aq) + 2e → Fe(s)
2+ – Eored = −0,450 V
Fe3+(aq) + 3e– → Fe(s) Eored = −0,0430 V

LIVRO OLÍMPICO – VOLUME I


61

Resposta: Para a resolução deste problema, será necessário calcular o


potencial padrão do ferro.

Fe2+(aq) + 2e– → Fe(s) Eored = −0,450 V

Fe3+(aq) + 3e– → Fe(s) Eored = −0,0430 V


Multiplicando a primeira semirreação por quatro e invertendo e multiplicando
por quatro a segunda semirreação, temos:

4Fe(s) → 4Fe2+(aq) + 8e– ∆G10 = −8 x F x (+0,450)


4Fe3+(aq) + 12e– → 4Fe(s) ∆G20 = −12 x F x (−0,0430)
0 0
4Fe3+(aq) + 4e– → 4Fe2+(aq) ∆Gtotal = −4 x F x Etotal

0
∆Gtotal = ∆G10 + ∆G20

0
−4 x F x Etotal = −8 x F x (+0,450) − 12 x F x (−0,0430)

0 −3,60 + 0,516
Etotal = = +0,771 V
−4
Cálculo do potencial padrão da semirreação O2(g) + 4H+(aq) + 4Fe2+(aq) →
4Fe3+(aq) + 2H2O(l):

O2(g) + 4H+(aq) + 4e– → 2H2O(l) ∆G10 = −4 x F x (+1,23)


4Fe (aq) → 4Fe (aq) + 4e
2+ 3+ – ∆G20 = −4 x F x (−0,771)
O2(g) + 4H+(aq) + 4Fe2+(aq) → 4Fe3+(aq) + 2H2O(l)
0
∆Gtotal = ∆G10 + ∆G20

0
−12 x F x Etotal = −4 x F x (+1,23) − 4 x F x (−0,771)

0 −4,92 + 3,084
Etotal = = +0,153 V
−12
0 0
Cálculo da constante de equilíbrio: ∆Geletroquímico = ∆Gtermodinâmico
0
−n x F x Etotal = −R x T x ln K eq
0
−n x F x Etotal
ln K eq =
−R x T
−12 x 96500 x (+0,153)
ln K eq =
−8,314 x 298
−177174
ln K eq =
−2477,572
ln K eq = +71,51

K eq = e(+71,51)

LIVRO OLÍMPICO – VOLUME I


62

66
(U. S NATIONAL CHEMISTRY OLYMPIAD) For the galvanic cell Zn(s)ǀZn2+(aq)
ǀǀ Cu2+(aq)ǀCu(s) the standard cell potential is 1,10 V. What is the potential at °C
if the concentrations of Zn2+ and Cu2+ are 2,5 M and 0,10 M respectively?
a) 1,14 V
b) 1,10 V
c) 1,06 V
d) 1,02 V

Resposta: Alternativa C.
Para a resolução desta questão, como as concentrações das espécies
químicas estão diferentes de 1,0 mol/L, será necessário aplicar a equação de
Nernst.
Equação iônica para o funcionamento da pilha:

Zn(s) + Cu+2 +2
(aq) → Cu(s) + Zn(aq)

RxT
E = E0 − x ln Q
nxF
1
8,314 x 298 a⏞
Cu x a Zn+2
E = +1,10 − x ln
2 x 96500 a⏟
Zn x a Cu+2
1
1
8,314 x 298 a⏞
Cu x 2,50
E = +1,10 − x ln = +1,06 V
2 x 96500 a⏟
Zn x 0,10
1

67
(U. S NATIONAL CHEMISTRY OLYMPIAD) Questions A and B should be
answered with reference to the voltaic cell shown and these half-reactions.
Rh+3(aq) + 3e- → Rh(s) Eº = + 0,80 V
Cu+(aq) + 1e- → Cu(s) Eº = + 0,52 V

A – What is the direction of electron flow in the external circuit if the


concentrations of Cu+ and Rh3+ are each 1 M?
a) from the Rh anode to the Cu cathode
b) from the Rh cathode to the Cu anode
c) from the Cu anode to the Rh cathode
d) from the Cu cathode to the Rh anode

LIVRO OLÍMPICO – VOLUME I


63

B – What is the voltage of this cell if the concentrations of Cu+ and Rh3+ are
each 1 M?
a) 0,28 V
b) 0,76 V
c) 1,32 V
d) 2,36 V

Resposta do Item A: Alternativa C.


Rh+3(aq) + 3e- → Rh(s) Eº = + 0,80 V
Cu+(aq) + 1e- → Cu(s) Eº = + 0,52 V

Invertendo a semirreação do cobre e multiplicando por três:

Rh+3(aq) + 3e- → Rh(s) ∆G10 = −3 x F x (+0,80)


3 Cu(s) → 3 Cu (aq) + 3e
+ - ∆G20 = −3 x F x (−0,52)
Rh+3(aq) + 3 Cu(s) → Rh(s) + 3 Cu+(aq)
0
∆Gtotal = ∆G10 + ∆G20
0
−3 x F x Etotal = −3 x F x (+0,80) − 3 x F x (−0,52)

0
Etotal = + 0,28 V

Rh+3(aq) = agente oxidante (cátodo) e Cu(s) = agente redutor (ânodo).


O fluxo de elétrons sai do ânodo para o cátodo.
Resposta do Item B: Alternativa A.
Rh+3(aq) + 3e- → Rh(s) Eº = + 0,80 V
Cu+(aq) + 1e- → Cu(s) Eº = + 0,52 V

Invertendo a semirreação do cobre e multiplicando por três:

Rh+3(aq) + 3e- → Rh(s) ∆G10 = −3 x F x (+0,80)


3 Cu(s) → 3 Cu+(aq) + 3e- ∆G20 = −3 x F x (−0,52)
Rh+3(aq) + 3 Cu(s) → Rh(s) + 3 Cu+(aq)
0
∆Gtotal = ∆G10 + ∆G20

0
−3 x F x Etotal = −3 x F x (+0,80) − 3 x F x (−0,52)

0
Etotal = + 0,28 V

LIVRO OLÍMPICO – VOLUME I


64

68
(INSTITUTO MILITAR DE ENGENHARIA) Construiu-se uma célula
eletrolítica de eletrodos de platina, tendo como eletrólito uma solução aquosa
de iodeto de potássio. A célula operou durante um certo intervalo de tempo
sob corrente constante de 0,2A. Ao final da operação, o eletrólito foi
completamente transferido para um outro recipiente e titulado com solução
0,1M de tiossulfato de sódio. Sabendo-se que foram consumidos 25 mL da
solução de tiossulfato na titulação, determine o tempo durante o qual a célula
operou.
S4O6-2 + 2e- → 2 S2O3-2 Eº = + 0,08 V
I2 + 2e- → 2 I- Eº = + 0,54 V

Resposta: Para a resolução do problema, será necessário inverter a primeira


semirreação.

2 S2O3-2 → S4O6-2 + 2e- ∆G10 = −2 x F x (−0,08)


I2 + 2e- → 2 I- ∆G20 = −2 x F x (+0,54)
2 S2O3-2 + I2 → S4O6-2 + 2 I- (Equação iônica para o funcionamento da pilha)
0
∆Gtotal = ∆G10 + ∆G20

0
−2 x F x Etotal = −2 x F x (−0,08) − 2 x F x (+0,54)

0
Etotal = −0,08 + 0,54 = +0,46 V

Cálculo do número de mol de tiossulfato:


ntiossulfato = 0,10 x 25 x 10−3 = 2,50 x 10−3 mol
Cálculo do número de mol de iodo molecular, a partir da equação iônica:
2 S2O3-2 + I2 → S4O6-2 + 2 I-
ntiossulfato niodo
=
2 1
2,50 x 10−3 mol niodo
=
2 1
niodo = 1,25 x 10−3 mol
Cálculo da carga produzida, a partir de iodo produzido.
I2 → 2I − + 2e−
96500 C
Q = 1,25 x 10−3 x 2 mol de elétrons mol x
1 mol de elétrons
Q = 241,25 C
241,25
Cálculo do tempo: tempo = 0,20
= 120,250 s

LIVRO OLÍMPICO – VOLUME I


65

69
(INSTITUTO MILITAR DE ENGENHARIA) Dada a reação Cu + 2 HCl →
CuCl2 + H2, assinale a afirmativa correta sabendo-se que os potenciais -
padrão de redução do cobre e do hidrogênio são respectivamente 0,34 e 0,00
V.
a) A reação produz corrente elétrica
b) A reação não ocorre espontaneamente
c) A reação ocorre nas pilhas de Daniell
d) O cobre é o agente oxidante
e) O hidrogênio sofre oxidação

Resposta: Alternativa B.

Cu + 2 HCl → não há formação de produtos.

O cobre não é atacado pela presença do ácido clorídrico. Diante desta


afirmativa, o processo não é espontâneo (Eº < 0 e ΔGº > 0).

70
(ITT-JEE) A ddp para a reação, 2Cu+(aq) → Cu(s) + Cu+2(aq) é +0,36 V a 298 K.
Calcule a constante de equilíbrio para esta reação.

Resposta: Cálculo da constante de equilíbrio químico, a partir da seguinte


0 0
relação termodinâmica. ∆Geletroquímico = ∆Gtermodinâmico
0
−n x F x Etotal = −R x T x ln K eq
0
−n x F x Etotal
ln K eq =
−R x T
−2 x 96500 x (+0,36)
ln K eq =
−8,314 x 298
ln K eq = +28,04

K eq = 𝑒 (+28,04)

LIVRO OLÍMPICO – VOLUME I


66

71
(ITT-JEE) Us the van der Waals equation of state to calculate the pressure at
35ºC that would result if the chlorine produced from electrolysis of 10 kg
molten NaCl is compressed into a tank with a volume of 75 L. Assume 96%
efficiency of the elestrolysis, a = 6,50 atm x L² x mol-2, b = 0,0562 L/mol.

Resposta: É bastante interessante apresentar o processo de eletrólise ígnea


do cloreto de sódio.

NaCl(s) → Na+ −
(l) + Cl(l)

Semirreação que ocorre no ânodo: 2Cl− (l) → Cl2(g) + 2e


Semirreação que ocorre no cátodo: Na+ −


(l) + 1e → Na (s)
Multiplicando a semirreação do sódio por dois e somando as semirreações:

2NaCl(s) → 2Na+ −
(l) + 2Cl(l)
2Cl−
(l) → Cl2(g) + 2e

+
2Na(l) + 2e− → 2Na(s) +
2NaCl(s) → 2Na(s) + Cl2(g)

Diante desta análise da eletrólise ígnea do cloreto de sódio, observa-se que


há a produção de sódio metálico e também do gás cloro.

Para um rendimento de 96%, o número de mol de gás cloro será calculado


pela relação estequiométrica.

2 mol de NaCl − − − − − 1 mol de Cl2(g) 𝑥 0,96


10000
mol − − − − − − nCl2
58,5

10000
0,96 𝑥
58,5
nCl2 = = 82,05 mol
2

Expressão da equação dos gases de van der Waals e aplicando os dados.

a
{preal + 2 } x {Vmolar − b} = R x T
Vmolar

6,50 75
{preal + 2} x{ − 0,0562} = 0,08206 x 308
75 82,05
( )
82,05

preal = 21,68 atm

LIVRO OLÍMPICO – VOLUME I


67

72
(ITT-JEE) Considere as seguintes equações eletroquímicas apresentadas a
seguir:
2 IrCl3−
6 + 3HCOOH → 2Ir + 3CO2 + 12Cl + 6H
− +

Dados:
CO2 + 2H3 O+ + 2e− → HCOOH + 2H2 O E 0 = −0,20 V
3− − −
IrCl6 + 3e → Ir + 6Cl E 0 = +0,77 V
a) Determine o potencial padrão desta célula.
b) Esta reação como está escrita, é termodinamicamente espontânea?
Explique.

Resposta:

Item a) Para a determinação da ddp padrão, será necessário inverter e


multiplicar por três a segunda semirreação e multiplicar por dois a segunda
semirreação.

3HCOOH + 6H2 O → 3CO2 + 6H3 O+ + 6e−


2IrCl3− −
6 + 6e → 2Ir + 12Cl

+
3−
3HCOOH + 6H2 O + 2IrCl6 → 3CO2 + 6H3 O+ + 2Ir + 12Cl−
0
∆Gtotal = ∆G10 + ∆G20

0
−6 x F x Etotal = −6 x F x (+0,20) − 6 x F x (+0,77)

0
Etotal = +0,20 + 0,77 = +0,97 V

Item b) Observando o valor da ddp padrão, o processo é sim espontâneo uma


0
vez que se trata de um valor positivo (Etotal > 0).

73
(ITT-JEE) Magnésio metálico é produzido comercialmente de maneira isolada
através do cloreto de magnésio, através da eletrólise conforme a equação
química a seguir: MgCl2(s) → Mg (s) + Cl2(g) . Qual é a massa de
magnésio que pode ser produzido se a corrente elétrica for igual a 430 A em
1 hora?

Resposta: Para a resolução deste problema, é importante ressaltar e


apresentar o processo da eletrólise ígnea do cloreto de magnésio.

MgCl2(s) → Mg +2 −
(l) + 2 Cl(l)

Semirreação que ocorre no ânodo: 2Cl−


(l) → Cl2(g) + 2e

Semirreação que ocorre no cátodo: Mg +2 −


(l) + 2e → Mg (s)

Somando as semirreações:

MgCl2(s) → Mg +2 −
(l) + 2 Cl(l)
2Cl−
(l) → Cl2(g) + 2e

Mg +2 −
(l) + 2e → Mg (s) +
MgCl2(s) → Mg (s) + Cl2(g)

LIVRO OLÍMPICO – VOLUME I


68

Cálculo da massa de magnésio produzido, através da eletrólise ígnea.


96500 C
24,0 g − − − − − − 2 mol de elétrons x
mol de elétrons
Massa − − − − − − 3600 x 430 C

3600 x 430 x 24
Massa = = 192,50 g
2 x 96500

74
(GRILLO) Calcule a fem para uma pilha de Daniell à 25ºC, quando as
concentrações das quantidades de matéria para o sulfato de cobre e para o
sulfato de zinco são respectivamente iguais a 0,50 mol/L e 0,10 mol/L. Calcule
novamente a fem para a mesma pilha de Daniell à 25ºC, levando em
consideração que o coeficiente de atividade médio para o sulfato de cobre e
sulfato de zinco são respectivamente iguais a 0,068 e 0,15. Informação para
a resolução do problema:
Zn+2 −
(aq) + 2e → Zn(s) Eº = - 0,76 V
+2 −
Cu(aq) + 2e → Cu(s) Eº = + 0,34 V

Resposta: Para a primeira parte do problema, será necessário calcular o


potencial-padrão, invertendo a semirreação do zinco.

Cu+2 −
(aq) + 2e → Cu(s) (ânodo)
Zn(s) → Zn+2
(aq) + 2e

(cátodo)
Zn(s) + Cu+2 +2
(aq) → Cu(s) + Zn(aq) (equação química global)

0
∆Gtotal = −2 x F x (+0,76) + [−2 x F x (+0,34)]
0
−2 x F x Etotal = −2 x F x (+0,76) + [−2 x F x (+0,34)]
0
Etotal = +0,76 + 0,34 = +1,10 V

Cálculo do potencial aplicando a equação de Nernst com relação à


concentração da quantidade de matéria:

RxT
E = E0 − x ln Q
nxF

RxT [Zn+2 ]
E = E0 − x ln
nxF [Cu+2 ]

8,314 x 298 0,10


E = +1,10 − x ln = +1,121 V
2 x 96500 0,50

Cálculo do potencial levando em consideração as atividades químicas.

RxT
E = E0 − x ln Q
nxF
RxT aZn+2 x aCu
E = E0 − x ln
nxF aCu+2 x aZn

LIVRO OLÍMPICO – VOLUME I


69

Cálculo das atividades:


aZn+2 = ƔZn+2 x [Zn+2 ] = 0,15 𝑥 0,10 = 0,015

aCu+2 = ƔCu+2 x [Cu+2 ] = 0,068 𝑥 0,50 = 0,034

8,314 x 298 0,015


E = +1,10 − x ln = +1,111 V
2 x 96500 0,034

75
(ITT-JEE) Considere a célula galvânica,
Zn(s) |Zn+2 −
(aq) (0,04 M)‖Cl(aq) (0,005 M)||Cl2(g) (0,10 atm)|Pt (s) ⌉
Determine a fem da célula e a constante de equilíbrio para a reação
apresentada acima.

Resposta:

Expressão para o funcionamento da pilha.

Semirreação que ocorre no ânodo: Zn(s) → Zn+2(aq) + 2e


Semirreação que ocorre no cátodo: 2Cl−


(l) → Cl2(g) + 2e

Invertendo a segunda semirreação, temos:


Cl2(g) + 2e− → 2Cl− (aq) Eº = + 1,36 V
+2 −
Zn(s) → Zn(aq) + 2e Eº = + 0,76 V
Zn(s) + Cl2(g) → 2Cl− +2
(aq) + Zn(aq) (equação química global)

0
∆Gtotal = −2 x F x (+0,76) + [−2 x F x (+1,36)]
0
−2 x F x Etotal = −2 x F x (+0,76) + [−2 x F x (+1,36)]
0
Etotal = +2,12 V

Cálculo da ddp, aplicando a equação de Nernst:

RxT
E = E0 − x ln Q
nxF

RxT [Zn+2 ] x [Cl− ]2


E = E0 − x ln
nxF aZn x pCl2

8,314 x 298 0,04 x (0,005)2


E = +2,12 − x ln = +2,27 V
2 x 96500 1 x 0,10

LIVRO OLÍMPICO – VOLUME I


70

76
(OLIMPÍADA NORTE-NORDESTE DE QUÍMICA) Em princípio, uma bateria
poderia ser desenvolvida a partir de alumínio metálico e cloro gasoso.
a) escreva a equação balanceada que ocorre em uma bateria cujas
semirreações são: Al3+(aq) /Al(s) e Cl2(g)/2Cl- (aq).
b) diga que semirreação ocorre no ânodo e que semirreação ocorre no
cátodo.
c) calcule o potencial padrão para esta bateria (ΔE°).
d) se a bateria produz uma corrente de 0,75 A, quanto tempo ela irá operar
se o eletrodo de alumínio contiver 30,0 g do metal? (considere que há
quantidade suficiente de cloro).
Dados: Cl2(g) + 2e- → 2 Cl-(aq) E° = +1,36 V
Al (aq) + 3e → Al(s)
3+ - E° = -1,66 V
Constante de Faraday = 96500 C/mol

Resposta: Invertendo a semirreação do alumínio e multiplicando por dois.


2Al0(s) → 2Al3+(aq) + 6e- ∆G0 = −6 x F x (+1,66)

Multiplicando por três a semirreação do cloro.


3Cl2(g) + 6e- → 6Cl-(aq) ∆G0 = −6 x F x (+1,36)

Equação eletroquímica global:

2Al0(s) → 2Al3+(aq) + 6e- ∆G0 = −6 x F x (+1,66)


3Cl2(g) + 6e- → 6Cl-(aq) ∆G0 = −6 x F x (+1,36) +
2Al0(s) + 3Cl2(g) → 2Al3+(aq) + 6Cl-(aq)
0
∆Gtotal = −6 x F x (+1,66) + [−6 x F x (+1,36)]
0
−6 x F x Etotal = −6 x F x (+1,66) + [−6 x F x (+1,36)]
0
Etotal = +1,66 + 1,36 = +3,02 V

Item a) 2Al0(s) + 3Cl2(g) → 2Al3+(aq) + 6Cl-(aq)

Item b) ânodo = alumínio e cátodo = cloro.


0
Item c) Etotal = +3,02 V

Item d) Cálculo da massa de alumínio.


96500 C
27,0 g − − − − − − 3 mol de elétrons x
mol de elétrons
24,0 g − − − − − − Q

24 x 3 x 96500
Q= = 321666,67 C
27

Cálculo do tempo:

321666,67 1 hora
tempo = = 4,29 x 105 s x = 119,13 h
0,75 3600 s

LIVRO OLÍMPICO – VOLUME I


71

77
(OLIMPÍADA NORTE-NORDESTE DE QUÍMICA) Uma célula eletroquímica
é constituída por:
Eletrodo A - fio de platina que mergulha numa solução que contém íons Fe+3
e Fe+2, ambos com concentração 1 mol/L.
Eletrodo B - vareta de tálio mergulhada numa solução 1 mol/L de Tl+1.
Sabendo que os potenciais-padrão referentes às semirreações:
Tl+1 + e- → Tl
Fe+3 + e- → Fe+2 são, respectivamente, - 0,34 V e + 0,77 V, faça o que se
pede:
a) Mostre as semirreações que ocorrem no cátodo e no ânodo.
b) Escreva a equação da reação que ocorre na célula.
c) Calcule o potencial da célula.

Resposta: Invertendo a primeira semirreação e determinando a equação


eletroquímica global.

Tl0(s) → Tl+(aq) + 1e- ∆G0 = −1 x F x (+0,34)


Fe+3(aq) + 1e- → Fe+2(aq) ∆G0 = −1 x F x (+0,77) +
Tl0(s) + Fe+3(aq) → Tl+(aq) + Fe+2(aq)
0
∆Gtotal = −1 x F x (+0,34) + [−1 x F x (+0,77)]
0
−1 x F x Etotal = −1 x F x (+0,34) + [−1 x F x (+0,77)]
0
Etotal = +0,77 + 0,34 = +1,11 V

Item a)
Cátodo: Fe+3(aq) + 1e- → Fe+2(aq)
Ânodo: Tl0(s) → Tl+(aq) + 1e-

Item b) Equação eletroquímica global: Tl0(s) + Fe+3(aq) → Tl+(aq) + Fe+2(aq)


0
Item c) Cálculo do potencial da célula: Etotal = +0,77 + 0,34 = +1,11 V

78
(OLIMPÍADA NORTE-NORDESTE DE QUÍMICA) O citocromo, molécula que
representaremos como CyFe+2, reage com o ar que respiramos para fornecer
energia necessária para sintetizar adenosina trifosfato (ATP). O corpo usa
ATP como fonte de energia para outras reações. Com base nos potenciais de
redução:

O2(g) + 4 H+(aq) + 4e- → 2 H2O(l) Eº = +0,82 V


CyFe+3 + 1e- → CyFe+2 Eº = +0,22 V

Responda as questões abaixo:

a) Considerando que esta reação ocorra em uma célula eletroquímica,


indique os eletrodos (cátodo e ânodo) onde cada semirreação deve ocorrer.
b) Escreva a equação geral balanceada, e determine o potencial Eº
gerado nessa reação.

LIVRO OLÍMPICO – VOLUME I


72

Resposta: Invertendo a segunda semirreação e multiplicando por quatro.

O2(g) + 4 H+(aq) + 4e- → 2 H2O(l) Eº = +0,82 V


4 CyFe+2 → 4 CyFe+3 + 4e- Eº = +0,22 V
O2(g) + 4 H+(aq) + 4 CyFe+2 → 2 H2O(l) + 4 CyFe+3
0
∆Gtotal = −4 x F x (+0,82) + [−4 x F x (−0,22)]
0
−4 x F x Etotal = −4 x F x (+0,82) + [−4 x F x (−0,22)]
0
Etotal = +0,82 − 0,22 = +0,60 V

Item a)

Cátodo: O2(g) + 4 H+(aq) + 4e- → 2 H2O(l)


Ânodo: 4 CyFe+2 → 4 CyFe+3 + 4e-

Item b)

O2(g) + 4 H+(aq) + 4e- → 2 H2O(l) Eº = +0,82 V


4 CyFe+2 → 4 CyFe+3 + 4e- Eº = +0,22 V
O2(g) + 4 H+(aq) + 4 CyFe+2 → 2 H2O(l) + 4 CyFe+3
0
∆Gtotal = −4 x F x (+0,82) + [−4 x F x (−0,22)]
0
−4 x F x Etotal = −4 x F x (+0,82) + [−4 x F x (−0,22)]
0
Etotal = +0,82 − 0,22 = +0,60 V

79
(ITT-JEE) A produção comercial de alumínio metálico envolve a eletrólise do
Al2O3, obtida a partir da bauxita. A equação balanceada é a seguinte:
2 Al2O3 + C → 4 Al + 3 CO2
a) Quantos mols de alumínio são produzidos se 12 mols de elétrons
passam através da célula?
b) Quantos quilogramas de alumínio metálico são produzidos se a
corrente de 1250 A é atravessado por um período de 1,25 horas?

Resposta: Estudando a eletrólise ígnea, para a produção de alumínio


metálico:

Al2 O3(s) → 2Al+3 −2


(l) + 3O(l)

Semirreação que ocorre no cátodo: 2Al+3 − 0


(l) + 6𝑒 → 2Al(s)
3
Semirreação que ocorre no ânodo: 3O−2 0
(l) → 2 O2(g) + 6𝑒

Somando as semirreações, temos:


2Al+3 − 0
(l) + 6e → 2Al(s)
3
3O−2 0
(l) → 2 O2(g) + 6e

+
3 0
2Al+3 −2 0
(l) + 3O(l) → O2(g) + 2Al(s)
2

Item a) Cálculo do número de mol de alumínio metálico.

LIVRO OLÍMPICO – VOLUME I


73

2 mol de Al ---- 6 mol de elétrons


nAl ---------------- 12 mol de elétrons
nAl = 4 mol de alumínio metálico

Item b) Cálculo da massa de alumínio metálico.


96500 C
27,0 g − − − − − − 3 mol de elétrons x
mol de elétrons
Massa − − − − − − 1250 x 1,25 x 3600 C

Massa = 524,61 g (0,525 kg)

80
(INSTITUTO TECNOLÓGICO DA AERONÁUTICA) Considere os eletrodos
representados pelas semi-equações químicas seguintes e seus respectivos
potenciais na escala do eletrodo de hidrogênio (Eo) e nas condições-padrão:
I. In+(aq) + e-(CM) → In(s) EºI = - 0,14 V
II. In+2(aq) + e-(CM) → In+(aq) EºII = - 0,40 V
III. In+3(aq) + 2e-(CM) → In+(aq) EºIII = - 0,44 V
IV. In (aq) + e (CM) → In (aq)
+3 - +2 EºIV = - 0,49 V
Assinale a opção que contém o valor correto do potencial-padrão do eletrodo
representado pela semi-equação In+3(aq) + 3e-(CM) → In(s).
a) −0,30V
b) −0,34V
c) −0,58V
d) −1,03V
e) −1,47

Resposta: Alternativa B.

A partir da primeira e da terceira semirreação, temos:

In+(aq) + e-(CM) → In(s) EºI = - 0,14 V ∆G0 = −1 x F x (−0,14)


In+3(aq) + 2e-(CM) → In+(aq) EºIII = - 0,44 V ∆G0 = −2 x F x (−0,44)

Somando as semirreações:

In+(aq) + e-(CM) → In(s) EºI = - 0,14 V ∆G0 = −1 x F x (−0,14)


In+3(aq) + 2e-(CM) → In+(aq) EºIII = - 0,44 V ∆G0 = −2 x F x (−0,44)+
In+3(aq) + 3e-(CM) → In(s)
0
∆Gtotal = −1 x F x (−0,14) + [−2 x F x (−0,44)]
0
−3 x F x Etotal = +0,14 x F + 0,88 x F

0 1,02
Etotal =+ = −0,34 V
−3

LIVRO OLÍMPICO – VOLUME I


74

81
(CYPRUS NATIONAL COMPETITION FOR THE INTERNATIONAL
CHEMISTRY OLYMPIAD)
a) Calcular o decréscimo de massa do eletrodo de zinco, se uma
corrente de 0,15 A, passa através da célula galvânica abaixo, durante 1,5 h:
Zn(s)/Zn2+(aq)//Cu2+(aq)/Cu(s)
b) Calcule o potencial padrão de redução da célula abaixo, a 25°C,
sendo a concentração de Zn2+ igual a 0,1 mol/L e a de Ag+ igual a 0,01 mol/L
(E° = 1,56 V). Zn(s)/Zn2+(aq) //Ag+(aq)/Ag(s)
c) Dar a equação química da reação que ocorre espontaneamente na
célula galvânica que consiste das seguintes meia-células:
MnO2 + 4H+ + 2e- → Mn2+ + 2 H2O E° = 1,21 V
Ag+ + e- → Ag E° = 0,80 V

Resposta:

Item a) Semirreação do zinco: Zn+2 −


(aq) + 2e → Zn(s)

96500 C
65,4 g − − − − − − 2 mol de elétrons x
mol de elétrons
Massa − − − − − − 0,15 x 1,50 x 3600 C

Massa = 0,277 g

Item b) Aplicando a equação de Nernst, para a seguinte equação química


global: Zn(s) + 2 Ag+(aq) → Zn2+(aq) + 2Ag(s)

RxT
E = E0 − x ln Q
nxF
1
+2
8,314 x 298 a⏞Cu x [Zn ]
E = +1,56 − x ln +2 2
2 x 96500 a⏟Zn x [Ag ]
1
1
8,314 x 298 a⏞
Cu x 0,10
E = +1,56 − x ln 2
= +1,47 V
2 x 96500 Zn x (0,01)
a⏟
1

Item c) Invertendo e multiplicando por dois a semirreação da prata.

MnO2(s) + 4 H + + 2e− → Mn+2


(aq) + 2H2 O
2Ag (s) → 2 Ag +
(aq) + 2e −

MnO2(s) + 4H + + 2Ag (s) → Mn+2 +


(aq) + 2H2 O + 2Ag (aq)

0
∆Gtotal = −2 x F x (+1,21) + [−2 x F x (−0,80)]
0
−2 x F x Etotal = −2 x F x (+1,21) + [−2 x F x (−0,80)]
0
Etotal = +0,41 V

LIVRO OLÍMPICO – VOLUME I


75

82
(U. S NATIONAL CHEMISTRY OLYMPIAD) A galvanic cell is based on the
half-reactions;
Cr3+ + 3e– → Cr E° = –0,744 V
Ni2+ + 2e– → Ni E° = –0,236 V
a) Write the balanced equation for the overall cell reaction.
b) State which electrode increases in mass as the cell operates. Explain
your answer.
c) Calculate E°cell.
d) Determine the value of ΔG° for the cell reaction at 25°C.
e) Calculate the value of K for the cell reaction at 25°C.
f) Find the voltage of the cell at 25°C if [Cr3+] and [Ni2+] are both
changed to 0.010 M.

Resposta: Para a determinação da equação eletroquímica global, será


necessário inverter e multiplicar por dois a semirreação do cromo e multiplicar
por três a semirreação do níquel.
+3
2Cr(s) → 6e− + 2Cr(aq)
3Ni+2 −
(aq) + 6e → 3Ni(s)
2Cr(s) + 3Ni+2 +3
(aq) → 2Cr(aq) + 3Ni(s) (equação química global)

0
∆Gtotal = −6 x F x (+0,744) + [−6 x F x (−0,236)]
0
−6 x F x Etotal = −2 x F x (+0,744) + [−6 x F x (−0,236)]
0
Etotal = +0,508 V

Item a) Equação química para o funcionamento da pilha:


2Cr(s) + 3Ni+2 +3
(aq) → 2Cr(aq) + 3Ni(s)

Item b) O eletrodo que vai apresentar a sua massa aumentada trata-se do


eletrodo de níquel.

Item c) Etotal
0
= +0,508 V

Item d) Cálculo da variação da energia livre de Gibbs padrão:


0 0
∆Gtotal = −n x F x Etotal
0
∆Gtotal = −6 x 96500 x (+0,508) = −294132 J (−294,1 kJ)

∆G0total
Item e) Cálculo da constante de equilíbrio químico. ln K eq = −R x T
−294132
ln K eq =
− 8,314 x 298

ln K eq = +118,72

K eq = 𝑒 +118,72

Item f) Para a resolução deste item será necessário aplicar a equação de


Nernst: 2Cr(s) + 3Ni+2 +3
(aq) → 2Cr(aq) + 3Ni(s)

LIVRO OLÍMPICO – VOLUME I


76

RxT
E = E0 − x ln Q
nxF
8,314 x 298 a3Ni x [Cr +3 ]2
E = +0,508 − x ln 2
6 x 96500 aCr x [Ni+2 ]3
1
8,314 x 298 a⏞
3
Ni x (0,010)
2
E = +0,508 − x ln 2 3
6 x 96500 Cr x (0,010)
a⏟
1

83
(INSTITUTO MILITAR DE ENGENHARIA) Dadas as reações de meia célula:
Cu2+ + e- → Cu+ E0 = + 0,153 V
I2 + 2e- → 2I- E0 = + 0,536 V
pede-se:
a) Escrever a equação que representa a reação global da célula.
b) Calcular o potencial de eletrodo global (E0).
c) Calcular a energia livre para a reação (G0), considerando que 1 mol de
elétrons percorreu a célula eletroquímica.

Resposta: Para a determinação da equação eletroquímica global, será


necessário inverter a primeira semirreação e multiplicar por dois.

2Cu+ − +2
(aq) → 2e + 2Cu(aq)
0 −
I2(aq) + 2e− → 2I(aq)
2Cu+ 0 +2 −
(aq) + I2(aq) → 2Cu(aq) + 2I(aq) (equação química global)

0
∆Gtotal = −2 x F x (−0,153) + [−2 x F x (+0,536)]
0
−2 x F x Etotal = −2 x F x (−0,153) + [−2 x F x (+0,536)]
0
Etotal = +0,383 V

Item a) Equação global: 2Cu+ 0 +2 −


(aq) + I2(aq) → 2Cu(aq) + 2I(aq)

0
Item b) Etotal = +0,383 V

Item c) Cálculo da variação da energia livre de gibbs:


0
∆Greação = −n x F x E 0
0
∆Greação = −2 x 96500 x (+0,383)
0
∆Greação = −73919 J

84
(INSTITUTO MILITAR DE ENGENHARIA)
a) Determinar o potencial padrão de uma célula formada por eletrodos de Cu
e Cd, onde os eletrólitos são sais de Cu2+ e Cd2+.
b) Indicar o eletrodo positivo, o anodo e o metal que se reduz.
Dados:
Cu2+(aq) + 2e- → Cu(s) ε° = + 0,337 V
Cd2+(aq) + 2e- → Cd(s) ε° = - 0,403 V

LIVRO OLÍMPICO – VOLUME I


77

Resposta: Para a determinação da equação eletroquímica global, será


necessário inverter a semirreação do cádmio.

2e− + Cu+2 0
(aq) → Cu(S)
Cd0(S) → 2e− + Cd+2
(aq)
Cd0(S) + Cu+2 0 +2
(aq) → Cu(S) + Cd(aq) (equação química global)

0
∆Gtotal = −2 x F x (+0,337) + [−2 x F x (+0,403)]
0
−2 x F x Etotal = −2 x F x (+0,337) + [−2 x F x (+0,403)]
0
Etotal = +0,740 V

Item b) Indicar o eletrodo positivo, o anodo e o metal que se reduz.

Ânodo = cádmio (Cd)


Eletrodo positivo = cobre (Cu)
Metal que apresenta sua massa reduzida = cádmio (Cd)
85
(CONCURSO PARA DOCENTE – IFRJ - MODIFICADA) Amálgama dentário
do mercúrio com uma liga contendo prata, estanho e cobre, além de zinco ou
outros elementos em menores proporções. As utilizações desse tipo de
amálgama na restauração de dentes apresentam duas desvantagens: a cor
e o efeito de dor, causado quando acidentalmente se morde um talher de
alumínio ou um papel confeccionado com alumínio. Assim, responda ao que
se questiona:
a) O exemplo descrito refere-se a uma pilha galvânica ou a uma célula
eletrolítica? Justifique, identificando as reações catódica e anódica, bem
como a reação global, balanceada da pilha.
b) Calcule a força eletromotriz (fem).
c) Determine o valor da Energia Livre de Gibbs (ΔG°), considerando as
seguintes informações:
Al+3(aq) + 3e- → Al(s) ΔE° = - 1,66 V
Hg+2(aq) + 2e- → 2 Hg(amálgama de Ag) ΔE° = + 0,85 V
F = 96485,33 C.mol-1
d) Determine a constante de equilíbrio (Keq).

Resposta:

Item a) O processo descrito refere-se a uma pilha galvânica, pelo fato deste
efeito ser espontâneo.

Invertendo e multiplicando por dois a primeira semirreação:

2 Al(s) → 2 Al+3(aq) + 6e- ∆G° = −6 x F x (+1,66)

Multiplicando a segunda semirreação por três:

3 Hg+2(aq) + 6e- → 3 Hg(amálgama de Ag) ∆G° = −6 x F x (+0,85)

LIVRO OLÍMPICO – VOLUME I


78

Somando as duas semirreações:

2 Al(s) → 2 Al+3(aq) + 6e-


3 Hg+2(aq) + 6e- → 3 Hg(amálgama de Ag) +
2 Al(s) + 3 Hg+2(aq) → 2 Al+3(aq) + 3 Hg(amálgama de Ag)

Semirreação de oxidação: 2 Al(s) → 2 Al+3(aq) (Ânodo)


Semirreação de redução: 3 Hg+2(aq) → 3 Hg (Cátodo)

Item b) Cálculo da força eletromotriz:

0
−6 x F x Etotal = −6 x F x (+ 1,66) + [−6 x F x (+ 0,85)]
0
−6 x F x Etotal = −6 x F x (+ 1,66) + [−6 x F x (+ 0,85)]

0
Etotal = +1,66 + 0,85 = + 2,51 V

Item c) Cálculo da energia livre de Gibbs padrão (ΔG°): ∆G° = −6 x F x E°

96500 C
∆G° = −6 mol de elétrons x x (+2,51)
1 mol de elétrons

∆G° = −1453290 J (−1453,29 kJ)

Item d) Cálculo da constante de equilíbrio (Keq):

∆G° = −6 x F x E° = −R x T x ln K eq

− 1453290 = − 8,314 x (25 + 273) x ln K eq

− 1453290
ln K eq = = 586,58
− 2477,572

K eq = e+586,58

86
(OLIMPÍADA BRASILEIRA DE QUÍMICA) “O Brasil possui um dos mais
eficientes ciclos de reciclagem de alumínio do mundo. De acordo com a
Associação Brasileira do Alumínio (Abal), o índice supera os 35% ante cerca
de 29% da média mundial. Segundo o consultor de Marketing da Alcoa,
Eduardo Lima, vem crescendo ano-a-ano no mundo o uso de alumínio
primário reciclado em relação ao metal primário, passando de 17% em 1960
para 33% em 2004. A estimativa para 2020 é a de que o porcentual pule para
40%.” (fonte: http://sustentar.net/2013/sem-categoria/reciclagem-de-
aluminio-deve-saltar-para-40) A reciclagem do alumínio é de grande
importância ambiental e energética, uma vez que a sua reciclagem
economiza cerca de 95% de energia elétrica.
a) Equacione a reação da eletrólise ígnea do óxido de alumínio. Indique os
produtos obtidos no cátodo e no ânodo da cuba eletrolítica.
b) Determine a massa de alumínio produzida em uma cuba eletrolítica com
corrente constante de 8000 A durante 150 horas.

LIVRO OLÍMPICO – VOLUME I


79

Resposta:

Item a) Al2 O3(s) → 2Al+3 −2


(l) + 3O(l)
Semirreação que ocorre no cátodo: Al+3 − 0
(l) + 3e → Al(s)
1
Semirreação que ocorre no ânodo: O−2
(l) → 2 O2(g) + 2e

Multiplicando a primeira semirreação por 2 e multiplicando a segunda


semirreação por 3.

Semirreação que ocorre no cátodo: 2Al+3 − 0


(l) + 6e → 2Al(s)
3
Semirreação que ocorre no ânodo: 3O−2
(l) → O2(g) + 6e

2

Item b) Somado as semirreações apresentadas:

2Al+3 − 0
(l) + 6e → 2Al(s)
3
3O−2
(l) → 2 O2(g) + 6e

+
−2 3
2Al+3 0
(l) + 3O(l) → 2Al(s) + O2(g)
2

Produto formado no cátodo: alumínio metálico

Produto formado no ânodo: oxigênio gasoso

Item b) Cálculo da massa de alumínio metálico produzido:

96500 C
54 g de de Al(s) − − − − 6 mol de e− x
1 mol de e−
3600 s
mAl(s) − − − − − − − − 8000 x 150 h x
1h
8000 x 150 x 3600 x 54
mAl(s) =
6 x 96500

mAl(s) = 402901,55 g (≅ 403 kg)

87
(INSTITUTO MILITAR DE ENGENHARIA) Deseja-se depositar uma camada
de 0,85 g de níquel metálico no catodo de uma célula eletrolítica, mediante a
passagem de uma corrente elétrica de 5 A através de uma solução aquosa
de nitrato de níquel. Assinale a opção que apresenta o tempo necessário para
esta deposição, em minutos.
a) 4,3
b) 4,7
c) 5,9
d) 9,3
e) 17,0

LIVRO OLÍMPICO – VOLUME I


80

Resposta: Alternativa D.

Reação de dissociação do nitrato de níquel: Ni(NO3 )2(aq) → Ni+2


(aq) +
2 NO−
3(aq)

Semirreação que ocorre no cátodo: Ni+2 −


(aq) + 2 e → Ni(s)

Quantidade de carga (Q):


96500 C
58,70 g − − − − − − 2 mol de elétrons x
mol de elétrons
0,85 g − − − − − − Q

0,85 x 96500 x 2
Q= = 2794,72 C
58,70

Cálculo do tempo:

Q 2794,72 C 1 min
tempo = = = 558,94 s x = 9,31 minutos
i 5,0 A 60 s

88
(GRILLO) Considere as seguintes semirreações de redução para um aparato
que se trata de uma pilha galvânica, apresentando os seguintes dados a
seguir:
Sn+2(aq) (0,01 mol.L-1) + 2e- → Sn(s) E° = - 0,14 V
Cu+2(aq) (0,50 mol.L-1) + 2e- → Cu(s) E° = + 0,52 V
A partir dessas informações apresentadas, calcule E°, E, ∆G°, ∆G e Keq.

Resposta: Cálculo do potencial padrão (E°), invertendo a primeira semirreação


do estanho:

Sn(s) → Sn+2(aq) + 2e- ∆G° = −2 x F x (+0,14)


Cu+2(aq) + 2e- → Cu(s) ∆G° = −2 x F x (+0,52) +
Sn(s) + Cu (aq) → Sn (aq) + Cu(s)
+2 +2

0
−2 x F x Etotal = −2 x F x (+0,14) + [−2 x F x (+0,52)]
0
−2 x F x Etotal = −2 x F x (+0,14) + [−2 x F x (+0,52)]
0
Etotal = +0,14 + 0,52 = + 0,66 V

Cálculo da variação da energia livre de Gibbs padrão (∆G°):

∆G° = −n x F x E°

96500 C
∆G° = −2 mol de elétrons x x (+0,66)
mol de elétrons

∆G° = −127380 J (−127,38 kJ)

Cálculo do potencial padrão fora das condições padrões (E), a partir da


equação de Nernst:

RxT
E = E° − x lnQ
nxF

LIVRO OLÍMPICO – VOLUME I


81

RxT [Sn+2
(aq) ]
E = E° − x ln
nxF [Cu+1
(aq) ]

8,314 x (25 + 273) 0,01


E = + 0,66 − x ln
2 x 96500 0,50

⏟ 0,20
− 3,91
E = + 0,71 V

Cálculo da variação da energia livre de Gibbs (∆G): ∆G = ∆G° + R x T x lnQ

[Sn+2
(aq) ]
∆G = ∆G° + R x T x ln 2
[Cu+1
(aq) ]
0,01
∆G = −127380 + 8,314 x (25 + 273) x ln
0,50

⏟ 0,20
− 3,91
J kJ
∆G = − 137067,31 (−137,07 )
mol mol

89
(GRILLO) Observe os valores para a ddp padrão (Eº) das seguintes
semirreações:
Ag+(aq) + 1e- → Ag(s) E° = +0,800 V
AgBr(s) + 1e → Ag(s) + Br (aq)
- - E° = +0,0713 V
Descreva como deve ser usado estes valores para a determinação do Kps.

Resposta: Invertendo a primeira semirreação.

Ag(s) → Ag+(aq) + 1e- ∆G° = −1 x F x (−0,800)


AgBr(s) + 1e- → Ag(s) + Br-(aq) ∆G° = −1 x F x (+0,0713) +
AgBr(s) → Ag+(aq) + Br-(aq)
0
−1 x F x Etotal = −1 x F x (−0,800) + [−1 x F x (+0,0713)]
0
−1 x F x Etotal = −1 x F x (−0,800) + [−1 x F x (+0,0713)]
0
Etotal = −0,7287 V

Cálculo do valor do produto de solubilidade:

∆G° = −1 x F x E° = −R x T x ln K ps

− 1 x 96500 x (−0,7287) = − 8,314 x (25 + 273) x ln K ps

K ps = e−28,38

K ps = 4,72 x 10−13

LIVRO OLÍMPICO – VOLUME I


82

90
(GRILLO) Uma solução de cloreto de alumínio apresenta coeficiente de
atividade médio igual a 0,96, pela lei de Debye Huckell. A partir desta
informação, calcule a massa de cloreto de alumínio a partir de 800 gramas de
água pura e também o valor das atividades dos íons alumínio e cloreto.

Resposta: AlCl3(aq) → Al+3 −


(aq) + 3 Cl(aq)

Cálculo da força iônica (I), a partir da utilização da lei de Debye-Huckell:

logγ±
x = −|z1 x z2 | x A x √I

log (0,96) = −|3 x (−1)| x 0,509 x √I

−1,16 x 10−2 = −3 x 0,509 x √I

√I = 1,16 x 10−2
2
(√I) = (1,16 x 10−2 )2

I = 1,35 x 10−4

Relação estequiométrica com a moladidade através da reação de dissociação


do cloreto de alumínio: AlCl3(aq) → Al+3 −
(aq) + 3 Cl(aq)

Chamando de W a concentração molal da solução, temos:

1 mol de AlCl3(aq) − −1 mol de Al+3 −


(aq) − − − 3 mol de Cl(aq)
W mol de AlCl3(aq) − − W mol de Al+3 −
(aq) − − 3W mol de Cl(aq)

Cálculo da massa de cloreto de alumínio, a partir da força iônica:

1
I= x {zi2 x WAl+3 + zi2 x 3 x WCl− }
2
1
I= x {(3)2 x W + (−1)² x 3W} = 6W
2
mAlCl3
1,35 x 10−4 = 6 x (kg)
<MM>AlCl3 x msolvente

mAlCl
1,35 x 10−4 = 6 x 133,5 x 0,800
3

1,35 x 10−4 x 133,5 x 0,800


mAlCl3 = = 2,40 𝑥 10−3 𝑔
6

Cálculo da molalidade (W):


mAlCl3 2,40 x 10−3
< MM >AlCl3 133,5 𝑚𝑜𝑙
W= (kg)
= = 2,25 x 10−5
msolvente 0,800 𝑘𝑔

LIVRO OLÍMPICO – VOLUME I


83

Analisando o íon (cátion) alumínio:

aAl+3 = γ± +3
x x [Al ] = 0,96 x 2,25 x 10
−5
= 2,16 x 10−5

Analisando o íon (ânion) cloreto: 𝑎𝐶𝑙− = γ± −


x 𝑥 [𝐶𝑙 ]

aCl− = 0,96 x 3 x 2,25 x 10−4 = 6,48 x 10−4

Observação: A equação apresentada pela questão trata-se da lei de Debye-


Huckell. Trata-se de uma lei em que é possível calcular as atividades iônicas
das espécies químicas, válidas para soluções aquosas a 25ºC e muito
diluídas. Trata-se de uma lei que confronta as previsões teóricas com os
dados experimentais. Os valores apresentados e calculados para estas duas
situações são bastante próximas e sua concordância, a lei, é simplesmente
excelente.

Importante ressaltar que este ponto bem específico da química, mas


especificamente da físico-química é bastante cobrado e abordado na IChO,
referente aos capítulos relacionado à soluções e eletroquímica.

91
(ITT-JEE) Calculate Eº for Ag2S + 2e- → 2 Ag(s) + S-2 if Kps of Ag2S = 6 x 10-
50 and Eº
Ag+/Ag = +0,80 V.

Resposta: Cálculo do valor do potencial padrão a partir da constante do


produto de solubilidade.

Ag2S(s) ⇄ 2 Ag+(aq) + S-2(aq)

∆G° = −2 x F x E° = −R x T x ln K ps

− 2 x 96500 x E° = − 8,314 x (25 + 273) x ln(6,0 𝑥 10−50 )

280801,80
E° = = −1,45 V
−193000

Informação do problema: Ag+(aq) + 1e- → Ag(s) Eº = +0,80 V

Para a determinação do potencial será necessário multiplicar a semirreação


da prata por dois. Diante desta afirmativa, temos:

Ag2S(s) → 2 Ag+(aq) + S-2(aq) ∆G° = −2 x F x (−1,45)


2 Ag+(aq) + 2e- → 2 Ag(s) ∆G° = −2 x F x (+0,80)
Ag2S(s) + 2e- → 2 Ag(s) + S-2(aq)

0
−2 x F x Etotal = −2 x F x (−1,45) + [−2 x F x (+0,80)]
0
−2 x F x Etotal = −2 x F x (−1,45) + [−2 x F x (+0,80)]
0
Etotal = −1,45 + 0,80 = − 0,65 V

LIVRO OLÍMPICO – VOLUME I


84

92
(U. S NATIONAL CHEMISTRY OLYMPIAD) What is the oxidation number of
Re in Mg(ReO4)2?
a) + 4
b) + 5
c) + 6
d) + 7

Resposta: Alternativa D.

Sabendo que o número de oxidação do magnésio é igual a (+2) e do oxigênio


igual a (-2), temos:

+2 + 2x − 16 = 0
x = +7
93
(U. S NATIONAL CHEMISTRY OLYMPIAD) What is coeficiente of I2(s) when
the reaction below is balanced with smallest whole number coefficients?
a) 2
b) 3
c) 4
d) 6

Cr2 O−2 − + +3
7(aq) + I(aq) + H(aq) → I2(s) + Cr(aq) + H2 O(l)

Resposta: Alternativa D.

Para esta equação química o dicromato é o agente oxidante enquanto o


iodeto é o agente redutor. Diante disso, a equação balanceada fica da
seguinte maneira:

2 Cr2 O−2 − + +3
7(aq) + 12 I(aq) + 28 H(aq) → 6 I2(s) + 4 Cr(aq) + 14 H2 O(l)

94
(QUALITYING EXAMINATIONS FOR APPLICATIONS FOR JAPANESE
GOVERNMENT SCHOLARSHIPS) Under acidic conditions, manganese
peroxide reacys with hydrogen peroxid as shown below.
MnO− + +2
4(aq) + H2 O2 + H(aq) → Mn(aq) + O2(s) + H2 O(l)

Resposta: Para esta equação química o íon permanganato é o agente


oxidante enquanto a água oxigenada é o agente redutor. Diante disso, a
equação balanceada fica da seguinte maneira:

2MnO− + +2
4(aq) + 5H2 O2 + 6H(aq) → 2Mn(aq) + O2(s) + 8H2 O(l)

95
(GRILLO) Considere a equação iônica não – balanceada apresentada a
seguir: Fe+2 −2 + +3 +3
(aq) + Cr2 O7(aq) + H(aq) → Fe(aq) + Cr(aq) + H2 O(l)
a) Que elemento se oxida?
b) Que elemento se reduz?
c) Qual é o agente oxidante?

LIVRO OLÍMPICO – VOLUME I


85

d) Qual é o agente redutor?


e) Qual é o valor do somatório dos coeficientes estequiométrico?

Resposta:

Item a) ferro
Item b) cromo
Item c) dicromato
Item d) A equação iônica balanceada fica da seguinte maneira:

6Fe+2 −2 + +3 +3
(aq) + Cr2 O7(aq) + 14H(aq) → 6Fe(aq) + 2Cr(aq) + 7H2 O(l)

Soma dos coeficientes estequiométricos: 6 + 1 + 14 + 6 + 2 + 7 = 36


96
(INSTITUTO TECNOLÓGICO DA AERONÁUTICA) Considere as reações
envolvendo o sulfeto de hidrogênio representadas pelas equações seguintes:
I) 2 H2S(g) + H2SO3(aq) → 3 S(s) + 3 H2O(l)
II) H2S(g) + 2 H+(aq) + SO4-2(aq) → SO2(g) + S(s) + 2 H2O(l)
III) H2S(g) + Pb(s) → PbS(s) + H2(g)
IV) 2 H2S(g) + 4 Ag(s) + O2(g) → Ag2S(s) + 2 H2O(l)
Nas reações representadas pelas equações acima, o sulfeto de hidrogênio é
agente redutor em:
a) Apenas I
b) Apenas I e II
c) Apenas III
d) Apenas III e IV
e) Apenas IV

Resposta: Alternativa B.

Analisando cada equação química:

Equação química I) S-2 → S0 + 2 elétrons; para este caso a reação é de


oxidação e o sulfeto de hidrogênio é o agente redutor.

Equação química II) S-2 → S+4 + 6 elétrons; para este caso a reação é de
oxidação e o sulfeto de hidrogênio é o agente redutor.

Equação química III) S-2 → S-2; não se tratando de uma reação de


oxirredução.

Equação química IV) S-2 → S-2; não se tratando de uma reação de


oxirredução.

LIVRO OLÍMPICO – VOLUME I


86

97
(ITT-JEE) Nas CNTP, 1,12 litros de hidrogênio é obtido a partir de uma vazão
em que a corrente é igual a 965 C em uma solução. Calcule o valor da
corrente.
+
Resposta: Semirreação do hidrogênio. 2H(aq) + 2𝑒 − → H2

Quantidade de carga (Q):


96500 C
22,4 L − − − − − − 2 mol de elétrons x
mol de elétrons
1,12 L − − − − − − Q

Q = 9650 C
Q 9650
Cálculo da corrente: i = tempo = 965 A = 10 A

98
(ITT-JEE) Eºcell do Zn/Zn+2 // Cu+2/Cu é 1,10 V a 25ºC. Calcule a constante de
equilíbrio para esta reação: Zn + Cu+2 +2
(aq) → Cu + Zn(aq) .

0 0
Resposta: ∆Geletroquímico = ∆Gtermodinâmico
0
−n x F x Etotal = −R x T x ln K eq
0
−n x F x Etotal
ln K eq =
−R x T
−2 x 96500 x 1,10
ln K eq =
−8,314 x 298
ln K eq = +85,69

K eq = e(+85,69)

99
(ITT-JEE) Calcule qual deve ser a massa de prata depositada através de uma
carga de 965 C de eletricidade, a partir de uma solução de nitrato de prata.

Resposta: Semirreação que ocorre no cátodo: Ag + −


(aq) + 1e → Ag (s)

96500 C
108 g − − − − − 1 mol de elétrons x
mol de elétrons
Massa − − − − − 965 C

108 𝑥 965
Massa = = 1,08 𝑔
96500

LIVRO OLÍMPICO – VOLUME I


87

100
(ITT-JEE) Calcule o número de oxidação do enxofre no Na2S4O6.

Resposta: Sabendo que o número de oxidação do sódio é igual a (+1) e do


oxigênio igual a (-2), temos:

+(2 x 1) + 4x − 12 = 0
x = + 2,50
101
(ITT-JEE) Calcule o número de oxidação do ferro no K4[Fe(CN)6.

Resposta: Sabendo que o número de oxidação do potássio é igual a (+1) e


do cianeto igual a menos um (-1), temos: +(4 x 1) + x − 1x 6 = 0
x=+2
102
(U. S. NATIONAL CHEMISTRY OLYMPIAD) 2 Cr(s) + 3 Cu2+(aq) → 2 Cr3+(aq)
+ 3 Cu(s). Which expression gives the value for ∆G˚ in kJ.mol–1 for this reaction
at 25°C?
a) −6 x 8,31 x 0,43 x 1000
−6 x 96500 x 0,43 x 1000
b) 8,31
−6 x 96500 x 0,43
c)
1000
−6 x 8,31 x 0,43
d) 1000

Resposta: Alternativa C.

A expressão que representa a variação da energia livre de Gibbs padrão em relação


ao processo eletroquímico é dado pela seguinte equação matemática:

J
∆G° = − n x F x E° = −6 x 96500 x 0,43 ( )
mol
−6 x 96500 x 0,43 kJ
Dividindo a expressão por 1000, temos: ∆G° = ( )
1000 mol

103
(U. S. NATIONAL CHEMISTRY OLYMPIAD) 2 Cr(s) + 3 Cu2+(aq) → 2 Cr3+(aq)
+ 3 Cu(s). What is the voltage for this cell when [Cu2+] = 1,0 mol.L-1 and [Cr3+]
= 0,010 mol.L-1?
a) 1,2
b) 0,87
c) 0,47
d) 0,39

LIVRO OLÍMPICO – VOLUME I


88

Resposta: Alternativa C.

Aplicando a equação de Nernst, temos:


3
[Cr +3 ]2 x 𝑎𝐶𝑢
RxT RxT
E = E° − x lnQ = E° − x ln { 2 }
nxF nxF 𝑎𝐶𝑟 x [Cu+2 ]3

Onde as atividades dos sólidos são unitários, aCr = aCu = 1.

RxT [Cr +3 ]2 x a3Cu


E = E° − x ln { 2 }
nxF aCr x [Cu+2 ]3

8,314 x (25 + 273) (0,010)2 x (1)3


E = + 0,43 − x ln { }
6 x 96500 (1)2 x (1)3

8,314 x 298
E = + 0,43 − x ln(10−4 )
6 x 96500

E = + 0,43 + 0,039 = +0,469 V

104
(GRILLO) As reações envolvidas no processo de oxirredução e seus
respectivos potenciais padrões de redução são os seguintes:
PbSO4(s) + 2e- → Pb(s) + SO42-(aq) Eo = - 0,36 V
PbO2(s) + 4 H+(aq) + SO42-(aq) + 2e- → PbSO4(s) + 2 H2O(l) Eo = + 1,64 V
a) Escreva a reação de oxirredução global da pilha.
b) Calcule o valor da variação da energia livre de Gibbs padrão, Go.
Esse resultado é esperado para uma pilha? Por quê?
c) Calcule G, a 25oC quando o pH é 1 e a concentração de sulfato,
[SO4 ] é de 5,0 x 10-3 mol.L-1.
2-

Resposta:

Item a) Invertendo a primeira semirreação, temos:


Pb(s) + SO42-(aq) → Pb SO4(s) + 2e- E° = + 0,36 V
PbO2(s) + 4H+(aq) + SO42-(aq) + 2e- → PbSO4(s)+ 2 H2O(l) E° = + 1,64 V
Pb(s) + PbO2(s) + 2 SO42-(aq) + 4H+(aq) → 2 PbSO4(s)+ 2 H2O(l)
E° = + 0,36 + 1,64 = + 2,00 V

Item b) Cálculo da variação de energia livre de Gibbs padrão, ΔG°:

∆G° = −n x F x E°
C
∆G° = −2 mol x 96500 x (+2,00 V)
mol
∆G° = −386000 J

Sim, o resultado é esperado, uma vez que caracteriza a espontaneidade da


pilha apresentando um valor negativo (ΔG° < 0 e E° > 0), o que se trata de
uma das principais características da pilha.

Item c) Cálculo da concentração da quantidade de matéria de [H+], a partir do


pH: pH = - log[H+]
1 = - log[H+]

LIVRO OLÍMPICO – VOLUME I


89

[H+] = 10-1 mol.L-1

Utilizando a equação termodinâmica que relaciona a energia livres de Gibbs,


temos: ∆G = ∆G° + RTlnQ

a2PbSO4 x a2H2 O
∆G = ∆G° + RTln { }
aPb x aPbO2 x [SO4 −2 ]² x [H + ]4

∆G
(1)² x (1)²
= −386000 + 8,314 x 298 x ln { }
1 x 1 x (5,0 x 10−3 )2 x (10−1 )4

1
∆G = −386000 + 8,314 x 298 x ln { }
25,0 x 10−10

∆G = −336926,79 J (−336,93 kJ)

105
(OLIMPÍADA BRASILEIRA DE QUÍMICA) Ouro metálico dissolve em água
régia, uma mistura de ácido clorídrico e ácido nítrico concentrados e, na
química do ouro, as seguintes reações são importantes:
Au+3(aq) + 3e- → Au(s) Eºredução = +1,498 V
AuCl4-(aq) + 3e- → Au(s) + 4Cl-(aq) Eºredução = +1,002 V
Utilizando as semirreações acima e a semirreação:
NO3-(aq) + 4H+(aq) + 3e- → NO(g) + 2 H2O(l) Eºredução = +0,96 V
Responda às questões (a), (b) e (c).
a) Dê a equação calibrada da reação entre o ouro e o ácido nítrico, para
formar Au+3 e NO(g) e calcule a fem-padrão (Eº) associada a esta reação. Esta
reação é espontânea?
b) Dê a equação da reação entre o ouro e o ácido clorídrico, formando
AuCl4-(aq) e H2(g) e calcule a fem-padrão (Eº) associada a esta reação. Esta
reação, em condições-padrão, é espontânea?
c) Dê a reação entre o ouro e a água régia para dar AuCl4-(aq) e NO(g) e
calcule a fem-padrão (Eº) associada a esta reação. Esta reação, em
condições-padrão, é espontânea?

Resposta:

Item a) Invertendo a semirreação do ouro:

Au(s) → Au+3(aq) + 3e- ∆G0 = −3 x F x (−1,498)


NO3 (aq) + 4H (aq) + 3e → NO(g) + 2 H2O(l) ∆G0 = −3 x F x (+0,96)
- + -

Au(s) + NO3-(aq) + 4H+(aq) → Au+3(aq) + NO(g) + 2 H2O(l)


0
−3 x F x Etotal = −3 x F x (−1,498) + [−3 x F x (+0,96)]
0
−3 x F x Etotal = −3 x F x (−1,498) + [−3 x F x (+0,96)]
0
Etotal = −1,498 + 0,96 = − 0,538 V

Para um potencial negativo, o processo não é espontâneo.

LIVRO OLÍMPICO – VOLUME I


90

Item b)
Invertendo a semirreação e multiplicando por dois:

2Au(s) + 2Cl-(aq) → 2AuCl4-(aq) + 6e- ∆G0 = −6 x F x (+1,002)

Multiplicando a semirreação de redução do hidrogênio por três:

6H+(aq) + 6e- → 3H2(g) ∆G0 = −6 x F x (+0,00)

2Au(s) + 2Cl-(aq) → 2AuCl4-(aq) + 6e- ∆G0 = −6 x F x (+1,002)


6H+(aq) + 6e- → 3H2(g) ∆G0 = −6 x F x (+0,00) +
2Au(s) + 2Cl-(aq) + 6H+(aq) → 3H2(g) + 2AuCl4-(aq)

0
−6 x F x Etotal = −6 x F x (+1,002) + [−6 x F x (+0,00)]
0
−6 x F x Etotal = −6 x F x (+1,002) + [−6 x F x (+0,00)]
0
Etotal = +1,002 + 0,00 = +1,002 V

Para um potencial padrão positivo, o processo é espontâneo.


Item c) Invertendo a sem
Au(s) + 4Cl-(aq) → AuCl4-(aq) + 3e- ∆G0 = −3 x F x (−1,002)
NO3-(aq) + 4H+(aq) + 3e- → NO(g) + 2 H2O(l) ∆G0 = −3 x F x (+ 0,96)
Au(s) + 4Cl-(aq) + NO3-(aq) + 4H+(aq) + 3e- → AuCl4-(aq) + NO(g) + 2 H2O(l)
0
−3 x F x Etotal = −3 x F x (−1,002) + [−3 x F x (+0,96)]
0
−3 x F x Etotal = −3 x F x (−1,002) + [−3 x F x (+0,96)]
0
Etotal = −0,042 V

Para um potencial negativo, o processo não é espontâneo.


106
(GRILLO) Calcule as atividades dos cátions e ânions de uma solução aquosa
de cloreto de cálcio, que apresenta molalidade igual a 3 x 10-4 mol.kg-1
obedecendo à lei de Debye-Huckel.

Resposta: Ionização do cloreto de cálcio: CaCl2(aq) → Ca+2 −


(aq) + 2 Cl(aq)

1 mol de CaCl2(aq) → 1 mol de Ca+2 −


(aq) + 2 mol de Cl(aq)
mol mol mol
3,0 x 10−4 − − − 3,0 x 10−4 − − 6,0 x 10−4
kg kg kg
1
Cálculo da força iônica (I): I = 2 x {zi2 x WCa+2 + zi2 x 3 x WCl− }

1
I= x {(+2)2 x 3,0 𝑥 10−4 + (−1)² x 6,0 𝑥 10−4 }
2
1
I= x {12,0 𝑥 10−4 + 6,0 𝑥 10−4 } = 9,0 𝑥 10−4
2

LIVRO OLÍMPICO – VOLUME I


91

Cálculo do coeficiente médio de atividade, a partir da utilização da lei de


Debye-Huckell: logƔ± = −|z1 x z2 | x A x √I

logƔ± = −|2 x (−1)| x 0,509 x √9,0 𝑥 10−4

logƔ± = −2 x 0,509 x 3,0 𝑥 10−2

logƔ± = −3,054 x 10−2


−2 )
Ɣ± = 10(−3,054 x 10 = 0,932

Cálculo das atividades dos íons:

Analisando o íon (cátion) cálcio:

aCa+2 = 0,932 x 3,0 x 10−4 = 2,80 x 10−4

Analisando o íon (ânion) cloreto:

aCl− = 0,932 x 6,0 x 10−4 = 5,59 x 10−4

107
(ITT-JEE) (a) When a current of 150 mA is used for 8,0 hr, what volume of
fluorine gas at N.T.P can be produced from a molten mixture of potassium and
hydrogen fluoride?
(b) If the same current is passed for same intervalo f time, how many litre of
oxygen gas could be produced from electrolysis of water?

Resposta:

Item a) Equação iônica: F2(g) + 2𝑒 − → 2F(aq)


Cálculo da carga: Q = i x t

Q = 150 x 10−3 x 8 x 3600 = 4320 C


96500 𝐶
22,4 L − − − − − − 2 mol de elétrons x
1 𝑚𝑜𝑙 𝑑𝑒 𝑒𝑙é𝑡𝑟𝑜𝑛𝑠
Volume − − − − − 4320 C
Volume = 0,501 L

+ 1
Item b) Eletrólise da água: H2 O(l) → 2H(aq) + 2 O2(g) + 2e−

1 96500 C
x 22,4 L − − − − − − 2 mol de elétrons x
2 1 mol de elétrons
Volume − − − − − − − 150 x 10−3 x 8 x 3600
48384
Volume = = 0,251 L
193000

LIVRO OLÍMPICO – VOLUME I


92

108
(ITT-JEE) Na aqueous solution of Na2SO4 was electrolysed for 30 min; 25 mL
of O2 was produced at the anode over water at 22ºC at a total pressure of 722
torr. Determine the strenght of current that was used to produce the oxygen
gas. Vapour pressure of water at 22ºC is 19,83 torr.

Resposta: Analisando a eletrólise aquosa do sulfato de sódio.

Reação de ionização do sulfato de sódio:



−2
⏞ 2Na+
Na2 SO4 (aq) → (aq) + SO4 (𝑎𝑞)

+ −
Reação de ionização da água: H2 O(l) → H(aq) + OH(aq)

Quem descarrega com melhor facilidade no cátodo, Na+ +


(aq) ou H(aq) ?
+
2H(aq) + 2e− → H2(g)

Quem descarrega com melhor facilidade no ânodo, SO4 −2 −


(𝑎𝑞) ou OH(aq) ?
− 1
2OH(aq) → H2 O(l) + O2(g) + 2e−
2
Multiplicando a segunda semirreação por dois e somando todas as reações, temos:

−2
Na2 SO4 (aq) → ⏞ 2Na+ (aq) + SO4 (𝑎𝑞)
+ −
2H2 O(l) → 2H(aq) + 2OH(aq)
+
2H(aq) + 2e− → H2(g)
− 1
2OH(aq) → H2 O(l) + O2(g) + 2e− +
2
Na2 SO4 (aq) + 2 H2 O(l)
+ −2 + 1
→ 2 Na
⏟ + SO4 + H2 + 2 H + O
(aq) (aq) (l) (aq)
2 2(g)
Na2 SO4 (aq)

Realmente, na eletrólise aquosa do sulfato de sódio, há a produção ½ mol de


oxigênio molecular.
vapor
Cálculo da pressão de oxigênio puro: POseco
2
= Patm − PH2 O

POseco
2
= 722 torr − 19,83 torr = 702,17 torr
702,17
x 25 x 10−3
Cálculo do número de mol de oxigênio molecular: nO2 = 760
0,08206 x 295

nO2 = 9,54 x 10−4 mol

Cálculo da corrente utilizada:

1 96500 C
x 22,4 L − − − − − − 2 mol de elétrons x
2 1 mol de elétrons
9,54 x 10−4 mol − − − − Q
Q = 0,205 A

LIVRO OLÍMPICO – VOLUME I


93

109
(GRILLO) Considere os seguintes potenciais padrões à 25ºC:
Pd+2(aq) + 2e- → Pd(s) E 0 = +0,83 V
PdCl4-2(aq) + 2e- → Pd(s) + 4Cl- E 0 = +0,64 V
Item a) Calcule a constante de equilíbrio para a seguinte reação
eletroquímica: Pd+2(aq) + 4Cl- → PdCl4-2(aq).
Item b) Calcule a variação da energia livre de Gibbs padrão.
δE V
Item c) Sabendo que (δT) = +2,30 x 10−4 𝐾, calcule o valor da
p
variação da entalpia padrão.
Item d) Calcule a entropia padrão.

Resposta: Invertendo a segunda semirreação eletroquímica, temos:

Pd(s) + 4Cl- → PdCl4-2(aq) + 2e- ∆G0 = −2 x F x (−0,64)


Pd+2(aq) + 2e- → Pd(s) ∆G0 = −2 x F x (+0,83)
0 0
Pd+2(aq) + 4Cl- → PdCl4-2(aq) ∆Gtotal = −2 x F x Etotal

0
−2 x F x Etotal = −2 x F x (−0,64) + [−2 x F x (+0,83)]
0
−2 x F x Etotal = −2 x F x (−0,64) + [−2 x F x (+0,83)]
0
Etotal = +0,19 V

Item b) Cálculo da variação da energia livre de Gibbs padrão:


∆G° = −n x F x E°
C
∆G° = −2 mol x 96500 x (+0,19 V)
mol
∆G° = −36670 J

Item a) Cálculo da Keq: ∆G° = −36670 J = −R x T x ln K eq

−36670 J = −8,314 x 298 x ln K eq


−36670
ln K eq =
− 2477,572
−36670
ln K eq =
− 2477,572
ln K eq = 14,80

K eq = e(+14,80)

Item c) Cálculo da variação da entalpia padrão:

o
δE
∆Hreação = −n x F x [E 0 − ( ) ]
δT p

o
∆Hreação = −2 x 96500 x [0,19 − (+ 2,30 x 10−4 )]

o
J
∆Hreação = −23441,78 (processo exotérmico)
mol

LIVRO OLÍMPICO – VOLUME I


94

Item d) Cálculo da entropia:


δE ∆S0
Primeira maneira de cálculo: ( ) =
δT p nxF

∆S 0
+ 2,30 x 10−4 =
2 x 96500
J
∆S 0 = + 44,39
K
o o o
Segunda maneira de cálculo: ∆Greação = ∆Hreação − 𝑇 𝑥 ∆Sreação

o
−36670 + 23441,78
∆Sreação =
− 298
J
∆S 0 = + 44,39
K
110
(ITT-JEE) The standard electromotive force of the cell: Fe | Fe2+(aq) || Cd2+ | Cd
is 0,0372 V The temperature coefficient of emf is –0,125 V.K–1. Calculate the
quantities ΔG°, ΔH° and ΔS° at 25°C.
Resposta: Para esta reação eletroquímica, há a participação de dois (2)
elétrons.
Cálculo da variação da energia livre de Gibbs padrão:
∆G° = −n x F x E°
C
∆G° = −2 mol x 96500 x (+0,0372 V)
mol
∆G° = −7179,60 J

∆So
reação δE
Cálculo da variação da entropia padrão: nxF
= (δT)
p

o
∆Sreação
= −0,125
2 x 96500
o
J
∆Sreação = −24125
K
A partir da equação de Gibbs − Helmholtz, temos:
o o o
∆Greação = ∆Hreação − T x ∆Sreação
o
−7179,60 = ∆Hreação − T x (−24125)
o
∆Hreação = −7196429,6 J (−7196,43 kJ)

LIVRO OLÍMPICO – VOLUME I


95

CAPÍTULO 02

CINÉTICA QUÍMICA
- EXERCÍCIOS

ALEXANDRE
VARGAS GRILLO

LIVRO OLÍMPICO – VOLUME I


96

SUMÁRIO
SUMÁRIO - CAPÍTULO 03 – CINÉTICA QUÍMICA – PÁGINA
1. OLIMPÍADA BRASILEIRA DE QUÍMICA
2. OLIMPÍADA BRASILEIRA DE QUÍMICA
3. OLIMPÍADA BRASILEIRA DE QUÍMICA
4. OLIMPÍADA BRASILEIRA DE QUÍMICA
5. OLIMPÍADA MUNDIAL DE QUÍMICA
6. OLIMPÍADA BRASILEIRA DE QUÍMICA
7. OLIMPÍADA BRASILEIRA DE QUÍMICA
8. OLIMPÍADA BRASILEIRA DE QUÍMICA
9. OLIMPÍADA BRASILEIRA DE QUÍMICA
10. OLIMPÍADA BRASILEIRA DE QUÍMICA
11. OLIMPÍADA BRASILEIRA DE QUÍMICA
12. OLIMPÍADA BRASILEIRA DE QUÍMICA
13. OLIMPÍADA BRASILEIRA DE QUÍMICA
14. (GRILLO) QUESTÃO DE PRÓPRIA AUTORIA
15. OLIMPÍADA BRASILEIRA DE QUÍMICA
16. 22TH ICHO, INTERNATIONAL CHEMISTRY OLYMPIAD
17. 23TH ICHO, INTERNATIONAL CHEMISTRY OLYMPIAD
18. 25TH ICHO, INTERNATIONAL CHEMISTRY OLYMPIAD
19. OLIMPÍADA CEARENSE DO ENSINO SUPERIOR DE QUÍMICA
20. OLIMPÍADA CEARENSE DO ENSINO SUPERIOR DE QUÍMICA
21. OLIMPÍADA CEARENSE DO ENSINO SUPERIOR DE QUÍMICA
22. CONCURSO PARA DOCENTE – IFRJ
23. CONCURSO PARA DOCENTE - IFRJ
24. CONCURSO PARA DOCENTE - IFRJ
25. CONCURSO PARA DOCENTE – IFRJ
26. 27TH ICHO, INTERNATIONAL CHEMISTRY OLYMPIAD
27. INSTITUTO MILITAR DE ENGENHARIA
28. PROGRAMA OLIMPÍADA URUGUAYA DE QUÍMICA
29. PROGRAMA OLIMPÍADA URUGUAYA DE QUÍMICA
30. PROGRAMA OLIMPÍADA URUGUAYA DE QUÍMICA
31. INSTITUTO TECNOLÓGICO DA AERONÁUTICA
32. U.S. NATIONAL CHEMISTRY OLYMPIAD
33. U.S. NATIONAL CHEMISTRY OLYMPIAD – SEGUNDA FASE
34. CONCURSO PARA DOCENTE - IFRJ – MODIFICADA
35. OLIMPÍADA BRASILEIRA DE QUÍMICA
36. U.S. NATIONAL CHEMISTRY OLYMPIAD
37. EXAME NACIONAL DE CURSOS – ENGENHARIA QUÍMICA
38. EXAME NACIONAL DE CURSOS – ENGENHARIA QUÍMICA
39. INSTITUTO MILITAR DE ENGENHARIA
40. (GRILLO) QUESTÃO DE PRÓPRIA AUTORIA
41. (GRILLO) QUESTÃO DE PRÓPRIA AUTORIA
42. INSTITUTO MILITAR DE ENGENHARIA
43. OLIMPÍADA BRASILEIRA DE QUÍMICA
44. INSTITUTO MILITAR DE ENGENHARIA
45. INSTITUTO TECNOLÓGICO DA AERONÁUTICA
46. 37TH ICHO, INTERNATIONAL CHEMISTRY OLYMPIAD
47. 40TH ICHO, INTERNATIONAL CHEMISTRY OLYMPIAD
48. 40TH ICHO, INTERNATIONAL CHEMISTRY OLYMPIAD
49. 39TH ICHO, INTERNATIONAL CHEMISTRY OLYMPIAD
50. 36TH ICHO, INTERNATIONAL CHEMISTRY OLYMPIAD
51. OLIMPÍADA BRASILEIRA DE QUÍMICA

LIVRO OLÍMPICO – VOLUME I


97

52. OLIMPÍADA NORTE-NORDESTE DE QUÍMICA


53. U.S. NATIONAL CHEMISTRY OLYMPIAD
54. U.S. NATIONAL CHEMISTRY OLYMPIAD
55. OLIMPÍADA BRASILEIRA DE QUÍMICA
56. OLIMPÍADA PORTUGUESA DE QUÍMICA
57. U.S. NATIONAL CHEMISTRY OLYMPIAD
58. INSTITUTO MILITAR DE ENGENHARIA
59. OLIMPÍADA ARGENTINA DE QUÍMICA
60. U.S. NATIONAL CHEMISTRY OLYMPIAD
61. U. S. NATIONAL CHEMISTRY OLYMPIAD
62. PROGRAMA OLIMPÍADA URUGUAYA DE QUÍMICA
63. OLIMPÍADA PORTUGUESA DE QUÍMICA
64. OLIMPÍADA PORTUGUESA DE QUÍMICA
65. OLIMPÍADA PERUANA DE QUÍMICA
66. OLIMPÍADA PERUANA DE QUÍMICA
67. INSTITUTO MILITAR DE ENGENHARIA
68. ITT-JEE
69. OLIMPÍADA NORTE–NORDESTE DE QUÍMICA
70. (GRILLO) QUESTÃO DE PRÓPRIA AUTORIA
71. OLIMPÍADA PIAUIENSE DE QUÍMICA
72. ITT-JEE
73. ITT-JEE
74. ITT-JEE
75. ITT-JEE

LIVRO OLÍMPICO – VOLUME I


98

1
(OLIMPÍADA BRASILEIRA DE QUÍMICA - MODIFICADA) Moléculas de
butadieno podem acoplar para formar C8H12. A expressão da velocidade para
esta reação é v = k.[C4H6]² e a constante de velocidade estimada é 0,014
L.mol-1.s-1. Se a concentração inicial de butadieno é 0,016 mol.L-1, determine
o tempo necessário para que a concentração decaia para 0,0016 mol.L-1.

Resposta: Para a resolução desta questão será necessário primeiramente


saber qual a cinética que deve ser estudada. Observando a unidade da
velocidade específica (L x mol-1 x s-1), trata-se da cinética de segunda ordem.
A partir disso é possível calcula o tempo necessário para o decaimento.

1 1
= +kxt
[C4 H6 ]Final [C4 H6 ]Inicial

1 1
= + 0,014 x t
0,0016 0,016

t = 40178,57 s

2
(OLIMPÍADA BRASILEIRA DE QUÍMICA) Observa-se que uma reação de
decomposição de primeira ordem tem as seguintes constantes de velocidade
nas temperaturas mencionadas:
T (°C) k (10-3 s-1)
0 2,46
20,0 25,1
40,0 576
Estime a energia de ativação.

Resposta: Refazendo a tabela fornecida pelo problema:

T (°C) T (K) k (s-1) 1/T Ln k


0 273 2,46 x 10-3 0,003663 - 6,01
20,0 293 25,1 x 10-3 0,003413 - 3,68
40,0 313 576 x 10-3 0,003195 - 0,55

Plotando um gráfico ln k versus 1/T, temos:

-1

-2

-3
Ln k

-4

-5

-6

0.0032 0.0033 0.0034 0.0035 0.0036 0.0037


1/T

LIVRO OLÍMPICO – VOLUME I


99

Cálculo da energia de ativação (Eat): Através do coeficiente angular da reta,


temos:

− Eat ∆y
=
R ∆x
− Eat − 0,55 − (−6,01)
=
8,314 0,003195 − 0,003663
− Eat 5,46
=
8,314 −0,00468
J
Eat = 96996,97
mol

3
(OLIMPÍADA BRASILEIRA DE QUÍMICA) Na reação abaixo ocorre à
transferência de um átomo de oxigênio do NO2 para o CO: CO(g) + NO2(g) →
CO2(g) + NO(g). Esta reação foi estudada a 267°C obtendo-se os seguintes
dados:

Concentração (mol.L-1) Velocidade Inicial


Experimento
[CO] [NO2] (mol.L-1.s-1)
1 5,0 x 10-4 0,35 x 10-4 5,65 x 10-10
2 5,0 x 10-4 0,70 x 10-4 1,13 x 10-9
3 1,0 x 10-3 0,35 x 10-4 1,13 x 10-9

A partir destes dados, determine:


a) A expressão da velocidade;
b) A ordem de reação em relação a cada reagente;
c) Calcule a constante de velocidade, expressando-a com suas unidades.

Resposta:

Item a) Cálculo da ordem de reação para o CO, entre o experimento I e III:


a
1,0 x 10−3 1,13 x 10−9
( ) =
5,0 x 10−4 5,65 x 10−10

(2)a = 2

a=1

Cálculo da ordem de reação para o NO2, entre o experimento I e II:

b
0,35 x 10−4 5,65 x 10−10
( ) =
0,70 x 10−4 1,13 x 10−9

1 b 1
( ) =
2 2

b=1

Equação da velocidade da reação: vreação = k x [CO] x [NO2 ]

LIVRO OLÍMPICO – VOLUME I


100

Item b)
Ordem de reação em relação ao monóxido de carbono = 1
Ordem de reação em relação ao dióxido de nitrogênio = 1
Ordem de reação global = 1 + 1 = 2

Item c) Cálculo da constante cinética (k), a partir do experimento I:


vreação = k x [CO]x [NO2 ]

5,65 x 10−10 = k x 5,0 x 10−4 x 0,35 x 10−4

L
k = 3,23 x 10−2
mol x min

4
(OLIMPÍADA BRASILEIRA DE QUÍMICA) Uma das reações que ocorrem
nos motores de carro e sistemas de exaustão é: NO2(g) + CO(g) → NO(g) +
CO2(g). Os dados experimentais para esta reação são os seguintes:
[NO2] inicial [CO] inicial Velocidade inicial
Experimento
(mol.dm-3) (mol.dm-3) (mol.dm-3.s-1)
1 0,10 0,10 0,0050
2 0,40 0,10 0,0800
3 0,10 0,20 0,0050
a) Escreva a equação da lei de velocidade desta reação, considerando o
seguinte mecanismo para esta reação:
Etapa 1: NO2 + NO2 → NO3 + NO
Etapa 2: NO3 + CO → NO2 + CO2
b) Qual a etapa determinante da reação? Justifique.
c) Desenhe um diagrama de energia (energia versus caminho da reação) para
esta reação.

Resposta:

Item a) Cálculo da ordem de reação em relação ao dióxido de nitrogênio, entre


o experimento 1 e 2:

0,40 a 0,0800
( ) =
0,10 0,0050

(4)a = 16

a=2

Cálculo da ordem de reação em relação ao monóxido de carbono, entre o


0,20 b 0,0050
experimento 1 e 3: (0,10) = 0,0050

b=0

Equação da velocidade: vreação = k x [NO2 ]2

Item b) A etapa 1 é a etapa determinante, uma vez que nesta etapa o


catalisador encontra-se como reagente (NO2) e o intermediário (NO3) como
produto.

LIVRO OLÍMPICO – VOLUME I


101

Item c) Gráfico que representa a Energia versus caminho da reação (C.R.).

5
(OLIMPÍADA MUNDIAL DE QUÍMICA) Uma determinada substância sofre
decomposição segundo uma cinética de primeira ordem, e sua dependência
em relação à temperatura segue uma lei empírica chamada de equação de
Arrhenius. Os tempos de meia-vida determinados a 95ºC e 85ºC foram 15,4
minutos e 57,8 minutos, respectivamente. A partir destes dados:
a) Calcule a energia de ativação e, supondo que esta permaneça constante,
independente da temperatura, estime o tempo de meia-vida a 25ºC.
b) Estime também a energia de ativação por meio de um gráfico do logaritmo
natural da constante de velocidade versus o inverso da temperatura (em
Kelvin).

Resposta:

85°C ln 2
Item a) Cálculo da constante cinética a T = 85°C: t1/2 = k

ln 2 ln 2
k 85°C = 85°C
= = 0,012 min−1
t1/2 57,8

95°C ln 2
Cálculo da constante cinética a T = 95°C: t1/2 = k

ln 2 ln 2
k 95°C = 95°C
= = 0,045 min−1
t1/2 15,4

k Eat 1 1
Cálculo da energia de ativação (Eat): ln (k95°C ) = R
x [T −T ]
85°C 2 1

0,045 min−1 Eat 1 1


ln ( −1 )= x[ − ]
0,012 min 8,314 (85 + 273) (95 + 273)

Eat 1 1
ln(3,75) = x[ − ]
8,314 358 368

ln(3,75) x 8,314 1,32 x 8,314


Eat = =
1 1 1 1
[358 − 368] [358 − 368]

LIVRO OLÍMPICO – VOLUME I


102

10,32 J
Eat = = 144774,19
1 1 mol
[ − ]
358 368

Cálculo da constante cinética a 25°C, a partir dos dados referentes a 95°C:

k 25°C Eat 1 1
ln ( )= x[ − ]
k 95°C R T2 T1

k 25°C 144774,19 1 1
ln ( −1
)= x[ − ]
0,045 min 8,314 (95 + 273) (25 + 273)

k 25°C 144774,19 1 1
ln ( −1
)= x[ − ]
0,045 min 8,314 368 298

k 25°C
( ) = e(−11,11)
0,045 min−1

k 25°C = 6,73 x 10−7 min−1

Caso a constante cinética a 25°C fosse calculada a partir de 85°C, temos:

k 25°C Eat 1 1
ln ( )= x[ − ]
k 85°C R T2 T1

k 25°C 144774,19 1 1
ln ( ) = x [ − ]
0,012 min−1 8,314 (85 + 273) (25 + 273)

k 25°C 144774,19 1 1
ln ( ) = x [ − ]
0,012 min−1 8,314 358 298

k 25°C
( ) = e(−9,79)
0,012 min−1
k 25°C = 6,72 x 10−7 min−1

25°C ln 2 ln 2
Cálculo do tempo de meia-vida a 25°C: t1/2 = k
= 6,72 x 10−7 =
1,03 x 106 min

dln k Eat
Item b) Sabendo que a equação de Arrhenius é dada por: dt
= R x T²

Eat
(− )
k =Axe R.T

Aplicando a função logarítmica natural (ln), temos: lnk = lnA +


Eat
(− )
ln [e R.T ]

Eat
lnk = lnA −
RxT
𝐸
Chamando ln k = y, ln A = a (coeficiente linear), 1/T = x e a razão (− 𝑇𝑎𝑡 )
será o coeficiente angular (m). Logo, a energia de ativação pode ser calculado

LIVRO OLÍMPICO – VOLUME I


103

∆𝑦 𝐸𝑎𝑡
a partir da seguinte relação matemática: 𝑚 = ∆𝑥 = − 𝑇
, conforme pode
ser visualizado a partir do gráfico abaixo.

6
(OLIMPÍADA BRASILEIRA DE QUÍMICA) O cobre-64 é usado na forma de
acetato de cobre(II), no tratamento de tumores cerebrais. Se a meia-vida
desse radioisótopo é de 12,8 horas, a quantidade que restará, após 2 dias e
16 horas, de uma amostra com 15,0 mg de acetato de cobre (II) estará entre:
a) 0,1 e 0,5 mg
b) 0,5 e 1,0 mg
c) 1,0 e 2,0 mg
d) 2,0 e 3,0 mg
e) 3,0 e 5,0 mg

Resposta: Alternativa A.

Para um processo radioativo a cinética é de primeira ordem. Diante disso


será necessário converter o tempo total para hora.

24 horas
tempo = 2 dias x + 16 horas = 64 h
1 dia

Cálculo da constante radioativa a partir da equação do tempo de meia-vida


para uma cinética de primeira ordem:
ln 2
t1/2 =
λ
ln 2
λ = 12,8 h−1

Cálculo da massa final, aplicando a equação cinética de primeira ordem:


mfinal
ln ( ) = −λ x tempo
minicial
mfinal ln 2
ln ( )=− x 64
15,0 12,8
mfinal = 0,469 mg

LIVRO OLÍMPICO – VOLUME I


104

7
(OLIMPÍADA BRASILEIRA DE QUÍMICA) Assinale a opção que corresponde
à fração de substrato que reagiu, em uma reação de primeira ordem, após
um período de quatro vezes a meia-vida:
a) 15/16
b) 1/16
c) 7/8
d) 3/4

Resposta: Alternativa A.

m
Para um processo em que a cinética é de primeira ordem: ln (m final ) =
inicial
−k x t
Informação do problema: tempo = 4 x t1/2

mfinal
Juntando essas duas informações, temos: ln ( ) = −k x 4 x t1/2
minicial

m ln 2
ln (m final ) = −k x 4 x k
inicial

mfinal
ln ( ) = −ln24
minicial

minicial −1
ln ( ) = −ln24
mfinal
minicial
−1 x ln ( ) = −ln24
mfinal
minicial
= 16
mfinal
m 1
Fração que não reagiu: m final = 16
inicial

1 15
Fração que reagiu: 1 − =
16 16

8
(OLIMPÍADA BRASILEIRA DE QUÍMICA) Para a reação:
2NO(g) + Cl2(g) → 2 (NO)Cl(g). Equação de velocidade é dada por: v = k[NO]2.
[Cl2]. Se as concentrações de NO e Cl2, no início da reação são, ambas, iguais
a 0,02 mol.dm-3, então, a velocidade desta reação, quando a concentração
de NO houver diminuído para 0,01 mol.dm-3 será igual a:
a) 1,0 x 10-4 k
b) 1,5 x 10-4 k
c) 5,0 x 10-4 k
d) 1,5 x 10-6 k
e) 5,0 x 10-6 k

LIVRO OLÍMPICO – VOLUME I


105

Resposta: Alternativa D.

Para a resolução desta questão será necessário levar em consideração a


tabela de equilíbrio químico. Base de cálculo: 0,02 mol/L de monóxido de
nitrogênio e 0,02 mol/L de cloro, ambos em fase gasosa.

2NO(g) Cl2(g) → 2 (NO)Cl(g)


Início 0,02 0,02 0
Reage 2X X 2X
Equilíbrio 0,01 0,02-X 2X
Cálculo do valor de X: 0,02 – 2X = 0,01
X = 0,005 mol/L
Logo, no equilíbrio vai haver em cada reagente:
mol mol
[NO] = 0,01 e [Cl2 ] = 0,02 − 0,005 = 0,015
L L
Cálculo do volume da reação: vreação = k x (0,01)2 x 0,015 =
1,50 x 10−4 k
9
(OLIMPÍADA BRASILEIRA DE QUÍMICA) Obtém-se uma reta quando, para
uma reação A → B, de primeira ordem, constrói-se um gráfico de:
a) In [A] versus t
b) In [A] versus 1/t
c) [A] versus t
d) [A] versus 1/t
e) 1/[A] versus t
Obs: [A]: concentração de A; t = tempo

Resposta: Alternativa A.

Estudando a reação com a cinética de primeira ordem.

d [A]
− dt = k x [A] (Equação diferencial com cinética de primeira
ordem)
1
Separando as variáveis: [A] 𝑥 d[A] = − k x dt

[A]final tfinal
1
∫ d[A] = − ∫ k. dt
[A]inicial [A] 0

ln[A]final = ln[A]inicial − k x
⏟ ⏟t
eixo y eixo x

LIVRO OLÍMPICO – VOLUME I


106

10
(OLIMPÍADA BRASILEIRA DE QUÍMICA) Supondo que a seguinte reação
ocorra por meio de um mecanismo de uma única etapa elementar nas duas
direções: A + 2B ⇌ AB2. Sendo k1 e k2 as constantes, a uma determinada
temperatura, para as reações direta e inversa, respectivamente, assinale a
alternativa correta.
a) k1/k2 = k = [AB2]/[A][B]2
b) k1/k2 = k = [A][B]2/[AB2]
c) k2/k1 = k = [AB2]2/[A][B]
d) k2/k1 = k = [A][B]2/[AB2]2
e) k2 + k1 = k’ = [AB2]/[A][B]2

Resposta: Alternativa A.

Equação da velocidade da reação direta: vdireta = k1 x [A] x [B]2


Equação da velocidade da reação inversa: vinversa = k 2 x [AB2 ]

Condição de equilíbrio: vdireta = vinversa

k1 x [A] x [B]2 = k 2 x [AB2 ]


k1 [AB2 ]
=
k
⏟2 [A] x [B]2
constante de
equilíbrio

11
(OLIMPÍADA BRASILEIRA DE QUÍMICA) Para a reação genérica abaixo, a
298 K: 3X2Y + WZ3 → produtos. Foram obtidos os seguintes dados cinéticos:
Concentração inicial Velocidade inicial
Experimento
[X2Y]0 [WZ3]0 (mol/L x s)
I 1,72 2,44 0,68
II 3,44 2,44 5,44
III 1,72 0,1 2,8 x 10-2
IV 2,91 1,33 ?
a) Em relação a cada reagente, determine a ordem da reação. Determine,
também, a ordem global da reação.
b) A partir das informações da tabela, determine a Lei da Velocidade para a
reação.
c) A partir dos dados, determine o valor da Constante de Velocidade para a
reação genérica acima.
d) Utilizando os dados fornecidos, calcule a velocidade de reação para o
experimento IV.
e) A velocidade de reação aumenta por um fator de 100 na presença de um
catalisador, a 298K. A energia de ativação aumentará, diminuirá ou
permanecerá a mesma? Justifique.

LIVRO OLÍMPICO – VOLUME I


107

Resposta:
Item a)
Determinação da ordem de reação (a) em relação ao reagente X2Y, entre o
experimento I e II:
3,44 a 5,44
( ) =
1,72 0,68
2a = 8
a=3
Determinação da ordem de reação (b) em relação ao reagente WZ3, entre o
experimento I e III:

2,44 b 0,68
( ) =
0,10 2,8 x 10−2
24,4b ≅ 24,4
b=1
Ordem global do processo cinético: a + b = 3 + 1 = 4
Item b) Lei de velocidade: vreação = k x [X 2 Y]3 x [WZ3 ]1

Item c) Cálculo da velocidade específica, a partir do experimento I:

vreação = k x [X 2 Y]3 x [WZ3 ]1

mol mol 3 mol


0,68 = k x (1,72 ) x 2,44
Lxs L L
L3
k = 5,48 x 10−2
mol3 x s

Item d) Cálculo da velocidade no experimento IV: vreação =


k x [X 2 Y]3 x [WZ3 ]1
mol
vreação = 5,48 x 10−2 x (2,91)3 x 1,33 = 1,79
Lxs
Item e) Aumentando a velocidade da reação química com a utilização de um
catalisador específico vai ocasionar uma diminuição da energia de ativação.

LIVRO OLÍMPICO – VOLUME I


108

12
(OLIMPÍADA BRASILEIRA DE QUÍMICA) A ingestão de alimentos
gordurosos pode causar uma elevação no índice de colesterol no indivíduo e,
como consequência, geram-se obstruções nas artérias. Um dos exames mais
utilizados para verificar tais obstruções é a cintilografia do miocárdio. Para
realizá-lo, o paciente recebe uma dose de contraste que contém tecnécio
metaestável (Tc-99). Esse isótopo emite radiação gama, com uma constante
de decaimento igual a 3,2 × 10–5 s-1. Considerando um paciente que recebeu
uma quantidade de contraste às 14 horas de uma segunda feira, e sabendo
que após 8 meias-vidas a radiação volta ao nível seguro, assinale a
alternativa que indica em qual dia da semana e hora isto irá acontecer com o
paciente.
a) 14 horas da quarta-feira
b) 08 horas da manhã da quarta-feira
c) 14 horas da quinta-feira
d) 12 horas da quarta-feira
e) 20 horas da quarta-feira

Resposta: Alternativa A.
Analisando o decaimento com relação à massa do tecnécio em oito meias-
vidas.
X m0 X m0 X m0 X m0 X m0 X m0 X m0 X m0

m0 → ⏞
→ ⏞
→ ⏞
→ ⏞
→ ⏞
→ ⏞
→ ⏞

2 4 8 16 32 64 128 256
Sendo o X correspondente ao tempo de meia-vida.
m
Cálculo do tempo para o decaimento da massa de tecnécio: ln (m final ) =
inicial
−λ x t
𝑚0
ln ( 256 ) = −3,20 x 10−5 x t
𝑚0

t = 173286,79 s (≅ 48 horas)
Logo, o dia da semana será na quarta-feira às 14 horas.
13
(OLIMPÍADA BRASILEIRA DE QUÍMICA)
a) Propõe-se que a formação de C2H5Cl(g) se dar via mecanismo de reação
em duas etapas seguintes.
Etapa 1: C2H4(g) + HCl(g) → C2H5+(g) + Cl−(g) (Etapa lenta)
Etapa 2: C2H5+(g) + Cl−(g) → C2H5Cl(g) (Etapa rápida)
Escreva a lei de velocidade para a reação que é consistente com o
mecanismo de reação acima. Justifique a sua resposta.
b) Identifique um dos intermediários no mecanismo de reação acima.

LIVRO OLÍMPICO – VOLUME I


109

Resposta:

Item a) A etapa dominante para o estudo da cinética química é a etapa lenta,


logo a lei de velocidade é dada por: vreação = k x [C2 H4 ] x [HCl]

Item b) Os intermediários para o mecanismo apresentado são os seguintes:


Cl- e o C2H5+(g), uma vez que são produtos da primeira etapa e reagente na
etapa seguinte.
14
(GRILLO) Considere o seguinte mecanismo proposto para a reação a seguir:
AH + B ⇄ BH+ + A-
A- + AH → P
Encontre a lei de velocidade, expressando o valor da constante de velocidade
em função das constantes de velocidade das etapas elementares.

Resposta:
Analisando a primeira equação química reversível:

[BH+ ] x [A− ]
Kc = [AH] x [B]
(Equação I)

Analisando a segunda equação química da velocidade:

vreação = k x [A− ] x [AH] (Equação II)

Isolando a concentração da quantidade de matéria do reagente [A-] a partir


da equação I de equilíbrio, pois se refere do intermediário:
[BH + ] x [A− ] = K c x [AH] x [B]

Kc x [AH] x [B]
[A− ] = (Equação III)
[BH+ ]

Substituindo a concentração da quantidade de matéria de [A-] da equação III


na equação II: vreação = k x [A− ] x [AH]

K c x [AH] x [B]
vreação = k x x [AH]
[BH + ]

k x K c x [AH]2 x [B]
vreação =
[BH + ]

15
(OLIMPÍADA BRASILEIRA DE QUÍMICA) O ozônio (O3) é um gás de cor
azul claro, instável e altamente reativo, utilizado para a purificação de água.
O processo de ozonização da água é uma forma de tratamento oxidativo
que serve para degradar moléculas orgânicas que estejam na água como
contaminante. É um processo muito utilizado na indústria, mas ultimamente
acoplados em filtros caseiros de água, a fim de melhorar a qualidade da água
consumida pelas pessoas. O ozônio também se forma fotoquimicamente na

LIVRO OLÍMPICO – VOLUME I


110

troposfera da Terra e se decompõe de acordo com a equação: 2 O3(g) → 3


O2(g). Esta reação ocorre via proposta de mecanismo em duas etapas:
Etapa 1: O3 (g) ⇌ O2 (g) + O(g), rápida e reversível
Etapa 2: O3 (g) + O(g) ⟶ 2O2 (g), lenta
Qual lei de velocidade é consistente com o mecanismo proposto?
[O3 ] 𝑘[O3 ]2
a) − =
∆𝑡 [O2 ]
[O3 ]
b) − = 𝑘[O3 ]
∆𝑡
[O3 ]
c) − = 𝑘[O3 ]2
∆𝑡
[O3 ] 𝑘[O3 ]2
d) − =
∆𝑡 [O2 ]3
[O3 ] 𝑘[O2 ]
e) − =
∆𝑡 [O]

Resposta: Alternativa A.

Analisando as etapas sugeridas pelo problema, o [O] é o agente


intermediário.

d[O]
= +k1 x [O3 ] − k 2 x [O2 ] x [O] − k 3 x [O3 ] x [O]
dt

Sabendo que a concentração da quantidade de matéria do intermediário é


pequena e durante o processo cinético sua formação na primeira etapa e seu
consumo na segunda etapa ocorre de maneira muito rápida, diante disso sua
derivada é igual a zero.

0 = +k1 x [O3 ] − k 2 x [O2 ] x [O] − k 3 x [O3 ] x [O]

k 2 x [O2 ] x [O] + k 3 x [O3 ] x [O] = + k1 x [O3 ]

+ k1 x [O3 ]
[O] =
(k 2 x [O2 ] + k 3 x [O3 ])
vreação = k 3 x [O3 ] x [O]

𝑑 [𝑂3 ]
− = k 3 x [O3 ] x [O]
𝑑𝑡

d [O3 ] +k1 x [O3 ]


− = k 3 x [O3 ] x
dt (k 2 x [O2 ] + k 3 x [O3 ])
𝑘
d [O3 ] +⏞ k1 x k 3 x [O3 ]²
− =
dt k 2 x [O2 ] + k 3 x [O3 ]

LIVRO OLÍMPICO – VOLUME I


111

d [O3 ] + k x [O3 ]²
− =
dt k 2 x [O2 ] + k 3 x [O3 ]

16
(22th IChO, INTERNATIONAL CHEMISTRY OLYMPIAD) Nitramide NO2NH2
decomposes slowly in aqueous solution according to the reaction: NO2NH2 →
d [N2 O]
N2O(g) + H2O. The experimental kinetic law is as follows: dt
=
[N ON𝐻 ]
k [H2 𝑂+]2
3

a) What is the apparent order of this reaction in a buffered solution?


b) Which of the following mechanisms is the most appropriate for the
interpretation of this kinetic law? Justify your answer.
Mechanism 1: NO2NH2 → N2O + H2O (rate limiting step)
Mechanism 2:
NO2NH2 + H3O+ ⇌ NO2NH3 + + H2O (rapid equilibrium)
NO2NH3+ → N2O + H3O+ (rate limiting step)
Mechanism 3:
NO2NH2 + H2O ⇌ NO2NH3- + H3O+
NO2NH– → N2O + OH– (rate limiting step)
H3O+ + OH– → 2 H2O (very fast reaction)
Resposta:

Item a) A ordem aparente do NO2NH2 é de primeira ordem, conforme


apresentado na equação da velocidade apresentada no enunciado.
Item b) Analisando o mecanismo 1: este mecanismo está apresentado
apenas por uma etapa, diante disso a equação da velocidade é a seguinte:
vreação = k x [NO2 NH2 ]. Este mecanismo não condiz com a proposta
apresentada.
Analisando o mecanismo 2:
NO2NH2 + H3O+ ⇌ NO2NH3 + + H2O (etapa rápida)
NO2NH3+ → N2O + H3O+ (etapa lenta)
A equação da velocidade é a seguinte: vreação = k x [NO2 NH3+ ]

Analisando o intermediário, [NO2 NH3+ ]:


d [NO2 NH3+ ]
= +k1 x [NO2 NH2 ] x [H3 O+ ]
dt
− k 2 x [NO2 NH3+ ] x [H2 O] − k 3 x [NO2 NH3+ ]

d [NO2 NH3+ ]
=0
dt

LIVRO OLÍMPICO – VOLUME I


112

0 = +k1 x [NO2 NH2 ] x [H3 O+ ] − k 2 x [NO2 NH3+ ] x [H2 O]


− k 3 x [NO2 NH3+ ]

+k 2 x [NO2 NH3+ ] x [H2 O] + k 3 x [NO2 NH3+ ]


= +k1 x [NO2 NH2 ] x [H3 O+ ]

[NO2 NH3+ ] x (+k 2 x [H2 O] + k 3 ) = +k1 x [NO2 NH2 ] x [H3 O+ ]

+k1 x [NO2 NH2 ] x [H3 O+ ]


[NO2 NH3+ ] =
(k2 x [H2 O]+ k3 )

Substituindo na equação da velocidade: vreação = k x [NO2 NH3+ ]


+k1 x [NO2 NH2 ] x [H3 O+ ]
vreação = k x (k2 x [H2 O]+ k3 )

Diante desta análise, este mecanismo não condiz com a lei de velocidade
proposta.
Analisando o mecanismo 3:
NO2NH2 + H2O ⇌ NO2NH- + H3O+ (etapa rápida)
NO2NH– → N2O + OH– (etapa lenta)
H3O+ + OH– → 2 H2O (etapa muito rápida)
Equação da velocidade: vreação = k x [NO2 NH− ]

Expressão da constante de equilíbrio para a etapa rápida: K c =


[NO2 NH− ] x [H3 O+ ]
[NO2 NH2 ] x ⏟
aH 2 O
1

[NO2 NH − ] x [H3 O+ ]
Kc =
[NO2 NH2 ]
K c 𝑥 [NO2 NH2 ]
[NO2 NH − ] =
[H3 O+ ]

Equação da velocidade: vreação = k x [NO2 NH− ]


constante
k⏞x K c x [NO2 NH2 ]
vreação =
[H3 O+ ]
k x [NO2 NH2 ]
vreação =
[H3 O+ ]

Condiz com o mecanismo apresentado pela questão.

LIVRO OLÍMPICO – VOLUME I


113

17
(24th IChO, INTERNATIONAL CHEMISTRY OLYMPIAD) The dissociation of
N2O4(g) to give NO2(g) is a first order process with a specific rate constant of
5,3×104 s-1 at 298 K. Starting with an initial concentration of 0,10 M, how many
seconds would it take for 20% of the original N2O4 to decompose?
Resposta:

Equação química da decomposição do tetróxido de dinitrogênio: N2 O4(g) →


2 NO2(g)

Cálculo do tempo para o decaimento da massa de tecnécio:


[N O ]
ln ([N 2O 4] final ) = −k x t
2 4 inicial

[N2 O4 ]final = 0,80 𝑥 [N2 O4 ]inicial


0,80 𝑥 [N2 O4]inicial
ln ( [N2 O4 ]inicial
) = −5,30 x 10+4 x t

ln(0,80) = −5,30 x 10+4 x t


ln(0,80)
t = −5,30 x 10+4 = 4,21 x 10−6 s

18
(25th IChO, INTERNATIONAL CHEMISTRY OLYMPIAD) 131I is a radioactive
isotope of iodine (β–emitter) used in nuclear medicine for analytical
procedures to determine thyroid endocrine disorders by scintigraphy. The
decay rate constant, k, of 131I is 9,93 × 10-7 s-1. Questions:
a) Write the decay reaction of 131I.
b) Calculate the half-life of 131I expressed in days.
c) Calculate the time necessary (expressed in days) for a sample of 131I to
reduce its activity to 30 % of the original value.

Resposta:

Item a) 131I53 → 0β-1 + 131X54

ln 2 0,693 1 hora 1 dia


Item b) t1/2 = λ
= 9,93 x 10−7 = 697885,20 s x 3600 s x 24 horas

t1/2 = 8,07 dias


I
Item c) Cálculo do tempo: ln (I final ) = −9,93 x 10−7 x t
inicial

ln(0,30) = −9,93 x 10−7 x t


ln(0,30) 1 hora 1 dia
t= −7
= 1,21 x 106 s x x = 14,03 dias
−9,93 x 10 3600 s 24 horas

LIVRO OLÍMPICO – VOLUME I


114

19
(OLIMPÍADA CEARENSE DO ENSINO SUPERIOR DE QUÍMICA) A
decomposição térmica da fosfina (PH3) em fósforo e hidrogênio molecular
trata-se de uma reação de primeira ordem. A meia vida da reação é 35,0 s à
680°C. Determine o tempo necessário para a decomposição de 95% da
fosfina.
a) t = 151,3 s
b) t = 250 s
c) t = 50 s
d) t = 15,01 s
e) t = 500 s

Resposta: Alternativa A.

Equação química: 4 PH3(g) → P4(g) + 6 H2(g)


Cálculo da constante cinética (k), a partir do tempo de meia-vida (t1/2): 35,0 =
ln2
k

ln2
k= = 1,98 x 10−2 s −1
35,0

Informação fornecida pelo problema: [Fosfina]FINAL = 0,05 x [Óxido]INICIAL

[Fosfina]Final
ln ( ) = −k x tempo
[Fosfina]Inicial

0,05 x [Fosfina]Inicial
ln ( ) = −0,0198 x tempo
[Fosfina]Inicial

ln(0,05 ) = −0,0198 x tempo

ln(0,05 )
tempo = = 151,3 s
−0,0198

20
(OLIMPÍADA CEARENSE DO ENSINO SUPERIOR DE QUÍMICA) Um
recipiente contém uma mistura dos compostos A e B que se decompõem
segundo uma cinética de primeira ordem. As meias-vidas são de 50,0 minutos
para A e 18,0 minutos para B. Se as concentrações de A e B forem iguais no
início, que tempo será necessário para que a concentração de A seja quatro
vezes a concentração de B?
a) 62 min
b) 56 min
c) 68 min
d) 32 min
e) 45 min

LIVRO OLÍMPICO – VOLUME I


115

Resposta: Alternativa B.
𝑙𝑛2
Cálculo da velocidade específica (k) para o gás A: 𝑘 𝐴 = 𝑡
1⁄
2

ln 2
𝑘𝐴 = = 1,39 𝑥 10−2 𝑚𝑖𝑛−1
50 𝑚𝑖𝑛
𝑙𝑛2
Cálculo da velocidade específica para o gás B: 𝑘 𝐵 = 𝑡
1⁄
2

𝑙𝑛2
𝑘𝐵 = = 3,85 𝑥 10−2 𝑚𝑖𝑛−1
18 𝑚𝑖𝑛

Dividindo a equação do gás A, [A]Final = [A]Inicial x e−k x tempo , pela


equação do gás B, [B]Final = [B]Inicial x e−k x tempo , temos:

[A]Final [A]Inicial x e−k x tempo


=
[B]Final [B]Inicial x e−k x tempo

Informação do problema: [A]Inicial = [B]Inicial e [A]Final = 4 x [B]Final

[A]Final [B]Inicial x e−k x tempo


=
[B]Final [B]Inicial x e−k x tempo
−2
4 x [B]Final [B]Inicial x e−1,39 x 10 x tempo
=
[B]Inicial [B]Inicial x e−3,85 x 10−2 x tempo
−2
e−1,39 x 10 x tempo
4= −3,85 x 10−2 x tempo
e
−2 + 3,85 x 10−2 ) x tempo
4 = e(−1,39 x 10

Aplicando o logaritmo neperiano, temos: ln 4 =


(−1,39 x 10−2 + 3,85 x 10−2 ) x tempo
ln{e }

1,386= 2,47 x 10−2 x tempo

1,386
tempo = = 56,11 min
2,47 x 10−2

21
(OLIMPÍADA CEARENSE DO ENSINO SUPERIOR DE QUÍMICA) O isótopo
34
15P é utilizado para localizar tumores no cérebro e em estudos de formação
de ossos e dentes. Uma bancada de laboratório foi contaminada com 100 mg
desse isótopo, que possui meia-vida de 14,3 dias. O tempo mínimo, expresso
em dias, para que a radioatividade caia a 0,1% do seu valor original é igual a:
a) 86
b) 114
c) 124
d) 157
e) 143

LIVRO OLÍMPICO – VOLUME I


116

Resposta: Alternativa E.

Tempo de meia−vida
32 32
15Pinicial → 15Pfinal

Cálculo da constante radioativa (λ), a partir do tempo de meia-vida (t1/2):


ln 2
14,30 = λ
ln 2
λ= = 4,85 x 10−2 dias−1
14,30

[ 32P ]
Cálculo do tempo (t): ln ( 3215P final ) = −λ x tempo
[ 15 inicial ]

0,001 x [ 32
15Pinicial ]
ln ( 32 ) = −4,85 x 10−2 x tempo
[ 15Pinicial ]

ln(0,001) = −4,85 x 10−2 x tempo

ln(0,001) −6,91
tempo = −2
= = 142,47 dias
−4,85 x 10 −4,85 x 10−2

22
(CONCURSO PARA DOCENTE - IFRJ) A decomposição química de um
composto A apresenta cinética de primeira ordem. Um estudo sobre o
decréscimo de concentração do composto versus o tempo é mostrado na
seguinte tabela:
[A] (mol.L-1) Tempo (horas)
25,00 0
18,95 1
10,88 3
6,25 5
De acordo com os dados obtidos, calcule:
a) o tempo de meia-vida para o decaimento do composto;
b) o valor da constante de velocidade;
c) concentração da substância A, decorridos 4 horas de reação;
d) o tempo necessário para a decomposição de 99% do composto.

Resposta:

ln 2
Item a) Cálculo do tempo de meia-vida para o composto A: t 1⁄ = k
=
2
ln 2
0,227
= 2,50 h
Item b) Cálculo da constante de velocidade (k):

6,25
ln ( ) = −k x 5
25,0

k = 0,277 h−1

[A]Final
Item c) Cálculo da concentração de A, decorridos 4 horas: ln ( 25,0
)=
−0,277 x 4

LIVRO OLÍMPICO – VOLUME I


117

mol
[A]Final = 25 x e− (4 x 0,277) = 8,25
L

Item d) Cálculo do tempo para uma decomposição de 99%:

[A]Final = 0,01 x [A]Inicial.

0,01 x [A]Inicial
ln ( ) = −0,277 x tempo
[A]Inicial

ln(0,01) = −0,277 x tempo

tempo = 16,62 h

23
(CONCURSO PARA DOCENTE - IFRJ) A reação de decomposição do
pentóxido de dinitrogênio (N2O5) possui uma constante de velocidade a 25°C
igual a 3,38 x 10-4.s-1. Considerando que a equação que representa a reação
é: 2 N2O5(g) → 4 NO2(g) + O2(g).
a) Calcule o tempo de meia vida do N2O5.
b) Construa um gráfico qualitativo, do qual possa extrair o valor da
constante.

Resposta:

ln 2
Item a) Cálculo do tempo de meia-vida para o composto A: 𝑡1⁄ = 𝑘
=
2
ln 2
3,38 𝑥 10−4
= 2050,73 𝑠

Item b) Desenvolvendo a equação para a cinética de primeira ordem, temos:

ln[N2 O5 ]Final − ln[N2 O5 ]Inicial = −k x tempo

Chamando: ln[N2O5]Final de eixo y; ln[N2O5]Inicial como o coeficiente linear; t


como o eixo x e k (constante cinética) será o coeficiente angular. O gráfico
qualitativo ficará da seguinte maneira:

LIVRO OLÍMPICO – VOLUME I


118

24
(CONCURSO PARA DOCENTE - IFRJ) Esta reação, expressa na equação
a seguir, foi realizada na temperatura de 20°C, com concentração inicial de
NO3(g), igual a 0,05 mol.L-1. Decorridos 60 minutos, foi verificado que a
concentração de NO3 passou a ser de 0,0358 mol.L-1. NO3(g) → NO2(g) + ½
O2(g). Sabendo que a decomposição de NO3(g) segue uma cinética de segunda
ordem, determine o seguinte:
a) O valor da constante cinética;
b) A concentração de NO3(g) decorridos 145 minutos;
c) O tempo necessário para que reste 1% da concentração inicial do
reagente.

Resposta:

Item a) Cálculo da constante cinética para uma reação com cinética de


1 1
segunda ordem: 0,0358 − 0,05 = k x 60
L
k = 0,132
mol. min

Item b) Cálculo da concentração da quantidade de matéria após 145 minutos:


1 1
[NO ]
− = 0,132 x 143
3 Final 0,05

mol
[NO3 ]Final = 2,55 x 10−2
L

Item c) Informação do problema: [NO3 ]Final = 0,01 x [NO3 ]Inicial

1 1
− = 0,132 x tempo
0,01 x [NO3 ]Final [NO3 ]Inicial

99
= 0,132 x tempo
[NO3 ]Final

99
= 0,132 x tempo
0,05

tempo = 15000 min

25
(CONCURSO PARA DOCENTE - IFRJ) Dados cinéticos foram levantados a
uma determinada temperatura para esta reação: C2H5I(g) → C2H4(g) + HI(g).
Com base nesses dados, construíram-se os seguintes gráficos:

LIVRO OLÍMPICO – VOLUME I


119

O efeito da temperatura sobre essa mesma reação foi também estudado. A


tabela a seguir mostra os resultados obtidos. Para esta reação:

Temperatura (oC) 300 325 375 400


Tempo de meia vida (h) 86,69 13,85 0,54 0,13

a) Descreva a lei de velocidade e justifique sua resposta;


b) Calcule os parâmetros de Arrhenius;
c) Determine a constante de velocidade da reação e a temperatura em
que os dados empregados na construção dos gráficos foram obtidos.

Resposta:

Item a) Através do gráfico que relaciona ln[C2H5I] versus o tempo (t) está
linearizado, o que comprova que a reação de decomposição apresenta um
comportamento cinético de primeira ordem.

Item b) Cálculo dos parâmetros de Arrhenius.

ln 2
Cálculo da constante cinética (k) para a 300°C: k 300°C = =
86,69
−1
0,0080 h

Cálculo da constante cinética (k) para a temperatura igual a 400°C:

ln 2
k 400°C = = 5,33 h−1
0,13

k300°C Eat 1
Cálculo da energia de ativação: ln (k400°C ) = R
. [(400+273) −
1
(300+273)]

J
Eat = 298479,82
mol

Cálculo do fator do fator de frequência (A), a partir da equação de Arrhenius,


Eat
(− )
a uma temperatura de 400oC: k 400°C = A. e R.T

− 208479,82
{( )}
5,33 = A. e 8,314 x (400+273)

LIVRO OLÍMPICO – VOLUME I


120

A = 8,06 x 1016 h−1

Caso, o fator de frequência fosse calculado a uma temperatura de 300oC,


temos:
Eat
(− )
k 300°C = A x e RxT

− 208479,82
{( )}
0,0080 = A x e 8,314 x (300+273)
A = 8,10 x 1016 h−1

Conclusão: tanto faz o cálculo do fator de frequência pode ser realizado para
qualquer temperatura dado pelo problema.

[C H I]
Item c) A partir do gráfico ln[C2H5I] versus o tempo, temos: ln [C 2H 5I] Final =
2 5 Inicial
− k x tempo

ln[C2 H5 I]Final − ln[C2 H5 I]Inicial = −k x tempo

Para a equação y = - 0,9163 – 0,2704.x, a constante cinética é igual a k =


0,2704 h-1.

Cálculo da temperatura para a constante cinética igual a k = 0,2704 h-1:

k Eat 1 1
ln ( )= x [( ) − ( )]
k 300°C 8,314 300 + 273 T

0,2704 298479,82 1 1
ln ( )= x[ − ]
0,0080 8,314 573 T

T = 623,13 K

Caso a temperatura fosse calculado em função de k400°C, temos:

k Eat 1 1
ln ( )= x [( ) − ( )]
k 400°C 8,314 400 + 273 T

0,2704 298479,82 1 1
ln ( )= x[ − ]
5,33 8,314 673 T

T = 623,13 K

LIVRO OLÍMPICO – VOLUME I


121

26
(27th IChO, INTERNATIONAL CHEMISTRY OLYMPIAD) Rate constants kc
for reaction (c) were measured at various temperatures in a simulation
experiment in order to obtain its kinetic parameters. On the basis of the data
given below answer the following questions:
T(°C) 25 40
kc / mol.dm .s
-3 -1 1,29×10 -4 2,50×10-4
a) Write the equation for calculating the activation energy of reaction (c)
and fínd the value.
b) Assign the overall reaction order of reaction (c).

Resposta: Item a) Para a determinação da energia de ativação, será


necessária utilizar a equação de Arrhenius. Esta equação relaciona a
velocidade específica com a temperatura.

k 25°C Eat 1 1
ln ( 40°C )= x [( )−( )]
k 8,314 40 + 273 25 + 273

1,29 x 10−4 Eat 1 1


ln ( −4 )= x[ − ]
2,50 x 10 8,314 313 298

1,29 Eat
ln ( )= x[− 1,61 x 10−4 ]
2,50 8,314

−0,66 x 8,314 = Eat x[− 1,61 x 10−4 ]

J
Eat = 34082,24
mol

Item b) Para uma unidade de velocidade específica correspondente a mol/dm³


x s, a cinética corresponde à ordem zero.

27
(INSTITUTO MILITAR DE ENGENHARIA) Sabe – se que dois compostos A
e B reagem em solução de acordo com a estequiometria A + B → C + D, que
segue uma cinética de primeira ordem em relação a A quanto a B, com
velocidade específica de reação k = 10-3 L.mol-1.s-1. Em um recipiente, são
adicionados 2 mols de cada um dos reagentes e um solvente adequado até
completar 1 L de solução. Considerando que A é totalmente solúvel e B tem
uma solubilidade igual a 0,10 mol.L-1, obtenha a taxa de reação (v em mol.L-
1.s-1) em função da conversão de A, dada por 𝑋 = 2−𝑛𝐴 (onde n é o número
A
2
de mols de A em um dado instante).
a) 0 v = 0,002(1 − X), 0 ≤ X ≤ 0,95 e v = 0,004(1 − X)2 , 0,95 ≤
X≤1
b) v = 0,0002(1 − X), 0 ≤ X ≤ 0,05 e v = 0,002(1 − X)2 , 0,05 ≤
X≤1
c) v = 0,004(1 −
X)2 durante todo o processo, pois a reação se dá em solução
d) v = 0,0004(1 − X)2 , 0 ≤ X ≤ 0,05 e v = 0,004(1 − X)2 , 0,05 ≤
X≤1
e) v = 0,0002(1 − X), 0 ≤ X ≤ 0,95 e v = 0,004(1 − X)2 , 0,95 ≤
X≤1

LIVRO OLÍMPICO – VOLUME I


122

Resposta: Alternativa E.
2−n
Desenvolvendo a equação: X = 2 A
nA = 2 − 2X
nA = 2(1 − X)
A = totalmente solúvel e B = solúvel a 0,10 mol/L

Pelo fato da reação química apresentar uma proporção 1:1 pela equação
química A(g) + B(g) → C(g) + D(g), para que a concentração da quantidade de
matéria chegue a 0,10 mol/L, a espécie B será igual a sua solubilidade, ou
seja, 0,10 mol/L.
0
Para nA = 0; 2 = 1 − 𝑋, logo X = 1 (conversão de 100%)

0,10
Para nA = 0,10; 2
= 1 − 𝑋, logo X = 0,95 (conversão de 95%)

2
Para nA = 2; 2 = 1 − 𝑋, logo X = 0 (limite máximo)

Analisando a relação do número de mol de A com o limite da conversão:

(A) 0,10 ≤ nA ≤ 2 ⇒ 0,10 ≤ X ≤ 0,95

Equação da velocidade: v = k. [A]. [B]


v = 10−3 x 2 x (1 − X) x 0,10
v = 2,0 x 10−3 . (1 − X)

(B) 0 ≤ nA ≤ 0,10 ⇒ 0,95 ≤ X ≤ 1,0

Equação da velocidade: v = k. [A]. [B]


v = 10−3 x 2 x (1 − X) x 2 x (1 − X)
v = 4 x 10−3 x (1 − X)²
28
(PROGRAMA OLIMPÍADA URUGUAYA DE QUÍMICA) A partir de un estudio
cinético se logró determinar la constante de velocidad de la reacción CO (g) +
NO2(g) → CO2(g) + NO(g) a diferentes temperaturas.
a) Cuánto vale la energía de activación de la reacción?
b) Calcula los valores de k que faltan para completar la tabla.

Temperatura (k) K (L x mol-1 x s-1)


600 0,028
650 --------
700 1,30
750 -------
800 23
Resposta:

Item a) Cálculo da energia de ativação, aplicando a equação de Arrhenius.


0,028 Eat 1 1
ln ( )= x[ − ]
23 8,314 800 600

ln(1,22 x 10−3 ) x 8,314 = Eat x (−4,17 x 10−4 )

LIVRO OLÍMPICO – VOLUME I


123

−0,66 x 8,314 = Eat x [− 1,61 x 10−4 ]

J
Eat = 1,34 𝑥 105
mol

Levando em consideração que a energia de ativação será constante no


0,028
intervalo que compreende de 600 à 800K, temos: ln ( k
) =
1,34 𝑥 105 1 1
x[ − ]
8,314 650 600

0,028
ln ( ) = −2,066
k
0,028
= 𝑒 (−2,066)
k
L
k = 0,221
mol x s
1,30 1,34 𝑥 105 1
Cálculo da velocidade específica à 750K: ln ( k
) = 8,314
x [750 −
1
700
]

0,028
ln ( ) = −1,534
k
1,30
= 𝑒 (−1,534)
k
L
k = 6,03
mol x s

29
(PROGRAMA OLIMPÍADA URUGUAYA DE QUÍMICA) Para estudiar la
cinética de la reacción de descomposición R → productos se midió la
concentración del reactivo R a lo largo del tiempo. Se obtuvieron los datos
que se muestran en la tabla, a partir de los que se determinó que la reacción
sigue una cinética de segundo orden.
a) Calcula el valor de la constante de velocidad de la descomposición
de R.
b) Calcula el tiempo medio de reacción de R en estas condiciones.

Tempo (s) [R] (mol x L-1)


20 0,971
50 0,926
100 0,870
200 0,769
500 0,571
1000 0,400
1500 0,308
2000 0,250
3000 0,182
5000 0,118

LIVRO OLÍMPICO – VOLUME I


124

Resposta:

Item a) Cálculo da velocidade específica (k), levando em consideração que


a cinética é de segunda ordem.

1 1
= + k x tempo
[R]final [R]inicial
1 1
0,118
= 0,971 + k x 5000

L
k = 1,49 x 10−3
mol x s

Item b) Cálculo do tempo de meia-vida para uma cinética de segunda


ordem.

1 1
t1/2 = = = 691,18 s
k x [R]inicial 0,971 x 1,49 x 10−3

30
(PROGRAMA OLIMPÍADA URUGUAYA DE QUÍMICA) Considera la
reacción en fase gaseosa de conversión del metil isonitrilo en acetonitrilo:
H3C–NC(g) → H3C–CN(g). En el gráfico se muestra cómo varía la presión
parcial del metil isonitrilo con el tiempo, cuando se reacomoda a 198°C. En el
gráfico (b) se muestra la variación del logaritmo de dicha presión.

a) Cuál es el orden de reacción? Escribe la ley de velocidad correspondiente.


b) Estima el valor de la constante de velocidad de la conversión a 198°C.
c) Calcula el tiempo medio de la conversión de metil isonitrilo a acetonitrilo a
dicha temperatura.
d) Estima la presión parcial de acetonitrilo que hay en la mezcla gaseosa,
luego de que transcurren siete horas de reacción.

Resposta:

Item a) A partir do segundo gráfico à direita, que relaciona o logaritmo


neperiano da pressão de CH3NC versus o tempo, a cinética é de primeira
ordem.

Lei da velocidade: vreação = k x [CH3 NC]

Item b) Cálculo da velocidade específica, a partir do gráfico b.

LIVRO OLÍMPICO – VOLUME I


125

[CH3 NCA]Final
ln ( ) = −k x tempo
[CH3 NC]Inicial

4 − 5 = −k x 20000

−1
k= = 5,0 x 10−5 s−1
−20000
ln2 0,693
Item c) Cálculo do tempo de meia-vida: t1/2 = = = 13860 s
k 5,0 x 10−5

PFinal
Item d) Aplicando a equação cinética de primeira ordem: ln ( ) =
PInicial
−k x tempo

PFinal
ln ( ) = −5,0 x 10−5 x 7 x 3600
150 torr
PFinal
ln ( ) = −1,26
150 torr
PFinal
𝑒 (−1,26)
=⏟
150 0,284

PFinal = 42,55 torr

31
(INSTITUTO TECNOLÓGICO DA AERONÁUTICA) Dado o seguinte
mecanismo reacional, constituído de duas etapas elementares (I e II).

Escreva a expressão para a taxa de variação temporal da concentração do:


a) reagente A.
b) intermediário M.
c) produto C.

Resposta:

Item a) Determinação da taxa da variação temporal com relação a A:

d[A]
dt
= − k1 x [A] + k −1 x [M] − k 2 x [M] x [A]

Item b) Determinação da taxa temporal em relação ao intermediário M:

d[M]
dt
= + k1 x [A] − k −1 x [M] − k 2 x [M] x [A]

Item c) Determinação da taxa temporal em relação a C:

d[C]
dt
= + k 2 x [M] x [A]

LIVRO OLÍMPICO – VOLUME I


126

32
(U.S. NATIONAL CHEMISTRY OLYMPIAD) O seguinte mecanismo foi
proposto para a reação:

Usando a aproximação de estado estacionário, derivar a lei de taxa prevista


por este mecanismo.

Resposta: O mecanismo proposto ocorre em duas etapas, sendo a primeira


rápida (reação reversível) e a segunda etapa correspondente à lenta (reação
irreversível). Analisando a taxa temporal do intermediário, através das duas
reações apresentadas pelo problema,
d[O]
= + k1 x [O3 ] − k −1 x [O2 ] x [O] − k 2 x [O] x [O3 ]
dt

Como a concentração da espécie intermediária é praticamente constante,


d[O]
temos: dt = 0

d[O]
. dt
= + k1 x [O3 ] − k −1 x [O2 ] x [O] − k 2 x [O] x [O3 ]

0 = + k1 x [O3 ] − k −1 x [O2 ] x [O] − k 2 x [O] x [O3 ]

k −1 x [O2 ] x [O] + k 2 x [O] x [O3 ] = + k1 x [O3 ]

[O] x {k −1 x [O2 ] + k 2 x [O3 ]} = + k1 x [O3 ]

k1 x [O3 ]
[O] =
k −1 x 2 ] + k 2 x [O3 ]
[O

Equação da taxa de reação: vreação = k 2 x [O] x [O3 ]

Substituindo a concentração do intermediário, [O], na equação de taxa,


temos:

vreação = k 2 x [O] x [O3 ]

k1 x [O3 ]
vreação = k 2 x x [O3 ]
k −1 x [O2 ] + k 2 x [O3 ]
k 2 x k1 x [O3 ]2
=
k −1 x [O2 ] + k 2 x [O3 ]

k 2 x k1 x [O3 ]2
vreação =
k −1 x [O2 ] + k 2 x [O3 ]

LIVRO OLÍMPICO – VOLUME I


127

33
(U.S. NATIONAL CHEMISTRY OLYMPIAD – SEGUNDA FASE) Pentóxido
de dinitrogênio gasoso, N2O5, decompõe para formar oxigênio gasoso e
dióxido de nitrogênio com uma taxa inicial a 25°C, conforme a tabela abaixo:
[N2O5] (mol.L-1) Taxa (mol.L-1min-1)
0,150 3,42 x 10-4
0,350 7,98 x 10-4
0,650 1,48 x 10-3
a) Escreva a equação balanceada para esta reação.
b) Use os dados fornecidos para escrever a lei de taxa e calcule o valor
de k para esta reação. Mostre os cálculos.
c) Calcule o tempo necessário para que a concentração de N2O5
diminua de 0,150 mol.L-1 para 0,050 mol.L-1.
d) A taxa inicial para a reação de uma amostra 0,150 mol.L-1 a 40°C é
2,37 x 10–3 mol.L–1.min–1. Determine a energia de ativação para esta reação.
Resposta:

Item a) Equação química: N2O5(g) → 2 NO2(g) + ½ O2(g)

Item b) Cálculo da ordem de reação (a) a partir da tabela fornecida pelo


0,350 a 7,98 x 10−4
problema, entre o experimento I e II: (0,150) = 3,42 x 10−4

(2,33)a = 2,33

a = 1 (Reação de decomposição que apresenta cinética de primeira ordem)

Cálculo da constante cinética (k), através do experimento I: vreação =


k x [N2 O5 ]

3,42 x 10−4 = k x 0,150

k = 0,00228 min−1

Item c) A partir da expressão que relaciona a concentração versus o tempo


para um processo que apresenta cinética de primeira ordem.
[N O ]
ln ([N 2O 5] Final ) = −k x tempo
2 5 Inicial

0,050
ln ( ) = −0,00228 x tempo
0,150

tempo = 481,85 min

Item d) Cálculo da velocidade específica (k2) a 40°C: vreação = k x [N2 O5 ]

2,37 x 10−3 = k 2 x (0,150)

k 2 = 0,0158 min−1

Utilizando a equação de Arrhenius e considerando a constante dos gases (R)


igual a 8,314 J.mol-1K-1, temos: k1 (25°C) = 0,00228 min-1 e k2 (40°C) = 0,0158
min-1

LIVRO OLÍMPICO – VOLUME I


128

0,00228 Eat 1 1
ln ( )= x {( )−( )}
0,0158 8,314 313 298

Eat 1 1
ln(0,144) = x {( )−( )}
8,314 313 298

Eat
− 1,94 = x (− 1,61 x 10−4 )
8,314

8,314 x 1,94 J
Eat = −4
= 100181,12
1,61 x 10 mol

34
(CONCURSO PARA DOCENTE - IFRJ – MODIFICADA) O seguinte
mecanismo foi proposto para a obtenção do ácido hipobromoso:

Etapa 1: HBr + O2 ⇌ HOOBr


k2

Etapa 1: HOOBr + HBr →
⏞ 2 HOBr

Onde k1 e k´1 são, respectivamente, na etapa 1, as constantes de velocidade


das reações direta e inversa e k2 é a constante de velocidade da reação na
etapa 2. Baseando-se no mecanismo proposto, determine a lei de velocidade
da reação de formação de HOBr.

Resolução: Analisando a cinética em função do intermediário, HOOBr,


através da sua taxa temporal. O mecanismo proposto ocorre em duas etapas,
sendo a primeira rápida (reação reversível) e a segunda etapa corresponde à
lenta (reação irreversível). Analisando a taxa temporal em função do
intermediário, através das duas reações apresentadas pelo problema.

d[HOOBr]
= + k1 x [HBr] x [O2 ] − k1′ x [HOOBr]
dt
− k 2 x [HOOBr] x [HBr]

𝑑[𝐻𝑂𝑂𝐵𝑟]
Admitindo que a espécie intermediária, 𝑑𝑡
não apresente variação ao
d[HOOBr]
longo do processo cinético, temos: dt
= 0.

d[HOOBr]
= + k1 x [HBr] x [O2 ] − k1′ x [HOOBr]
dt
− k 2 x [HOOBr] x [HBr]

0 = + k1 x [HBr] x [O2 ] − k1′ x [HOOBr] − k 2 x [HOOBr] x [HBr]

+ k1 x [HBr] x [O2 ] = k1′ x [HOOBr] + k 2 x [HOOBr] x [HBr]

+ k1 x [HBr] x [O2 ] = [HOOBr] x {k1′ + k 2 x [HBr]}

k1 x [HBr] x [O2 ]
[HOOBr] =
{k1′ + k 2 x [HBr]}

Cálculo da taxa cinética d[HOOBr]/dT, substituindo expressão do


intermediário na equação da taxa em função do HOOBr:

LIVRO OLÍMPICO – VOLUME I


129

1 d[HOOBr]
+ x = +k 2 x [HOOBr]x [HBr]
2 dt
+ k1 x [HBr] x [O2 ]
Sabendo que [HOOBr] = , temos:
{k′1 + k2 x [HBr]}

1 d[HOOBr] k 2 x k1 x [HBr]² x [O2 ]


+ x =
2 dt {k1′ + k 2 x [HBr]}

d[HOOBr] 2 x k 2 x k1 x [HBr]² x [O2 ]


=
dt {k1′ + k 2 x [HBr]}

35
(OLIMPÍADA BRASILEIRA DE QUÍMICA) Considere as assertivas abaixo,
que se referem à ação dos catalisadores:
I. Alteram a velocidade da reação;
II. Diminuem a energia de ativação;
III. Transformam as reações em reações espontâneas;
IV. Deslocam o equilíbrio da reação para o lado dos produtos.
Estão corretas, somente as assertivas:
a) I e II
b) I e III
c) I e IV
d) II e III
e) III e IV

Resposta: Alternativa A.

Item I) Correto. Com a utilização do catalisador, a velocidade da reação


aumenta.
Item II) Correto. Com a utilização do catalisador, a energia de ativação
diminui.
Item III) Incorreto. A utilização do catalisador não influencia em reações a
serem ou não espontâneas.
Item IV) Incorreto. A utilização do catalisador não desloca o equilíbrio da
reação para nenhum dos lados, apenas deixa a reação em equilíbrio mais
rapidamente.

36
(U.S. NATIONAL CHEMISTRY OLYMPIAD) A 651,5 K, o tempo de meia-vida
para a decomposição de primeira ordem um determinado óxido é de
aproximadamente 6,03 minutos, e uma energia de ativação igual a 52
kcal.mol-1. A partir desta informação, assinale a alternativa do tempo
aproximado em segundos, necessário para que o referido óxido se
decomponha em 75%, a 723 K.
a) 10,0 segundos
b) 10,5 segundos
c) 12,4 segundos
d) 12,8 segundos
e) 13,8 segundos

Resposta: Alternativa E.

LIVRO OLÍMPICO – VOLUME I


130

Cálculo da constante cinética para um tempo de meia-vida igual a 6,03


minutos a 651,50 K:

ln2 ln 2
k 651,5 K = = = 0,115 min−1
t 1⁄ 6,03 min
2

Cálculo da velocidade específica a 723 K, utilizando a equação de Arrhenius:


k1 (651,50 K) = 0,115 min-1 e k2 (723 K) = ?

k 651,5 K Eat 1 1
ln ( 723 K )= x [( )−( )]
k R 723 651,5

0,115 52000 1 1
ln ( 723 K
)= x [( )−( )]
k 1,987 723 651,5

0,115
ln ( ) = −3,97
k 723 K
0,115
= e−3,97
k 723 K
0,115
k 723 K = = 6,12 min−1
1,88 x 10−2

Informação fornecida pelo problema: [Óxido]Final = 0,25 x [Óxido]Inicial


Cálculo do tempo (t) em minutos, a partir da cinética de primeira ordem:

0,25 x [óxido]Inicial
ln ( ) = −k x tempo
[óxido]Inicial

ln(0,25) = − 6,12 x tempo

ln(0,25) 60 s
tempo = = 0,226 min x ( ) = 13,56 s
−6,12 1 min

37
(EXAME NACIONAL DE CURSOS – ENGENHARIA QUÍMICA) Para a
reação de decomposição do ozônio 2O3(g) → 3O2(g) foi proposto um
mecanismo em duas etapas, a primeira envolvendo a reação de equilíbrio O3
⇄ O2 + O e a segunda envolvendo a reação irreversível O + O3 → 2 O2.
a) Desenvolva a equação da velocidade de reação em termos de O2
para o mecanismo proposto.
b) Mostre que a cinética da reação é de segunda ordem em relação ao
O3 e de ordem -1 em relação ao O2.

Resposta:
Item a) Analisando a primeira reação – reação reversível:

[O2 ] x [O] k
Expressão da constante de equilíbrio: K = [O3 ]
= k1′
1

K x [O3 ]
Isolando o intermediário ([O]): [O] = [O2 ]

LIVRO OLÍMPICO – VOLUME I


131

k1 [O3 ]
[O] = x
k′1 [O2 ]
(Equação A)

Analisando a segunda reação – reação irreversível:

Expressão da velocidade da reação:


𝑣𝑟𝑒𝑎çã𝑜 = k x [O] 𝑥 [𝑂3 ] (Equação B)

Substituindo a equação A na equação B, temos: 𝑣𝑟𝑒𝑎çã𝑜 = k 2 x [O] 𝑥 [𝑂3 ]

k1 [O3 ]
vreação = k 2 x x x [O3 ]
k1′ [O2 ]
k1 [O3 ]2
vreação = k2 x ′ x
k1 [O2 ]

Sabendo que a velocidade da reação em relação ao O2 é dada por 𝑣𝑟𝑒𝑎çã𝑜 =


1 𝑑[𝑂2 ] 𝑘1 [𝑂3 ]2
+2𝑥 𝑑𝑡
. Sabendo que 𝑣𝑟𝑒𝑎çã𝑜 = k 2 x 𝑘1′
𝑥 [𝑂 ] , temos:
2

1 d[O2 ] k1 [O3 ]2
+ x = k2 x ′ x
2 dt k1 [O2 ]

d[O2 ] 2 x k1 x k 2 x [O3 ]2
=
dt k1′ x [O2 ]

Item b) Observando a equação em relação a velocidade de formação do


Oxigênio, observamos que
d[O2 ] 2 x k1 x k2 x [O3 ]2
dt
= k′1 x [O2 ]
.

A ordem de reação em relação ao [O3] é igual a dois e em relação ao [O2] é


igual a -1.

38
(EXAME NACIONAL DE CURSOS – ENGENHARIA QUÍMICA) Calcule a
constante de equilíbrio a 350°C para a reação H2(g) + I2(g) ⇄ 2 HI(g), a partir
dos dados abaixo.
Reação Eat (kJ.mol-1) A (L.mol-1.s-1)
Sentido 1 165,1 1,6 x 1011
Sentido 2 186,0 1,2 x 1010
Eat é a energia de ativação e A é o fator de frequência ou pré-exponencial.

Resposta: Cálculo da constante cinética para a reação direta, ou seja,


k1
H2(g) + I2(g) → 2HI(g)

Eat
(− )
k1 = A1 x e R.T

165100
(− )
k1 = 1,6 x 1011 x e 8,314 x (350+273) = 1,6 x 1011 x e(−31,87)

LIVRO OLÍMPICO – VOLUME I


132

L
k1 = 2,30 x 10−3
mol x s

Cálculo da constante cinética para a reação inversa, ou seja, 2HI(g)


k2
→ H2(g) + I2(g):

Eat
(− )
k 2 = A2 x e R.T

186000
(− )
k 2 = 1,20 x 1010 x e 8,314 x (350+273) = 1,20 x 1010 x e(−35,91)

L
k 2 = 3,04 x 10−6
mol x s

Sabendo que a constante de equilíbrio é a razão das velocidades específicas,


k
o valor da constante de equilíbrio será: K = k1
2
−3 L
2,30 x 10
K= mol x s
L
3,04 x 10−6
mol x s

k1
K= = 756,58
k2

39
(INSTITUTO MILITAR DE ENGENHARIA) A reação abaixo segue a mesma
cinética do decaimento radioativo. A → 2B + ½C. Ao se acompanhar
analiticamente o desenvolvimento desta reação na temperatura T1, obtêm-se
o Gráfico 1, o qual estabelece uma relação entre a concentração molar da
substância A no meio reacional e o tempo de reação.

Ao se conduzir esta mesma reação em diversas temperaturas, obtêm-se


diferentes valores para a constante de velocidade de reação k, conforme os
dados da Tabela 1.

Tabela 1 - Efeito da temperatura na constante de velocidade (k).


Temperatura (°C) 25 45 55 65
Constante de velocidade, k 3,2 x 10- 5,1 x 10- 1,7 x 10- 5,2 x 10-
(s-1) 5 4 3 3

LIVRO OLÍMPICO – VOLUME I


133

Finalmente, com um tratamento matemático dos dados da Tabela 1, pode-se


construir o Gráfico 2, o qual fornece uma relação entre a constante de
velocidade e a temperatura. Com base nas informações fornecidas,
considerando ainda que ln 2 = 0,69 e que a constante universal dos gases é
igual a 8,3 J.mol-1.K-1, determine:
a) a temperatura T1;
b) a energia de ativação, em kJ/mol, da reação.

Resolução: Item a) Cálculo da temperatura T1: Para a determinação da


temperatura T1, será calculado a constante cinética a partir do gráfico 1 e
comparado com a tabela 1 fornecida pelo problema. A reação de
decomposição obedece a uma cinética química de primeira ordem, uma vez
que a constante cinética se encontra na unidade (s-1), conforme pode ser
observado também na própria tabela.

[A]Final
ln ( ) = −k x tempo
[A]Inicial

0,010
No intervalo entre 0 e 400 segundos, temos: ln ( ) = − 400 x k
0,020
1
ln ( ) = − 400 x k
2

k = 1,73 x 10−3 s−1

Observando a tabela 1, a temperatura correspondente é de 55°C.

Item b) Através de dois pontos do gráfico 2, temos:

Eat 1 1
Utilizando a equação de Arrhenius: ln k1 − ln k 2 = R
x {T − T }
1

Eat
− 5,25 − (−7,5) = x {0,0031 − 0,0029}
8,3

J
Eat = 9,33 x 104
mol

LIVRO OLÍMPICO – VOLUME I


134

40
(GRILLO) Considere uma reação hipotética em fase aquosa que apresenta
cinética de segunda ordem, do tipo A(aq) + B(aq) → P(aq). Este processo foi
conduzido inicialmente em 0,050 mol.L-1 de Q e 0,080 mol.L-1 em B. Após o
decorrer da reação, a concentração da quantidade de matéria de A caiu para
0,020 mol.L-1 em um período de uma hora. Determine a constante cinética em
L.mol-1.s-1.

Resposta: Tabela de equilíbrio químico:

A(aq) (mol.L-1) B(aq) (mol.L-1) P(aq) (mol.L-1)


Início 0,050 0,080 0

Reage 0,030 = α 0,030 0,030
Equilíbrio 0,020 0,050 0,030

Sabendo que a equação matemática que expressa a cinética de segunda


ordem para uma reação do tipo A(aq) + B(aq) → P(aq) em que as concentrações
de ambos os reagentes são distintas, temos:

1 [A]0 x ([B]0 − α)
k x tempo = x ln [ ]
[B]0 − [A]0 ([A]0 − α) x [B]0

1 0,050 x (0,080 − 0,030)


kx1= x ln [ ]
0,080 − 0,050 (0,050 − 0,030) x 0,080

1 0,050 x (0,080 − 0,030)


kx1= x ln [ ]
0,030 (0,050 − 0,030) x 0,080

1 (0,050)2 0,00250
k= x ln [ ] = 33,33 x ln [ ]
0,030 (0,020) x 0,080 0,0016
= 33,33 x ln(1,5625)

L 1h L
k = 14,38 x( ) = 0,00413
mol x h 3600 s mol x s

1
(GRILLO) Considere a reação de segunda ordem do tipo A(aq) + 2B(aq) → P(aq).
Este processo apresentou inicialmente 0,075 mol.L-1 em A e 0,080 mol.L-1 em
B. Após um período de aproximadamente uma hora, a concentração da
quantidade de matéria de A foi igual a 0,045 mol.L-1. Através destas
informações, determine a constante de velocidade em L.mol-1.s-1.

Resolução: Tabela de equilíbrio químico:

A(aq) (mol.L-1) 2B(aq) (mol.L-1) P(aq) (mol.L-1)


Início 0,075 0,080 0

Reage 0,030 = α 2 x 0,030 = 0,060 0,030
Equilíbrio 0,045 0,020 0,030

Sabendo que a cinética de segunda ordem para uma reação do tipo A(aq) +
B(aq) → P(aq) em que as concentrações dos reagentes são distintas é dada
pela equação matemática abaixo:

LIVRO OLÍMPICO – VOLUME I


135


k x tempo =
(2 x [A]0 − [B]0 ) x ([B]0 − 2α) x [B]0
1 [A]0 x ([B]0 − 2α)
+ 2
x ln [ ]
(2 x [A]0 − [B]0 ) ([A]0 − α) x [B]0

Cálculo da constante cinética:


k x tempo =
(2 x [A]0 − [B]0 ) x ([B]0 − 2α) x [B]0
1 [A]0 x ([B]0 − 2α)
+ x ln [ ]
(2 x [A]0 − [B]0 )2 ([A]0 − α) x [B]0

kx1
2 x (0,030)
=
(2 x 0,075 − 0,080) x (0,080 − 2 x 0,030) x 0,080
1 0,075 x (0,080 − 2 x 0,030)
+ 2
x ln [ ]
(2 x 0,075 − 0,080) (0,075 − 0,030) x 0,080

0,060
k =
(0,070) x (0,020) x 0,080
1 0,075 x (0,080 − 0,060)
+ 2
x ln [ ]
(0,070) (0,045) x 0,080

0,060 1 0,075 x 0,020


k= + 2
x ln [ ]
(0,070) x (0,020) x 0,080 (0,070) (0,045) x 0,080

3 1 0,075
k= + x ln [ ]
0,0056 0,0049 (0,045) x 4

L 1h L
k = 357,04 x = 0,099
mol x h 3600 s mol x s

42
(INSTITUTO MILITAR DE ENGENHARIA) Considere a reação de
decomposição da nitramida em solução aquosa: NH2 NO2(aq) → N2 O(g) +
H2 O(l). Sabendo-se que a lei de velocidade, determinada
[NH2 NO2 ]
experimentalmente, é dada pela expressão v = k x [H3 O+ ]
, foram
propostos três possíveis mecanismos para a reação:

MECANISMO I:
NH2 NO2 + H2 O ⇌ NHNO−2 + H3 O
+
(equilíbrio rápido)
− −
NHNO2 → N2 O + OH (etapa lenta)
H3 O+ + OH − → 2 H2 O (etapa rápida)

MECANISMO II: NH2 NO2 → N2 O + H2 O (etapa elementar)

MECANISMO III:
NH2 NO2 + H3 O+ ⇌ NH3 NO+2 + H2 O (equilíbrio rápido)
NH3 NO+
2 ⇌ N 2 O + H3 O+
(etapa lenta)

LIVRO OLÍMPICO – VOLUME I


136

Com base nas informações acima, determine se cada mecanismo proposto é


compatível com a expressão da velocidade experimental, fundamentando
suas respostas.

Resposta: Primeiramente analisaremos o mecanismo II que é bastante


direto. Como se trata de uma reação elementar, a equação da velocidade é
dada pela seguinte expressão: vreação = k x [NH2 NO2 ]1 . Diante da
equação fornecida pelo problema, o mecanismo II não condiz com a lei de
velocidade sugerida pelo problema.

Analisando o mecanismo III: Apresentando as equações químicas


apresentadas pelo enunciado, NH2 NO2 + H3 O+ ⇌ NH3 NO+ 2 + H2 O
(equilíbrio rápido) e NH3 NO+ +
2 → N2 O + H3 O (etapa lenta), o [NH3 NO2 ]
+

é o intermediário, por isso realizando o estudo do estado estacionário em


função do próprio intermediário temos:

d[NH3 NO+
2]
= + k a x [NH2 NO2 ] x [H3 O+ ]
dt
− k b x [NH3 NO+ +
2 ] x [H2 O] − k c x [NH3 NO2 ]

Sabendo que a concentração da quantidade de matéria do intermediário é


pequena e sua respectiva derivada é igual a zero, temos:

d[NH3 NO+
2]
= + k a x [NH2 NO2 ] x [H3 O+ ]
dt
− k b x [NH3 NO+ +
2 ] x [H2 O] − k c x [NH3 NO2 ]

0 = + k a x [NH2 NO2 ] x [H3 O+ ] − k b x [NH3 NO+


2 ] x [H2 O]
− k c x [NH3 NO+ 2 ]

Isolando a concentração da quantidade de matéria do intermediário:

k a x [NH2 NO2 ] x [H3 O+ ]


[NH3 NO+
2] =
k b x [H2 O] + k c

Equação da velocidade do mecanismo III: vreação = k 𝑐 x [NH3 NO+ 1


2] .

Substituindo a expressão da concentração da quantidade de matéria do


intermediário na equação da velocidade de reação, temos:

vreação = k 𝑐 x [NH3 NO+


2]
1

k a x [NH2 NO2 ] x [H3 O+ ]


vreação = k 𝑐 x
k b x [H2 O] + k c

Levando em consideração que a concentração da quantidade de matéria do


[H3 O+ ] é baixa e a atividade da água líquida é igual a 1, a equação da
velocidade da reação pode ser reescrita da seguinte maneira: vreação =
ka x [NH2 NO2 ] x [H3 O+ ]
k𝑐 x kb x [H2 O]+kc

LIVRO OLÍMPICO – VOLUME I


137

k a x k 𝑐 x [NH2 NO2 ]
vreação =
kb + kc

Portanto para esta lei de velocidade não condiz com o enunciado.

Analisando o mecanismo I: A equação da taxa de velocidade para o


mecanismo I é dada pela seguinte equação: vreação = k 𝑐 x [NHNO− 1
2] .

Observando as etapas e concluindo que a concentração de [NHNO2 ] é o
intermediário e realizando o seu estado estacionário, temos:

d[NHNO−
2]
= + k a x [NH2 NO2 ] x [H2 O] − k b x [NHNO− +
2 ] x [H3 O ]
dt
− k c x [NHNO− 2]

d[NHNO−
2]
Analisando o estado estacionário do intermediário igual a zero, dt
=
0:

d[NHNO−
2]
= + k a x [NH2 NO2 ] x [H2 O] − k b x [NHNO− +
2 ] x [H3 O ]
dt
− k c x [NHNO− 2]

0 = + k a x [NH2 NO2 ] x [H2 O] − k b x [NHNO− +


2 ] x [H3 O ]

− k c x [NHNO2 ]

Isolando a concentração da quantidade de matéria do intermediário:


kb x [NH2 NO2 ] x [H2 O]
[NHNO− 2] = +
kb x [H3 O ]+kc

Levando em consideração que a concentração da quantidade de matéria do


[H3 O+ ] é baixa e a atividade da água líquida é igual a 1, a equação da
velocidade da reação pode ser reescrita da seguinte maneira:
k b x [NH2 NO2 ] x [H2 O]
vreação = k 𝑐 x
k b x [H3 O+ ] + k c

kb x kc
vreação = x [NH2 NO2 ] = k ′ x [NH2 NO2 ]
k
⏟b + k c
k′

Portanto para esta lei de velocidade condiz com o enunciado, condiz com o
mecanismo proposto.

43
(OLIMPÍADA BRASILEIRA DE QUÍMICA) O dióxido de cloro (ClO2) reage
com íons hidróxido para produzir uma mistura de íons clorato e clorito,
conforme representado pela equação não balanceada a seguir: ClO2(aq) + OH-
(aq) → ClO3 (aq) + ClO2 (aq) + H2O(l). Os dados de velocidade inicial de reação
- -

apresentados na tabela abaixo foram determinados à temperatura constante.


Experimento [ClO2]/(mol.L-1) [OH-]/(mol.L-1) (mol.L-1.s-1)
1 0,0150 0,0250 1,30 x 10-3
2 0,0150 0,0500 2,60 x 10-3
3 0,0450 0,0250 1,17 x 10-2
Com base nesses dados, indique a alternativa que apresenta o valor da
constante de velocidade, k (em L² x mol-2 x s-1), da reação.

LIVRO OLÍMPICO – VOLUME I


138

a) 231
b) 139
c) 118
d) 69
e) 185

Resposta: Alternativa A.

Determinação da ordem de reação (a) em relação ao reagente ClO2, entre o


experimento 1 e 3:

0,0450 a 1,17 x 10−2


( ) =
0,0150 1,30 x 10−3
3a = 9
a=2
Determinação da ordem de reação (a) em relação ao reagente OH-, entre o
experimento 1 e 2:
0,0500 a 2,60 x 10−3
( ) =
0,0250 1,30 x 10−3
2a = 2
b=1
Ordem global do processo cinético: a + b = 2 + 1 = 3

Lei de velocidade: vreação = k x [ClO2 ]2 x [OH − ]1

Cálculo da velocidade específica, a partir do experimento I:


vreação = k x [ClO2 ]2 x [OH − ]1

1,30 x 10−3 = k x (0,0150)2 x 0,0250


L2
k = 231,11 mol2 x s

44
(INSTITUTO MILITAR DE ENGENHARIA) Um cientista prepara uma amostra
de 1,1 g do isótopo C11 do carbono de extrema pureza. Esse isótopo é
radioativo, iniciando seu decaimento após a preparação (instante inicial t0 =
0). Sabendo-se que sua meia-vida é de 21 minutos, calcule a massa restante
de C11 no instante t = 1 h e 31 min.

Resposta: O decaimento radioativo ocorre a partir de uma cinética de


primeira ordem. Diante disso, será necessário calcular primeiramente a
constante cinética (λ).

Tempo = 1 h 31 min corresponde a 61 minutos.


ln 2
Cálculo da constante radioativa: t1/2 = λ

LIVRO OLÍMPICO – VOLUME I


139

0,693
λ= min−1
21
mfinal
Cálculo da massa restante: ln ( ) = −λ x tempo
minicial

mfinal 0,693
ln ( )=− x 61
1,1 21

mfinal (−
0,693
x 61)
= e 21
1,1

mfinal = 0,147 g

45
(INSTITUTO TECNOLÓGICO DA AERONÁUTICA) Considere a reação
genérica A + 2B →C, cuja lei de velocidade é dada por v = k[A]α[B]β. Em um
estudo cinético, foram obtidas as velocidades da reação em cinco
experimentos distintos, em que as concentrações das espécies A e B
variaram conforme a tabela abaixo.
Experimento [A] (mol/L) [B] (mol/L) v (mol.L-1.min-1)
1 0,025 0,01 2,5 x 10-6
2 X 0,02 2,0 x 10-5
3 0,025 0,005 1,25 x 10-6
4 0,1 0,005 Y
5 0,05 0,01 1,0 x 10-5
Com base nos experimentos, assinale a opção que apresenta os valores
corretos de α
β, k, X e Y, respectivamente.
a) 1; 1; 1 x 10-2; 1,0 x 10-1 e 5,0 x 10-6
b) 1; 2; 1,0; 1,0 x 10-3 e 5,0 x 10-4
c) 1; 2; 1,0; 1,0 x 10-3 e 5,0 x 10-4
d) 2; 1; 0,4; 2,5 x 10-3 e 2,0 x 10-3
e) 2; 1; 0,4; 5,0 x 10-2 e 2,0 x 10-5

Resposta: Alternativa D.

Determinação da ordem de reação (α) em relação ao reagente A, entre o


experimento 1 e 5:
0,05 α 1,0 x 10−5
( ) =
0,025 2,50 x 10−6
2α = 4
α=2
Determinação da ordem de reação (β) em relação ao reagente B, entre o
experimento 1 e 3:

0,01 β 2,50 x 10−6


( ) =
0,005 1,25 x 10−6
2β = 2

LIVRO OLÍMPICO – VOLUME I


140

β=1
Ordem de reação global do processo cinético: a + b = 2 + 1 = 3
Determinação da constante cinética (k), a partir do experimento 2: v =
k x [A]α [B]β

2,5 x 10−6 = k x (0,025)2 x 0,01

2,5 x 10−6 L2
k= = 0,40
6,25 x 10−6 mol2 x min

Determinação do valor de X, a partir do experimento 1: v = k x [A]α [B]β

2,0 x 10−5 = 0,40 x X 2 x 0,02

mol
X = 0,05
L
Determinação do valor de Y, a partir do experimento 4: v = k x [A]α [B]β

Y = 0,40 x (0,10)2 x 0,005

mol
Y = 2,0 x 10−5
L x min

46
(37TH ICHO, INTERNATIONAL CHEMISTRY OLYMPIAD) Ozone (O3) is a
formo f oxygen. It is a natural componente of the stratosphere, where it Shields
the earth from life-destrying ultravioleta radiation. On absorbing light in this
region, ozone is converted to dioxygen molecules. For the overall reaction of
ozone decomposition, 2 O3 → 3O2. One of the proposed mechanisms is
expressed as
O3 ⇌ O + O 2
O3 + O → 2 O 2
According to the above mechanism what are the differentail rate equations for
the formation (or consumption) of O3, O2 and O at time t, assuming step 2 is
irreversible.
Resposta: Analisando a primeira reação – reação reversível:
[O2 ] x [O] k
Expressão da constante de equilíbrio: K = [O3 ]
= k1′ , em que k1 é a
1
velocidade específica da reação direta e k´1 é a velocidade específica da
reação inversa. Além disso, importante afirmar que a primeira etapa é a rápida
e a segunda etapa é a lenta.

d[O]
Determinação do estudo em função do intermediário, dt
.

d[O]
= k1 x [O3 ] − k 2 x [O] x [O2 ] − k 3 x [O] x [O3 ]
dt

0 = k1 x [O3 ] − k 2 x [O] x [O2 ] − k 3 x [O] x [O3 ]

LIVRO OLÍMPICO – VOLUME I


141

k1 x [O3 ] = k 2 x [O] x [O2 ] + k 3 x [O] x [O3 ]

k1 x [O3 ] = [O] x (k 2 + [O2 ] + k 3 x [O3 ])

k1 x [O3 ]
[O] =
k 2 + [O2 ] + k 3 x [O3 ]

Expressão da velocidade da reação: vreação = k x [O] x [O3 ]

1 d[O3 ] 1 d[O2 ]
vreação = − x =+ x = k 2 x [O] x [O3 ]
2 dt 3 dt

d[O3 ] k1 x [O3 ]
− = 2 x k2 x x [O3 ]
dt k 2 + [O2 ] + k 3 x [O3 ]

d[O3 ] k1 x [O3 ]²
− dt
= 2 x k2 x k2 +[O2 ]+k3 x [O3 ]
(em relação ao consumo de O3)

d[O2 ] k1 x [O3 ]²
+ dt
= 3 x k2 x k2 +[O2 ]+k3 x [O3 ]
(em relação à formação de O2)

47
(40TH ICHO, INTERNATIONAL CHEMISTRY OLYMPIAD) Embora o íon
ditionato seja, termodinamicamente, um razoável reagente redutor ele não
reage com oxidantes, em solução à temperatura ambiente. A 75°C,
entretanto, ele pode ser oxidado em soluções ácidas. Uma série de
experimentos cinéticos foram realizados, usando o bromo como oxidante. As
velocidades iniciais (v0) da reação foram determinadas em uma série de
experimentos a 75°C.

[Br2]0 [Na2S2O6]0 [H+]0 v0


(mmol/dm ) (mol/dm ) (mol/dm3) (nmol dm–3 s−1)
3 3

0,500 0,0500 0,500 640


0,500 0,0400 0,500 511
0,500 0,0300 0,500 387
0,500 0,0200 0,500 252
0,500 0,0100 0,500 129
0,400 0,0500 0,500 642
0,300 0,0500 0,500 635
0,200 0,0500 0,500 639
0,100 0,0500 0,500 641
0,500 0,0500 0,400 511
0,500 0,0500 0,300 383
0,500 0,0500 0,200 257
0,500 0,0500 0,100 128
Determine a ordem da reação em relação ao Br2, ao H+ e ao S2O62−, a
equação da velocidade experimental e o valor e a unidade da constante de
velocidade.

Resposta:

Equação da velocidade: vreação = k x [Br2 ]α x [Na2 S2 O6 ]β x [H + ]γ

LIVRO OLÍMPICO – VOLUME I


142

0,300 α 635
Cálculo da ordem de reação α entre o experimento 6 e 7: (0,400) = 642

(0,75)α = 0,99

α x ln 0,75 = ln 0,99

ln 0,99 −1,0 𝑥 10−2


α= = = 0,035
ln 0,75 −0,288
0,0200 β 252
Cálculo da ordem de reação β entre o experimento 3 e 4: (0,0300) = 387

(0,667)β = 0,651

β x ln 0,75 = ln 0,651
β = 1,06
0,300 γ 383
Cálculo da ordem de reação γ entre o experimento 10 e 11: (0,400) = 511

(0,75)γ = 0,749

γ =1
Equação da velocidade: vreação =
[Br ]0,035 [Na ]1,06 [H + ]1,0
kx 2 x 2 S2 O6 x
Cálculo da velocidade específica:
mol
640 x 10−9
dm3 x s
mol 0,035 mol 1,06 mol
= k x (0,500 ) x (0,0500 ) x 0,500
dm³ dm³ dm³
(dm3 )1,095
k = 3,140 x 10−5
mol1,095 x s
48
(40TH ICHO, INTERNATIONAL CHEMISTRY OLYMPIAD) Um experimento
foi realizado para acompanhar a absorbância em 275 nm, com concentrações
iniciais: [Na2S2O6] = 0,0022 mol/dm3, [HClO4] = 0,70 mol/dm3, e a temperatura
foi de 75 °C. Uma curva cinética de pseudo primeira ordem foi encontrada
com uma meia-vida de 10 horas e 45 minutos. Calcule a constante de
velocidade da reação.

Resposta:
Cálculo da constante cinética (k), a partir de um processo de pseudo-primeira
ordem.

ln2 = k x t1/2

ln2 = k x (600 + 45)


0,693
k= = 1,07 x 10−3 min−1
645

LIVRO OLÍMPICO – VOLUME I


143

49
(39TH ICHO, INTERNATIONAL CHEMISTRY OLYMPIAD) Muitas reações
químicas apresentam comportamento cinético instável. Em diferentes
condições (concentrações e temperatura) tais reações podem ocorrer de
vários modos: estável, oscilatório ou caótico. Muitas destas reações incluem
etapas elementares auto catalíticas. Considere um simples mecanismo de
reação envolvendo etapa auto catalítica:
k1
⏞ 3X
B + 2X →
k2
X + D→ ⏞P
(В e D são reagentes, X é um intermediário e P é um produto).

Deduza a equação da velocidade usando aproximação do estado


estacionário. Encontre as ordens:
(i) ordem da reação parcial com respeito a B;
(ii) ordem da reação parcial com respeito a D;
(iii) ordem total da reação.

Resposta: Estudo em função do intermediário (X):


d[X]
= − k1 x [B] x [X]2 − k 2 x [X] x [D]
dt
0 = − k1 x [B] x [X]2 − k 2 x [X] x [D]

− k1 x [B] x [X]2 = k 2 x [X] x [D]


k 2 x [D]
[X] =
− k1 x [B]
Equação da velocidade: vreação = k 2 x [X] x [D]

Substituindo a equação do intermediário na equação da velocidade.


k 2 x [D]
vreação = k 2 x x [D]
− k1 x [B]
k 22 x [D]²
vreação = −
k1 x [B]
(i) ordem da reação parcial com respeito a B = -1.
(ii) ordem da reação parcial com respeito a D = 2.
(iii) ordem total da reação = -1+2 = +1

50
(36TH ICHO, INTERNATIONAL CHEMISTRY OLYMPIAD) Considere que a
oxidação catalítica do CO sobre uma superfície de Pd com igual superfície
ocorre da seguinte forma: na primeira etapa, em um rápido equilíbrio, CO e
O2 adsorvidos formam CO2 adsorvido
CO(ads) + 0,50 O2(ads) ⇌ CO2(ads). Na segunda etapa lenta, CO2 é então
k2
dessorvido da superfície: CO2(ads.) →
⏞ CO2(g). Deduza a fórmula para a
velocidade da reação de formação de CO2(g) - formação em função das
pressões parciais dos componentes da reação.

LIVRO OLÍMPICO – VOLUME I


144

Resposta: Estudo em função do intermediário (X).


dPCO2(ads) 1/2
= + k1 x PCO x PO2 − k −1 x PCO2 − k 2 x PCO2
dt
K1 e k-1 são as velocidades específicas da reação direta e inversa.
1/2
0 = + k1 x PCO x PO2 − k −1 x PCO2 − k 2 x PCO2
1/2
+ k1 x PCO x PO2 = k −1 x PCO2 + k 2 x PCO2
1/2
PCO2 x (k −1 + k 2 ) = + k1 x PCO x PO2
1/2
+ k1 x PCO x PO2
PCO2 =
k −1 + k 2
Equação da velocidade: vreação = k 2 x PCO2

Substituindo a equação do intermediário na equação da velocidade.


1/2
+ k1 x PCO x PO2
vreação = k 2 x
k −1 + k 2
51
(OLIMPÍADA BRASILEIRA DE QUÍMICA) Considerando a reação hipotética
A + B ⇌ C, que ocorre na direção direta, numa única etapa e cujo perfil
energético está representado na figura ao lado. Responda:
a) Que reação é mais rápida no equilíbrio, a direta ou a inversa?
b) O equilíbrio favorece aos produtos ou aos reagentes?
c) Em geral, como um catalisador alteraria o perfil energético dessa
reação?
d) Um catalisador afetaria a razão entre as constantes de velocidade
das reações direta e inversa?
e) Como varia a constante de equilíbrio dessa reação, com a
temperatura? Justifique suas respostas

Resposta:

Item a) Não há reação mais rápida, pois como o processo está em equilíbrio,
suas velocidades ficam iguais.
Item b) Não é possível responder com precisão esta alternativa. Caso fosse
fornecido o valor da constante de equilíbrio ou a variação da energia livre de
gibbs, seria possível responder este item.

LIVRO OLÍMPICO – VOLUME I


145

Item c) O catalisador não modifica em momento algum o perfil energético de


um determinado processo. Para esta reação apresentada, o processo é
endotérmico, ou seja, ΔH > 0.
Item d) Não. A presença do catalisador não afeta a razão das constantes de
velocidade. A constante de velocidade é função apenas da temperatura,
conforme estudado por Arrhenius.
Item e) A constante de equilíbrio varia sim com a temperatura, conforme pode
ser observado a partir da equação de van´t Hoff.

dln K eq ∆H 0
=
dT RT²

52
(OLIMPÍADA NORTE-NORDESTE DE QUÍMICA) Cloreto de sulfurila,
SO2Cl2, se decompõe em fase gasosa, produzindo SO2(g) e Cl2(g). A
concentração do SO2Cl2, foi acompanhada em uma experiência e verificou-
se que o gráfico do ln (logarítmo) de [SO2Cl2] contra o tempo é linear e que,
em 240 segundos, a concentração caiu de 0,400 mol/L para 0,280 mol/L.
a) Qual a constante de velocidade da reação SO2Cl2(g) → SO2(g) +
Cl2(g)?
b) Qual a meia-vida desta reação?

Resposta:

Item a) Como a relação do logaritmo neperiano versus o tempo é linear, a


cinética é de primeira ordem.

[SO2 Cl2 ]Final


ln ( ) = −k x tempo
[SO2 Cl2 ]Inicial

Cálculo da velocidade específica:

0,280
ln ( ) = −k x 240
0,400

−0,35667
k= = 1,49 𝑥 10−3 𝑠 −1
− 240
ln 2 0,693
Item b) Cálculo do tempo de meia-vida: t1/2 = k
= 1,49 x 10−3 =
466,31 s

53
(U.S. NATIONAL CHEMISTRY OLYMPIAD) If a reaction A → B has the rate
law k[A]2, which graph produces a straight line?
a) 1/[A] vs time
b) ln[A] vs time
c) [A]2 vs time
d) 1/ln[A] vs time

Resposta: Alternativa A.

LIVRO OLÍMPICO – VOLUME I


146

Lei da taxa de velocidade: v = k x [A]2 . Esse indicativo afirma que a cinética


é de segunda ordem.

d[A]
− = k x [A]2
dt
1
x d[A] = − k x dt
[A]2

[A]final tfinal
1
∫ 2
x d[A] = − k x ∫ dt
[A]inicial [A] 0
1 [A]final
− | = −k x tempo
[A] [A]
inicial

1 1
= + k. t
[A]final [A]inicial
1
Logo, a relação é [A] x tempo.

54
(U.S. NATIONAL CHEMISTRY OLYMPIAD) A compound decomposes with
a first-order rate constant of 0,00854 s–1. Calculate the concentration after 5,0
minutes for an initial concentration of 1,2 M.
a) 0,010 M
b) 0,093 M
c) 0,92 M
d) 1,1 M

Resposta: Alternativa B.

O problema leva em consideração que a cinética seja de primeira ordem.


Diante disso, temos:

[A]Final
ln ( ) = −k x tempo
[A]Inicial

Cálculo da concentração da quantidade de matéria após cinco minutos.

[A]Final 60
ln ( ) = −0,00854 x 5 min x
1,20 1 min

[A]Final
ln ( ) = −2,562
1,20

[A]Final
= e(−2,562)
1,20

mol
[A]Final = 1,20 x e(−2,562) = 0,0926
L

LIVRO OLÍMPICO – VOLUME I


147

(OLIMPÍADA BRASILEIRA DE QUÍMICA) O conhecimento e o estudo da


velocidade das reações são de grande interesse industrial, pois permitem
reduzir custos e aumentar a produtividade dos processos fabris. Sabe-se que
as reações químicas ocorrem com velocidades diferentes e estas podem ser
alteradas. Para exemplificar, considere a reação abaixo representada: 5 Br-
(aq) + BrO3 (aq) + 6 H (aq) → 3 Br2(aq) + 3 H2O(ℓ). A representação matemática
– +

de velocidade desta reação é: v = k[Br-][BrO3–][H+]2. Assim sendo, para esse


caso, qual afirmação está correta?
a) A ordem geral é 12.
b) Dobrando a concentração de Br- e BrO3– e reduzindo a metade da
concentração de H+ a velocidade de reação não se altera.
c) A unidade da constante de velocidade, k, é mol.dm–3.s–1.
d) Uma alteração na concentração de Br- ou BrO3– não afeta a velocidade de
reação.
e) Dobrando a concentração de Br- ou BrO3– e reduzindo a metade da
concentração de H+ a velocidade de reação não se altera.

Resposta: Alternativa B.

Item a) Alternativa falsa. Ordem global = 1 + 1 + 2 = 4. A ordem de reação


global é o somatório das ordens parciais. Diante disso, a ordem global é igual
a quatro (4).

Item b) Alternativa Verdadeira.

Chamando a concentração da quantidade de matéria de [Br-] = x; [BrO3–] = y


e [H+] = z, temos:

Primeira situação: vreação = k . x . y . z 2


z 2
Segunda situação: vreação = k . 2x . 2y . (2)

vreação = k . x . y . z 2

Item c) Alternativa Falsa.

vreação = k x [Br − ] x [BrO− + 2


3 ] x [H ]

mol mol mol mol 2


= k x x x ( )
dm3 x s dm³ dm³ dm³

(dm³)3
k=
mol3 x s

Item d) Alternativa Falsa.

Variando a concentração da quantidade de matéria de brometo e de bromato,


a velocidade vai modificar sim.

Item e) Alternativa Falsa.

Chamando a concentração da quantidade de matéria de [Br-] = x; [BrO3–] = y


e [H+] = z, temos:
Primeira situação: vreação = k . x . y . z 2

LIVRO OLÍMPICO – VOLUME I


148

Dobrando a concentração da quantidade de matéria de bromato.

z 2
Segunda situação: vreação = k . x . 2y . (2)

1
vreação = . k . x . y . z 2
2

56
(OLIMPÍADA PORTUGUESA DE QUÍMICA) Considere o seguinte
mecanismo proposto para a decomposição do peróxido de hidrogênio:
H2O2(aq) + I-(aq) → H2O(l) + IO-(aq)
H2O2(aq) + IO-(aq) → H2O(l) + I-(aq) + O2(g)
a) Escreva a reação global.
b) Diga qual é o catalisador desta decomposição. Justifique.
c) Quais são os intermediários desta reação? Justifique.

Resposta:

Item a) Somando as equações químicas apresentadas pelo problema:

H2O2(aq) + I-(aq) → H2O(l) + IO-(aq)


H2O2(aq) + IO-(aq) → H2O(l) + I-(aq) + O2(g)
2 H2O2(aq) → 2 H2O(l) + O2(g)

Esta equação corresponde à decomposição do peróxido de hidrogênio.

Item b) O catalisador é o iodeto (I-), uma vez que na primeira equação química
ele é um reagente e na segunda equação química é o produto.

Item c) O intermediário é o hipoiodito (IO-), uma vez que na primeira equação


química ele é um produto e na segunda equação química é um reagente.

57
(U.S. NATIONAL CHEMISTRY OLYMPIAD) One of the steps in the
manufacture of nitric acid is the oxidation of ammonia shown in this equation:
4NH3(g) + 5O2(g) → 4NO2(g) + 6H2O(g). If gaseous water appears at a rate of
0,025 mol·min–1, at what rate does ammonia disappear?
a) 0,0040 mol·min–1
b) 0,017 mol·min–1
c) 0,038 mol·min–1
d) 0,150 mol·min–1

Resposta: Alternativa B.

A relação entre a velocidade da reação com os participantes químicos está


apresentado pela seguinte relação matemática:

1 d [NH3 ] 1 d [O2 ] 1 d [NO3 ]


vreação = − x =− x =+ x
4 dt 5 dt 4 dt
1 d [H2 O]
=+ x
6 ⏟ dt
0,025

LIVRO OLÍMPICO – VOLUME I


149

d [NH3 ] 1
− = + x 0,025 x 4
dt 6

d [NH3 ] mol
− = 0,0167
dt min

58
(INSTITUTO MILITAR DE ENGENHARIA) Em 1889, o químico sueco August
Svante Arrhenius demonstrou que, para uma reação com energia de ativação
constante Ea, a variação da velocidade específica, k, com a temperatura é
expressa pela equação: k = A. e(-Ea/R.T), onde: A é o fator de freqüência, R é a
constante universal dos gases, e o número de neper, que é a base dos
logaritmos neperianos e T é a temperatura kelvin. Certa reação obedece a
uma lei velocidade onde os valores de k são 0,00001 e 0,00010 L.mol–1.s–1, a
312,50 K e 357,14 K, respectivamente. Usando essas informações, calcule:
a) a ordem da reação;
b) a temperatura na qual a reação é 10 vezes mais lenta que a 312,50
K.

Resposta:

Item a) A partir da informação fornecida pelo problema em que a velocidade


𝐿
específica apresenta unidade igual a , a cinética é de segunda ordem.
𝑚𝑜𝑙 𝑥 𝑠

Item b) Cálculo da energia de ativação, aplicando a equação de Arrhenius:


L
k (312,50 K) = 0,00001
mol x s
L
k (357,14 K) = 0,0001
mol x s
k (312,50 K) Eat 1 1
ln ( )= x[ − ]
k (357,14 K) R 357,14 312,50
0,00001 Eat 1 1
ln ( )= x[ − ]
0,0001 8,314 357,14 312,50
𝐽
Eat = 47861,91
𝑚𝑜𝑙
Determinação da temperatura (T):
L
k (312,50 K) = 0,00001
mol x s
L
k (T) = 0,000001
mol x s
k (312,50 K) Eat 1 1
ln ( )= x[ − ]
k (T) R T 312,50
0,00001 47861,91 1 1
ln ( )= x[ − ]
0,000001 8,314 T 312,50
T = 277,78 K

LIVRO OLÍMPICO – VOLUME I


150

59
(OLIMPÍADA ARGENTINA DE QUÍMICA)
II. A decomposição do N2O5 (em NO2 e O2) é uma reação de primeira
ordem e a 35°C o valor de sua constante de velocidade (k) é 0,0086 min-1.
a) Calcule o tempo de meia-vida.
b) Se partirmos de 4 mol de N2O5 e foram transcorridos 321,6 minutos
desde o início da reação de decomposição, calcule a quantidade de N2O5 que
fica sem se decompor ao fim desse período de tempo.
III. Por sua vez, a decomposição do NO2 pode ser representada pela
equação: 2 NO2(g) → 2 NO(g) + O2(g) , e para esta, k = 4,87 x 10-3 mol-1.L.s-1 a
65°C e a energia de ativação tem um valor de 1,039 x 105 J.mol-1. Tendo em
conta esta informação:
c) Indique qual a ordem total da reação de decomposição do NO2.
d) Calcule a constante de velocidade da reação a 100°C.

Resposta: Equação química: N2O5(g) → 2 NO2(g) + ½ O2(g)

Item a) Sabendo que a cinética para este processo de decomposição é de


primeira ordem, temos:
ln 2 0,693
t1/2 = = = 80,58 min
k 0,0086 min−1
Item b)
[N2 O5 ]Final
ln ( ) = −k x t
[N2 O5 ]Inicial

[N2 O5 ]Final
ln ( ) = −0,0086 x 321,6
4,0 mol

[N2 O5 ]Final = 4,0 x e−2,76 = 0,252 mol

Item c) A partir da informação fornecida pelo problema em que a velocidade


𝐿
específica apresenta unidade igual a 𝑚𝑜𝑙 𝑥 𝑠, a cinética é de segunda ordem.

Item d) Cálculo da constante de velocidade à T = 100ºC:


L
k (65°C) = 4,87 x 10−3
mol x s
k (100°C) =?
J
Eat = 1,039 x 10+5
mol
k (65°C) Eat 1
Aplicando a equação de Arrhenius, temos: ln (k (100°C)) = R
x [373 −
1
338
]

4,87 x 10−3 1,039 x 10+5 1 1


ln ( )= x[ − ]
k (100°C) 8,314 373 338

4,87 x 10−3
= 𝑒 −3,47
k (100°C)

LIVRO OLÍMPICO – VOLUME I


151

4,87 x 10−3 L
k (100°C) = −2
= 0,156
3,11 𝑥 10 mol x s
60
(U.S. NATIONAL CHEMISTRY OLYMPIAD) Gaseous dinitrogen pentoxide,
N2O5, decomposes to form nitrogen dioxide and oxygen gas with the initial
rate data at 25˚C given in the table.
[N2O5] (mol.L-1) Rate (mol.L-1min-1)
0,150 3,42 x 10-4
0,350 7,98 x 10-4
0,650 1,48 x 10-3

a) Write a balanced equation for this reaction.


b) Use the data provided to write the rate law and calculate the value of k for
this reaction. Show all calculations.
c) Calculate the time required for the concentration of a 0,150 mol.L-1 sample
of N2O5 to decrease to 0,050 mol.L-1.
d) The initial rate for the reaction of a 0,150 mol.L-1 sample is 2,37×10–3 mol.L–
1.min–1 at 40˚C. Determine the activation energy for this reaction.

Resposta:
Item a) Equação química que representa a decomposição do pentóxido de
dinitrogênio:
1
N2 O5(g) → 2 NO2(g) + O2(g)
2
Item b) Determinação da ordem de reação em reação ao pentóxido de
dinitrogênio:

0,350 β 7,98 x 10−4


( ) =
0,150 3,42 x 10−4
(2,33)β = 2,33

β=1
Cálculo do valor da velocidade específica (k), a partir do experimento I:
vreação = k x [N2 O5 ]

3,42 x 10−4 = k x 0,150

0,00228
k=
min
[N O ]
Item c) Cálculo do tempo: ln ([N 2O 5] Final ) = −k x t
2 5 Inicial

0,050
ln ( ) = −0,00228 x tempo
0,150
tempo = 481,85 min

Item d) Aplicando a equação de Arrhenius, temos:

LIVRO OLÍMPICO – VOLUME I


152

k (25°C) Eat 1 1
ln ( )= x[ − ]
k (40°C) R 318 298

2,28 x 10−3 Eat 1 1


ln ( −3 )= x[ − ]
2,37 x 10 8,314 318 298

2,28 x 10−3 Eat 1 1


ln ( −3 )= x[ − ]
2,37 x 10 8,314 318 298
J
Eat = 1525,10
mol
61
(U.S. NATIONAL CHEMISTRY OLYMPIAD) At a given temperature a first-
order reaction has a rate constant of 3,33 x 10-3 s-1. How much time is required
for the reaction to be 75% complete?
a) 100 s
b) 210 s
c) 420 s
d) 630 s

Resposta: Alternativa C.
[A]
Cálculo do tempo: ln ([A] Final ) = −k x t
Inicial

Informação do problema: [A]Final = 0,25 x [A]Inicial


0,25 x [A]Inicial
ln ( ) = −0,00333 x tempo
[A]Inicial

ln(0,25) = −0,00333 x tempo


tempo = 416,30 s

62
(PROGRAMA OLIMPÍADA URUGUAYA DE QUÍMICA) En la siguiente
figuraI se muestra el perfil de la reacción A → C.

En el diagrama, señala cuál es la energía de activación de la reacción.


Además, responde a las siguientes preguntas:
a) Cuántos intermediarios de reacción y cuántos estados de transición hay?
b) Cuál es el paso rápido y cuál es el lento?
c) La reacción es exotérmica o endotérmica?

LIVRO OLÍMPICO – VOLUME I


153

Resposta:

Item a) Número de intermediários e de estados de transição estão indicados


no diagrama a seguir.

Item b) A etapa mais rápida e a mais lenta está representada no diagrama a


seguir.

Item c) O processo é exotérmico, conforme pode ser observado no diagrama


a seguir.
.

63
(OLIMPÍADA PORTUGUESA DE QUÍMICA) A cinética da reação de
hidrogenação de benzotiofeno (BT) a 2,3 – dihidrobenzotiofeno (DHBT),
utilizando como catalisador um complexo de ródio, foi realizada a diferentes
temperaturas, variando as concentraçõs dos reagentes e do catalisador. As
velocidades iniciais de reação foram determinadas para cada caso, obtendo-
se a seguinte tabela:
[catalisador] [BT] (mol.dm- [H2] (mol.dm- T v (mol.dm-3s-
(mol.dm-3) 3) 3) (ºC) 1)

LIVRO OLÍMPICO – VOLUME I


154

6,0 x 10-4 5,0 x 10-2 2,3 x 10-3 125 10,3 x 10-7


8,0 x 10-4 5,0 x 10-2 2,3 x 10-3 125 13,7 x 10-7
6,0 x 10-4 5,0 x 10-2 3,0 x 10-3 125 13,6 x 10-7
6,0 x 10-4 1,0 x 10-2 2,3 x 10-3 125 10,3 x 10-7
6,0 x 10-4 5,0 x 10-2 2,3 x 10-3 110 3,7 x 10-7

a) Determine a ordem da reação em relação às concentrações de


catalisador, BT e H2 (as ordens de reação devem ser números inteiros).
b) Estabeleça a lei de velocidade experimental para esta reação.
c) Calcule as constantes de velocidade para as temperaturas de 125ºC
e 110ºC.

Resposta:

Item a) Determinação da ordem de reação em relação ao catalisador, entre o


experimento 1 e 2:
α
8,0 x 10−4 13,7 x 10−7
( ) =
6,0 x 10−4 10,3 x 10−7

(1,33)α = 1,33

α=1
Determinação da ordem de reação em relação ao BT, entre o experimento 1
e 4:
β
1,0 x 10−2 10,3 x 10−7
( ) =
5,0 x 10−2 10,3 x 10−7

(1,33)β = 1

β=0
Determinação da ordem de reação em relação ao H2, entre o experimento 1
e 3:
γ
3,0 x 10−3 13,6 x 10−7
( ) =
2,3 x 10−3 10,3 x 10−7

(1,31)γ = 1,31

γ=1
Item b) Equação da velocidade:
vreação = k x [catalisador]1 x [BT]0 x [H2 ]1

Cálculo da velocidade específica à T = 125ºC, a partir do experimento 1:

10,3 x 10−7 = k x 6,0 x 10−4 x 2,3 x 10−3


10,3 x 10−7 dm³
k = 6,0 x 10−4 x 2,3 x 10−3 = 0,746 mol x s

Item c) Cálculo da velocidade específica à T = 110ºC, a partir do


experimento 5:

LIVRO OLÍMPICO – VOLUME I


155

3,7 x 10−7 = k x 6,0 x 10−4 x 2,3 x 10−3


3,7 x 10−7 dm³
k = 6,0 x 10−4 x 2,3 x 10−3 = 0,268 mol x s

64
(OLIMPÍADA PORTUGUESA DE QUÍMICA) Um estudante de doutoramento
foi incubido de testar as propriedades catalíticas do novo material, designado
por AM-53, na redução de NO (gás nocivo, formado, por exemplo, nos
motores de combustão). A reação que se pretende catalisar é: 2 NO(g) → O2(g)
+ N2(g). Na presença de AM-53, a concentração inicial de NO reduz-se de 0,43
mol x dm-3 a 0,09 mol x dm-3 em 120 segundos. No ensaio de controle (sem
AM-53), ao fim do mesmo tempo, a concentração de N2 formado era de 0,17
mol x dm-3.
a) A partir destes resultados, diga, justificando, se o AM-53 é um bom
catalisador para esta reação.
b) Se a concentração inicial de N2 fosse diferente de zero, qual a
variação mais provável observada na velocidade média desta reação (em
qualquer dos ensaios)? Justifique.
c) Qual o efeito previsível do aumento da temperatura na velocidade
desta reação? Justifique.

Resposta:
Item a) Para analisar se o catalisador AM-53 apresenta bom rendimento, será
necessário determinar a velocidade do óxido neutro e do gás nitrogênio.

1 |0,09−0,43| mol
Para o óxido neutro: vmédia = − 2 𝑥 120
= − 0,00142 dm3 x s

0,17 mol
Para o gás nitrogênio: vmédia = + 120 = + 0,00142 dm3 x s

Observando o cálculo da velocidade média para ambos os gases, observa-


se que a velocidade é igual, tanto a de consumo quanto a de formação. Diante
disso, o AM-53 não é um bom catalisador para este processo.

Item b) A partir da lei de Guldberg-Waage, a velocidade de uma reação


química é diretamente proporcional à concentração da quantidade de matéria
dos reagentes. Diante desta afirmativa, a concentração da quantidade de
matéria de gás nitrogênio não influencia na reação química.

Item c) Quanto maior a temperatura, maior será a probabilidade de encontro


dos reagentes químicos, o que consequentemente aumenta a probabilidade
de choques efetivos. Isso tudo diante do aumento da velocidade da reação.

65
(OLIMPÍADA PERUANA DE QUÍMICA) O N2O5 se decompõe segundo a
seguinte equação química: 2 N2O5(g) → 4 NO2(g) + O2(g), mediante uma
cinética de primeira ordem, sendo sua constante de velocidade de 8,5 x 10 -3
s-1. Se a pressão inicial do N2O5(g) é de 240 mmHg, qual é a pressão depois
de 5 minutos?
a) 12,7 mmHg
b) 18,7 mmHg
c) 25,5 mmHg
d ) 36,6 mmHg

LIVRO OLÍMPICO – VOLUME I


156

Resposta: Alternativa B.

Equação química: 2 N2O5(g) → 4 NO2(g) + O2(g)


P
A partir da equação cinética de primeira ordem: ln ( final ) = −k x tempo
Pinicial
Pfinal
ln ( ) = −k x tempo
240 mmHg

Pfinal 60 s
ln ( ) = −8,50 x 10−3 x 5 min x
240 mmHg 1 min

Pfinal = 18,74 mmHg

66
(OLIMPÍADA PERUANA DE QUÍMICA) Determine como verdadeiro (V) ou
falso (F) as seguintes afirmações:
I) A constante de velocidade, de qualquer reação química pode ser expressa
em mol-1∙L∙s-1;
II) As unidades da constante k, depende da reação ou da lei de velocidade;
III) A constante de velocidade sempre é o mesmo valor durante toda a reação;
IV) O valor da constante k é independente da temperatura a que a reação se
realiza;
V) A ordem total da reação é sempre igual a soma dos coeficientes
estequiométricos dos reagentes.
VI) A ordem total de qualquer reação química é sempre um número inteiro.
a) VVVFFF
b) FVFVFV
c) FFVVVF
d) FVVFFF

Resposta: Alternativa D.

I) Falso. A constante de velocidade depende da ordem de reação


determinada.
II) Verdadeiro.
III) Verdadeiro.
IV) Falso. Segundo o sestudo de Svante Arrhenius, a constante de velocidade
é dependente da temperatura.
V) Falso. Caso a reação seja elementar a afirmativa é verdadeira. Caso seja
uma reação não – elementar, a afirmativa torna-se falsa.
VI) Falso. A ordem de reação pode sim ser um número fracionário.

LIVRO OLÍMPICO – VOLUME I


157

67
(INSTITUTO MILITAR DE ENGENHARIA) À temperatura de 147°C, a
decomposição do peróxido de diterbutila, em fase gasosa, obedece à
equação: (CH3)3COOC(CH3)3 → 2 CH3COCH3 + C2H6. O estudo cinético
dessa reação pela medida da pressão total da mistura em função do tempo,
a volume constante, mostrou que a mesma é de 1ª ordem. Com base na
tabela abaixo, calcular a velocidade média da reação em relação de di-t-
butila, no intervalo de 0 a 10 minutos, em mol.L –1.min –1 .
T(min) 0 6 10 14 22
P(mmHg) 180 200 210 220 240
Dado: Constante dos gases ideais (R) = 62,50 mmHg.L / mol.K.

Resposta: Para a resolução deste problema será necessário utilizar a tabela


de equilíbrio químico, uma vez que a tabela fornecida pelo problema está em
função da pressão total. Diante disso, será necessário calcular e formular uma
nova tabela em função da pressão parcial do reagente A.

Equação química: (CH3)3COOC(CH3)3 (A) → 2 CH3COCH3 (2B) + C2H6 (C)


Base de cálculo: p0 para a pressão inicial do peróxido de diterbutila.

Equação química A → 2B + C
Início p0 0 0
Reage p 2p p
Equilíbrio p0-p 2p p

Logo no equilíbrio a pressão total será a seguinte: ptotal = p0 − p + 2p +


p
ptotal = p0 + 2p
Na tabela fornecida pelo problema, a pressão inicial é igual a 180 mmHg (p0).
ptotal = 180 + 2p
Cálculo da pressão p no estado de equilíbrio químico:
Para o tempo 0 minutos (Pressão total = 180 mmHg): 180 = 180 + 2p
p=0
Para o tempo 6 minutos (Pressão total = 200 mmHg): 200 = 180 + 2p
p = 10 mmHg
Para o tempo 10 minutos (Pressão total = 210 mmHg): 210 = 180 + 2p
p = 15 mmHg
Para o tempo 14 minutos (Pressão total = 220 mmHg): 220 = 180 + 2p
p = 20 mmHg
Para o tempo 22 minutos (Pressão total = 240 mmHg): 240 = 180 + 2p
p = 30 mmHg

LIVRO OLÍMPICO – VOLUME I


158

Diante de todas essas informações, será possível determinar e organizar uma


nova tabela para a determinação da pressão parcial de A.

T(min) 0 6 10 14 22
Ptotal 180 200 210 220 240
PA = p0-p 180-0 180-10 180-15 180-20 180-30
PA 180 mmHg 170 mmHg 165 mmHg 160 mmHg 150 mmHg

Cálculo da velocidade média do reagente:

∆nA nfinal
A − ninicial
A
vditerbutila = =
∆t t final − t inicial

Pfinal x V Pinicial x V
( AR x T ) − ( A R x T )
vditerbutila = −
10 min − 0 min
165 x V 180 x V
( )−( )
62,50 x 420 62,50 x 420
vditerbutila =
10 min − 0 min
Como o sistema afirma que o volume é constante (V = 1,0 L), temos:
165 x 1 180 x 1
( )−( )
62,50 x 420 62,50 x 420
vditerbutila =
10 min − 0 min
mol
vditerbutila = −5,71 x 10−5
min
O sinal negativo é apenas um indicativo que o reagente está sendo
consumido no processo cinético.
68
(ITT-JEE) The gaseous reaction A(g) → 2B(g) + C(g) is found to be first-order
with respect to A. If the reaction is started with pA = 90 mmHg, the pressure
after 10 min is found to be 180 mmHg. The rate constant of the reaction is:
a) 1,15 x 10–3 s–1
b) 2,30 x 10–3 s–1
c) 3,45 x 10–3 s–1
d) 4,60 x 10–3 s–1

Resposta: Alternativa A.
Para a resolução deste problema será necessário utilizar a tabela de equilíbrio
químico.
Equação química: A(g) → 2B(g) + C(g)
Base de cálculo: p0 para a pressão inicial do peróxido de diterbutila.

Equação química A → 2B + C
Início 90 mmHg 0 0
Reage X 2X X
Equilíbrio 90-X 2X X

LIVRO OLÍMPICO – VOLUME I


159

Logo no equilíbrio a pressão total será a seguinte: ptotal = 90 − X + 2X +


X
Sabendo que pressão total é igual a 180 mmHg, temos:
ptotal = 90 − X + 2X + X
Na tabela fornecida pelo problema, a pressão inicial é igual a 180 mmHg (p0).
180 = 90 + 2X
X = 45 mmHg
Cálculo da velocidade específica, sabendo que a cinética é de primeira
ordem:
Pfinal
ln ( ) = −k x tempo
Pinicial

45 𝑚𝑚𝐻𝑔
ln ( ) = −k x (10 𝑥 60)
90 mmHg

− ln 0,50
k= = 0,00115 𝑠 −1
600

69
(OLIMPÍADA NORTE-NORDESTE DE QUÍMICA) Dado o gráfico abaixo, que
representa a energia, em Kcal/mol, posta em jogo no decorrer de uma reação
química:

a) Indique a grandeza que deve ser representada no eixo dos "x";


b) Indique a grandeza que deve ser representada no eixo dos "y";
c) Classifique a reação, segundo a variação de energia ocorrida no
processo;
d) Indique o efeito da adição de um catalisador, no perfil do gráfico;
e) Calcule a energia de ativação da reação "direta";
f) Calcule a variação de entalpia (DH) nesta reação.

Resposta:

Item a) Caminho da reação, podendo ser representado por uma equação


química na forma: A(g) + B(g) → C(g) + D(g).

Item b) Função de estado entalpia (dH)

LIVRO OLÍMPICO – VOLUME I


160

Item c) Processo exotérmico, uma vez que a entalpia dos reagentes é maior
que a entalpia dos produtos, Hreagentes > Hprodutos.

Item d) No perfil do gráfico, a adição de um catalisador, substância que


aumenta a velocidade de uma reação química, diminui a energia de ativação.

Item e) Eat = 35 kJ – 20 kJ = + 15 kJ

Item f) Cálculo da entalpia do processo químico: ΔH = Hprodutos - Hreagentes

ΔH = 5 – 20 = - 15 kJ

70
(GRILLO) Considere a seguinte reação de síntese da sacarose com a
presença da água no estado líquido, conforme pode ser observado pela
equação química não balaneada, C12H22O11(aq) + H2O(l) → C6H12O6(aq). A
cinética química desta reação foi estudada e os dados obtidos foram
colocados em uma tabela pelos meus alunos da Qim-241 do curso técnico de
Química - IFRJ – Campus Nilópolis.

Tempo (minutos) [SACAROSE] (mol.L-1)


0 0,316
39 0,274
80 0,238
140 0,190
210 0,146

a) Prove que estes dados referem-se a uma cinética de primeira ordem.


b) Calcule a constante cinética.

Resposta:

Item a) Calculando o logaritmo neperiano para cada concentração da


sacarose, a tabela fica da seguinte maneira:

Tempo (minutos) [C12H22O11] (mol.L-1) ln [C12H22O11]


0 0,316 1,152
39 0,274 1,295
80 0,238 1,435
140 0,190 1,661
210 0,146 1,924

Gráfco plotado que relaciona ln(sacarose) versus o tempo.

LIVRO OLÍMPICO – VOLUME I


161

-1.1

-1.2

-1.3

-1.4

Ln [sacarose]
-1.5

-1.6

-1.7

-1.8

-1.9

-2.0
0 50 100 150 200 250

Tempo (minutos)

A partir dos dados fornecidos, como a relação ln [sacarose] versus o tempo é


uma reta, logo a reação química apresenta cinética de primeira ordem.

Item b) Cálculo da velocidade específica: ln[sacarose]final −


ln[sacarose]inicial = − k x tempo

− 1,924 − (−1,152) = − k x 210

k = 0,00368 min−1

71
(OLIMPÍADA PIAUIENSE DE QUÍMICA) Para a decomposição do ácido
iodídrico (HI) sobre uma superfície de platina, a energia de ativação tem valor
de 59 kJ/mol entre as temperaturas de 400 e 500 K. Qual a relação entre as
velocidades a 500 K e a 400 K, desta reação catalisada?

Resposta: Equação química da decomposição do iodeto de hidrogênio:

2 HI(g) → H2(g) + I2(g)

Para a resolução deste problema, será necessário utilizar a equação de


Arrhenius.

k (500 K) Eat 1 1
ln ( )= x[ − ]
k (400 K) R 400 500

k (500 K)
ln ( ) = 7096,46 x (0,0005)
k (400 K)

k (500 K)
= 𝑒 +3,55 = 34,81
k (400 K)

LIVRO OLÍMPICO – VOLUME I


162

72
(ITT-JEE) At 380ºC, the half-life period for the first order decomposition of
H2O2 is 360 min. The energy of activation of the reaction is 200 kJ/mol.
Calculate the time required for 75% decomposition at 450ºC.
Resposta: Equação química da decomposição do peróxido de hidrogênio:
1
H2 O2(aq) → H2 O(l) + O2(g)
2
Cálculo da velocidade específica à 380ºC:
0,693 1
k 3800 C = 360
= 0,001925 min

Cálculo da velocidade específica à 450ºC:


0,001925 200000 1 1
ln ( )= x[ − ]
k 4500 C 8,314 450 + 273 380 + 273
0,001925
ln ( ) = −3,57
k 4500 C
0,001925
= e(−3,57)
k 4500 C
0,001925
k 4500 C = = 0,0684 min−1
0,02815
Informação do problema: [H2 O2(aq) ] = 0,25 x [H2 O2(aq) ]
final inicial

[H2 O2(aq) ]final


ln ( ) = −k x tempo
[H2 O2(aq) ]inicial

0,25 𝑥 [H2 O2(aq) ]


inicial
ln ( ) = −0,0684 x t
[H2 O2(aq) ]inicial

ln 0,25
t= = 20,27 min
−0,0684

73
(ITT-JEE) The rate constant of a reaction is 1,50 x 107 s-1 at 50ºC and 1,50 x
107 s-1 at 100ºC. Evaluate the Arrhenius parameters A and Eat.
Resposta: Cálculo da energia de ativação, a partir da equação de Arrhenius:
1,50 x 10+7 Eat 1 1
ln ( +7 )= x[ − ]
4,50 x 10 8,314 100 + 273 50 + 273
1,50
ln ( ) x 8,314 J
4,50
Eat = −4
= 22009
− 4,15 x 10 mol
Cálculo do valor do fator de frequência à 100ºC:

LIVRO OLÍMPICO – VOLUME I


163

Eat
k = A x e− R x T
k 4,50 x 10+7
A= Eat = 22009

e− R x T e 8,314 x (100+273)

4,50 x 10+7 4,50 x 10+7


A= 22009 = = 5,44 x 1010 s−1

8,314 x (100+273)
8,27 x 10−4
e
74
(ITT-JEE) The half-life of a radioisotope is four hours. If the initial masso f the
isotope was 200 g, the mass remaining after 24 hours undecayed is:
a) 3,125 g
b) 2,084 g
c) 1,042 g
d) 4,167 g

Resposta: Alternativa A.
ln 2
Cálculo da constante radioativa: t1/2 =
λ

ln 2 −1
λ= h
4
Cálculo da massa final, após 24 horas:

mfinal
ln ( ) = −λ x tempo
minicial

mfinal ln 2
ln ( )=− x 24
200 4

mfinal = 3,125 g

75
(ITT-JEE) The time required for 10% completion of a first order reaction at 298
K is equal to that required for its 25% completion at 308K. If the pre-
exponential factor for the reaction is 3,56 x 109 s-1, calculate its rate constant
at 318 K, and also the energy of activation.
Resposta: Primeiro passo para a resolução do problema:
[A]inicial = 0,90 x [A]inicial
[𝐴]final
ln ( ) = −k 298 x tempo
[A]inicial

0,90 𝑥 [𝐴]inicial
ln ( ) = −0,0684 x t
[A]inicial

ln 0,90
tempo =
−k 298

Segundo passo para a resolução do problema:

LIVRO OLÍMPICO – VOLUME I


164

[A]inicial = 0,75 x [A]inicial


[A]final
ln ( ) = −k 308 x tempo
[A]inicial

0,75 x [A]inicial
ln ( ) = −0,0684 x t
[A]inicial

ln 0,75
tempo =
−k 308
ln 0,90 ln 0,75
Igualando os tempos: −k = −k308
298

k 308 ln 0,75
=
k 298 ln 0,90
k 308
= 2,73
k 298
k308 Eat 1 1
Cálculo da energia de ativação: ln ( ) = x[ − ]
k298 8,314 298 308

ln(2,73)x 8,314 J
Eat = = 76637,47
1 1 mol
[298 − 308]
Eat
Cálculo da velocidade específica à 318 K: k = A x e−R x T
76637,47

k = 3,56 x 10+9 x e 8,314 x 318

k = 9,17 x 10−4 s−1

LIVRO OLÍMPICO – VOLUME I


165

CAPÍTULO 03

TERMODINÂMICA -
EXERCÍCIOS

ALEXANDRE
VARGAS GRILLO

LIVRO OLÍMPICO – VOLUME I


166

1. DEDUÇÃO DA ENTALPIA A PARTIR DA PRIMEIRA LEI DA


TERMODINÂMICA
2. DEDUÇÃO DA EXPANSÃO ADIABÁTICA REVERSÍVEL
3. DEDUÇÃO DA EXPANSÃO ADIABÁTICA REVERSÍVEL,
RELACIONANDO A PRESSÃO COM O VOLUME
4. OLIMPÍADA BRASILEIRA DE QUÍMICA
5. OLIMPÍADA BRASILEIRA DE QUÍMICA
6. OLIMPÍADA BRASILEIRA DE QUÍMICA
7. OLIMPÍADA BRASILEIRA DE QUÍMICA
8. GRILLO (QUESTÃO DE PRÓPRIA AUTORIA)
9. OLIMPÍADA BRASILEIRA DE QUÍMICA
10. OLIMPÍADA BRASILEIRA DE QUÍMICA
11. OLIMPÍADA BRASILEIRA DE QUÍMICA
12. OLIMPÍADA BRASILEIRA DE QUÍMICA
13. OLIMPÍADA BRASILEIRA DE QUÍMICA
14. OLIMPÍADA ARGENTINA DE QUÍMICA
15. U. S. NATIONAL CHEMISTRY OLYMPIAD
16. GRILLO (QUESTÃO DE PRÓPRIA AUTORIA)
17. INSTITUTO MILITAR DE ENGENHARIA
18. INSTITUTO MILITAR DE ENGENHARIA
19. U. S. NATIONAL CHEMISTRY OLYMPIAD
20. GRILLO (QUESTÃO DE PRÓPRIA AUTORIA)
21. OLIMPÍADA BRASILEIRA DE QUÍMICA
22. PROGRAMA OLIMPÍADA URUGUAYA DE QUÍMICA
23. OLIMPÍADA PIAUIENSE DE QUÍMICA
24. OLIMPÍADA PIAUIENSE DE QUÍMICA
25. OLIMPÍADA MINEIRA DE QUÍMICA
26. OLIMPÍADA MINEIRA DE QUÍMICA
27. OLIMPÍADA DE QUÍMICA DO RIO DE JANEIRO
28. OLIMPÍADA DE QUÍMICA DO RIO DE JANEIRO
29. RELAÇÃO DA VARIAÇÃO DA ENERGIA LIVRE DE GIBBS COM
O QUOCIENTE REACIONAL
30. OLIMPÍADA BRASILEIRA DE QUÍMICA
31. U. S. NATIONAL CHEMISTRY OLYMPIAD
32. OLIMPÍADA DE QUÍMICA DO RIO DE JANEIRO
33. OLIMPÍADA DE QUÍMICA DO RIO DE JANEIRO
34. 29TH ICHO, INTERNATIONAL CHEMISTRY OLYMPIAD
35. OLIMPÍADA BRASILEIRA DE QUÍMICA
36. OLIMPÍADA BRASILEIRA DE QUÍMICA
37. OLIMPÍADA PORTUGUESA DE QUÍMICA
38. OLIMPÍADA PORTUGUESA DE QUÍMICA
39. OLIMPÍADA BRASILEIRA DE QUÍMICA
40. OLIMPÍADA BRASILEIRA DE QUÍMICA
41. OLIMPÍADA BRASILEIRA DE QUÍMICA
42. GRILLO (QUESTÃO DE PRÓPRIA AUTORIA)
43. OLIMPÍADA NORTE-NORDESTE DE QUÍMICA
44. U. S. NATIONAL CHEMISTRY OLYMPIAD
45. U. S. NATIONAL CHEMISTRY OLYMPIAD
46. OLIMPÍADA PORTUGUESA DE QUÍMICA
47. OLIMPÍADA NORTE-NORDESTE DE QUÍMICA
48. GRILLO (QUESTÃO DE PRÓPRIA AUTORIA)
49. OLIMPÍADA BRASILEIRA DE QUÍMICA
50. U. S. NATIONAL CHEMISTRY OLYMPIAD
51. U. S. NATIONAL CHEMISTRY OLYMPIAD
52. OLIMPÍADA BRASILEIRA DE QUÍMICA

LIVRO OLÍMPICO – VOLUME I


167

53. INSTITUTO MILITAR DE ENGENHARIA


54. INSTITUTO TECNOLÓGICO DA AERONÁUTICA – ADAPTADO
55. GRILLO (QUESTÃO DE PRÓPRIA AUTORIA)
56. CONCURSO PARA DOCENTE – IFRJ
57. SELETIVA PARA A 37TH INTERNATIONAL CHEMISTRY
OLIMPIAD
58. INSTITUTO MILITAR DE ENGENHARIA
59. INSTITUTO MILITAR DE ENGENHARIA
60. INSTITUTO MILITAR DE ENGENHARIA
61. OLIMPÍADA BRASILEIRA DE QUÍMICA
62. GRILLO (QUESTÃO DE PRÓPRIA AUTORIA)

LIVRO OLÍMPICO – VOLUME I


168

1
Dedução da entalpia a partir da primeira lei da termodinâmica.

Resposta: A entalpia é uma função de estado, ou seja, a entalpia só está


relacionada com o estado inicial e final de um processo termodinâmico. A
função entalpia calcula a quantidade de calor inicial e final em processos
químicos, representado por reação químicas. Levando em consideração dois
pontos, A (estado inicial) e B (estado final), a partir da primeira lei da
termodinâmica temos:
∆U = Q + W
∆U = Q − p∆V
UB − UA = Q − p(VB − VA )
(UA + pVA ) = Q
UB + pVB − ⏟

HB HA

∆H = Q p

Esta relação termodinâmica afirma que a entalpia calcula a quantidade de


calor à pressão constante.
Estudando de forma infinita, temos: dH = n x Cp 𝑥 𝑑𝑇

Para 1 mol de um gás ideal.


dH
Cp = ( dT ) . Esta relação termodinâmica afirma que a função de estado
p
entalpia é função da temperatura.
2
Dedução da expansão adiabática reversível.
Resposta: A partir da primeira lei da termodinâmica e sabendo que o processo
termodinâmico é caracterizado por ser adiabático, pelo fato de não haver
troca de calor entre o sistema e a vizinhança, temos a seguinte relação:
∆U = Q + W
∆U = 0 + W
∆U = W
n x CV x dT = −p x dV
Realizando a separação das variáveis e considerando que os gases
apresentam comportamento ideal, temos:
nxRxT
n x CV x dT = − x dV
V
1 R 1
x dT = − x x dV
T CV V
Aplicando a integral, temos:

LIVRO OLÍMPICO – VOLUME I


169

T2 V2
1 R 1
∫ x dT = − x∫ x dV
T1 T CV V1 V

R
ln T2 − ln T1 = − x ln V2 − ln V1
CV
R
ln T2 − ln T1 = − x ln V2 − ln V1
CV

T2 R V2
ln =− x ln
T1 CV V1

A partir da relação entre as capacidades caloríficas, temos: Cp − CV = R


C C R
Dividindo a equação por CV , temos: Cp − CV = C
V V V

Cp R
−1=
CV CV
C C R
Chamando a relação Cp = γ, temos: Cp − 1 = C
V ⏟V V
𝛾

R
𝛾−1=
CV
T R V
Substituindo essa última relação na equação ln T2 = − C x ln V2 , temos:
1 V 1

T2 V2
ln = −(𝛾 − 1) x ln
T1 V1

T2 V2 −(𝛾−1)
ln = ln ( )
T1 V1

T2 V1 (𝛾−1)
ln = ln ( )
T1 V2
𝛾−1 𝛾−1
T1 𝑥 V1 = T2 𝑥 V2

3
Dedução da expansão adiabática reversível, relacionando a pressão com o
volume.
Resposta: A partir da relação termodinâmica apresentada no exercício 2 e
considerando que os gases apresentam comportamento ideal, temos:
𝛾−1 𝛾−1
T1 𝑥 V1 = T2 𝑥 V2

p1 x V1 γ−1 p2 x V2 γ−1
x V1 = x V2
nxR nxR
p1 x V1 𝛾−1 p2 x V2 𝛾−1
𝑥 V1 = 𝑥 V2
nxR nxR

LIVRO OLÍMPICO – VOLUME I


170

𝛾 𝛾
p1 x V1 𝑥 V1 𝑥 V1−1 = p2 x V2 𝑥 V2 𝑥 V2−1
𝛾 𝛾
p1 x V1 = p2 x V2

4
(OLIMPÍADA BRASILEIRA DE QUÍMICA) A partir das entalpias das reações
dadas abaixo:
2 C(grafite) + 2 H2(g) → C2H4(g) ΔHº = + 52,0 kJ
C2H4Cl2(g) → Cl2(g) + C2H4(g) ΔHº = + 116,0 kJ
Podemos concluir que a entalpia molar de formação (em kJ/mol) do C2H4Cl2(g),
será igual a:
a) -64 kJ/mol
b) + 64 kJ/mol
c) -168 kJ/mol
d) + 168 kJ/mol
e) + 220 kJ/mol

Resposta: Alternativa A.

Para a resolução deste problema, será necessário utilizar a lei de Hess.


Diante disso, será necessário inverter a segunda equação química
termodinâmica. Diante disso:

2 C(grafite) + 2 H2(g) → C2H4(g) ΔHº = + 52,0 kJ


Cl2(g) + C2H4(g) → C2H4Cl2(g) ΔHº = - 116,0 kJ

2 C(grafite) + 2 H2(g) + Cl2(g) → C2H4Cl2(g)


ΔHº = + 52,0 kJ + (- 116,0 kJ)
ΔHº = - 64,0 kJ
5
(OLIMPÍADA BRASILEIRA DE QUÍMICA) O acetileno ou etino (C2H2) é um
gás de grande uso comercial, sobretudo em maçaricos de oficinas de
lanternagem. Assinale a opção que corresponde à quantidade de calor
liberada pela combustão de 1 mol de acetileno, à 25ºC, de acordo com a
reação abaixo: 2 C2H2(g) + 5 O2(g) → 4 CO2(g) + 2 H2O(g). Dados: ΔHºf: C2H2(g)
= + 227 kJ/mol; CO2(g) = - 394 kJ/mol; H2O(g) = - 242 kJ/mol.
a) 204 kJ
b) 409 kJ
c) 863 kJ
d) 1257 kJ
e) 2514 kJ

Resposta: Alternativa D.

Para a resolução deste problema, será necessário calcular a variação da


entalpia de combustão do gás acetileno. Diante disso, temos:
0 0
0
∆Hreação = 4 x HCO 2
+ 2 x HH 2O
− 2 x HC02 H2 − 5 x HO0 2

0
∆Hreação = 4 x (−394) + 2 x (−242) − 2 x (227) − 5 x 0

LIVRO OLÍMPICO – VOLUME I


171

0
∆Hreação = 4 x (−394) + 2 x (−242) − 2 x (227) − 5 x 0

0
∆Hreação = −2514 kJ

Este valor corresponde para 2 mol. Logo para 1 mol, temos:


0
∆Hreação −2514 kJ
= = −1257 kJ, para 1 mol.
2 mol 2 mol

6
(OLIMPÍADA BRASILEIRA DE QUÍMICA) Assinale a opção que apresenta
valores de ΔH e ΔS para uma reação que ocorre espontaneamente a
qualquer temperatura:
a) ΔH < 0 e ΔS < 0
b) ΔH > 0 e ΔS < 0
c) ΔH > 0 e ΔS = 0
d) ΔH = 0 e ΔS < 0
e) ΔH < 0 e ΔS > 0

Resposta: Alternativa E.

A equação de Gibbs é conduzida à pressão constante e temperatura


constante. A variação da energia livre de Gibbs é definida a partir do trabalho
útil, representado pela seguinte equação termodinâmica:

∆G = −Wútil

Relacionando com as demais funções de estado, temos:

∆G = ∆H − T x ∆S

Para que um processo seja definido como espontâneo, a variação de energia


livre de Gibbs deve estar representado da seguinte maneira:

∆G < 0 (processo espontâneo)

∆G > 0 (processo não-espontâneo)

∆G = 0 (processo em equilíbrio)

Logo, a partir da equação termodinâmica, ∆G = ∆H − T x ∆S, o processo


deve ser exotérmico (∆H < 0) e entropia deverá ser maior que zero (∆S >
0).

LIVRO OLÍMPICO – VOLUME I


172

7
(OLIMPÍADA BRASILEIRA DE QUÍMICA) Explosivos produzem, em geral,
um grande volume de gases como produtos. A nitroglicerina detona de acordo
com a seguinte reação: 2 C3H5N3O9(l) → 6 CO2(g) + 3 N2(g) + 5 H20(g) + ½ O2(g).
Se 1 g de nitroglicerina sofre uma explosão, o volume de gases produzidos,
se a pressão total é de 1 atm e a temperatura 500°C será de:
a) 1 L
b) 2 L
c) 3 L
d) 4 L
e) 5 L

Resposta: Alternativa B.
1
Cálculo do número de mol da nitroglicerina: n = 227 mol

A partir da equação química, para cada dois mol de nitroglicerina há a


formação de 15,50 mol de gases. Diante disso, temos:

2 mol − − − − − 15,50 mol de gases


1
mol − − − − ngases
227
15,50
ngases = mol
454
Cálculo do volume dos gases produzidos, aplicando a equação dos gases
ideais: pV = nRT
15,50
x 0,08206 x 773
V = 454 = 2,16 L
1
8
(GRILLO) Calcule a temperatura adiabática de chama da queima de gás
metano a 1 atm com: (a) oxigênio puro e (b) ar atmosférico.

Espécie H°298 (cal.mol-1) Cpmédio (cal/mol.K)


CH4(g) - 17895 9,85
O2(g) 0 9,17
N2(g) 0 5,95
H2O(g) - 57798 10,04
CO2(g) - 94050 14,78

Resposta:

Item a) Considerando o oxigênio puro: CH4(g) + 2 O2(g) → CO2(g) + 2 H2O(g)

Cálculo da variação de entalpia padrão da reação de combustão do gás


metano (CH4):

0 0 0
∆Hreação = ∑ Hprodutos − ∑ Hreagentes

LIVRO OLÍMPICO – VOLUME I


173

0 0 0
0
∆Hreação = [1 x HCO 2
+ 2 x HH 2O
] − [1 x HCH 4
+ 2 x HO0 2 ]

0
∆Hreação = [1 x (−94050) + 2 x (−57798)] − [1 x (−17895)
+ 2 x 0]

0
cal
∆Hreação = [−94050 − 115596] − [−17895] = −191751
mol

Análise do balanço térmico:

Cálculo da temperatura adiabática de chama com a presença de O 2: Q p =


∑ ∆H
298 298 T
Qp = ∫ (1 x 9,85)dT + ∫ (2 x 9,17)dT + ∫ (1 x 14,78)dT
298 298 298
T
+ ∫ (2 x 10,04)dT + (−191751)
298

Q p = 0 + 0 + 14,78. (T − 298) + 20,08. (T − 298) − 191751

Q p = (T − 298) x [14,78 + 20,08] − 191751

Sabendo que o processo é adiabático (Q = 0), temos:

0 = (T − 298) x [14,78 + 20,08] − 191751

T = 5798,60 K

Item b) Cálculo do número de mol de gás nitrogênio (N2), considerando que


a composição do ar atmosférico seja igual a 20% de O2(g) e 80% de N2(g):

2 mol de O2 --------- 20%


nN2 --------------------- 80%
nN2 = 8,00 mol de N2(g)

Equação química da combustão completa com a presença de N2(g):

CH4(g) + 2 O2(g) + 8 N2(g) → CO2(g) + 2 H2O(g) + 8 N2(g)

0 0 0
∆Hreação = ∑ Hprodutos − ∑ Hreagentes

0 0 0
0
∆Hreação = [1 x HCO 2
+ 2 x HH 2O
0
+ 8 x HN 2
] − [1 x HCH 4
+ 2 x HO0 2
0
+ 8 x HN 2
]

LIVRO OLÍMPICO – VOLUME I


174

0
∆Hreação = [1 x (−94050) + 2 x (−57798) + 8 x 0)]
− [1 x (−17895) + 2 x 0 + 8 x 0]
0
∆Hreação = [−94050 − 115596] − [−17895]

0
cal
∆Hreação = −94050 − 115596 + 17895 = −191751
mol

Estudo do balanço térmico, com a presença do nitrogênio gasoso:

Cálculo da temperatura adiabática de chama (T) com a presença do ar


atmosférico: 𝑄𝑝 = ∑ ∆𝐻
298 298 298
Qp = ∫ (1 x 9,8)dT + ∫ (2 x 9,17)dT + ∫ (8 x 5,95)dT
298 298 298
T T
+ ∫ (1 x 14,78)dT + ∫ (8 x 5,95)dT
298 298
T
+ ∫ (2 x 10,04)dT + (−191751)
298

Q p = 14,78. (T − 298) + 20,08. (T − 298) + 47,6. (T − 298)


− 191751

Sabendo que o processo é adiabático (Q = 0), temos:

0 = (T − 298) x [14,78 + 20,08 + 47,6] − 191751

191751 = (T − 298) x [82,46]

T = 2623,38 K

9
(OLIMPÍADA BRASILEIRA DE QUÍMICA) A queima de metano na presença
de oxigênio pode produzir três produtos distintos, contendo carbono: fuligem
(partículas muito pequenas de grafite), monóxido de carbono gasoso e
dióxido de carbono gasoso.
a) Escreva três equações químicas equilibradas, correspondendo às
reações de metano gasoso com oxigênio que levam a cada um dos produtos
acima citados. Em todos os casos admita que o outro produto é a água, H2O(l);
b) Determina as entalpias padrões de cada uma das reações do item
(a);
c) Por que, havendo oxigênio em quantidade suficiente, o CO2(g) é o
produto "carbônico" predominante na combustão do metano? Dados: ΔHf0
(kJ.mol-1): CO(g) = -110,5; CO2(g) = -393,5; H2O(l) = -285,83; CH4(g) = -74,81.

LIVRO OLÍMPICO – VOLUME I


175

Resposta:

Item a) Equação química (a) da combustão completa do gás metano:


CH4(g) + O2(g) → CO2(g) + 2 H2O(l)

Equação química (b) de combustão incompleta do gás metano:


CH4(g) + 3/2 O2(g) → CO(g) + 2 H2O(l)

Equação química (c) de combustão incompleta do gás metano:


CH4(g) + O2(g) → C(s) + 2 H2O(l).

Item b) Cálculo da variação de entalpia da combustão completa do gás


metano da equação química (a):

0 0 0
∆Hreação = ∑ Hprodutos − ∑ Hreagentes

0 0 0
0
∆Hreação = [1 x HCO 2
+ 2 x HH 2O
] − [1 x HCH 4
+ 1 x HO0 2 ]

0
∆Hreação = [1 x (−393,5) + 2 x (−285,83)] − [1 x (−74,81)
+1x0]

0
kJ
∆Hreação = −890,35
mol

Cálculo da variação de entalpia da combustão incompleta do gás metano da


0 0 0
equação química (b): ∆Hreação = ∑ Hprodutos − ∑ Hreagentes

0 0 0 3
0
∆Hreação = [1 x HCO + 2 x HH 2O
] − [1 x HCH + x HO0 2 ]
4 2

0
∆Hreação = [1 x (−110,5) + 2 x (−285,83)] − [1 x (−74,81)
3
+ x0]
2

0
kJ
∆Hreação = −607,35
mol

Cálculo da variação de entalpia de combustão incompleta do gás metano da


equação química (c):

0 0 0
∆Hreação = ∑ Hprodutos − ∑ Hreagentes

0
∆Hreação = [1 x HC0 + 2 x HH
0
2O
0
] − [1 x HCH 4
+ 1 x HO0 2 ]

0
∆Hreação = [1 x (0) + 2 x (−285,83)] − [1 x (−74,81) + 1 x 0 ]

0
kJ
∆Hreação = −496,85
mol

LIVRO OLÍMPICO – VOLUME I


176

Item c) O dióxido de carbono (CO2) é o produto mais predominante pelo fato


de ser o mais exotérmico, o que pode ser observado no valor obtido na
equação química do item b.

10
(OLIMPÍADA BRASILEIRA DE QUÍMICA) O trióxido de boro é um composto
utilizado como aditivo de fibra óptica, na produção de vidro de borossilicato,
entre outros. Esse composto é obtido pela desidratação do ácido bórico,
porém também é possível consegui-lo a partir das seguintes etapas de
reação:
Etapa 1: B2O3(s) + 3 H2O(g) → 3 O2(g) + B2H6(g) 2035 ΔH/kJ
Etapa 2: H2O(l) → H2O(g) 44 ΔH/kJ
Etapa 3: H2(g) + 1/2 O2(g) → H2O(l) - 286 ΔH/kJ
Etapa 4: 2 B(s) + 3 H2(g) → B2H6(g) 36 ΔH/kJ
Tendo como base as reações acima e suas respectivas entalpias, calcule a
entalpia geral de formação do trióxido de boro e informe sua equação global.

Resposta: Para a resolução do problema, será necessário aplicar a Lei de


Hess.

Invertendo a etapa 1:
3 O2(g) + B2H6(g) → B2O3(s) + 3 H2O(g) - 2035 ΔH/kJ

Multiplicando por três e invertendo a etapa 2:


3 H2O(g) → 3 H2O(l) 3x (- 44) ΔH/kJ

Multiplicando por três e invertendo a etapa 3:


3 H2O(l) → 3 H2(g) + 3/2 O2(g) 3x (+ 286) ΔH/kJ

Etapa 4: 2 B(s) + 3 H2(g) → B2H6(g) 36 ΔH/kJ

Somando as equações químicas e as entalpias:

3 O2(g) + B2H6(g) → B2O3(s) + 3 H2O(g) 2035 ΔH/kJ


3 H2O(g) → 3 H2O(l) 3x (- 44) ΔH/kJ
3 H2O(l) → 3 H2(g) + 3/2 O2(g) 3x (+ 286) ΔH/kJ
2 B(s) + 3 H2(g) → B2H6(g) 36 ΔH/kJ +
2 B(s) + 3/2 O2(g) → B2O3(s)

ΔHTOTAL = - 2035 ΔH/kJ + 3x (- 44) ΔH/kJ + 3x (+ 286) ΔH/kJ + 36 ΔHTOTAL = -


1273 ΔH/kJ

11
(OLIMPÍADA BRASILEIRA DE QUÍMICA) Analise as proposições para
previsão da ocorrência de uma transformação química, sob pressão e
temperatura constantes.
I. ΔH > 0 e ΔS > 0 reação não espontânea e ΔG < 0;
II. ΔH < 0 e ΔS > 0 reação espontânea e ΔG < 0;
III. ΔH > 0 e ΔS < 0 reação não espontânea e ΔG > 0;
IV. ΔH < 0 e ΔS < 0 reação não espontânea e ΔG > 0;
V. ΔH < 0 e ΔS < 0 reação espontânea e ΔG = 0.
a) Apenas I e II são corretas
b) Apenas I e III são corretas
c) Apenas I e IV são corretas

LIVRO OLÍMPICO – VOLUME I


177

d) Apenas II e III são corretas


e) Apenas II e IV são corretas

Resposta: Alternativa D.

A equação de Gibbs é apresentada da seguinte maneira?

∆G = ∆H − T x ∆S

Para que um processo seja definido como espontâneo, a variação de energia


livre de Gibbs deve estar representado da seguinte maneira:

∆G < 0 (processo espontâneo)

∆G > 0 (processo não-espontâneo)

∆G = 0 (processo em equilíbrio)

Logo, a partir da equação termodinâmica, ∆G = ∆H − T x ∆S, será


necessário analisar cada o processo.

Item I) Falso. Para um processo não espontâneo, ∆G > 0.


Item II) Verdadeiro.
Item III) Verdadeiro.
Item IV) Falso.

12
(OLIMPÍADA BRASILEIRA DE QUÍMICA) Assinale a opção que apresenta a
equação química da reação, cuja entalpia é a entalpia padrão de formação
do gás etano (C2H6):

a) 2 C(g) + 6 H(g) → C2H6(g)


b) 2 C(s) + 6 H(g) → C2H6(g)
c) 2 C(g) + 3 H2(g) → C2H6(g)
d) 2 C(s) + 3 H2(g) → C2H6(g)
e) CH4(g) + H2(g) → C2H6(g)

Resposta: Alternativa D.

A entalpia de formação do gás etano (ΔHf) ocorre segundo as substâncias na


sua forma mais estável. Logo, a reação de formação refere-se a seguinte
equação química balanceada: 2 C(s) + 3 H2(g) → C2H6(g).

LIVRO OLÍMPICO – VOLUME I


178

13
(OLIMPÍADA BRASILEIRA DE QUÍMICA) Os produtos da combustão do
H2S(g) são H2O(g) e SO2(g). Usando as informações dadas nas equações
termoquímicas abaixo:
H2(g) + S(s) → H2S(g) ΔH = - 21 kJ
S(s) + O2(g) → SO2(g) ΔH = - 297 kJ
H2(g) + ½ O2(g) → H2O (g) ΔH = - 242 kJ
Conclui-se que a energia desprendida na combustão de 1 mol de H2S(g) é:
a) – 67 kJ
b) 34 kJ
c) – 560 kJ
d) – 34 kJ
e) – 518 kJ

Resolução: Alternativa E.

Equação química: H2S(g) + 3/2 O2(g) → SO2(g) + H2O(g)

Invertendo a equação química:


H2S(g) → H2(g) + S(s) ΔH = + 21 kJ
S(s) + O2(g) → SO2(g) ΔH = - 297 kJ
H2(g) + ½ O2(g) → H2O (g) ΔH = - 242 kJ

Somando as equações químicas apresentadas:

H2S(g) + 3/2 O2(g) → SO2(g) + H2O(g)

ΔHTOTAL = + 21 kJ + (- 297 kJ) + (- 242 kJ) = - 518 kJ

14
(OLIMPÍADA ARGENTINA DE QUÍMICA) Em um compartimento de uma
bomba calorimétrica (volume constante) rodeado de 945 g de água, a
combustão de 1,048 g de benzeno, C6H6(l), elevou a temperatura da água
desde 23,640°C a 32,692°C. A capacidade do calorímetro é de 891 kJ/°C e
a da água é 4,184 J/g.°C.
a) Escreva a equação química balanceada para a reação de
combustão do benzeno.
b) Calcule ΔUcombustão (C6H6) expresso em kJ/mol.

Resposta:

Item a) Equação química da combustão do benzeno líquido:

C6H6(l) + 15/2 O2(g) → 6 CO2(g) + 3 H2O(l).

Item b) Como o problema pede a variação da energia interna e o processo


ocorre a volume constante, pela primeira lei da termodinâmica, o trabalho
realizado será igual a zero. Então: ∆U = Q + W = Q + 0

∆U = Q

O calor calculado será o sistema formado pelo calorímetro + água.

∆U = Q + W

LIVRO OLÍMPICO – VOLUME I


179

∆U = Q + 0 = Q V

∆U = (m x c x ∆T)calorímetro + (m x c x ∆T)água

J
∆U = [891 x (32,692 − 23,640)°C]
°C
J
+ [945 g x 4,18 (32,692 − 23,640)°C]
g x °C

J
∆U = [891 x (32,692 − 23,640)°C]
°C
J
+ [945 g x 4,18 (32,692 − 23,640)°C]
g x °C

∆U = 8065,33 J + 35756,30 J = 43821,64 J

Se para:

1,048 g --------- 43821,64 J


78 g -------------- QV
QV = 3,26 x 106 J

Como o processo é exotérmico, o valor da quantidade de calor a volume


constante é igual a QV = - 3,26 x 106 J.

15
(U. S. NATIONAL CHEMISTRY OLYMPIAD) Which point in the phase
diagram best represents supercritical conditions?

a) A
b) B
c) C
d) D

Resposta: Alternativa B.

O ponto crítico é a região que especifica a região vapor-líquido crítico de um


material, a partir do qual a distinção entre fase a líquida e gasosa não existe,
conforme pode ser observado no diagrama apresentado a seguir.

LIVRO OLÍMPICO – VOLUME I


180

16
(GRILLO) Considere a seguinte situação termodinâmica: em um recipiente
são colocados 10 mol de gás nitrogênio molecular a uma temperatura igual a
27°C por um pistão a 40 atmosferas de pressão. Depois de um certo
momento do processo, a pressão do sistema é reduzida para 10 atm e o gás
se expande de forma adiabaticamente. Sabendo que a capacidade calorífica
a volume constante é igual a 20,8 J.mol-1.K-1, calcule cada item abaixo:
a) O volume inicial
b) A temperatura final e o volume final
c) A quantidade de calor
d) a variação da energia interna
e) a variação da entalpia
f) o trabalho realizado

Resposta:

Item a) Considerando que o gás nitrogênio (N2) apresenta comportamento


ideal, temos:

Situação Inicial:

Pinicial = 40 atm
Vinicial = ?
Tinicial = 27 + 273 = 300 K

Situação Final:

PFinal = 10 atm
VFinal = ?
TFinal = ?

Cálculo do volume inicial: 40 x Vinicial = 10 x 0,08206 x 300

Vinicial = 6,15 L

Item b) Para o cálculo da temperatura final, sendo primeiramente necessário


calcular a capacidade calorífica a pressão constante (Cp), coeficiente
termodinâmico (γ) e o volume final (V2):

Cp − CV = R

Cp − 20,8 = 8,314

LIVRO OLÍMPICO – VOLUME I


181

J
Cp = 20,8 + 8,314 = 29,11
mol x K
Cp 29,11
Cálculo da constante termodinâmica (γ): γ = C = 20,8
= 1,40
V
γ γ
Cálculo do volume final (VFinal): Pinicial x Vinicial = Pfinal x Vfinal

1,40
40 x (6,15)1,40 = 10 x Vfinal

1,40 40 x (6,15)1,40
Vfinal =
10

1,40
40 x (6,15)1,40
Vfinal = √
10

1,40
Vfinal = √50,87 = 16,55 L

γ−1 γ−1
Cálculo da temperatura final (TFINAL): Tinicial x Vinicial = Tfinal x Vfinal

γ−1
Tinicial x Vinicial Vinicial γ−1
Tfinal = γ−1 = Tinicial x ( )
Vfinal Vfinal

Vinicial γ−1
Tfinal = Tinicial x ( )
Vfinal

6,15 1,40−1
Tfinal = 300 x ( )
16,55

Tfinal = 300 x (0,372)0,40 = 202 K

Item c) Sabendo que o processo é adiabático, a quantidade de calor é igual


a zero (Q = 0).

Item d) Cálculo da variação da energia interna: ∆U = n x CV x ∆T

J
∆U = 10 mol x 20,8 x (202 − 300)
mol x K

∆U = −20384 J

Item e) Cálculo da variação da entalpia (ΔH): ∆H = n x Cp x ∆T

J
∆H = 10 mol x 29,11 x (202 − 300)
mol x K

∆H = −28528 J

Item f) Através da primeira lei da termodinâmica, temos: ∆U = 0 + W

∆U = W = - 20384 J

LIVRO OLÍMPICO – VOLUME I


182

17
(INSTITUTO MILITAR DE ENGENHARIA) Considere o diagrama de fase
hipotético representado esquematicamente na figura ao lado. O que
representa os pontos A, B, C, D e E?

Resposta:

A – Equilíbrio sólido-vapor
B – Ponto Triplo
C – Equilíbrio sólido-líquido
D – Equilíbrio líquido-vapor
E – Ponto Crítico

Observação:
Pc – Pressão crítica
Tc – Temperatura crítica

18
(INSTITUTO MILITAR DE ENGENHARIA) Em um gráfico de pressão versus
volume, represente o diagrama do ciclo idealizado por Carnot (máquina
térmica) para uma transformação cíclica, ininterrupta, e sem perdas de calor
e trabalho, e vice-versa. Identifique e denomine as quatro etapas dessa
transformação cíclica.

Resposta: O ciclo de Carnot é constituído de quatro processos reversíveis


sucessivos.

1 – Isotérmico

2 – Adiabático

3 – Isotérmico

4 – Adiabático

Ou seja, no ciclo de Carnot há duas isotermas e duas adiabáticas.

LIVRO OLÍMPICO – VOLUME I


183

19
(U. S. NATIONAL CHEMISTRY OLYMPIAD) A entalpia de combustão do
octano líquido, C8H18(l) para a formação de produtos gasosos, é – 5090 kJ.mol-
1. Use este valor para responder as questões abaixo, assumindo uma

temperatura de 100°C.
a) Escreva a equação balanceada para a combustão completa do
octano líquido.
b) Determine a entalpia molar de formação, ∆H°f, para o estado líquido.
Dados: H°CO2 = - 393,5 kJ.mol-1; H°H2O = - 241,8 kJ.mol-1.
c) Calcule o valor da energia interna, para a reação de combustão.
d) Se ∆G° para a combustão do octano é – 5230 kJ.mol-1, calcule o
valor da ∆S°.

Resposta:

Item a) Equação química da combustão do octano líquido:

C8H18(l) + 25/2 O2(g) → 8 CO2(g) + 9 H2O(g).

Item b) Cálculo da entalpia molar de formação do octano líquido:

0 0 0
∆Hreação = ∑ Hprodutos − ∑ Hreagentes

0 0 25
0
∆Hreação = [8 x HCO + 9 x HH 2O
] − [1 x HC08 H18 + x HO0 2 ]
2 2

25
− 5090 = [8 x (−393,5) + 9 x (−241,8)] − [1 x HC08 H18 + x 0]
2

− 5090 = [8 x (−393,5) + 9 x (−241,8)] − 1 x HC08 H18 ]

− 5090 = −5324,20 − HC08 H18

kJ
HC08 H18 = −234,20
mol

Item c) Cálculo da energia interna, levando em consideração que os gases


apresentam comportamento ideal.

∆H = ∆U + (nprodutos – nreagentes) x R x T

- 5090000 = ∆U + (17 – 12,5) x (8,314) x (100 + 273)

∆U = - 5090000 – 13955,089 J.mol-1

∆U = - 5103,95 kJ.mol-1

Item d) Cálculo da entropia (∆S°): ∆G° = ∆H° - T x ∆S°

- 5230 + 5090 = - (100 + 273) x ∆S°

∆S° = + 0,375 kJ.mol-1.K-1

LIVRO OLÍMPICO – VOLUME I


184

20
(GRILLO) Óxido de zinco e carbono, ambos a 25°C, são alimentados em
quantidades estequiométricas em um reator operado a 1,0 atm, onde ocorre
a seguinte reação química: ZnO(s) + C(s) → Zn(g) + CO(g). A reação é completa
e os produtos saem do reator a 1,0 atm e a uma temperatura de 1000°C.
Calcule o calor que é adicionado ou removido do reator, por quilograma de
zinco.
Espécie Cp [J/(mol.K)] ΔHo298 [kJ.mol-1]
ZnO(s) 52,5 -350,5
C(s) 19,8 0
Zn(g) 20,9 130,4
CO(g) 31,6 -110,5

Resposta: Equação química: ZnO(s) + C(s) → Zn(g) + CO(g)

Cálculo da variação de entalpia a 298 K:

0 0 0
∆Hreação = ∑ Hprodutos − ∑ Hreagentes

0 0
0
∆Hreação 0
= [1 x HZn + 1 x HCO ] − [1 x HZnO + 1 x HC0 ]

0
∆Hreação = [1 x (130,40) + 1 x (−110,50)] − [1 x (−350,5)
+1x0]

0
kJ
∆Hreação = + 370,40
mol

Estudo do balanço térmico:

Cálculo da entalpia em relação ao ZnO(s):


T 298
∆H1 = ∫T 2 1 x Cp x dT = ∫298 1 x Cp x dT = 0
1

Cálculo da entalpia em relação ao C(s):


T 298
∆H2 = ∫T 2 1 x Cp x dT = ∫298 1 x Cp x dT = 0
1

Cálculo da entalpia em relação ao Zn(g):


T 1273
∆H3 = ∫T 2 1 x Cp x dT = ∫298 1 x 20,90 x dT
1

1273
∆H3 = ∫298 20,90 x dT

∆H3 = 20,90 x T|1273


298

J
∆H3 = 20,90 x (1273 − 298) = 20377,50 mol

LIVRO OLÍMPICO – VOLUME I


185

Cálculo da entalpia em relação ao CO(g):


𝑇 1273
∆𝐻4 = ∫𝑇 2 1 𝑥 𝐶𝑝 𝑥 𝑑𝑇 = ∫298 1 x 31,60 x dT
1

1273
∆H4 = ∫298 31,60 x dT

∆H3 = 31,60 x T|1273


298

J
∆H3 = 31,60 x (1273 − 298) = 30810
mol

Sabendo que o processo ocorre a pressão constante, então: Q p = ∑ ∆H

0
Q p = ∆Hreação + ∆H1 + ∆H2 + ∆H3 + ∆H4

kJ kJ kJ
Q p = 370,40 + 0 + 0 + 20,38 + 30,81
mol mol mol
kJ
Q p = 421,59
mol

Cálculo da quantidade de calor para 1 quilograma de zinco gasoso:

65,4 g de Zn ---------------------- 421,59 kJ


1000 g de Zn ---------------------- Qp
Qp = ΔH = 6446,33 kJ

21
(OLIMPÍADA BRASILEIRA DE QUÍMICA) Para a seguinte reação: NO(g) +
CO(g) → ½ N2(g) + CO2(g) ΔH = - 374 kJ. As condições que favorecem a
máxima conversão de reagentes em produtos são:
a) Baixa temperatura e alta pressão
b) Baixa temperatura e baixa pressão
c) Alta temperatura e baixa pressão
d) Alta temperatura e alta pressão
e) Apenas alta temperatura

Resposta: Alternativa A.
O problema é aplicado para o princípio de Le Chatelier. Diante disso, como o
equilíbrio deve ser deslocado para a direita, as circunstâncias que isso deve
ocorrer são as seguintes:
Item a) Diminuição da temperatura, uma vez que o processo é exotérmico;
Item b) Aumento da pressão do sistema reacional.

LIVRO OLÍMPICO – VOLUME I


186

22
(PROGRAMA OLIMPÍADA URUGUAYA DE QUÍMICA) A una temperatura
dada y, para cantidades dadas de produtos y reactivos, la energia livre de
Gibbs para el processo representado por la seguiente ecuación vale 586
kJ/mol: 2Fe(s) + 3/2O2(g) → Fe2O3(s). Señale cuál de las siguientes
afirmaciones es correta:
a) El sistema está em equilíbrio
b) Se formará espontaneamente Fe2O3
c) Se decompondrá espontaneamente Fe2O3
d) El processo será rápido
e) Ninguna de las anteriores

Resposta: Alternativa C.

Item a) Falso.
Determinação da constante de equilíbrio, a partir da relação
termodinâmica da energia livre de Gibbs e levando em consideração
que a temperatura seja igual a 298 K.

−586000 = 8,314 x 298 x ln K

ln K = −236,52

K = 1,90 𝑥 10−103

Para este valor, o sistema não está em equilíbrio termodinâmico.

Item b) Falso. Para uma variação de energia livre de Gibbs maior que
zero, o processo é não espontâneo para a reação direta.

Item c) Verdadeiro.

Item d) Não é possível afirmar se o processo é rápido, uma vez que não foi
mencionado a utilização do catalisador.
23
(OLIMPÍADA PIAUIENSE DE QUÍMICA) Calcule a pressão exercida por 5
mols de gás carbônico (CO2) em um vaso com um volume de 1 litro a 47°C,
usando a equação de van der Waals. Calcule também a pressão do gás,
assumindo um comportamento ideal. Dados para o CO2: a = 3,592 atm x L-2
x mol-2; b = 0,0427 L x mol-1.
Resposta: Cálculo da pressão considerando comportamento ideal:
n x R x T 5 x 0,08206 x (47 + 273)
pideal = = = 131,30 atm
V 1
Cálculo da pressão considerando comportamento real:
a
(preal + 2 )x (Vmolar − b) = R x T
Vmolar

LIVRO OLÍMPICO – VOLUME I


187

3,592 5
(preal + 2 ) x ( − 0,0427) = 0,08206 x (47 + 273)
5 1
(1)

preal = 5,15 atm


24
(OLIMPÍADA PIAUIENSE DE QUÍMICA) Se o volume ocupado pelas
moléculas de CO2 é desprezível, então calcule a pressão exercida por um mol
de gás carbônico a 273 K.
Resposta: Como o problema afirma que o volume das moléculas é
desprezível, será necessário levar em consideração que o gás apresenta
comportamento ideal.
n x R x T 1 x 0,08206 x 273
pideal = = = 22,40 atm
V 1
25
(OLIMPÍADA MINEIRA DE QUÍMICA) O ácido nítrico, HNO3, é uma
substância utilizada para a fabricação de fertilizantes e explosivos. Este ácido
pode ser obtido de acordo com a reação química indicada pela equação
termoquímica abaixo:
NH3(g) + 2 O2(g) → HNO3(aq) + H2O(l) ΔHº = - 447 kJ; ΔGº = - 79,6 kJ

De acordo com as informações acima, assinale a alternativa incorreta:


a) Nas condições padrão, a reação de obtenção do ácido nítrico é exotérmica.
b) Termodinamicamente, o processo de obtenção do ácido nítrico é
espontâneo nas condições padrão.
c) Nas condições indicadas, o valor da variação de entropia ∆S° para a
reação, a 25oC, é −1,233 kJ.K−1 .
d) A produção de 0,5 mol de ácido nítrico a partir da amônia, NH3, e oxigênio,
O2, resulta na liberação de ∆Ho = −894 kJ de energia.
Resposta: Alternativa D.
Item a) Verdadeiro. Como a variação da entalpia é negativa, o processo se
caracteriza como exotérmico.
Item b) Verdadeiro. Como a variação da energia livre de Gibbs é negativa, a
reação direta de obtenção do ácido nítrico é um processo termodinâmico
espontâneo.
Item c) Verdadeiro.
A partir da equação de Gibbs, temos: ∆G° = ∆H° − T x ∆S°
−79,6 = −447 − (25 + 273) x ∆S°
−79,6 + 447 = −298 x ∆S°

LIVRO OLÍMPICO – VOLUME I


188

kJ
∆S° = −1,23
K
Item d) Falso.
A partir de uma relação estequiométrica, temos:
1 mol de ácido ------ (-447 kJ)
0,50 mol -------------- ΔH
ΔH = - 223,50 kJ

26
(OLIMPÍADA MINEIRA DE QUÍMICA) O processo de produção de amônia
(NH3), a partir dos gases hidrogênio (H2) e nitrogênio (N2), foi de difícil
desenvolvimento e demandou vários anos de estudo. Este método recebeu o
nome de processo Haber-Bosch, em homenagem aos seus idealizadores. A
equação química abaixo descreve a síntese da amônia: 3/2 H2(g) + ½N2(g) ⇌
NH3(g). Supondo que os gases nitrogênio e hidrogênio sejam colocados em
um reator fechado, onde não ocorra nenhuma troca de matéria com a
vizinhança e na presença de um catalisador apropriado, responda às
questões que se seguem:
a) Considerando o princípio de Le Chatelier, descreva qual será a nova
situação do equilíbrio químico no interior do reator, se sua pressão interna for
duplicada.
b) Calcule a pressão que 17 g do gás amônia exercem em um reator
de volume igual a 5 L e operando a 427ºC. (R = 8,314 J.K-1.mol-1)

Resposta:

Item a) A partir do princípio de Le Chatelier, 3/2 H2(g) + ½ N2(g) ⇌ NH3(g),


aumentando a pressão interna para o sistema reacional, o equilíbrio será
deslocado para a direita, ou seja, aumentando a produção de gás amoníaco.

Item b) Para a determinação do cálculo da pressão, será utilizado a equação


dos gases ideais.
17
ideal
n x R x T (17) x 0,08206 x (427 + 273)
p = = = 11,49 atm
V 5
27
(OLIMPÍADA DE QUÍMICA DO RIO DE JANEIRO) A reação 2A + 3B ⇌ C +
2D, onde todas as substâncias se encontram no estado gasoso, ocorre com
perda de calor. Após certo tempo, o sistema entra em equilíbrio. Indique a
alternativa que apresenta três maneiras de aumentar a produção de C e D.
a) utilizar um catalisador, aumentar a pressão e aumentar a concentração de
A.
b) reduzir a concentração de B, aumentar a temperatura e reduzir a pressão.
c) utilizar um catalisador, aumentar o volume e reduzir a temperatura.
d) aumentar a concentração de C, reduzir o volume e aumentar a
temperatura.
e) aumentar a concentração de A, aumentar a pressão e reduzir a
temperatura.

Resposta: Alternativa E.

LIVRO OLÍMPICO – VOLUME I


189

Para aumentar a produção de C ou D, o equilíbrio químico deverá ser


deslocado para a direita. Diante disso, será colocado a alguns pontos para
que o equilíbrio seja deslocado para a direita, segundo o princípio de Le
Chatellier. Equação química: 2A + 3B – calor ⇌ C + 2D
- Aumentar a pressão do sistema reacional;
- Diminuição da temperatura do sistema reacional;
- Aumentando a pressão parcial de A ou até mesmo de B;
- Diminuir o volume do sistema reacional.
28
(OLIMPÍADA DE QUÍMICA DO RIO DE JANEIRO) Considere o seguinte
sistema no qual o NO2 é um gás que tem coloração castanho-avermelhada e
o N2O4 é um gás incolor. N2O4(g) ⇌ 2 NO2(g). Num sistema em frasco fechado
sob condições de equilíbrio, contendo a mistura de NO2 e de N2O4, a
concentração de NO2 aumenta quando ocorre:
a) aumento da pressão total
b) diminuição da pressão total
c) aumento da concentração do catalisador
d) diminuição da concentração do catalisador
e) aumento do pH da solução

Resposta: Alternativa E.
Segundo o princípio de Le Chatellier, para aumentar a produção de dióxido
de nitrogênio, será necessário diminuir a pressão do sistema reacional.
29
Faça a relação entre a variação da energia livre de Gibbs em função do
quociente reacional. Faça também a relação com a constante de equilíbrio.

Resposta: Para o estudo da variação da energia livre de Gibbs, será


necessário aplicar a definição do potencial químico. Levando em
consideração que a relação da energia livre de Gibbs com o potencial químico
é dado pela seguinte equação:

dG
( ) =μ − μreagentes
dT p,T ⏟produtos
diferença de potencial

Então, considere a seguinte equação química representada da seguinte


maneira: aA(g) + bB(g) ⇌ cC(g) + dD(g)

dG
( ) =μ − μreagentes
dT p,T ⏟produtos
diferença de potencial

dG
( ) = μ0produtos + R x T ln aprodutos
dT p,T
− (μ0reagentes + R x T ln areagentes )

LIVRO OLÍMPICO – VOLUME I


190

dG
( ) = μ0produtos − μ0reagentes + R x T ln aprodutos
dT p,T
− R x T ln areagentes

dG aprodutos
( ) = μ0produtos − μ0reagentes + R x T ln
dT p,T areagentes

dG 0 0
aprodutos
( ) =μ⏟produtos − μreagentes + R x T ln
dT p,T a⏟reagentes
∆G0
Q
∆Greação = ∆G0 + R x T ln Q

Quando o sistema estiver em equilíbrio, ∆Greação = 0. Logo, Q = Keq.

0 = ∆G0 + R x T ln K eq

∆G0 = −R x T ln K eq

30
(OLIMPÍADA BRASILEIRA DE QUÍMICA) A 500ºC, NO reage com Cl2, para
formar NOCl, segundo a reação: 2 NO + Cl2 ⇌ 2 NOCl, Kc = 2,1 x 10³. Em
qualquer mistura destas três espécies, em equilíbrio, podemos afirmar que:
a) A concentração de pelo menos uma das espécies, NO ou Cl2, será
muito maior que a concentração de NOCl
b) A concentração de pelo menos uma das espécies, NO ou Cl2, será
muito menor que a concentração de NOCl
c) A concentração de NOCl será exatamente 2100 vezes o produto das
concentrações de NO ou Cl2
d) A concentração de ambos, NO e Cl2, será muito maior que a
concentração de NOCl
e) A concentração de ambos, NO e Cl2, será muito menor que a
concentração de NOCl

Resposta: Alternativa B.

Equação química: 2 NO(g) + Cl2(g) ⇌ 2 NOCl(g)

[NOCl]2
Expressão da constante de equilíbrio: K c = [NO]2 x [Cl
2]

Sabendo que a constante de equilíbrio é igual a 2100, temos:

[NOCl]2
= 2100
[NO]2 x [Cl2 ]

[NOCl]2 = 2100 x [NO]2 x [Cl2 ]

[NOCl] = [NO] x √2100 x [Cl2 ]

Diante desta relação, o equilíbrio químico está deslocado para a direita e, a


concentração da quantidade de matéria de NOCl será maior em comparação
ao monóxido de nitrogênio e também em relação ao cloro gasoso.

LIVRO OLÍMPICO – VOLUME I


191

31
(U. S. NATIONAL CHEMISTRY OLYMPIAD) The equilibrium constant for the
reaction N2O4(g) ⇌ 2NO2(g) is 6,10 × 10–3 at 25˚C. Calculate the value of K for
this reaction: NO2(g) ⇌ 1/2N2O4(g).
a) 327
b) 164
c) 12,8
d) 3,05 × 10–3

Resposta: Alternativa C.

Para a resolução deste problema, será necessário inverter a equação química


e dividir por dois.

N2O4(g) ⇌ 2NO2(g) K eq = 6,10 x 10−3

1
2NO2(g) ⇌ ½ N2O4(g) K′eq = = 12,80
√6,10 x 10−3

32
(OLIMPÍADA DE QUÍMICA DO RIO DE JANEIRO) A que temperatura as
moléculas de O2, teriam a mesma velocidade média dos átomos de He à
27ºC?
a) 216ºC
b) 300ºC
c) 1000ºC
d) 2127ºC
e) 2700ºC

Resposta: Alternativa D.

A questão leva em consideração que a teoria cinética dos gases, através da


velocidade média quadrática (c²).
3xRxT
c2 = <MM>

O problema leva em consideração que a velocidade média quadrática das


moléculas de oxigênio é igual a velocidade média quadrática do gás nobre
hélio.

cO2 2 = cHe
2

3 x R x T 3 x R x 300
=
32,0 4,0
32,0 x 300
T= 4
= 2400 K (2127ºC)

LIVRO OLÍMPICO – VOLUME I


192

33
(OLIMPÍADA DE QUÍMICA DO RIO DE JANEIRO) Em um recipiente de aço
inox com capacidade de 1,0 L foram colocados 0,500 mol de H2 e 0,500 mol
de I2. A mistura alcança o equilíbrio quando a temperatura atinge 430oC. As
concentrações de H2, I2 e HI na situação de equilíbrio, à 430oC, sabendo que
a constante de equilíbrio para a reação H2(g) + I2(g) ⇌ 2HI(g) é igual a 50,3, são
respectivamente iguais a:
a) 0,39; 0,39; 0,78
b) 0,11; 0,11; 0,39
c) 0,11; 0,11; 0,78
d) 0,39; 0,39; 0,39
e) 0,39; 0,39; 0,11

Resposta: Alternativa A.

Para a resolução deste problema será necessário aplicar a tabela de


equilíbrio junto com uma base de cálculo.

Base de cálculo: 0,500 mol de H2(g) e I2(g) no estado inicial.

Substância H2(g) (mol) I2(g) (mol) 2HI(g) (mol)


Início 0,500 0,500 0
Reage X X 2X
Equilíbrio 0,500-X 0,500-X 2X
[HI]2
Expressão da constante de equilíbrio: K eq = [H
2 ] x [I2 ]
(2X)2
K eq =
(0,500 − X) x (0,500 − X)

(2X)2
50,3 =
(0,500 − X)2

Tirando a raiz quadrada da equação acima, temos:

(2X)2
√50,3 = √
(0,500 − X)2

2X
7,09 =
0,500 − X

X = 0,39

Diante disso, as concentrações das quantidades de matéria das espécies


químicas são iguais a:

mol
[HI] = 2 𝑥 0,390 = 0,780
L

mol
[H2 ] = 0,500 − 0,390 = 0,110
L
mol
[I2 ] = 0,500 − 0,390 = 0,110
L

LIVRO OLÍMPICO – VOLUME I


193

34
(29TH ICHO, INTERNATIONAL CHEMISTRY OLYMPIAD)
a) Quando um gás ideal e monoatômico expande reversivelmente de
um volume V1 para o volume V2, em que o sistema apresentado pelo trabalho
V
W = − ∫V 2 p x dV, em que W é o trabalho e p é a pressão do gás.
1
Determine o trabalho realizado quando um mol de um gás ideal expande
isotermicamente de V1 = 1,0 dm³ para V2 = 20,0 dm³ à T = 300,0 K
b) Determine a quantidade de calor adicionado ao gás durante o
processo apresentado no item a).

Resposta do item a) Levando em consideração que o gás apresenta


comportamento ideal, temos:
V2
nxRxT
W = −∫ x dV
V1 V

V2
1
W = −n x R x T ∫ x dV
V1 V

V
W = −n x R x T 𝑥 ln (V2 ). Substituindo os valores na equação, temos:
1

20
W = −1 x 8,314 x 300 𝑥 ln ( ) = −7471,95 𝐽
1,0

Resposta do item b) A partir da primeira lei da termodinâmica e sabendo que


o processo ocorre à temperatura constante, temos:

∆U
⏟ =Q+W
0

0=Q+W

Q = −W = −(−7471,95 J) = +7471,95 J

LIVRO OLÍMPICO – VOLUME I


194

35
(OLIMPÍADA BRASILEIRA DE QUÍMICA) A termodinâmica é uma área do
conhecimento que fundamenta grande parte dos conhecimentos estudados
da Química. Por exemplo, as variações de energia envolvidas nas
transformações da matéria podem ser determinadas por meio das três leis da
termodinâmica, para dar continuidade aos estudos dentro da área da química.
Desta forma, baseado nas informações apresentadas abaixo, determine a
variação de entalpia, a 200ºC, da reação:
CH3COOH(g) + 2 O2(g) → 2 CO2(g) + 2 H2O(g)
CH3COOH(g) + 2 O2(g) → 2 CO2(g) + 2 H2O(l) ΔHº = - 871,5 kJ/mol (25ºC)
H2O(l) → H2O(g) ΔHº = + 44 kJ/mol (25ºC)
CH3COOH(l) → CH3COOH(g) ΔHº = + 30,2 kJ/mol (25ºC)

Substância CH3COOH(g) O2(g) CO2(g) H2O(l) H2O(g)


Cp/R 14,9 3,53 4,46 9,055 4,038

Resposta: Para a determinação da entalpia padrão (25ºC; 1 atm), será


necessário utilizar a lei de Hess.
CH3COOH(g) + 2 O2(g) → 2 CO2(g) + 2 H2O(l) ΔHº = - 871,5 kJ/mol
2 H2O(l) → 2 H2O(g) ΔHº = 2 x (+ 44 kJ/mol)
CH3COOH(g) → CH3COOH(l) ΔHº = - 30,2 kJ/mol +
CH3COOH(g) + 2 O2(g) → 2 CO2(g) + 2 H2O(g) ΔHº = - 813,7 kJ/mol

Cálculo da capacidade calorífica à pressão constante média:


Cpmédio = 2 x (4,46R) + 2 x (4,038R) – 1 x (14,9R) – 2 x (3,53R)
Cpmédio = – 4,964 x R
Cálculo do valor da entalpia à 200ºC, aplicando a equação de Kirchhoff:
T2 =473K
0
∆H200℃ = ∆H + ∫ Cpmédio x dT
T1 =298K

T2 =473K
∆H200℃ = −813700 + ∫ (−4,964 x R) x dT
T1 =298K

T2 =473K
∆H200℃ = −813700 + ∫ (−4,964 x 8,314) x dT
T1 =298K

∆H200℃ = −813700 − 41,27 x (473 − 298)


J J
∆H200℃ = −820922 (−820,92 )
mol mol

LIVRO OLÍMPICO – VOLUME I


195

36
(OLIMPÍADA BRASILEIRA DE QUÍMICA) Para um recipiente foram
transferidos, a 25oC, os gases NO2 e SO2. A reação que ocorre entre esses
gases é representada abaixo: NO2(g) + SO2(g) → NO(g) + SO3(g). Decorrido
algum tempo, a pressão parcial dos gases presentes no recipiente é: NO,
0,7 bar; NO2, 10–6 bar; SO2, 0,005 bar e SO3, 0,05 bar. Indique a alternativa
que apresenta a variação de energia de Gibbs da reação, rG (em kJ mol-1).
Dados de valores padrão para 25oC:
Gás fHo/kJ mol–1 So/J K–1 mol–1
NO + 90,25 + 210,76
NO2 + 33,18 + 240,06
SO2 − 296,83 + 248,22
SO3 − 395,72 + 256,76
a) + 3,42
b) – 41,82
c) – 35,63
d) – 3,42
e) + 4,18

Resposta: Alternativa A.
Equação química: NO2(g) + SO2(g) → NO(g) + SO3(g)
Cálculo da variação da entalpia:
0 0 0 0 0
∆Hreação = 1 x HNO + 1 x HSO3
− 1 x HNO 2
− 1 x HSO 2

0
kJ
∆Hreação = + 90,25 − 395,72 − 33,18 + 296,83 = −41,82
mol
Cálculo da variação da entropia:
0 0 0 0 0
∆Sreação = 1 x SNO + 1 x SSO 3
− 1 x SNO 2
− 1 x SSO 2

0
∆Sreação = + 210,76 + 256,76 − 240,06 − 248,22

0
J
∆Sreação = −20,76
mol x K
Cálculo da variação da energia livre de Gibbs padrão:
0 0 0
∆Greação = ∆Hreação − T x ∆Sreação
0
∆Greação = −41820 + T x 20,76
0
Para T = 25ºC: ∆Greação = −41820 + 298 x 20,76

0
J
∆Greação = −35633,52
mol
Cálculo da variação da energia livre de Gibbs fora das condições padrões:
PNO x PSO3
∆Greação = ∆G0 + R x T x ln
PNO2 x PSO2

LIVRO OLÍMPICO – VOLUME I


196

PNO PSO3
x 0
0 P0
∆Greação = ∆G + R x T x ln PNO P
P
, para P0 = 1 bar.
2 x SO2
P0 P0

0,7 0,05
x 1
∆Greação = −35633,52 + 8,314 x 298 x ln 1−6
10 0,005
1 x 1
J
∆Greação = −35633,52 + 39050,05 = +3416,52
mol
J kJ
∆Greação = +3416,52 (+ 3,42 )
mol mol
37
(OLIMPÍADA PORTUGUESA DE QUÍMICA) Quinta Porta: uma surpresa!
A Quarta porta abre para um corredor mal iluminado, ao fundo do qual há
uma porta entreaberta. Os nossos amigos percorrem o corredor e entram de
rompante numa sala que, afinal, é uma sala de aulas da sua Escola, onde
está a decorrer a prova Global de Química do 10º ano! O tempo está a acabar
e ainda têm de resolver este problema: "Considere o equilíbrio em fase
gasosa: 2SO2(g) + O2(g) ⇌ 2SO3(g) (reacção exotérmica).
Num recipiente fechado, com a capacidade de 10,0 dm3, introduziram-se a
uma dada temperatura 6,0 mol de dióxido de enxofre e 3,0 mol de oxigénio
molecular. Atingido o equilíbrio, verificou-se que havia no reactor 4,5 mol de
trióxido de enxofre.
1) Calcule a concentração das diferentes espécies químicas presentes no
equilíbrio.
2) Indique, justificando, qual o sentido em que evoluirá o sistema por:
a) aumento da temperatura do reactor;
b) aumento da pressão total do sistema por diminuição do volume do reactor;
c) aumento da pressão por adição de um gás inerte;
d) adição de um catalisador.

Resposta:

Item 1) Para a realização deste item, será necessário utilizar a tabela de


equilíbrio químico, levando em consideração que a base de cálculo será a
seguinte: 0,60 mol x L-1 de dióxido de enxofre, 0,30 mol x L-1 de oxigénio
molecular no estado inicial e 0,45 mol x L-1 de trióxido de enxofre.

Equação química: 2SO2(g) + O2(g) ⇌ 2SO3(g)

Substância 2SO2(g) O2(g) ⇌ 2SO3(g)


Início 0,60 0,30 0
Reage 2X X 2X
Equilíbrio 0,60-2X 0,30 - X 0,45
Cálculo do valor de X, a partir da concentração de trióxido de enxofre.

0 + 2X = 0,45
X = 0,225 mol x L-1

Concentração da quantidade de matéria para as espécies químicas:

LIVRO OLÍMPICO – VOLUME I


197

[SO2] = 0,60 – 2 x 0,225 = 0,15 mol x L-1

[O2] = 0,30 - X = 0,30 – 0,225 = 0,075 mol x L-1

Item 2)
a) Como o processo é exotérmico, aumentando a temperatura do
sistema, o equilíbrio será deslocado para a esquerda, favorecendo dióxido de
enxofre e oxigênio molecular.
b) Aumentando a pressão do sistema reacional, o equilíbrio será
deslocado para a direita, favorecendo a produção de trióxido de enxofre.
c) Não influencia no deslocamento do equilíbrio químico.
d) A adição do catalisador não influencia no equilíbrio químico.

38
(OLIMPÍADA PORTUGUESA DE QUÍMICA) Para a seguinte reacção N2O4(g)
⇌ 2 NO2(g) ∆H = +58,2 kJ. Qual das seguintes alterações nas condições de
reacção produzirá um aumento da concentração de NO2(g)?
a) Aumentar a temperatura
b) Diminuir o volume do recipiente onde ocorre a reacção.
c) Diminuir a temperatura.
d) Aumentar a pressão
e) Como o sistema está em equilíbrio, não será possível provocar o
efeito desejado.

Resposta: Alternativa A.

N2O4(g) ⇌ 2 NO2(g) ∆H = +58,2 kJ

Para maior produção de dióxido de nitrogênio, podem ser considerados os


seguintes pontos:

- Aumento da pressão parcial do tetróxido de dinitrogênio;


- Diminuição da pressão total do sistema reacional;
- Aumento do volume do recipiente;
- Aumento da temperatura do sistema reacional, uma vez que o processo é
endotérmico.

LIVRO OLÍMPICO – VOLUME I


198

39
(OLIMPÍADA BRASILEIRA DE QUÍMICA) O tetróxido de dinitrogênio está
25% dissociado a 30ºC e 1 atm no equilíbrio: N2O4(g) ⇌ 2 NO2(g). Baseado
nesta informação e sabendo que ∆Hº = +57,2 kJ/mol no intervalo de
temperatura considerado, determine a constante de equilíbrio (Kp) da reação
a 90ºC.
Resposta: Para a resolução do problema, será necessário primeiramente
calcular a constante de equilíbrio à 30ºC, com grau de dissociação igual a
25%.
Equação química: N2O4(g) ⇌ 2 NO2(g)

Base de cálculo: n(mol) inicial de tetróxido de dinitrogênio.

Substância N2O4(g) ⇌ 2 NO2(g)


Início n 0
Reage nα 2nα
Equilíbrio n-nα 2nα

Cálculo do número de mol total: ntotal = n − nα + 2nα = n + nα


Expressão das pressões parciais:
n − nα 1−α 0,75
PNO2 = x ptotal = x ptotal = = 0,60 atm
n + nα 1+α 1,25
2nα 2α 0,50
PN2 O4 = x ptotal = x ptotal = = 0,40 atm
n + nα 1+α 1,25
Cálculo da constante de equilíbrio em função das pressões parciais em uma
T = 30ºC:
2
PNO 2
(0,40)2
Kp = = = 0,267
PN2 O4 0,60

Cálculo da constante de equilíbrio à 90ºC, aplicando a equação de van’t Hoff.


0
K 30
p
C
∆H 0 1 1
ln ( 900 C ) = x( − )
Kp R 90 + 273 30 + 273

0,267 57200 1 1
ln ( 0C )= x( − )
K 90
p
8,314 90 + 273 30 + 273

0C 0,267
K 90
p = = 11,35
0,0235

LIVRO OLÍMPICO – VOLUME I


199

40
(OLIMPÍADA BRASILEIRA DE QUÍMICA) A formação de um sólido a partir
de líquido e/ou de gases é uma das evidências de que ocorreu uma reação
química. Para demonstrar tal evidência, um recipiente de 2,50 L que contém
amônia gasosa a 0,78 atm e 18,5ºC foi conectado em outro recipiente de 1,40
L com cloreto de hidrogênio gasoso a 0,93 atm e 18,5ºC, respectivamente.
Sabe-se que a combinação desses gases leva a formação de cloreto de
amônio sólido, logo, qual a quantidade aproximada de massa formada desse
comp; osto, o gás que sobrou nos recipientes conectados e sua pressão?
a) 1,44 g; amônia
b) 3,19 g; cloreto de hidrogênio; 0,332 atm
c) 1,44 g; cloreto de hidrogênio; 0,332 atm
d) 1,44 g; amônia; 0,332 atm
e) 2,91 g; amônia; 0,166 atm

Resposta: Alternativa E.

Equação química: NH3(g) + HCl(g) → NH4Cl(s)

Determinação do número de mol para cada reagente químico, aplicando a


equação dos gases ideais.

Analisando o gás amoníaco:

0,78 x 2,50
nNH3 =
0,08206 x (18,5 + 273)

nNH3 = 8,15 x 10−2 mol (reagente em excesso)

Analisando o cloreto de hidrogênio:

0,93 x 1,40
nHCl =
0,08206 x (18,5 + 273)

nHCl = 5,44 x 10−2 mol (reagente limitante)

Cálculo da massa de cloreto de amônio, a partir da relação estequiométrica:

NH3(g) + HCl(g) → NH4Cl(s)

1 mol de HCl(g) -------- 1 mol de NH4Cl(s)


5,44 x 10-2 mol -------- nsólido
nsólido = 5,44 x 10-2 mol

Cálculo da massa de cloreto de amônio:

msólido = 5,44 x 10−2 x 53,5 = 2,91 g

LIVRO OLÍMPICO – VOLUME I


200

Gás que sobrou no recipiente → gás em excesso → NH3(g)

Cálculo do número de mol de gás amoníaco em excesso:

nNH3 = 8,15 x 10−2 𝑚𝑜𝑙 − 5,44 x 10−2 𝑚𝑜𝑙 = 2,71 x 10−2 𝑚𝑜𝑙

Cálculo da pressão (p) do gás remanescente no recipiente:

2,71 x 10−2 𝑥 0,0826 𝑥 (18,5 + 273)


p= = 0,166 atm
(2,50 + 1,40)

41
(OLIMPÍADA BRASILEIRA DE QUÍMICA) Óxidos metálicos podem ser
reduzidos pelo hidrogênio gasoso, produzindo os respectivos metais.
Exemplo: ZnO(s) + H2(g) → Zn(s) + H2O(g)
Prediga os efeitos de cada uma dos seguintes procedimentos sobre a posição
do equilíbrio, ou seja, para que lado o equilíbrio será deslocado em cada caso.
a) Adição de ZnO(s)
b) Adição de H2(g)
c) Retirada de Zn(s)
d) Retirada de vapor de água
Usando os dados termodinâmicos fornecidos abaixo, responda os quesitos
que seguem:
ΔHºf (298,15 K), ΔSºf (298,15 K), ΔGºf (298,15 K),
Espécie
kJ/mol J/mol.K kJ/mol
Início -241 188,825 -228,572
Reage - 348,28 43,64 -318,30
e) Em que direção o equilíbrio será deslocado por um aumento de
temperatura?
f) Calcule o valor de ΔGº para esta reação.
g) Esta reação deverá ocorrer espontaneamente ou não?
h) A constante de equilíbrio para esta reação é maior ou menor que 1?

Resposta:

Item a) Aumentando a quantidade de ZnO, o equilíbrio não vai sofrer


mudança, uma vez que se trata de um sólido e, por convenção a sua atividade
termodinâmica é igual a um.

Item b) Aumentando a quantidade de hidrogênio gasoso (concentração da


quantidade de matéria ou pressão parcial) o equilíbrio será deslocado para a
direita.

Item c) Diminuindo a quantidade de zinco metálico, o equilíbrio não vai sofrer


mudança, uma vez que se trata de um sólido e, por convenção a sua atividade
termodinâmica é igual a um.

Item d) Diminuindo a quantidade de vapopr de água (concentração da


quantidade de matéria ou pressão parcial) o equilíbrio será deslocado para a
direita.

Item e) Para a resolução deste item, será necessário inicialmente calcular a


variação da entalpia.

LIVRO OLÍMPICO – VOLUME I


201

0 0 0 0 0
∆Hreação = 1 x HZn + 1 x HH 2𝑂
− 1 x HZnO − 1 x HH 2

0
∆Hreação = + 0 − 241,818 + 348,28 + 0
0 0 0 0 0
∆Hreação = 1 x HNO + 1 x HSO3
− 1 x HNO 2
− 1 x HSO 2

0
kJ
∆Hreação = + 106,46
mol
Como o processo é endotérmico, com o aumento da temperatura do
sistema reacional, o equilíbrio será deslocado para a direita.
Item f)
0 0 0
∆Greação = 1 x GH 2𝑂
− 1 x GZnO
0
∆Greação = −228,572 − (−318,30)

0
kJ
∆Greação = + 89,73
mol
Item g) Não, pelo fato da variação da energia livre de Gibbs ser positivo.
Item h) Menor do que, o que pode ser observado a partir dos seguintes
cálculos.

0
kJ
∆Greação = −R x T x ln K eq + 89,73
mol
kJ
−8,314 x 298,15 x ln K eq = 89,73
mol
ln K eq = −36,20

K eq = 𝑒⏟(−36,20) < 1
1,60 𝑥 10−16

LIVRO OLÍMPICO – VOLUME I


202

42
(GRILLO) Usando as constantes de capacidade calorífica molar a pressão
constante, apresentada na tabela a seguir para o dióxido de carbono, calcule
a quantidade de calor a pressão constante e também a volume constante,
quando são utilizados 350 g desta mesma amostra gasosa de 300 à 500 K.
Substância C̅p [J/mol.K]
CO2(g) A B x 10³ C x 107 D x 109
5,166 15,177 - 95,78 2,260

Resposta:
350 g
Cálculo do número de mol de dióxido de carbono: n = g = 7,95 mol
44
mol

Expressão da equação matemática da capacidade calorífica a pressão


constante:

Cp = 5,166 − 15,177 x 103 x T − 95,78 x 107 x T 2


+ 2,260 x 109 x T 3

Analisando primeiramente o processo a pressão constante. Cálculo da


variação da entalpia, considerando a variação de temperatura igual a 300 –
500 K:

∆H = ∫ n x Cp x dT

500 K
∆H = ∫ 7,95 x (5,166 + 15,177 x 103 x T − 95,78 x 107 x T 2
300 K
+ 2,260 x 109 x T 3 ) x dT
500 K 500 K
∆H = 7,95 x {∫ 5,166 x dT + ∫ 15,177 x 103 x T x dT
300 K 300 K
500 K
−∫ 95,78 x 107 x T 2 x dT
300 K
500 K
+∫ 2,260 x 109 x T 3 x dT}
300 K

500

∆H = 7,95 x {5,166 x T|500 3
300 + 15,177 x 10 x ( )|
2 300
500
7
T3
− 95,78 x 10 x ( )|
3 300
500
9
T4
+ 2,260 x 10 x ( )| }
4 300

LIVRO OLÍMPICO – VOLUME I


203

∆H = 7,95 x {5,166 x (500 − 300)

(500)2 (300)2
+ 15,177 x 103 x [ − ]
2 2
(500)3 (300)3
− 95,78 x 107 x [ − ]
3 3
(500)4 (300)4
+ 2,260 x 109 x [ − ]}
4 4

∆H = 7,95 x {5,166 x 200 + 15,177 x 103 x 80000


− 95,78 x 107 x 3,26 x 107
+ 2,260 x 109 x 1,36 x 1010 }

∆H = 7,95 x {1033,2 + 1,21 x 109 − 3,12 x 1016 + 3,07 x 1019 }

J
∆H = 2,44 x 1020
mol

Analisando o processo a volume constante. Cálculo da capacidade calorífica


a volume constante (CV): Cp − CV = R

5,166 + 15,177 x 103 x T − 95,78 x 107 x T 2 + 2,260 x 109 x T 3


− CV = R

CV = 5,166 + 15,177 x 103 x T − 95,78 x 107 x T 2


+ 2,260 x 109 x T 3 − 8,314

CV = −3,148 + 15,177 x 103 x T − 95,78 x 107 x T 2


+ 2,260 x 109 x T 3

Cálculo da variação da energia interna, considerando a variação de


temperatura igual a 300 – 500 K: ∆U = ∫ n x CV x dT
500 K
∆U = ∫ 7,95 x (−3,148 + 15,177 x 103 x T
300 K
− 95,78 x 107 x T 2 + 2,260 x 109 x T 3 ) x dT
500 K 500 K
∆U = 7,95 x {∫ (−3,148) x dT + ∫ 15,177 x 103 x T x dT
300 K 300 K
500 K
−∫ 95,78 x 107 x T 2 x dT
300 K
500 K
+∫ 2,260 x 109 x T 3 x dT}
300 K

LIVRO OLÍMPICO – VOLUME I


204

500

∆U = 7,95 x {(−3,148) x T|500
300 + 15177 x ( )|
2 300
500
7
T3
− 95,78 x 10 x ( )|
3 300
500
T4
+ 2,260 x 109 x ( )| }
4 300

∆U = 7,95 x {−3,148 x (500 − 300)

(500)2 (300)2
+ 15,177 x 103 x [ − ]
2 2
(500)3 (300)3
− 95,78 x 107 x [ − ]
3 3
(500)4 (300)4
+ 2,260 x 109 x [ − ]}
4 4

∆U = 7,95 x {−629,6 + 15,177 x 103 x 80000


− 95,78 x 107 x 3,26 x 107
+ 2,260 x 109 x 1,36 x 1010 }

J
∆U = 2,44 x 1020
mol

43
(OLIMPÍADA NORTE-NORDESTE DE QUÍMICA) Alguns anos atrás, Texas
City (Texas – USA), foi abalada por uma explosão de um depósito de nitrato
de amônio, composto muito usado como fertilizante. Este composto, quando
aquecido, pôde decompor exotermicamente em N2O e água, conforme a
equação:
NH4NO3(s) → N2O(g) + 2 H2O(g) (Equação 1).
Se o calor liberado nesta reação ficar aprisionado, altas temperaturas sendo
atingidas, e assim, o NH4NO3 pode decompor explosivamente em N2, H2O e
O2.
2 NH4NO3(s) → 2 N2(g) + 4 H2O(g) + O2(g) (Equação 2).
Usando as informações fornecidas abaixo, responda:
a) Qual o calor liberado (à pressão constante de 1 atm e à temperatura
de 25°C) na primeira reação?
b) Se 8,00 kg de nitrato de amônio são misturados com alumínio em
pó, em excesso estequiométrico, qual a quantidade de calor produzida (à
pressão constante)?
c) Nitrato de amônio reage com alumínio em pó, produzindo Al2O3,
segundo a equação: 2 Al(s) + 3 NH4NO3(s) → 3 N2(g) + 6 H2O(g) + Al2O3(s). Se
8,00 kg de nitrato de amônio são misturados com alumínio em pó, em excesso
estequiométrico, qual a quantidade de calor produzida (à pressão constante)?
Dados a 25°C (kJ.mol-1): Al2O3(g) = - 1675,7; H2O(g) = - 241,8; NH4NO3(s) = -
365,6; N2O(g) = 82,0.

LIVRO OLÍMPICO – VOLUME I


205

Resposta:

Item a) Cálculo da quantidade de calor para a primeira reação química:


NH4NO3(s) → N2O(g) + 2 H2O(g)

0 0 0
∆Hreação = ∑ Hprodutos − ∑ Hreagentes

0 0 0 0
∆Hreação = [1 x HNO 2
+ 2 x HH 2O
] − [1 x HNH 4 NO3
]

0
∆Hreação = [1 x 82,0 + 2 x (−241,8)] − [1 x (−365,6)]

0
kJ
∆Hreação = −36,0
mol

Item b) Cálculo da quantidade de calor considerando 8,00 kg de nitrato de


amônio: 2 NH4NO3(s) → 2 N2(g) + 4 H2O(g) + O2(g)

0 0 0
∆Hreação = ∑ Hprodutos − ∑ Hreagentes

0
0
∆Hreação 0
= [2 x HN 2
+ 4 x HH 2O
+ 1 x HO0 2 ] − [2 x HNH
0
4 NO3
]

0
∆Hreação = [1 x 0 + 4 x (−241,8) + 1 x 0] − [2 x (−365,6)]

0
kJ
∆Hreação = −263,0
mol

Pela estequiometria da reação, temos:

160 g de NH4NO3 -------------- (- 263,00) kJ


8000 g de NH4NO3 ------------ Qp
Qp = - 13150,0 kJ

Item c) Cálculo da quantidade de calor considerando 8,00 kg de nitrato de


amônio: 2 Al(s) + 3 NH4NO3(s) → 3 N2(g) + 6 H2O(g) + Al2O3(s)

0 0 0
∆Hreação = ∑ Hprodutos − ∑ Hreagentes

0 0 0 0 0
∆Hreação = [3 x HN 2
+ 6 x HH 2O
+ 1 x HAl2 O3
] − [2 x HAl
0
+ 3 x HNH 4 NO3
]

0
∆Hreação = [3 x 0 + 6 x (−241,8) + 1 x (−1675,7)] − [2 x 0
+ 3 x (−365,6)]

0
kJ
∆Hreação = −2030,0
mol

Pela estequiometria da reação, temos:

240 g de NH4NO3 -------------- (- 2030,00) kJ


8000 g de NH4NO3 ------------ Qp
Qp = - 67666,67 kJ

LIVRO OLÍMPICO – VOLUME I


206

44
(U. S. NATIONAL CHEMISTRY OLYMPIAD) A combustão de etano produz
dióxido de carbono e água líquida a 25°C.
a) Escreva a equação para esta reação.
b) Sabendo que ∆H°combustão para o etano sob estas condições
é -1560 kJ.mol-1 de etano, calcule:
i.∆H°f para o etano;
Dado:

ΔH°
(kJ.mol-1)
CO2(g) - 393,5
H2O(l) - 285,8
ii.A energia de ligação para a ligação C = O.
Energia de
ligação (kJ.mol-1)
C - C 347
H - C 413
H - O 164
O = O 195
c) Sabendo que ∆G° = - 1467,5 kJ.mol-1. Calcule ∆S° para esta
reação em J.mol-1.K-1.

Resposta:

Item a) Equação química da combustão completa do etano:

C2H6(l) + 7/2 O2(g) → 2 CO2(g) + 3 H2O(l)

Item b) i

0 0 0
∆Hreação = ∑ Hprodutos − ∑ Hreagentes
0 0 7
0
∆Hreação = [(2 x HCO + 3 x HH 2O
) − (1 x HC02 H6 +
x HO0 2 )]
2 2
7
− 1560 = [2 x (−393,5) + 3 x (−285,8)] − [(1 x HC02 H6 + x 0)]
2
− 1560 = −787 − 857,40 − HC02 H6
− 1560 = −1644,40 − HC02 H6
HC02 H6 = −84,40 kJ

ii
Cálculo das entalpias tanto para os reagentes quanto para os produtos.

0 7
REAGENTES: Hreagentes = + [1 x 1 x HC−C + 6 x HC−H + 2 x HO=O ]

0
7
Hreagentes = + [347 + 6 x (413) + x 495]
2
0
kJ
Hreagentes = + 4557,50
mol

LIVRO OLÍMPICO – VOLUME I


207

PRODUTOS:
0
Hprodutos = −[2 x 2 x HC=O + 3 x 2 x HO−H ]
0
Hprodutos = −[4 x HC=O + 6 x HO−H ]
0
Hprodutos = −[4 x HC=O + 6 x (464)]
0
Hprodutos = −[4 x HC=O + 2784]

Finalizando:
0
∆Hreação = − 4 x HC=O − 2784 + 4557,50
− 1560 = − 4 x HC=O − 2784 + 4557,50
− 1560 = − 4 x HC=O + 1773,50
− 1560 − 1773,50 = − 4 x HC=O
− 1560 − 1773,50 kJ
HC=O = = + 833,37
−4 mol

Item c) Cálculo da entropia padrão (∆S°):

∆G° = ∆H° - T x ∆S°

- 1467500 = - 1560500 – (25 + 273) x ∆S°

∆S° = - 312,50 J.mol-1.K-1

LIVRO OLÍMPICO – VOLUME I


208

45
(U. S. NATIONAL CHEMISTRY OLYMPIAD) A entalpia de combustão do
octano líquido, C8H18(l) para a formação de produtos gasosos, é – 5090 kJ.mol-
1. Use este valor para responder as questões abaixo, assumindo uma

temperatura de 100°C.
a) Escreva a equação balanceada para a combustão completa do
octano líquido.
b) Determine a entalpia molar de formação, ∆H°f, para o estado líquido.
Dados: H°CO2 = - 393,5 kJ.mol-1; H°H2O = - 241,8 kJ.mol-1.
c) Calcule o valor da energia interna, para a reação de combustão.
d) Se ∆G° para a combustão do octano é – 5230 kJ.mol-1, calcule o
valor da ∆S°.

Resposta:

Item a) Equação química da combustão do octano líquido:

C8H18(l) + 25/2 O2(g) → 8 CO2(g) + 9 H2O(g).


0
Item b) Cálculo da entalpia molar de formação do octano líquido: ∆Hreação =
0 0 0 0
∑ Hprodutos − ∑ Hreagentes = [8 x HCO 2
+ 9 x HH 2O
]−
25
[1 x HC08 H18 + x HO0 2 ]
2
25
− 5090 = [8 x (−393,5) + 9 x (−241,8)] − [1 x HC08 H18 + x 0]
2
− 5090 = [8 x (−393,5) + 9 x (−241,8)] − 1 x HC08 H18 ]
− 5090 = −5324,20 − HC08 H18
kJ
HC08 H18 = −234,20
mol

Item c) Cálculo da energia interna, a partir da seguinte relação


estequiométrica: ∆H = ∆U + pV, levando em consideração que os gases
apresentam comportamento ideal (pV = nRT)

∆H = ∆U + (nprodutos – nreagentes) x R x T
- 5090000 = ∆U + (17 – 12,5) x (8,314) x (100 + 273)
∆U = - 5090000 – 13955,089 J.mol-1 = - 5103,95 kJ.mol-1

Item d) Cálculo da entropia (∆S°): ∆G° = ∆H° - T x ∆S°

- 5230 + 5090 = - (100 + 273) x ∆S°

∆S° = + 0,375 kJ.mol-1.K-1

LIVRO OLÍMPICO – VOLUME I


209

46
(OLIMPÍADA PORTUGUESA DE QUÍMICA) Dados um copo com água, outro
com HCl(aq) e outro com NaOH(aq) concentrados, todos a mesma temperatura,
como fazer para retirar, o mais rapidamente possível, um diamante retido num
cubo de gelo, usando estes líquidos. Justifique.

Resposta: Para retirar o diamante rapidamente que está preso no cubo de


gelo, será necessário misturar ácido clorídrico com hidróxido de sódio, o que
vai gerar uma reação de neutralização. Para este processo, trata-se de uma
reação exotérmica, logo colocando na presença do cubo de gelo, haverá o
derretimento de forma rápida e com isso o diamante ficará livre.

47
(OLIMPÍADA NORTE-NORDESTE DE QUÍMICA) Hidrogênio gasoso pode
ser produzido a partir da reação entre carvão e vapor d´água, como mostra a
reação: C(s) + H2O(g) → CO(g) + H2(g).
a) Calcule a ΔG° para esta reação a 25°C, considerando que C(s) é
grafite.
b) Calcule Kp para esta reação a 25°C.
c) Esta reação ocorre espontaneamente sob estas mudanças? Se não,
a que temperatura ela se tornará espontânea? Dado: R = 8,314 J.mol-1.K-1.

Espécie ΔHo298 [kJ.mol-1] ΔSo298 [J.mol-1.K-1] ΔGo298 [kJ.mol-1]


H2O(g) - 241,8 + 188,8 - 228,6
CO(g) - 110,5 + 197,7 - 137,2
C(s) 0 + 5,740 0
H2(g) 0 + 130,68 0

Resolução: Item a) Cálculo da variação de entalpia da reação a 298 K, a


partir da equação química:

C(s) + H2O(g) → CO(g) + H2(g)

Cálculo da variação de entalpia da reação a 298 K:

0 0 0
∆Hreação = ∑ Hprodutos − ∑ Hreagentes

0
0
∆Hreação = [1 x HCO 0
+ 1 x HH 2
] − [1 x HC0 + 5 x HH
0
2O
]

0
∆Hreação = [1 x (−110500) + 1 x (0)] − [0 + 1 x (−241800)]

0
J
∆Hreação = +131300
mol

Cálculo da variação de entropia a 298 K:

0 0 0
∆Sreação = ∑ Sprodutos − ∑ Sreagentes

0
0
∆Sreação = [1 x SCO 0
+ 1 x SH 2
] − [1 x SC0 + 5 x SH
0
2O
]

LIVRO OLÍMPICO – VOLUME I


210

0
∆Sreação = [1 x (+197,70) + 1 x (130,68)] − [5,740
+ 1 x (188,8)]

0
J
∆Sreação = +133,84
mol x K

Cálculo da variação da energia livre de Gibbs a 298 K:


0 0 0
∆Greação = ∆Hreação − T x ∆Sreação

0
∆Greação = 131300 − (25 + 273) x 133,84

0
∆Greação = 131300 − 298 x 133,84
0
J
∆Greação = + 91415,7
mol

Item b) Cálculo da constante de equilíbrio:

0
J
∆Greação = + 91415,7 = −R x T x lnK p
mol
J
+ 91415,7 = −8,314 x 298 x lnK p
mol
+ 91415,7
lnK p = = −36,90
−2477,572

K p = e−36,90

Item c) Cálculo da temperatura para que o processo passe a ser espontâneo:


0 0 0
∆Greação = ∆Hreação − T x ∆Sreação

131300 − T x 133,84 < 0

T x 133,84 > 131300

131300
T>
133,84

T > 981,02 K (Passa a ser espontânea, a partir de 981,02 K).

LIVRO OLÍMPICO – VOLUME I


211

48
(GRILLO) O calor específico do acetileno é representado através da seguinte
equação empírica: 7,33 + 12,62 x 10-3T – 3,89 x 10-6T-², em cal.mol-1.K-1, com
a temperatura em Kelvin. Calcule o calor específico médio em um intervalo
de temperatura que compreende entre 298 K e 500 K?

Resposta:

Sabendo que o cálculo da entalpia para uma temperatura diferente de 25°C


pode ser determinado através de duas equações termodinâmicas:
T
∆H = ∫T 2 Cp dT (Equação A)
1
∆H = Cpmédio x (T2 − T1 ) (Equação B)

T
Igualando as equações: Cpmédio x (T2 − T1 ) = ∫T 2 Cp dT
1
T
∫T 2 Cp dT
1
Cpmédio =
(T2 − T1 )
T
∫T 2 Cp dT
Cálculo do Cpmédio : Cpmédio = 1
(T2 −T1 )

500
∫298 (7,33 + 12,62 x 10−3 x T − 3,89 x 10−6 x T −2 ) dT
Cpmédio =
(500 − 298)
500 500
−3 T² T −2+1
7,33 x T|500
298 + 12,62 x 10 x 2| − 3,89 x 10−6 x |
298 (−2 + 1) 298
=
202
500
T² (−1) 500
7,33 x T|500
298 + 12,62 x 10 −3
x |
2 298 − 3,89 x 10 −6
x T |298
=
202
500
−3 T² 1 500
7,33 x T|500
298 + 12,62 x 10 x |
2 298 + 3,89 x 10 −6
x T|298
=
202

(500)2 (298)2 1 1
7,33 x (500 − 298) + 12,62 x 10−3 x [ −6
2 − 2 ] + 3,89 x 10 x (500 − 298)
=
202

7,33 x 202 + 12,62 x 10−3 x (80598) + 3,89 x 10−6 x (− 1,35 x 10−3 )


=
202

1480,66 + 1017,15 − 5,25 x 10−9


=
202

cal
Cpmédio = 12,36
mol x K

LIVRO OLÍMPICO – VOLUME I


212

Observação: A capacidade calorífica é uma propriedade termodinâmica


diretamente relacionada com a temperatura. É representada pelas
expressões: A + B.T + CT-2 ou A + BT + CT², com A, B e C valores constantes
(tabelados). Os cálculos são bastante trabalhosos, como foi observado na
primeira questão acima e a maioria dos exercícios é mais comum a utilização
do calor específico médio (<Cp>médio), tomando como base a temperatura de
25°C.

49
(OLIMPÍADA BRASILEIRA DE QUÍMICA/CANADIAN CHEMISTRY
OLYMPIAD FINAL SELECTION EXAMINATION) Uréia, CO(NH2)2, reage
com água produzindo dióxido de carbono e amônia. Os dados
termodinâmicos para os possíveis reagentes e produtos são dados abaixo
(negligencie a solubilidade do dióxido de carbono e da amônia em água
líquida).

Composto ΔH°f (kJ.mol-1) S°f (J.mol-1.K-1)


CO(NH2)2(s) - 333,51 104,60
H2O(l) - 285,83 69,91
H2O(g) - 241,82 188,83
CO2 (g) - 393,51 213,74
NH3(g) - 46,11 192,45

a) Considere a hidrólise de ureia com H2O(l) (reação A) e com H2O(g)


(reação B), respectivamente. Calcule ∆H 0 , ∆S 0 e ∆G0 a 25°C, para cada
reação e especifique se a reação é espontânea ou não.
b) Considerando que ambos, ∆H 0 e ∆S 0, são independentes da
temperatura, encontre a temperatura na qual a reação A ocorrerá
espontaneamente.
c) Calcule Kp a 25°C para cada reação, expressando esse valor em
unidades apropriadas.

Resposta:

Item a) Analisando a reação A:

Equação química: CO(NH2)2 + H2O(l) → CO2(g) + 2 NH3(g)


0 0
Cálculo da variação de entalpia (∆Hreação ) para a reação A: ∆Hreação =
0 0
∑ Hprodutos − ∑ Hreagentes

0 0 0 0 0
∆Hreação = [1 x HCO 2
+ 2 x HNH 3
] − [1 x HCO(NH2 )2
+ 1 x HH 2O
]

0
∆Hreação = [1 x (−393,51) + 2 x (−46,11)] − [1 x (−333,51)
+ 1 x (−285,83)]

0 kJ
∆Hreação = −485,73 + 619,34 = + 133,61 mol

(Processo endotérmico)

LIVRO OLÍMPICO – VOLUME I


213

0
Cálculo da variação de entropia (∆Sreação ) para a reação A:

0 0 0
∆Sreação = ∑ Sprodutos − ∑ Sreagentes

0 0 0 0 0
∆Sreação = [1 x SCO 2
+ 2 x SNH3
] − [1 x SCO(NH 2 )2
+ 1 x SH 2O
]

0
∆Sreação = [1 x (213,74) + 2 x (192,45)] − [1 x (104,60)
+ 1 x (69,91)]

0 J
∆Sreação = 598,64 − 174,51 = + 424,13 mol x K

0
Cálculo da variação de energia livre de Gibbs (∆Greação ) para a reação A:

0 0 0
∆Greação = ∆Hreação − T x ∆Sreação

0
∆Greação = + 133610 − (25 + 273) x 424,13

0 J
∆Greação = + 133610 − 126390,74 = + 7219,26 mol

(Processo não espontâneo)

Analisando a reação B:

Equação química: CO(NH2)2 + H2O(g) → CO2(g) + 2 NH3(g)


0
Cálculo da variação de entalpia (∆Hreação ) para a reação B:

0 0 0
∆Hreação = ∑ Hprodutos − ∑ Hreagentes

0 0 0 0 0
∆Hreação = [1 x HCO 2
+ 2 x HNH 3
] − [1 x HCO(NH2 )2
+ 1 x HH 2O
]

0
∆Hreação = [1 x (−393,51) + 2 x (−46,11)] − [1 x (−333,51)
+ 1 x (−241,82)]

0 kJ
∆Hreação = −485,73 + 575,33 = + 89,60 mol (Processo endotérmico)

0
Cálculo da variação de entropia (∆Sreação ) para a reação B:

0 0 0
∆Sreação = ∑ Sprodutos − ∑ Sreagentes

0 0 0 0 0
∆Sreação = [1 x SCO 2
+ 2 x SNH3
] − [1 x SCO(NH 2 )2
+ 1 x SH 2O
]

0
∆Sreação = [1 x (213,74) + 2 x (192,45)] − [1 x (104,60)
+ 1 x (188,83)]
0
∆Sreação = 598,64 − 293,43

LIVRO OLÍMPICO – VOLUME I


214

0
J
∆Sreação = + 305,21
mol x K
0
Cálculo da variação de energia livre de Gibbs (∆Greação ) para a reação B:

0 0 0
∆Greação = ∆Hreação − T x ∆Sreação

0
∆Greação = + 89600 − (25 + 273) x 305,21

0 J
∆Greação = + 89600 − 90952,58 = − 1352,58 mol
(Processo espontâneo)

Item b) Equação da energia livre de Gibbs para a reação A:


0 0 0
∆Greação = ∆Hreação − T x ∆Sreação

0
∆Greação = + 133610 − T x 424,13

0
Critério de espontaneidade: ∆Greação <0

+ 133610 − T x 424,13 < 0

424,13 x T > 133610

133610
T>
424,13

T > 315,02 K
0
Item c) Analisando a reação A: ∆Greação = −R x T x lnK

7219,26 = −8,314 x 298 x lnK

− 7219,26
lnK =
2477,572

K = e−2,91
0
Analisando a reação B: ∆Greação = −R x T x lnK

− 1352,58 = −8,314 x 298 x lnK

− 1352,58
lnK =
− 2477,572

K = e+0,546

LIVRO OLÍMPICO – VOLUME I


215

50
(U.S. NATIONAL CHEMISTRY OLYMPIAD) O primeiro passo para a
produção de silício com alta pureza para semicondutores está representado
pela seguinte equação química: SiO2(s) + 2 C(s) → Si(s) + 2 CO(g) ΔH° = + 689,9
kJ
0 0
a) Calcule ∆Hformação para SiO2(s). Dado: ∆Hformação para CO(g) = -
110,5 kJ.mol-1.
0
b) Calcule ∆Sreação para a produção de silício puro. Dado:
J J J
SC0 = 5,7 mol x K ; SCO
0 0
= +197,6 mol x K ; SSi = 18,8 mol x K
0
e SSiO =
J
41,8 .
mol x K
0
c) Determine ∆Greação para uma temperatura igual a 25°C.
d) Determine a constante de equilíbrio.
e) Calcule a temperatura em graus de Celsius, para que o processo
0 0
seja espontâneo. Assuma que ∆Hreação e ∆Sreação e não variam com a
temperatura.

Resposta:

Item a) Cálculo da variação da entalpia do dióxido de silício: SiO 2(s) + 2 C(s)


0 0 0
→ Si(s) + 2 CO(g), ∆Hreação = ∑ Hprodutos − ∑ Hreagentes

0 0 0
0
∆Hreação = [1 x HSi + 2 x HCO ] − [1 x HSiO 2
+ 2 x HC0 ]

0 0
∆Hreação = [1 x 0 + 2 x (−110,5)] − [1 x HSiO 2
+ 2 x 0]

0
+689,9 = −221,0 − HSiO 2

0 kJ
HSiO = −910,9
2 mol

Item b) Cálculo da variação da entropia padrão da reação: SiO2(s) + 2 C(s) →


Si(s) + 2 CO(g),

0 0 0
∆Sreação = ∑ Sprodutos − ∑ Sreagentes

0 0 0
0
∆Sreação = [1 x SSi + 2 x SCO ] − [1 x SSiO 2
+ 2 x SC0 ]

0
∆Sreação = [1 x (18,8) + 2 x (+197,6)] − [1 x (41,8) + 2 x (5,7)]

0
J
∆Sreação = 414,0 − 53,20 = + 360,8
mol x K
0
Item c) Cálculo da variação da entropia padrão da reação: ∆Greação =
0 0
∆Hreação − T x ∆Sreação

0
∆Greação = + 689900 − (25 + 273) x 360,80

0
∆Greação = + 89600 − 90952,58

LIVRO OLÍMPICO – VOLUME I


216

0 J kJ
∆Greação = − 582381,60 mol (− 582,4 mol
)
(Processo espontâneo)
0
Item d) Cálculo da constante de equilíbrio: ∆Greação = −R x T x lnK

− 582381,6 = −8,314 x 298 x lnK


− 582381,6
lnK = − 2477,572 = 235,06

K = e+235,06
0
Item e) Critério de espontaneidade: ∆Greação <0

0 0 0
∆Greação = ∆Hreação − T x ∆Sreação

0
∆Greação = + 689900 − T x 360,80

0
Critério de espontaneidade: ∆Greação <0

+ 689900 − T x 360,80 < 0

360,80 x T > 689900

689900
T>
360,80

T > 1911,66 K

51
(U.S. NATIONAL CHEMISTRY OLYMPIAD) Consider the thermodynamic
data given below:
Espécies ΔHºf (kJ x mol-1) Sºf (J x mol-1 x K-1)
H+(aq) 0 0
OH (aq)
- - 229,9 -
H2O(l) - 285,83 69,95
The autoionization of water can be described according to the equation below.
Its equilibrium constant, Kw, is 1,0  10-14 at 25ºC. H2O(l) ⇌ H+(aq) + OH-(aq)
a) Calculate ∆Hº for the autoionization of water.
b) Calculate ∆Gº (at 298 K) for the autoionization of water.
c) Calculate ∆Sº for the autoionization of water and rationalize its sign.
d) Calculate Sº for OH–(aq).
e) Calculate Kw at 50ºC.

Resposta:
Item a) Cálculo da variação de entalpia padrão (∆Hº):

0 0 0
∆Hreação = ∑ Hprodutos − ∑ Hreagentes

0 0 0 0
∆Hreação = [1 x HH + + 1 x HOH− ] − [1 x HH
2O
]

LIVRO OLÍMPICO – VOLUME I


217

0
∆Hreação = [1 x 0 + 1 x (−229,9)] − [1 x (−285,83)]

0
kJ
∆Hreação = +55,93
mol

Item b) Cálculo da variação da energia livre de Gibbs padrão (∆Gº):

0
∆Greação = −R x T x lnK

0
∆Greação = −8,314 x (25 + 273) x ln (10−14 )

0
J kJ
∆Greação = +79867,48 (79,87 )
mol mol

Item c) Cálculo da variação de entropia padrão:

0 0 0
∆Greação = ∆Hreação − T x ∆Sreação

0
79,87 = + 55,93 − (25 + 273) x ∆Sreação

0
79,87 − 55,93 J
∆Sreação =− = − 8,03 x 10−2
298 mol x K

Item d) Cálculo da entropia de OH- à 25°C:

0 0 0
∆Sreação = ∑ Sprodutos − ∑ Sreagentes

0 0 0 0
∆Sreação = [1 x SH + + 1 x SOH− ] − [1 x SH
2O
]

0
−80 = [1 x 0 + 1 x SOH − ] − [1 x (69,95)]

0 J J
SOH − = −80 + 69,95 = −10,05
mol x K mol x K

dlnK ∆H0
Item e) Aplicando a equação de Van´t Hoff: dT
= R x T²

+ ∆H° 1 1
lnK W1 − lnK W2 = x{ − }
R T2 T1

+ 55930 1 1
ln (10−14 ) − lnK W2 = x{ − }
8,314 (50 + 273) (25 + 273)

+ 55930 1 1
ln (10−14 ) − lnK W2 = x{ − }
8,314 323 298

+ 55930
ln (10−14 ) − lnK W2 = x (− 2,60 x 10−4 )
8,314

LIVRO OLÍMPICO – VOLUME I


218

−32,24 − lnK W2 = − 1,75

lnK W1 = − 30,49

K W1 = e−30,49 = 5,74 x 10−14

Segunda maneira de resolução, a partir da equação livre de Gibbs:


0 0 0
∆Greação = ∆Hreação − T x ∆Sreação

0
∆Greação = + 55930 − (50 + 273) x (−80)

0
J
∆Greação = + 55930 + 25840 = + 81770
mol
0
Cálculo da constante de ionização a 50°C: ∆Greação = −R x T x lnK W

81770 = −8,314 x (50 + 273) x ln K W

81770
ln K W = − = −30,44
2685,42
K W = e(−30,44) = 6,03 x 10−14

52
(OLIMPÍADA BRASILEIRA DE QUÍMICA) Durante a titulação de uma
solução de triiodeto (I3-) com uma solução de tiossulfato de sódio (Na2S2O3)
ocorrem as seguintes reações, com seus potenciais padrão de redução:
I3- + 2e- → 3 I- E° = + 0,54 V
S4O6-2 + 2e- → 2 S2O3-2 E° = + 0,08 V
a) Escreva a reação global da redução do triiodeto pelo íon tiossulfato
e calcule o potencial de redução padrão desta reação.
b) O íon triiodeto (I3-), em solução aquosa, permanece em equilíbrio
com o iodo molecular (I2) e íon iodeto (I-), de acordo com a equação I.
Equação química: I- + I2 → I3-. Os dados termodinâmicos, a temperatura de
25°C, para essas três espécies encontram-se na tabela abaixo.
Espécie ∆H°f (kJ.mol-1) ∆G°f (kJ.mol-1) ∆S°f (J.mol-1)
I- 55,2 - 51,6 111,3
I2 22,5 16,4 137,2
I- 3 51,5 - 51,4 239,3
Calcule o valor da constante de equilíbrio, K, para a reação apresentada na
equação I, utilizando os dados da tabela.

Resposta:

Item a) Invertendo a segunda reação química, temos:

I3- + 2e- → 3 I- E° = + 0,54 V ΔG° = -2 x F x (+ 0,54)


2 S2O3-2 → S4O6-2 + 2e- E° = + 0,08 V ΔG° = -2 x F x (- 0,08) +
I3- + 2 S2O3-2 → 3 I- + S4O6-2 ΔG°TOTAL = - 2 x F x (+ 0,54) + [-2 x F x (-
0,08)]

LIVRO OLÍMPICO – VOLUME I


219

ΔG°TOTAL = - 2 x F x (+ 0,54) - 2 x F x (- 0,08)


-2 x F x E° = -2 x F x (+ 0,54) –2 x F x (- 0,08)
E° = + 0,54 – 0,08 = + 0,46V
0
Item b) Cálculo da constante de equilíbrio (K): ∆Greação = −R x T x lnK

Equação química iônica: I- + I2 → I3-

0 0
∑ Gprodutos − ∑ Greagentes = −R x T x lnK

{(1 x − 51,4)} − {(1 x − 51,6) + (1 x 16,4)}


= − 8,314 x 298 x ln K

−51,4 + 35,2 = − 8,314 x 298 x ln K

16,2
ln K = = 6,54 x 10−3
2477,572
−3 )
K = e(−6,54 x 10

LIVRO OLÍMPICO – VOLUME I


220

53
(INSTITUTO MILITAR DE ENGENHARIA) Mistura-se a água contida em dois
recipientes, designados por A e B, de forma adiabática. Cada um contém a
mesma massa m de água no estado líquido. Inicialmente, as temperaturas
são T no recipiente A e T + ΔT no recipiente B. Após a mistura, a água atinge
a temperatura final de equilíbrio térmico. Mostre que a variação de entropia
do processo de mistura é positiva. Dado: onde ∆S = m x Cp x ln(T2/T1), onde
T2 e T1 são duas temperaturas em dois estados diferentes do processo e cp é
o calor específico da água, considerado constante.

Resposta: Considere dois sistemas caracterizados pelas letras A e B,


inseridos dentro de um reservatório.

água
Sistema A: calor específico de A = c e mA
água
Sistema B: calor específico de B = c e mB

Como o processo ocorre a pressão constante, Qp = H


Como o processo é adiabático, Qp = 0, logo: H = 0

Como são dois sistemas HA +HB = 0. (Equação A)

Desenvolvendo a equação A, temos: mA x CA x (Teq – TA) + mB x CB x (Taq –


TB) = 0

Como as massas são iguais (mA = mB) e o fluido é o mesmo nos dois
reservatórios (água), então a capacidade calorífica é a mesma (cA = cB).

m x c x (Teq – TA) + m x c x (Teq – TB) = 0


m x c x (Teq – TA) = – m x c x (Teq –TB)
Teq – TA = –Teq + TB
2 x Teq = TB + TA (Equação B)
T +T
Definiremos a equação B da seguinte forma Teq = B A e sabendo que a
2
entropia de todo o sistema é dado por: S = SA +SB, temos:

Ts Ts
∆S = m x cp x ln ( ) + m x cp x ln ( )
TA TB

Ts Ts
∆S = m x cp x {n ( ) x ( )}
TA TB

T2s
∆S = m x cp x {n (T )} (Equação C)
A x TB

Substituindo a equação B na equação C, temos:

T + TB 2
( A )
∆S = m x cp x ln {[ 2 ]}
TA x TB

(TA + TB )2
∆S = m x cp x ln { }
4 x TA x TB

LIVRO OLÍMPICO – VOLUME I


221

TA2 + 2 x TA x TB + TB2
∆S = m x cp x ln
4 x TA x TB

TA2 + 2 x TA x TB + TB2
∆S = m x cp x ln
4 x TA x TB

(TA +TB )2
∆S = m x cp x ln (Equação D)
2 x √TA x TB

Sabendo que pela desigualdade das médias, a média aritmética (M.A.) é


maior ou igual à média geométrica (M.G.), ou seja, M. A. ≥ M. G.

Colocando em relação a temperatura, a função fica da seguinte maneira:

TA + TB
≥ √TA x TB
2

Manipulando a equação D apresentada acima, temos:

(TA + TB )2
∆S = m x cp x ln
2 x √TA x TB
(TA + TB )
∆S = m x cp x ln x (TA + TB )
2 x √TA x TB

≥1
(TA + TB )
≥1
2 x √TA x TB

(TA + TB )
≥⏟
√TA x TB
⏟ 2
M.A M.G.

Logo ∆S > 0, pois as temperaturas são distintas.

54
(ITA – ADAPTADO) Certa quantidade de oxigênio (considerado aqui como
gás ideal) ocupa um volume V1 a uma temperatura T1 e pressão p1. A seguir,
toda essa quantidade é comprimida, por meio de um processo adiabático e
𝑣
quase estático, tendo reduzido o seu volume para 𝑣𝑓𝑖𝑛𝑎𝑙 = 𝑖𝑛𝑖𝑐𝑖𝑎𝑙
2
. Dê o
valor da expressão do trabalho realizado sobre esse gás.

Resposta: Sendo o oxigênio ser uma molécula diatômica, então a capacidade


calorífica a volume constante será expressa pela seguinte expressão: 𝐶𝑉 =
5𝑅
2
.

Cálculo da capacidade calorífica a pressão constante: 𝐶𝑝 − 𝐶𝑉 = 𝑅

5R
Cp − =R
2
5R 7R
Cp = +R =
2 2

LIVRO OLÍMPICO – VOLUME I


222

Cálculo da relação entre a capacidade calorífica a pressão constante e a


7R
C 7
volume constante (γ): γ = Cp = 2
5R = 5 = 1,40
V
2

Sabendo que a relação entre a pressão e o volume para um processo


adiabático é dado por: p x V γ = constante
γ γ
PInicial x VInicial = Pfinal x Vfinal

1,40 1,40
PInicial x VInicial = Pfinal x Vfinal

Aplicando a equação do trabalho adiabático, temos:

(Pfinal x Vfinal − Pinicial x Vinicial )


Wad =
1− γ

21,40
( 2 − 1) x (Pinicial x Vinicial )
Wad =
1 − 1,40

21,40 − 2
( 2 ) x (Pinicial x Vinicial )
Wad =
− 0,40

5
Wad = − x (20,40 − 1) x (Pinicial x Vinicial )
2

55
(GRILLO) Um mol de um gás a uma temperatura igual a 27°C é comprimido
isotermicamente e reversivelmente a partir de um volume inicial de 10 litros
para um volume final de 0,20 L. Calcule o trabalho realizado no sistema a
partir da equação do gás real (Vm – b)P = RT, sabendo que a constante de
Van der Waals de b é igual a 0,03 L/mol.

Resposta: Sabendo que o trabalho é definido como: dW = − pexterna x dV

A partir da equação dos gases mencionado pelo problema, temos:

RxT
P=
Vmolar − b

RxT
Substituindo na equação acima, temos: dW = − (V ) x dV
molar −b

V −b
Desenvolvendo a equação, temos: W = −R x T x ln (V2 −b)
1

V2 −b
Substituindo os valores proposto pelo problema: W = −R x T x ln ( )
V1 −b

0,20−0,03
W = − 8,314 x (27 + 273) x ln ( 10−0,03 )

W = − 8,314 x 300 x ln(1,70 x 10−2 ) = +10155 J

LIVRO OLÍMPICO – VOLUME I


223

56
(CONCURSO PARA DOCENTE - IFRJ) Num recipiente de 20 litros, inseriu-
se 1,0 mol de bicarbonato de sódio. Após algum tempo, estabeleceu-se o
equilíbrio apresentado a seguir. Ao longo de todo o processo, a temperatura
permaneceu constante, 25°C. Equação química balanceada: 2 NaHCO3(s) ⇄
Na2CO3(s) + H2O(g) + CO2(g).

ΔHo298 So298
Substância
(kJ.mol-1) (J.mol-1.K-1)
NaHCO3(s) - 947 102
Na2CO3(s) - 1133 136
H2O(g) - 242 189
CO2(g) - 393 213

Admitindo que todos os gases envolvidos apresentam comportamento de gás


ideal, determine a variação da energia livre de Gibbs, a variação da energia
interna do sistema reacional e também a constante de equilíbrio para o
sistema na temperatura em questão.

Resposta:

Equação química: 2 NaHCO3(s) ⇄ Na2CO3(s) + H2O(g) + CO2(g)


𝑜
Cálculo da variação da entalpia padrão (∆𝐻𝑟𝑒𝑎çã𝑜 ):

0 0 0
∆Hreação = ∑ Hprodutos − ∑ Hreagentes

0 0 0 0 0
∆Hreação = [1 x HNa 2 CO3
+ 1 x HH 2O
+ 1 x HCO 2
− 2 x HNaHCO 3
]

0
∆Hreação = [1 x (−1133) + 1 x (−242) + 1 x (−393)
− 2 x (−947)]

0 kJ
∆Hreação = −1768 + 1894 = +126
mol

(Processo Endotérmico)
𝑜
Cálculo da variação de entropia padrão (∆𝑆𝑟𝑒𝑎çã𝑜 ):

0 0 0
∆Sreação = ∑ Sprodutos − ∑ Sreagentes

0 0 0 0 0
∆Sreação = [1 x SNa 2 CO3
+ 1 x SH 2O
+ 1 x SCO 2
− 2 x SNaHCO 3
]

0
∆Sreação = [1 x 136 + 1 x 189 + 1 x 213 − 2 x 102]

0
J
∆Sreação = [538 − 204] = +334
mol x K
o
Cálculo da variação da energia livre de Gibbs (∆Greação ):

LIVRO OLÍMPICO – VOLUME I


224

0 0 0
∆Greação = ∆Hreação − T x ∆Sreação

0
∆Greação = 12600 − T x 334

0
∆Greação = 12600 − (25 + 273) x 334

0 J
∆Greação = 12600 − 99532 = + 26468 mol

(Processo não espontâneo)

Cálculo da constante de equilíbrio (K):

0 J
∆Greação = + 26468 mol = −R x T x ln K

J J
+ 26468 = −8,314 x 298 K x ln K
mol mol x K

− 26468 = 2477,572 x ln K

− 26468
ln K =
2477,572

ln K = −10,68

K = e(−10,68)

Cálculo da variação da energia interna do sistema (∆𝑈): ∆H = ∆U + pV

∆H = ∆U + ∆n x R x T

∆H = ∆U + (nprodutos − nreagentes ) x R x T

𝐽
126000 = ∆U + (2 − 0) x 8,314 x 298 K
𝑚𝑜𝑙 𝑥 𝐾

126000 − 4955,14 = ∆U

∆U = 121044,86 J (+121,04 kJ)

57
(SELETIVA PARA A 37TH INTERNATIONAL CHEMISTRY OLIMPIAD) Uma
mistura de 11,02 mmol de H2S e 5,48 mmol de CH4 foi colocada em um reator
com um catalisador de platina e, o equilíbrio H2S(g) + CH4(g) ⇄ H2(g) + CS2(g),
foi estabelecido em 700°C e 762 torr. Retirou-se o catalisador da mistura
reacional e em seguida a mistura foi resfriada. Através de análises no
equilíbrio encontrou-se 0,711 mmol de CS2. Determine:
a) As frações molares de cada substância envolvida;
b) As porcentagens molares;
c) As pressões parciais;
d) Kp e o ΔG° para a reação em 700°C.

Resposta:

LIVRO OLÍMPICO – VOLUME I


225

Equação química: 2 H2S(g) + CH4(g) ⇄ 4 H2(g) + CS2(g)

Base de cálculo: 11,02 mmol de H2S(g) e 5,48 mmol de CH4(g).

Tabela de equilíbrio químico:

2 H2S(g) CH4(g) ⇄ 4 H2(g) CS2(g)


(mmol) (mmol) (mmol) (mmol)
Início 11,02 5,48 0 0
Reage 2α α 4α α
Equilíbrio 11,02 - 2α 5,48 - α 4α α = 0,7

Cálculo do número de mol para cada espécie gasosa no equilíbrio:

nH2 S = 11,02 − 2α = 11,02 − 2 x (0,711)

nH2 S = 11,02 − 1,422 = 9,60 mmol

nCH4 = 5,48 − α = 5,48 − 0,711 = 4,77 mmol

nH2 = 4. α = 4 x (0,711) = 2,84 mmol

nCS2 = 0 + α = 0,711 mmol

Cálculo do número de mol total (nTOTAL):

nTOTAL = nH2 S + nCH4 + nH2 + nCS2

nTOTAL = 9,60 mmol + 4,77 mmol + 2,84 mmol + 0,711 mmol

nTOTAL = 17,92 mmol

Item a) Frações molares para cada espécie gasosa:

n H2 S 9,60 mmol
X H2 S = = = 0,536
nTOTAL 17,92 mmol

nCH4 4,77 mmol


X CH4 = = = 0,266
nTOTAL 17,92 mmol

nH2 2,84 mmol


X H2 = = = 0,158
nTOTAL 17,92 mmol

nCS2 0,711 mmol


X CS2 = = = 3,97 x 10−2
nTOTAL 17,92 mmol

Item b) Cálculo do percentual (%) para cada espécie gasosa:

n H2 S 9,60 mmol
X H2 S = = = 0,536 (53,6%)
nTOTAL 17,92 mmol

nCH4 4,77 mmol


X CH4 = = = 0,266 (26,6%)
nTOTAL 17,92 mmol

LIVRO OLÍMPICO – VOLUME I


226

nH2 2,84 mmol


X H2 = = = 0,158 (15,8%)
nTOTAL 17,92 mmol
nCS2 0,711 mmol
X CS2 = n = 17,92 mmol = 3,97 x 10−2 (3,97 %)
TOTAL

Item c e d) Cálculo das pressões parciais:

PH2 S = X H2 S x PTOTAL = 0,536 x 762 torr = 408,43 torr

PCH4 = X CH4 x PTOTAL = 0,266 x 762 torr = 202,69 torr

PH2 = X H2 x PTOTAL = 0,158 x 762 torr = 120,40 torr

PCS2 = X CS2 x PTOTAL = 3,97 x 10−2 x 762 torr = 30,25 torr

Equação da constante de equilíbrio em função das pressões parciais:

P4H x PCS2
K p = P2 2 x PCH4
H2 S

Cálculo da constante de equilíbrio em função das pressões parciais:

PH42 x PCS2 (120,40)4 x 30,25


Kp = = = 188,0
PH22 S x PCH4 (408,43)2 x 202,69

Cálculo da variação da energia livre de Gibbs (ΔG°): ∆G° = −R x T x lnK p

J
∆G° = −8,314 x (700 + 273)x ln(188)
mol x K
J kJ
∆G° = −42389,1 mol (−42,39 mol ) (Processso espontâneo)

58
(INSTITUTO MILITAR DE ENGENHARIA) Um mol de um gás ideal sofre uma
expansão adiabática reversível de um estado inicial cuja pressão é Pi e o
volume é Vi para um estado final em que a pressão é Pf e o volume é Vf.
𝐶
Sabe-se que 𝛾 = 𝐶𝑝 é o expoente de Poisson, em que Cp e CV são os
𝑉
respectivos calores molares a pressão e a volume constantes. Obtenha a
expressão do trabalho realizado pelo gás em função de Pi, Vi, Pf, Vf e γ.

Resposta: Segundo o problema, o gás sofre um processo de expansão


adiabática, logo, a quantidade de calor é igual a zero (Q = 0).

ΔU = Q + W

ΔU = 0 + W

ΔU = W

Sabendo que a variação da energia interna ocorre a volume constante, sua


expressão é dada pela seguinte relação: ∆𝑈 = 𝑛 𝑥 𝐶𝑉 𝑥 ∆𝑇.

LIVRO OLÍMPICO – VOLUME I


227

Para n = 1 mol, ∆𝑈 = 𝐶𝑉 𝑥 ∆𝑇 e sabendo que a relação entre as


capacidades caloríficas é dada pela seguinte equação termodinâmica:

Cp − CV = R.

W = ∆U

W = CV x ∆T

W = CV x (Tf − Ti )

Pf x Vf Pi x Vi
W = CV x [( )−( )]
R R
CV
W= x [Pf x Vf − Pi x Vi ]
R
CV
W= x [Pf x Vf − Pi x Vi ]
CP − CV

1
W= x [Pf x Vf − Pi x Vi ]
CP
CV − 1

1 1
W= x [Pf x Vf − Pi x Vi ] = x [Pf x Vf − Pi x Vi ]
CP γ−1
C − 1
⏟V
γ

59
(INSTITUTO MILITAR DE ENGENHARIA) Considere a proposta de um
processo para a obtenção da cementita, esquematizada abaixo.

Sabe-se que a energia livre de Gibbs molar está relacionada diretamente com
a constante de equilíbrio de uma reação química, conforme a seguinte
equação termodinâmica: ∆Greação = - R x T x lnKp. Determine as frações
molares na fase gasosa, na situação de equilíbrio, e avalie se o processo é
viável.

Fe(s) Fe3C(s) CH4(g) H2(g)


ΔHof (kJ.mol-1) 0 25,10 - 74,80 0
ΔSof (J.mol-1.K-1) 27,30 104,6 186,2 130,6

Resposta:

LIVRO OLÍMPICO – VOLUME I


228

Equação química: 3 Fe(s) + CH4(g) → Fe3C(s) + 2 H2(g)


o
Cálculo da variação da entalpia padrão (∆Hreação ):
0 0 0
∆Hreação = ∑ Hprodutos − ∑ Hreagentes

0 0 0 0 0
∆Hreação = [1 x HFe 3C
+ 2 x HH 2
− 3 x HFe − 1 x HCH 4
]

0
∆Hreação = [1 x 25,10 + 2 x 0 − 3 x 0 − 1 x (−74,80)]

0 kJ
∆Hreação = [25,10 + 74,80] = + 99,90 mol (Processo Endotérmico)

o
Cálculo da variação de entropia padrão (∆Sreação ):

0 0 0
∆Sreação = ∑ Sprodutos − ∑ Sreagentes

0 0 0 0 0
∆Sreação = [1 x SFe 3C
+ 2 x SH 2
− 3 x SFe − 1 x SCH 4
]

0
∆Sreação = [1 x 104,60 + 2 x (130,6) − 3 x 27,30 − 1 x 186,2]

0
∆Sreação = [365,80 − 268,1]

0
J
∆Sreação = +97,7
mol x K

Cálculo da variação da energia livre de Gibbs:


0 0 0
∆Greação = ∆Hreação − T x ∆Sreação

0
∆Greação = 99900 − T x 97,7

0
Para T = 25°C: ∆Greação = 99900 − (25 + 273) x 97,7

0 J
∆Greação = +70785,4 mol (Processõ não espontâneo)

Cálculo da constante de equilíbrio em fução das pressões parcias (Kp):


0 J
∆Greação = +70785,4 mol
J
− R x T x lnK p = +70785,4
mol
J
− 8,314 x (25 + 273) x lnK p = +70785,4
mol
− 70785,4
lnK p = = −28,57
2477,572

K p = e−28,57 = 3,91 x 10−13

Para uma constante de equilíbrio menor que 1, significa que a o equilíbrio


está deslocado para a esquerda, ou seja, o processo não é viável para a
produção de cementita.

LIVRO OLÍMPICO – VOLUME I


229

0
Critério de espontaneidade: ∆Greação ≤0

0
∆Greação = 99900 − T x 97,7 ≤ 0

97,7 x T ≥ 99900

99900
T≥ ≥ 1022,52 K
97,7

Cálculo das frações molares (X) das espécies gasosas.

Base de cálculo: 1 atm de gás metano.

3 Fe(s) + CH4(g) ⇌ Fe3C(s) + 2 H2(g)


Início - 1 atm - 0
Reage - α - 2α
Equilíbrio - 1-α - 2α
P2H2
Cálculo do grau de dissociação (α): K p =
PCH4

(2α)²
Kp =
1−α

4α²
Kp = = 3,91 x 10−13
1−α

Sabendo que o valor do x calculado será bem baixo, logo (1 – x) → 1.

4α² = 3,91 x 10−13

3,91 x 10−13
α=√
4

α = √9,77 x 10−14

α = 3,13 x 10−7

Cálculo das frações molares a partir das pressões parcias, levando em


consideração que a pressão total seja igual a 1 atm. Em relação ao gás
hidrogênio: PH2 = X H2 x PTOTAL

PH2
X H2 =
PTOTAL

PH2
X H2 =
PTOTAL

2. α
X H2 =
1

X H2 = 2 x (3,13 x 10−7 ) = 6,26 x 10−7

LIVRO OLÍMPICO – VOLUME I


230

Em relação ao gás metano: X CH4 + X H2 = 1

X CH4 = 1 − X H2

X CH4 = 1 − 6,26 x 10−7 ≅ 1

60
(INSTITUTO MILITAR DE ENGENHARIA) O oxigênio e o hidrogênio
combinam-se, em células de combustível, produzindo água líquida e gerando
corrente elétrica. O máximo trabalho elétrico útil que essas células produzem
é dado por ΔG° = - 237 x 10³ J.mol-1. Com base nos dados fornecidos, calcule
o ponto de ebulição da água. Aproxime ΔH por ΔH° e ΔS por ΔS°. Dados
termodinâmicos:

O2(g) H2(g) H2O(l) H2O(g)


S° = 206 J.mol- S° = 131 J.mol- S° = 70,0 J.mol- S° = 189,0 J.mol-
1.K-1 1.K-1 1.K-1 1.K-1

H° = -242x10³
J.mol-1
Resposta: Tabela fornecida pelo problema:

O2(g) H2(g) H2O(l) H2O(g)


S° = 206 J.mol- S° = 131 J.mol- S° = 70,0 J.mol- S° = 189,0 J.mol-
1.K-1 1.K-1 1.K-1 1.K-1

H° = -242x10³
J.mol-1
Equação química: H2(g) + ½ O2(g) → H2O(l)
o
Cálculo da variação de entropia padrão (∆Sreação ):

0 0 0
∆Sreação = ∑ Sprodutos − ∑ Sreagentes

0 1
0
∆Sreação = [1 x SH 2O
0
− (1 x SH + x SO0 2 )]
2 2

0
1
∆Sreação = [1 x 70 − 1 x 131 − x 206]
2
0
∆Svaporização = [70 − 131 − 103]

0 J
∆Svaporização = −164
mol x K
o
Cálculo da entalpia da água no estado líquido (HH 2 O(l)
), a 25°C:
0 0 0
∆Greação = ∆Hreação − T x ∆Sreação

o 1
0
∆Greação = {HH 2 O(l)
0
− (1 x HH + x HO0 2 )} − T x ∆Sreação
0
2 2

o 1
− 237000 = {HH 2 O(l)
− (1 x 0 + x 0)} − (25 + 273) x (−164)
2
o
− 237000 = HH 2 O(l)
+ 298 x 164

LIVRO OLÍMPICO – VOLUME I


231

o J kJ
HH 2 O(l)
= − 237000 − 48872 = −285872 (−285,87 )
mol mol

Cálculo da variação de entropia padrão para o processo de vaporização


(ΔS°), a partir da equação química: H2O(l) → H2O(g):

0 0 0
∆Svaporização = ∑ Sprodutos − ∑ Sreagentes

0 0 0
∆Svaporização = [SH 2 O(g)
− SH 2 O(l)
]

0 J
∆Svaporização = [1 x 189 − 1 x 70] = +119
mol x K

Cálculo da variação de entalpia padrão para o processo de vaporização


(ΔH°), a partir da equação química: H2O(l) → H2O(g):

0 0 0
∆Hvaporização = ∑ Hprodutos − ∑ Hreagentes

0 0 0
∆Hvaporização = [HH 2 O(g)
− HH 2 O(l)
]

0 J
∆Hvaporização = −242000 − (−285872) = + 43872
mol

Como o processo ocorre no equilíbrio líquido → gasoso, a variação da


energia livre de Gibbs é igual a zero, logo: ∆G = ∆H − Tebulição x ∆S

0 = 43872 − Tebulição x 119

43872
Tebulição = = 368,67 K
119

LIVRO OLÍMPICO – VOLUME I


232

61
(OLIMPÍADA BRASILEIRA DE QUÍMICA) As energias-livres de formação do
etileno (C2H4) e do etano (C2H6) a 25ºC e 1 atm são, respectivamente, + 68,06
kJ/mol e -32,85 kJ/mol. Suas entalpias-padrão de formação são + 52,23
kJ/mol e -84,58 kJ/mol. Pergunta-se:
a) Para a redução do etileno a eteno com H2, a 25ºC e 1 atm, determine
a variação da energia livre e a variação da entropia.
b) A reação é espontânea? Justifique.
c) Como a temperatura modificaria a espontaneidade da reação?

Resposta:
Item a) Dados do problema:
kJ
2C(grafite) + 2H2(g) → C2 H4(g) ∆Gf0 = +68,06 mol
kJ
2C(grafite) + 3H2(g) → C2 H6(g) ∆Gf0 = −32,85 mol

kJ
2C(grafite) + 2H2(g) → C2 H4(g) ∆Hf0 = +52,23
mol

kJ
2C(grafite) + 3H2(g) → C2 H6(g) ∆Hf0 = −84,58 mol

Cálculo da variação da energia livre de Gibbs, invertendo a primeira equação


termodinâmica:
kJ
C2 H4(g) → 2C(grafite) + 2H2(g) ∆Gf0 = −68,06 mol
kJ
2C(grafite) + 3H2(g) → C2 H6(g) ∆Gf0 = −32,85 mol +
kJ
C2 H4(g) + H2(g) → C2 H6(g) ∆Gf0 = −100,91
mol

kJ
C2 H4(g) → 2C(grafite) + 2H2(g) ∆Hf0 = −52,23 mol
kJ
2C(grafite) + 3H2(g) → C2 H6(g) ∆Hf0 = −84,58 mol +
kJ
C2 H4(g) + H2(g) → C2 H6(g) ∆Hf0 = −136,81
mol

0 0 0
Cálculo do valor da entropia: ∆Greação = ∆Hreação − T x ∆Sreação

kJ kJ 0
−100,91 = −136,81 − 298 x ∆Sreação
mol mol

0
kJ
∆Sreação = −0,120
mol x K

LIVRO OLÍMPICO – VOLUME I


233

Item b) Sim, uma vez que a variação da energia livre de Gibbs padrão é menor
que zero.
0 𝟎 𝟎
Item c) ∆Greação = ∆𝐇𝐫𝐞𝐚çã𝐨 − 𝐓 𝐱 ∆𝐒𝐫𝐞𝐚çã𝐨 <
⏟ 𝟎
𝐜𝐫𝐢𝐭é𝐫𝐢𝐨 𝐝𝐞
𝐞𝐬𝐩𝐨𝐧𝐭𝐚𝐧𝐞𝐢𝐝𝐚𝐝𝐞
kJ
−136,81 + T x 0,120 < 𝟎
mol

T < 1140,08 K
Modificando, para temperatura maior que 1140,08 K o processo se torna não-
espontâneo.
62
(GRILLO) Calcule o valor da entalpia padrão a uma temperatura igual a
125℃, a partir da seguinte reação química balanceada: C(grafite) + H2O(g) →
CO(g) + H2(g). Considere que a entalpia padrão deste processo a 25℃ seja
igual a 131,30 kJ.mol-1, caracterizando um processo endotérmico. Dados para
a resolução do problema:
Substância C̅pmédio [J/mol.K]
C(grafite) 8,64
H2O(g) 33,58
CO(g) 29,14
H2(g) 28,84

Resposta: Para a resolução do problema será necessário utilizar a lei de


kirchoff.

Variação da entalpia padrão (∆H°): C(grafite) + H2O(g) → CO(g) + H2(g)

0 0 0
∆Hreação = ∑ Hprodutos − ∑ Hreagentes

0
0
∆Hreação = [1 x HCO 0
+ 1 x HH 2
] − [1 x HC0 + 1 x HH
0
2O
]

0
kJ
∆Hreação = 131,30
mol

Cálculo da variação da capacidade calorífica médio a pressão constante:

∆C̅pmédio = ∑ C̅p médio


− ∑ C̅p médio
dos produtos dos reagentes

∆C̅pmédio = [1 x 29,14 + 1 x 28,8] − [1 x 8,64 + 1 x 33,58]

J
∆C̅pmédio = 57,94 − 42,22 = +15,72
mol x K

Cálculo da variação de entalpia para T = 125℃:


(125+273)
0
∆H = ∆Hreação +∫ C̅pmédio dT
(25+273)

LIVRO OLÍMPICO – VOLUME I


234

398
∆H = 131300 + ∫ (15,72) x dT
298

∆H = 131300 + 15,72 x (398 − 298)

J kJ
∆H = 131300 + (15,72 x 100) = +132872 (+132,87 )
mol mol

LIVRO OLÍMPICO – VOLUME I


235

APÊNDICE

APÊNDICE A – CONVERSÃO DE UNIDADE E CONSTANTES


FUNDAMENTAIS
a) Unidade de massa atômica (u): 1u = 1,6606 x 10-24 g
b) Número de Avogadro (N): 6,0221415 x 1023 mol-1
c) Constante de Boltzmann (k): 1,3806 x 10-23 m².kg.s-2.K-1
d) Constante de Faraday (F): 96485 C.mol-1
e) Constante de Planck (h): 6,626 x 10-34 J.s-1
f) Elétron-volt (1 eV): 1,60218 x 10-19 J
g) Massa do elétron (me-): 9,11 x 10-31 kg
h) Massa do prótron (mp+): 1,67 x 10-27 kg
i) Massa do nêutron (mn): 1,67 x 10-27 kg
j) Aceleração da gravidade (g): 9,98 m.s-2
k) Volume molar (Vm): 22,4 L.mol-1
l) Carga eletrônica (e-): 1,60218 x 10-19 coulombs
m) Valor do pi (π): 3,1415
n) Raio de Bohr: 5,29 x 10-18 J

ALGUNS FATORES DE CONVERSÃO IMPORTANTES COM BASE


NO SI - VOLUME
a) 1 litro = 10-3 m³
b) 1 litro = 1000 cm³
c) 1 mililitro = 10-3 L
d) 1 litro = 1 dm³
e) 1 litro = 1000 mL

ALGUNS FATORES DE CONVERSÃO IMPORTANTES COM BASE


NO SI - PRESSÃO
a) 1 atm = 760 mmHg
b) 1 atm = 760 torr
c) 1 atm = 101325 Pa
d) 1 atm = 1,01325 bar
e) 1 atm = 14,70 psia (libras por polegada quadrada)
f) 1 torr = 1 mmHg

LIVRO OLÍMPICO – VOLUME I


236

ALGUNS FATORES DE CONVERSÃO IMPORTANTES COM BASE


NO SI - COMPRIMENTO
a) 1 km = 1000 m
b) 1 m = 100 cm
c) 1 cm = 10-2 m
d) 1 m = 39,37 polegadas
e) 1 jarda = 0,9144 m
f) 1 milha = 1,609 km
g) 1 polegada = 2,54 cm
h) 1 mícron = 1,0 x 10-6 m
i) 1 Ângstrom = 1,0 x 10-10 m
j) 1 nanômetro = 1,0 x 10-9 m
k) 1 picômetro = 1,0 x 10-12 m

ALGUNS FATORES DE CONVERSÃO IMPORTANTES COM BASE


NO SI - MASSA

a) 1 kg = 1000 g
b) 1 g = 10-3 kg
c) 1 g = 1000 mg
d) 1 mg = 10-3 g
e) 1 libra = 453,59 g
f) 1 t = 1000 kg
g) 1 t = 106 g
h) 1u = 1,6606 x 10-24 g
i) 1 t = 2000 libras

ALGUNS FATORES DE CONVERSÃO IMPORTANTES COM BASE


NO SI - ENERGIA
a) 1 caloria = 4,18 J
b) 1 caloria = 4,13 x 10-2 atm.L
c) 1 Joule = 1,0 x 107 ergs
d) 1 elétron-volts = 1,6022 x 10-19 J
e) 1 elétron-volts = 96,485 kJ.mol-1
f) 1 atm.L = 101,325 J

LIVRO OLÍMPICO – VOLUME I


237

APÊNDICE B – TABELA DE CONSTANTES QUÍMICAS


Zero Absoluto 0 K - 273,15°C

Aceleração da gravidade g 9,81 m.s-2

Número de Avogadro N0 6,02 x 1023

Constante de Faraday F ≅ 96500 C.mol-1

Constante de Coulomb K 8,998 x 109 Nm2/C2

Carga do Elétron q 1,602 x 10-19 C

Constante Gravitacional G 6,673 x 10-11 Nm2/kg2

Massa do Elétron me 9,109 x 10-31 kg

Massa do Próton mp 1,673 x 10-27 kg

Constante de Planck h 6,626 x 10-34 J.s

Velocidade da luz no vácuo c 2,997 x 108 m/s

APÊNDICE C – CONSTANTE DOS GASES (R)

R = 0,08206 atm.L.mol-1.K-1
R = 0,08206 atm.dm³.mol-1.K-1
R = 82,06 atm.cm³.mol-1.K-1
R = 8,31451 Pa.m³.mol-1.K-1
R = 8,31451 kPa.m³.kmol-1.K-1
R = 8,314 J.mol-1.K-1
R = 1,98722 cal.mol-1.K-1
R = 1,987 Btu.lbmol-1.°R-1
R = 10,73 psia. ft³. lbmol-1.°R-1
R = 62,36 torr.L.mol-1.K-1
R = 62,36 mmHg.L.mol-1.K-1
R = 0,7302 ft³.atm.lbmol-1. °R-1

LIVRO OLÍMPICO – VOLUME I


238

APÊNDICE D – DISTINÇÃO ENTRE PILHA GALVÂNICA E ELETRÓLISE


Processo de
Propriedades Pilhas Galvânicas
Eletrólise
Processo não
Espontaneidade Processo espontâneo espontâneo
Quanto ao uso de
ponte salina Utiliza Não utiliza
Sai do ânodo para o Sai do ânodo para o
Fluxo de elétrons cátodo cátodo
Ânodo = Polo
Quanto a Polaridade
negativo Ânodo = Polo positivo
Cátodo = polo Cátodo = polo
Diferença de potencial
positivo (E > 0) negativo (E < 0)

APÊNDICE E – TABELA DE POTENCIAL-PADRÃO (E° DE REDUÇÃO)

REAÇÃO DE REDUÇÃO E° (VOLTS)


Li+ −
(aq) + 1e → Li(s) -3,05
K+ −
(aq) + 1e → K (s) -2,93
Ba+2 −
(aq) + 2e → Ba (s) -2,90
+2
Sr(aq) + 2e− → Sr(s) -2,89
Ca+2
(aq)

+ 2e → Ca(s) -2,87
+
Na(aq) + 1e− → Na(s) -2,71
Mg +2
(aq) + 2e− → Mg (s) -2,37
Be+2
(aq)

+ 2e → Be(s) -1,85
Al+3
(aq)

+ 3e → Al(s) -1,66
Mn+2
(aq)+ 2e− → Mn(s) -1,18

2H2 O(l) + 2e− → H2 (g) + 2OH(aq) -0,83
Zn+2
(aq)

+ 2e → Zn(s) -0,76
+3
Cr(aq) + 3e− → Cr(s) -0,74
Fe+2
(aq)

+ 2e → Fe(s) -0,44
Cd+2
(aq)

+ 2e → Cd(s) -0,40
PbSO4(s) + 2e− → Pb(s) + SO−2
4(aq) -0,31
Co+2 −
(aq) + 2e → Co(s) -0,28
Ni+2 −
(aq) + 2e → Ni(s) -0,25
Sn+2 −
(aq) + 2e → Sn(s) -0,14
Pb+2 −
(aq) + 2e → Pb(s) -0,13
+
𝟐𝐇(𝐚𝐪) + 𝟐𝐞− → 𝐇𝟐(𝐠) 0,00
Sn+4 − +2
(aq) + 2e → Sn(aq) +0,13
Cu+2 − +
(aq) + 1e → Cu(aq) +0,15
SO−2 + −
4(aq) + 4H(aq) + 2e → SO2(g) + 2H2 O(l) +0,20
AgCl(s) + 1e− → Ag (s) + Cl−
(aq) +0,22
+2 −
Cu(aq) + 2e → Cu(s) +0,34

LIVRO OLÍMPICO – VOLUME I


239


O2(g) + 2H2 O(l) + 4e− → 4OH(aq) +0,40

I2(s) + 2e− → 2I(aq) +0,53
MnO−2 − −
4(aq) + 2H2 O(l) + 3e → MnO2(s) + 4OH(aq) +0,59
+
O2(g) + 2H(aq) + 2e− → H2 O(aq) +0,68
Fe+3 − +2
(aq) + 1e → Fe(aq) +0,77
Ag + −
(aq) + 1e → Ag (s) +0,80
Hg +2 −
2(aq) + 2e → 2Hg (l) +0,85
2Hg +2 − +2
(aq) + 2e → Hg 2(aq) +0,92
+
NO− −
3(aq) + 4H(aq) + 3e → NO(g) + 2H2 O(l) +0,96

Br2(l) + 2e− → 2Br(aq) +1,07
+
O2(g) + 4 H(aq) + 4e− → 2H2 O(l) +1,23
MnO2(s) + 4H(aq)+
+ 2e− → Mn+2(aq) + 2H2 O(l) +1,23
−2 + −
Cr2 O7(aq) + 14H(aq) + 6e → Cr(aq) +3
+ 7H2 O(l) +1,33

Cl2(g) + 2e− → 2 Cl(aq) +1,36
Au+3 −
(aq) + 3e → Au(s) +1,50
MnO− + − +2
4(aq) + 8H(aq) + 5e → Mn(aq) + H2 O(l) +1,51
Ce+4 − +3
(aq) + 1e → Ce(aq) +1,61
+
H2 O2(g) + 2H(aq) + 2e− → 2 H2 O(l) +1,77
Co+3 − +2
(aq) + 1e → Co(aq) +1,82
+
O3(g) + 2H(aq) + 2e− → O2 + H2 O(l) +2,07

F2(g) + 2e− → 2 F(aq) +2,87
Fonte: Atkins, P. W.; Paula de, J. Físico-Química”, 8ªedição, volume um.
Livros Técnicos e Científicos - LTC, Rio de Janeiro, 2008.

Observação: Os valores dos potenciais-padrão foram determinados a partir


das condições padrões, ou seja, concentração da quantidade da matéria igual
a um (1 mol.L-1), caso esteja estudando para fases gasosas, a pressão é de
1 atm e temperatura igual a 25°C.

APÊNDICE F – CAPACIDADE CALORÍFICA MÉDIA À PRESSÃO


CONSTANTE (𝑐𝑃 )
𝐽
SUBSTÂNCIA 𝑐𝑃 ( )
𝑚𝑜𝑙 𝑥 𝐾
Al(s) 24,2
Nb(s) 24,6
Nb(l) 41,7
Nb2O5(s) 131,6
Al2O3(s) 77,2
Al2O3(l) 144,9
Fe(s) 25,1
Fe(l) 41,4
Fe2O3(s) 103,7
Ar(g) 20,8

LIVRO OLÍMPICO – VOLUME I


240

Cl2(g) 33,9
H2(g) 29,0
N2(g) 29,1
O2(g) 29,4
Fonte: Atkins, P. W.; Paula de, J. Físico-Química”, 8ªedição, volume um.
Livros Técnicos e Científicos - LTC, Rio de Janeiro, 2008.

Apêndice G - TABELA DE CONSTANTES CRIOSCÓPICAS E


EBULIOSCÓPICAS

CONSTANTE CONSTANTE
SOLVENTE
CRIOSCÓPICA EBULIOSCÓPICA
Água - 1,86 0,51
Benzeno - 5,12 2,53
Ácido acético - 3,90 2,93
Fenol - 7,40 3,04
Naftaleno - 6,94 5,80
Ácido fórmico - 2,77 -
Acetona - 1,71
Álcool comum 1,22 1,22
Cânfora - 40 -
Dissulfeto de carbono - 3,80 2,37
Tetracloreto de carbono - 30 4,95

APÊNDICE H – TABELA PERIÓDICA DOS ELEMENTOS QUÍMICOS

LIVRO OLÍMPICO – VOLUME I


241

Fonte: http://iupac.org/cms/wp-
content/uploads/2015/07/IUPAC_Periodic_Table-28Nov16.jpg (Acesso dia
15/01/2021).

LIVRO OLÍMPICO – VOLUME I


242

REFERÊNCIAS BIBLIOGRÁFICAS
1. Físico-Química: Uma aplicação aos materiais. Rupen Adamian, Ericksson
Rocha e Almendra. Rio de Janeiro: COPPE – UFRJ, 2002.
2. Curso de Termodinâmica Aplicada às Máquinas. Abrahão Izecksohn;
Imprensa Nacional Rio de Janeiro – 1943.
3. Introduction to the Thermodynamics of Materials Fourth Edition. David R.
Gaskell. Taylor & Francis, New York – London, 2003.
4. Curso de Física Básica 2 – Fluidos. H. Moysés Nussenzveig. São Paulo:
Edgard Blucher, 1981.
5. Principles of Physical Chemistry Four Edition. Samuel H. Maron; Carl F.
Prutton. Editora Collier Macmillian Student Editions, New York, 1965.
6. Química Geral – Volume 2. Linus Pauling. AO LIVRO TÉCNICO SA –
Indústria e Comércio. Rio de Janeiro, 1982.
7. Química Geral – Volume 2. Brady, J. E.; Humiston, G. E. tradução de
Cristina Maria Pereira dos Santos e Roberto Barros Faria. Segunda edição.
Rio de Janeiro. Livros técnicos e científicos Editora, 1986.
8. Físico-Química – Volume 1, Levine, I. N. Tradução e revisão técnica
Edilson Clemente da Silva, Oswaldo Esteves Barcia – Rio de Janeiro: LTC
2012.
9. Termodinâmica e Física da Estrutura da Matéria – Segunda Edição. Rui
Manuel A. Dilão. Escolar Editora, 2014.
10. Antonio Braz de Pádua; Cléia Guiotti de Pádua; João Lucas Correia Silva;
Ricardo Spagnuolo Martins; Felipe Barreiro Postali; Luiz Augusto Calvo
Tiritan.Termodinâmica clássica ou termodinâmica do equilíbrio: aspectos
conceituais básicos. Ciências Exatas e da Terra, Londrina, v. 29, n. 1, p. 57-
84, jan./jun. 2008.
11. Atkins, P. W. & Jones, L. Princípios de Química – Questionando a vida
moderna e o meio ambiente – Quinta Edição. Editora bookman, 2012.
12. Atkins, P. W. & Paula de, J. Físico-Química”, 8ªedição, volume um.
Livros Técnicos e Científicos - LTC, Rio de Janeiro, 2008.
13. Atkins, P. W.; Paula de, J. Físico-Química”, 10ªedição, volume um.
Livros Técnicos e Científicos - LTC, Rio de Janeiro, 2018.
14. Crockford, H. D. e Samuel B. Knight; tradução e revisão de Horácio
Macedo, Livre docente da UFRJ – Instituto de Química. Fundamentos
de Físico-Química. Rio de Janeiro, Livros Técnicos e Científicos, 1977.

LIVRO OLÍMPICO – VOLUME I


243

15. Moore, W. J. Físico-Química - tradução da quarta edição


americana: Tibor Rabockai [e outros]. Supervisão Ivo Jordan. Editora
da Universidade de São Paulo, 1976.
16. Castellan, G. W. Physical Chemistry, segunda edição. Addison –
Wesley Publishing Company, 1964.
17. Nabuco, João Roberto da Paciência & Barros. Físico-Química. Rio
de Janeiro, Primeira edição. Editora Ao livro técnico, 1979.
18. Nabuco, João Roberto da Paciência & Barros. Química Geral. Rio
de Janeiro, Primeira edição. Editora Ao livro técnico, 1979.
19. Mahan, Bruce M. Departamento de Química, Universidade da
Califórnia, Berkeley, Segunda edição, revisada. Editora Edgard
Bluncher LTDA, United States of America, 1978.
20. Wood, J. H.; Keenan, C. W.; Bull, W. E.; Bowman, N. S.
Fundamentals of college chemistry. A Harper International Edition,
1964.
21. Luiz, A. M. Termodinâmica – Teoria & Problemas. Livros Técnicos
e Científicos - LTC, Rio de Janeiro, 2012.
22. Warren, B. A. The First law of thermodynamics in a salty ocen.
Progress in Oceanography, page 149 – 167, 70 (2006).
23. Modell, M.; Reid, R. C. Thermodynamics and its applications. New
Jersey: Prendice-Hall, 1974.
24. Stevens, B. Chemical Kinetics. Physical Chemistry Textbooks,
1965.
25. Pádua, A. B.; Pádua, C. G.; Silva, J. L. C. A história da
termodinâmica, uma ciência fundamental. Londrina: EDUEL, 2008.
26. Ebbing, D. D., Química Geral. 5ª Ed. Rio de Janeiro, Livros
Técnicos e Científicos, V. 1, 1998. 569 p.
27. Kotz, J. C.; Treichel, P.J. Química e Reações Químicas. 3ª ed. Rio
de Janeiro, Livros Técnicos e Científicos, Volumes 1 e 2.
28. Russel, J. B.; Química Geral – Segunda Edição. Pearson Makron
Books, 1994.
29. Rogado, J. A Grandeza quantidade de matéria e sua unidade, o
mol: Algumas considerações sobre dificuldades de ensino e
aprendizagem. Ciência & Educação, v.10, n.1, p. 63-73, 2004.

LIVRO OLÍMPICO – VOLUME I


244

30. Levenspiel, O. Engenharia das reações químicas. São Paulo,


Edgard Blücher Ltda., 2000.
31. Fogler, H.S.; Elements of Chemical Reaction Engineering, Pearson
Education Inc, New Jersey - USA, 4th Ed. (2006).
32. Instituto Militar de Engenharia
33. Instituto Tecnológico da Aeronáutica
34. Olimpíada Brasileira de Química
35. Olimpíada de Química do Rio de Janeiro
36. Olimpíada Iberoamerica de Química
37. Olimpíada America - U. S. National Chemistry Olympiad.

LIVRO OLÍMPICO – VOLUME I

Você também pode gostar